ENARM 5

Descargar como pdf o txt
Descargar como pdf o txt
Está en la página 1de 222

 

1. CUÁL DE LOS SIGUIENTES ENUNCIADOS SOBRE UN CICLO MENSTRUAL NORMAL


ES FALSO:

EL AUMENTO DE LOS ESTRÓGENOS Y PROGESTERONA INDUCE EL FLUJO


1. 1.
MENSTRUAL.
EL INCREMENTO DE LA FSH EN EL DÍA 28 DEL CICLO ES EL RESULTADO DE
2. 2. LA PÉRDIDA DE LA RETROALIMENTACIÓN NEGATIVA POR LOS
ESTEROIDES.
LOS FOLÍCULOS NO DOMINANTES SE VUELVEN ATRÉSICOS A CAUSA DE LA
3. 3.
DISMINUCIÓN DE LA FSH.
4. 4. LA RELACIÓN LH/FSH SE INCREMENTA ANTES DE LA OVULACIÓN.
Gráfico de respuestas
Comentario

La menstruación se debe al descenso de estrógenos y progesterona al final del ciclo (causado por
la involución del cuerpo lúteo), y no al aumento.(R1)

2. Señale la respuesta que considere INCORRECTA en relación con Entamoeba


histolytica:

Los portadores asintomáticos, aunque existen, son relativamente infrecuentes en


1. 1.
comparación con los casos de amebiasis sintomática.
El absceso hepático amebiano suele resolverse con tratamiento médico, sin necesidad de
2. 2.
realizar drenaje quirúrgico ni punción.
3. 3. Las ulceraciones colónicas adquieren un aspecto típico "en matraz".
Los amebomas consisten en la formación de masas pseudotumorales que habitualmente
4. 4.
aparecen en ciego.
Gráfico de respuestas
Comentario

Los portadores asintomáticos de Entamoeba histolytica son frecuentes. Por eso debuta con cierta
frecuencia como absceso hepático sin haber dado clínica intestinal previamente. Recuerda,
además, que es de los pocos abscesos que responden bien al tratamiento médico sin precisar
drenaje de entrada.(R1)

3. Señale la afirmación FALSA respecto al manejo de la enfermedad de membrana hialina


los RNPT:

La administración prenatal de corticoides a la madre tiene más beneficios en los embarazos


1. 1.
de > 34 semanas.
El beneficio óptimo de la administración prenatal se observa a las 24-48 horas después de
2. 2.
iniciar el tratamiento.
La administración de surfactante exógeno mejora la supervivencia de la EMH, pero no
3. 3.
disminuye la incidencia de displasia broncopulmonar.
La canalización de la vena umbilical puede tener como complicación una trombosis de la
4. 4.
vena porta.
Gráfico de respuestas
Comentario

 
 
 
 
La enfermedad de membrana hialina es la causa más frecuente de distrés respiratorio en el recién
nacido pretermino. Se debe a un déficit de surfactante, que no alcanza la superficie pulmonar hasta
la semana 34-35. Este déficit causa un augmento de tensión superficial y una tendencia de los
pulmones al colapso.

En el bebé a término, hijo de madre hipertensa e hijo de madre adicta a los opiaceos, ocurre lo
contrario, se reduce el riesgo de sufrir enfermedad de la membrana hialina.(R1)

4. Paciente de 62 años de edad valorado por bocio. Aporta un ultrasonido en el que se


objetiva un aumento de la glándula tiroidea en el contexto de múltiples nódulos que van
desde los 0,5 cm a 2,5 cm y laboratorio que incluye determinación de TSH dentro de la
normalidad. ¿Cuál sería su diagnóstico y actitud inicial?:

1. 1. Bocio multinodular. Tratamiento con I-131.


2. 2. Bocio multinodular. PAAF guiada por ecografía de los nódulos dominantes o sospechosos.
3. 3. Bocio multinodular. Cirugía.
4. 4. Bocio multinodular. Gammagrafía tiroidea.
Gráfico de respuestas
Comentario

Esta pregunta repasa el diagnóstico del bocio multinodular y la actitud en el manejo de los nódulos
tiroideos en su contexto. Es importante recordar que, aunque la mayoría de los nódulos tiroideos
son benignos y nódulos sin características ecográficas sospechosas y menores de 1- 1,5 cm no
requieren estudio citológico, los nódulos dominantes en el contexto de un bocio multinodular tienen
la misma probabilidad de malignidad que un nódulo solitario y precisan estudio con PAAF.(R2)

5. En relación a la adolescencia marque lo FALSO:

1. 1. El “estirón” del crecimiento se inicia distalmente por manos pies y extremidades.


2. 2. Los cambios puberales de la fase 1 y 2 de Tanner coinciden con la adolescencia temprana.
En la adolescencia tardía se prefiere la relación de grupo a la relación con personas del
3. 3.
sexo opuesto.
La activación neurohormonal para el inicio de la pubertad es dependiente de los factores
4. 4.
culturales.
Gráfico de respuestas
Comentario

Pregunta de crecimiento y desarrollo. Fácil porque no hace mucho que ha pasado la adolescencia
tardía, en la que si recuerdan, cambia la preferencia del grupo por el interés por el sexo opuesto.
En la temprana es donde se prefiere la relación grupal.(R3)

6. A previously healthy 6-month-old girl is brought to the pediatrician's office by her


worried mother because she has noticed that the veins on her skull seem very swollen.
She says that the child eats less recently and that she is irritable. Physical examination
shows a bulged fontanel and diastatic sutures. Her growth curve with head circumference
is in the 98th percentile. What should be done next?

1. 1. CT scan of the brain


2. 2. Skull X ray
3. 3. Lumbar puncture
4. 4. Observe and schedule next follow-up appointment in two months

 
 
 
 
Gráfico de respuestas
Comentario
CT scan of the brain. Hydrocephalus in the nursing child produces diastasis of the fontanelles, so
the head circumference is enlarged (which can be seen in shown in the percentile curves). The
consequences for development are drastic if treatment is not applied (often a ventriculo-peritoneal
shunt). The first step for an adequate diagnosis is to perform an image test. CT scan of the brain is
the test of choice as an initial test which may be complemented by an MRI depending on the
findings.(R1)

7. ¿Cuál es el lugar de afectación más frecuente en una erupción vesicular por herpes
zoster?:

1. 1. Rama oftálmica del nervio trigémino.


2. 2. Dermatomas dorsales y lumbares.
3. 3. Dermatomas cervicales.
4. 4. Miembros inferiores.
Gráfico de respuestas
Comentario
Esta es una pregunta muy fácil sobre el herpes zóster, y sobre un aspecto muy preguntable. La
localización más frecuente es el tronco, y más específicamente dorsolumbar. Otro dato a recordar
es el peligro inminente de queratitis herpética en un zoster de la cara, cuando se compromete
punta y ala de la naríz (pregunta MIR), con la derivación inmediata a un oftalmólogo que ello
implica. Recuerda que en la cara la localización más frecuente del herpes zoster es el territorio
trigeminal, específicamente la primera rama (oftámica).(R2)

8. ¿En cuál de los siguientes cuadros otológicos nunca está abolido el reflejo
estapedial?:

1. 1. Otosclerosis.
2. 2. Neurinoma del VIII par craneal.
3. 3. Síndrome de Ménière.
4. 4. Post-estapedectomía.
Gráfico de respuestas
Comentario
El reflejo estapedial puede estar abolido en un neurinoma del VIII par por estar dañada la vía
aferente del reflejo, esto es, la vía auditiva. También está ausente si se afecta la vía eferente, en
este caso, si el nervio facial está dañado. En la otoesclerosis el estribo está fijo a la ventana oval
por lo que el reflejo puede estar ausente o invertido. En una estapedectomía se extrae el estribo
por lo que no puede existir el reflejo estapedial. En el síndrome de Meniere existe reclutamiento por
ser una hipoacusia coclear, en estos casos el reflejo estapedial aparece antes de lo esperado (Test
de Metz positivo).(R3)

9. Which of the following is not a typical feature of benign arteriolar nephrosclerosis?

1. 1. It is a long-term consequence of uncontrolled hypertension.


2. 2. It is characterized by hyperplasia of the vascular walls with fibrinoid necrosis.
It is often accompanied by signs of vascular involvement in other organs, such as left
3. 3.
ventricular hypertrophy.
4. 4. If present, proteinuria is usually mild.

 
 
 
 
Gráfico de respuestas
Comentario

Las lesiones producidas por la HTA a nivel renal son un tema que no debemos dejar de estudiar de
cara al ENARM, La nefroangioesclerosis BENIGNA traduce una HTA de larga evolución (opción 1
correcta) por lo que es razonable pensar que el ventrículo izquierdo presente hipertrofia (opción 3
correcta). Clínicamente existe proteinuria (opción 4 correcta) y el riñón es pequeño por la isquemia
crónica a la que se ve sometido por hialinosis de la arteriola aferente. La opción 2 es propia de la
nefroangioesclerosis MALIGNA que traduce una HTA acelerada o maligna con repercusión visceral
grave.(R2)

10. A full-term newborn is being examined in the delivery room. The child is large for
gestational age. He has an episode of severe hypoglycemia immediately after umbilical
cord ligation. Physical examination reveals low-set ears, prominent eyes and tongue and
a big umbilical hernia, with organomegaly. Vital signs are normal. What is the most likely
diagnosis?

1. 1. Down syndrome
2. 2. Edward's syndrome
3. 3. Beckwith-Wiedemann syndrome
4. 4. Hurler's disease
Gráfico de respuestas
Comentario
Beckwith-Wiedemann Syndrome. The child in this case has phenotypic traits of a chromosomal or
genetic disorder. Sons of mothers suffering from gestational diabetes are bigger than expected, but
their phenotype is normal, just like to healthy children. The association of macrosomia, umbilical
hernia and macroglossia is typical of Beckwith-Wiedemann Syndrome. Hypoglycemia is explained
by abnormal enlargement of insulin islets of the pancreas. The manifestations of lysosomal storage
diseases have a later age of onset.(R3)

11. En relación a la fibrosis quística (FQ), todas las afirmaciones siguientes son ciertas,
EXCEPTO una, ¿cuál?

El diagnóstico por despistaje neonatal se realiza mediante la determinación del


1. 1. tripsinógeno o tripsina inmunorreactivos séricos, que se eleva cuando existe afectación
pancreática.
El hallazgo de B. cepacea, P. aeruginosa o S. aureus en vía respiratoria sugiere fuertemente
2. 2.
el diagnóstico de fibrosis quística.
3. 3. La manifestación digestiva más característica es la hepatopatía, de gravedad variable.
4. 4. En las primeras semanas de vida, el test del sudor puede no ser fiable.
Gráfico de respuestas
Comentario

Todas las respuestas son ciertas salvo la 3. La hepatopatía no es la manifestación digestiva más
característica puesto que si existe, es de gravedad variable, y hay varias etiologías diferentes que
pueden producirlo. Sin embargo la insuficiencia pancreática exocrina con maldigestión y diarrea
crónica, el íleo meconial, las obstrucciones intestinales y el prolapso rectal recurrente en el niño
son fenómenos que nos hacen pensar inmediatamente en la posibilidad de que haya una FQ
subyacente.(R3)

 
 
 
 
12. Mujer de 67 años que está diagnosticada desde hace 20 años de estenosis mitral de
etiología reumática. Actualmente se encuentra en clase funcional III-IV de la NYHA y en
una ecocardiografía realizada recientemente se veía una válvula severamente calcificada,
con un pequeño flujo de regurgitación mitral asociado y con área estimada de 0,9 cm2.
¿Qué tipo de tratamiento propondría a esta paciente?:

Tratamiento con valvuloplastia percutánea para mejorar la clase funcional y


1. 1.
posteriormente sustitución valvular.
2. 2. Cirugía de sustitución valvular por prótesis biológica.
3. 3. Cirugía de sustitución valvular por prótesis mecánica bidisco.
4. 4. Cirugía de sustitución valvular por prótesis mecánica monodisco.
Gráfico de respuestas
Comentario
Pregunta importante, que sirve para repasar dos temas: la Estenosis Mitral y la elección del tipo de
prótesis. En este caso, la paciente presenta una EM severa y está en clase funcional III- IV, por
tanto está indicado el tratamiento mecánico. La anatomía valvular no es favorable (válvula
severamente calcificada) y además presenta regurgitación mitral asociada, así que la valvuloplastia
NO podría realizarse. Por tanto, el tratamiento de elección sería la cirugía de sustitución valvular.
Ahora debemos decidir qué tipo de prótesis estaría indicada (mecánica o biológica). Las prótesis
mecánicas tienen la ventaja de que son de duración ilimitada, y su inconveniente es que requieren
anticoagulación de por vida, por el alto riesgo embolígeno. Las prótesis biológicas son de duración
limitada (necesidad de sustituir la válvula en el 30% de los pacientes a los 10 años y en el 50% a
los 15 años), pero no requieren anticoagulación de por vida. Por todo esto, las prótesis mecánicas
son las más utilizadas en los pacientes de menos de 65- 70 años, y las prótesis biológicas sólo se
utilizan en pacientes ancianos (mayores de 70 años para la válvula mitral), o con
contraindicaciones para la anticoagulación. En este caso la paciente tiene 67 años y no presenta
contraindicaciones para el tratamiento anticoagulante, así que está indicada la sustitución valvular
por una prótesis mecánica (las bidisco son las más usadas).(R3)

13. La menopausia tardía aumenta el riesgo de desarrollar un cáncer de mama:

1. 1. Si hay antecedentes familiares de cáncer de mama.


2. 2. En casos de menarquia precoz.
3. 3. En todas las mujeres.
4. 4. En multíparas.
Gráfico de respuestas
Comentario

Pregunta muy fácil acerca de los factores de riesgo del cáncer de mama.

La menopausia tardía, al igual que la menarquia precoz, constituyen un aumento de la estimulación


estrogénica a los órganos sensibles y, consecuentemente, son un factor de riesgo para desarrollar
tumores estrogenodependientes independientemente de otros factores.(R3)

14. En un RN la glucemia es normal por lo siguientes mecanismos EXCEPTO:

1. 1. Gluconeogenesis.
2. 2. Elevación de insulina.
3. 3. Elevación de glucagon.
4. 4. Reservas de glucógeno.
Gráfico de respuestas

 
 
 
 
Comentario

Pregunta difícil sobre fisiología neonatal. La insulina no es una hormona que aumente en el RN por
el parto.(R2)

15. What should we recommend the family of a girl who has recently been diagnosed with
whooping cough?

1. 1. No prophylaxis is required
2. 2. 14-day preventive treatment with erythromicine - all household contacts
3. 3. DTP vaccine - all household contacts
4. 4. Keep the child hospitalized until symptoms disappear
Gráfico de respuestas
Comentario
14-day preventive treatment with erythromycin - all household contacts. Whooping cough is rare in
developed countries since the introduction of an effective vaccine. Nevertheless, we have to
consider population at risk (immigrants, marginalized or refusing to vaccination). When a child is
diagnosed, prophylaxis is required for household contacts. Best plan is a macrolide antibiotic for
two weeks.(R2)

16. Lactante de 6 meses de edad, cuya madre refiere que desde hace tres horas presenta
varias “ronchas ”de color rojo en la piel del abdomen y parte del tórax. Señale el posible
agente causal si sospecha de una picadura o mordedura:

1. 1. Araña.
2. 2. Mosquito.
3. 3. Acaro.
4. 4. Pulga.
Gráfico de respuestas
Comentario

Cuadro clínico típico de una picadura de pulga. Son generalmente pápulas en un patrón no
folicular, que pueden producir uriticaria papular, pruriginosas, generalmente en tórax y
abdomen.(R4)

17. Paciente masculino de 17 años, de raza negra. HIV (+). Serología para hepatitis B y C
(-), que desarrolla síndrome nefrótico. ¿Qué afectación glomerular sería la más
probable?:

1. 1. GN de cambios mínimos.
2. 2. GN focal y segmentaria.
3. 3. Enfermedad de Berger.
4. 4. Enfermedad de la membrana basal adelgazada.
Gráfico de respuestas
Comentario

La GN focal y segmentaria (GNFS) puede ser debida a múltiples causas. Entre ellas, destacan la
uropatía obstructiva, la heroína, el VIH, la obesidad y el trasplante renal. Clínicamente, la GNFS se
manifiesta como un síndrome nefrótico con niveles normales de complemento. También se sabe

 
 
 
 
que es más frecuente en la raza negra, como es el caso del paciente de la pregunta (respuesta 2
correcta).(R2)

18. Physical examination of a 3-day-old newborn reveals that he is hyporeactive and


slightly unresponsive to stimuli compared to the previous days. His mother says that he
has been quite weak for the past 2 hours and that it seems that he stops breathing
occasionally, with a bluish color appearing on his lips. Further and thorough physical
examination shows pallor, pin-point sized petechiae over his chest, shallow breathing
with tachypnea, a soft and depressible abdomen and a bulging fontanelle. He is afebrile.
He is a full-term newborn, with a weight of 3200 gr and vacuum-assisted vaginal delivery.
He has been adequately immunized, with no relevant medical history except for a positive
direct Coombs' test. What is the most likely diagnosis?

Intracranial hemorrhage due to thrombopenia secondary to idiopathic thrombocytopenic


1. 1.
purpura
2. 2. Intracranial hemorrhage due to hemorrhagic disease of the newborn
3. 3. Intracranial hemorrhage due to birth trauma
4. 4. Intracranial hemorrhage due to platelets antigen isoimmunization
Gráfico de respuestas
Comentario
Intracranial hemorrhage due to platelets antigen isoimmunization. A bulging fontanelle is a sign of
increased intracranial pressure in infants. The acute onset of the symptoms described in this case
speaks in favour of intracranial hemorrhage. The statement provides a clue: the positivity of the
direct Coombs test. Circulating antibodies induce an immune thrombocytopenia with subsequent
risk of fetal bleeding.(R4)

19. En relación con la placenta humana, NO es cierto que:

1. 1. Cada cotiledón recibe una rama de la arteria umbilical.


2. 2. La sangre materna llega a través de las arteriolas espirales.
3. 3. El CO2 y el O2 atraviesan la placenta por difusión.
4. 4. Los aminoácidos atraviesan la placenta por difusión y transporte activo.
Gráfico de respuestas
Comentario

Pregunta sobre la fisiología placentaria. Las arterias umbilicales llevan la sangre desde el feto
hasta la placenta, ramificándose en cada cotiledón. La circulación uteroplacentaria se inicia cuando
las arterias espirales entran en la cámara hemática e inyectan toda su sangre en el espacio
intervelloso. Los gases pasan por gradiente de concentración por medio de difusión simple. El paso
de la glucosa a través de la placenta se realiza por difusión facilitada. Los aminoácidos pasan la
barrera placentaria consumiendo energía por transporte activo, pero no por difusión facilitada, ya
que la concentración en el lado materno es menor; por lo tanto, la opción falsa es la 4.(R4)

20. A 4-year-old child with a history of atopic dermatitis who has recently suffered an
upper respiratory cough, presents with progressive respiratory distress. Physical exam
shows use of accessory respiratory muscles, "seal bark" cough and a marked inspiratory
stridor. Oxygen saturation is getting lower and the child is tachypneic (respiratory rate 50
rpm). What should be done as a first measure?

1. 1. Orotracheal intubation.

 
 
 
 
2. 2. Tracheostomy.
3. 3. Epinephrine administration.
4. 4. Chest X ray.
Gráfico de respuestas
Comentario
Epinephrine administration. The child is suffering from croup, also known as
laryngotracheobronchitis. This condition is often triggered by an acute viral respiratory infection. The
mucosal edema causes swelling with a hyperactive response. Airway obstruction is responsible for
the characteristic “seal bark cough, which usually worsens at night and with crying. Fever may also
be present. The first measure is to administer nebulized epinephrine, which reduces croup severity
within 10–30 minutes.(R3)

21. Un paciente con 63 años acude a su consulta por dolor lumbar sordo de dos meses
de evolución. Refiere haber presentado hematuria autolimitada en dos ocasiones que no
se ha acompañado de otra sintomatología. Los urocultivos han sido repetidamente
negativos. Presenta citologías en orina positivas, con una ecografía reno-vesico-
prostática que evidencia una lesión vesical de 2 cm, pediculada, en pared lateral
izquierda. Se le realiza un UR-TAC que evidencia una lesión ocupante de espacio en
pelvis renal derecha de 2,5 cm sugestiva de tumor urotelial aparentemente limitado a la
cavidad ureteral. En este caso le propondrá al paciente:

1. 1. Nefrectomía parcial RTU de la lesión vesical.


2. 2. Nefroureterectomía radical con rodete vesical derecho más RTU de la lesión vesical.
3. 3. Pieloureterectomía segmentaria.
4. 4. Nefroureterectomía radical con cistoprostatectomía radical.
Gráfico de respuestas
Comentario
En cuanto al tumor de urotelio superior se debe aplicar el tratamiento de elección, así como a la
lesión vesical, es decir, resección para comprobar si se trata de un tumor superficial o infiltrante y
actuar a continuación en consecuencia.(R2)

22. La causa mas frecuente de epistaxis en niños es:

1. 1. Trauma.
2. 2. Sinusitis.
3. 3. Pólipos.
4. 4. Trombocitopenia.
Gráfico de respuestas
Comentario

La causa más frecuente de epistaxis en niños es la traumática, por lo que la respuesta correcta es
la 1.(R1)

23. El síndrome de ovarios poliquísticos aumenta la incidencia de:

1. 1. Tumor ovárico.
2. 2. Hemorragia uterina por atrofia endometrial.
3. 3. Cáncer de endometrio.
4. 4. Mola hidatiforme.

 
 
 
 
Gráfico de respuestas
Comentario

Concepto muy importante que si domina el tema no se olvidará jamás. Al presentar ciclos
anovulatorios, las mujeres con SOP presentan un incremento de estrógeno, sin progestágeno, por
lo que no menstruan, aumentando de esta manera el riesgo de cáncer de endometrio.

No se confunda con tumores de ovario, ya que el SOP, disminuye el riesgo para este tipo de
neoplasia.(R3)

24. La cirugía moderna del cáncer gástrico, tiene fundamento en la disección de las
diferentes estaciones ganglionares. La extirpación del grupo Nº9, corresponde a:

1. 1. Arteria coronaria estomáquica.


2. 2. Arteria esplénica.
3. 3. Arteria hepática.
4. 4. Tronco celíaco.
Gráfico de respuestas
Comentario

Pregunta muy complicada sobre los relevos ganglionares. No se preocupes si las has fallado.(R4)

25. Acude a su consulta una paciente de 83 años con un bocio multinodular. No presenta
disnea, disfagia ni disfonía. Aporta hormonas tiroideas con T3, T4 y TSH normales. Al
elevar los brazos presenta ingurgitación yugular, plétora facial y síncope. ¿Cuál es el
tratamiento de elección?:

1. 1. Hormona tiroidea en dosis supresora.

 
 
 
 
2. 2. Radioyodo.
3. 3. Cirugía.
4. 4. Vigilancia periódica.
Gráfico de respuestas
Comentario

Esta pregunta hace referencia al tratamiento del bocio multinodular (BMN). Nos presentan a una
paciente diagnosticada de BMN con hormonas tiroideas dentro de límites normales. En esta
situación, el tratamiento correcto sería realizar seguimiento con determinación de hormonas y
ecografía de forma periódica. Sin embargo, cuando el paciente presenta síntomas compresivos
como la disfagia, disnea o signo de Pemberton, está indicada la intervención quirúrgica,
realizándose una tiroidectomía subtotal (respuesta correcta 3) El radioyodo se encontraría indicado
en caso de hiperfunción, y el tratamiento con dosis supresoras de levotiroxina parece poco
apropiado, ya que en el BMN casi nunca hace regresar el bocio.(R3)

26. ¿Cuál es la enfermedad asociada a virus papiloma humano 18?

1. 1. Verrugas planas.
2. 2. Cáncer cervical.
3. 3. Condiloma acuminado.
4. 4. Cáncer de piel.
Gráfico de respuestas
Comentario

Pregunta que no puede fallar para el nacional se ha preguntado en el ENARM tal cual en repetidas
ocasiones.

VPH 16 y 18 se asociana CaCu; respuesta 2 correcta.(R2)

27. A 6-year-old child is brought to the emergency department presenting with joint pain
and abdominal pain accompanied by nausea and one bloody diaper. Urinalysis shows 10
RBC/hpf, the rest is normal. She had an respiraory tract infection the past week. Physical
examination shows bilateral swollen lesions around her ankles that do not blanch when
digital pressure is applied. Which of the following appears as a complication of this
condition?

1. 1. Appendicitis
2. 2. Reye's syndrome
3. 3. Volvulus
4. 4. Intussusception
Gráfico de respuestas
Comentario
Intussusception. The condition described is Schönlein-Henoch purpura, a type of leucocytoclastic
vasculitis. The hallmark sign of this disease is the presence of cutaneous lesions on the lower limbs
and buttocks but it is a multisystemic disorder with renal involvement with transient hematuria and
bowel involvement, with bowel-wall edema that can cause an increase in peristalsis and
intussusception as a complication.(R4)

28. Mujer de 36 años, en el tercer embarazo, de 35 semanas de amenorrea. En la


exploración física llama la atención una altura uterina menor a la que corresponde. En el

 
 
 
 
ultrasonido se ve estancamiento del crecimiento de los diámetros abdominales respecto
a la eco anterior, con diámetro biparietal y longitud femoral normales. El usg-doppler fetal
muestra aumento de resistencia al flujo en arterias placentarias. El registro
cardiotocográfico basal es no reactivo. ¿Cuál sería la siguiente actuación para valorar el
estado fetal?

1. 1. Registro cardiotocográfico estresante.


2. 2. Determinación de HCG, alfa feto-proteína y estriol no conjugado.
3. 3. Inducción del parto.
4. 4. Amniocentesis tardía.
Gráfico de respuestas
Comentario

Nos presentan un caso de CIR asimétrico o tipo II, dado que la relación diámetro biparietal /
diámetro abdominal está aumentando (por estancamiento del crecimiento del DA) y nos piden que
prueba solicitaríamos para valorar el bienestar fetal en el tercer trimestre (35 semanas de
gestación). Entre las 4 opciones posibles la más indicada será realizar un RCTG estresante (o
prueba de Pose).

•   Los marcadores bioquímicos que nos apuntan (BHCG, AFP y estradiol conjugado) se
utilizan sobretodo en el screening prenatal de cromosomopatías en el 1º y 2º trimestre.
•   Claramente no estaría indicado una inducción de parto, dado que todavía no queda claro si
existe sufrimiento fetal grave (u otra indicación para la misma).
•   La amniocentesis tardía se suele realizar en la semana 32 para medir la madurez pulmonar
fetal, medir la bilirrubina en sospecha de isoinmunización Rh, evacuadora en caso de
polihidramnios o amnioinfusión en oligoamnios o el el tratamiento de la transfusión feto-
fetal de gemelares. Este tipo de amniocentesis no se usa por tanto para el diagnóstico
prenatal.

(R1)

29. Daniel sufre diabetes tipo I desde hace 12 años, y acude anualmente a su oftalmólogo
para revisar su fondo de ojo. Hasta ahora, no ha presentado ningún signo de retinopatía
diabética. En el momento en que aparezca, ¿cual de las siguientes es la lesión más precoz
que esperaría encontrar?

1. 1. Microaneurismas
2. 2. Exudados lipídicos
3. 3. Neovasos
4. 4. Hemorragia retiniana
Gráfico de respuestas
Comentario
Los microaneurismas son la lesión más precoz que podemos detectar en un fondo de ojo diabético.
A medida que avanza la enfermedad se añaden otras lesiones que están incluidas en la fase de
retinopatía diabética no proliferativa, como los exudados lipídicos, algodonosos, hemorragias
intrarretinianas e IRMAS. La presencia de Neovasos define a la retinopatía diabética como
Proliferativa, indicando la necesidad de Panfotocoagulación del paciente.(R1)

 
 
 
 
30. Ante un paciente en el que sospechemos participación etiológica de la flora Gram
positiva, ¿qué antimicrobiano NO utilizaremos?

1. 1. Tigeciclina.
2. 2. Linezolid.
3. 3. Colistina.
4. 4. Daptomicina.
Gráfico de respuestas
Comentario

Una pregunta difícil sobre antibióticos de reciente introducción en clínica.

La colistina es un antibiótico de la familia de las polimixinas. Se trata de una familia de péptidos


poco difusibles, con cierta toxicidad cuando se suministran por vía sistémica (neurotóxicos y
nefrotóxicos). Como mecanismo de acción, atacan las membranas celulares, destruyéndolas por
su efecto surfactante.

Su actividad está restringida a bacterias gramnegativas (respuesta 3 correcta), lo mismo que


sucede con el aztreonam. En clínica, la colistina se emplea con poca frecuencia. Suele restringirse
a infecciones muy graves, con riesgo vital y resistencias frente a otros antibióticos. Dado que
apenas se absorbe desde el tracto gastrointestinal, se utiliza por vía parenteral.(R3)

31. ¿Cuál de los siguientes es una característica de la forma juvenil de la leucemia


mieloide crónica?:

1. 1. Hepatoesplenomegalia en la mayoría de los casos.


2. 2. Buen control con el tratamiento con Hidroxiurea.
3. 3. Cromosoma Filadelfia.
4. 4. Supervivencia prolongada con tendencia a la evolución a mielofibrosis.
Gráfico de respuestas
Comentario

Pregunta de dificultad elevada. En la infancia se reconocen dos tipos de LMC: la forma adulta y la
denominada forma juvenil. Esta ultima es en realidad una leucemia mielomonocitica subaguda o
crónica de la infancia, es decir, un síndromes mielodisplásicos sin cromosoma Filadelfia (opción 3
falsa), pero con ciertos rasgos característicos como, la edad de los pacientes es inferior a 5 años,
la frecuente presencia de erupciones cutáneas, adenopatías u otras localizaciones leucémicas
extramedulares (opción 1 correcta), una menor leucocitosis y una mayor resistencia al tratamiento,
con un acortamiento de la supervivencia (opciones 2 y 4 falsas). En cambio la forma adulta, afecta
a niños mayores de 5 años, es Ph- positivo y resulta en todo indistinguible de la LMC del
adulto.(R1)

32. Hombre de 72 años que acude por disfagia progresiva a sólidos y líquidos de 4 meses
de evolución. Asimismo refiere perdida de 5 kg de peso, astenia e insomnio. ¿Cuál es la
primera exploración a realizar?:

Una radiografía de tórax en el que seguramente se observe un nivel hidroaéreo en esófago


1. 1.
cervical.
2. 2. Una gastroscopia.
3. 3. TC toraco-abdominal con contraste.
4. 4. Fibrobroncoscopia.

 
 
 
 
Gráfico de respuestas
Comentario

Ante esta situación, lo primero a descartar es una neoplasia esofágica por lo que la primera
exploración a realizar es una gastroscopia. Asimismo y en caso de que se tratase de una acalasia
(patología caracterizada por una relajación incompleta del esfínter esofágico inferior en relación a
la deglución), debe realizarse igualmente una gastroscopia para descartar fuese secundaria.(R2)

33. Indique en cuál de los siguientes parámetros NO se basa el pronóstico del cáncer de
mama:

1. 1. Número de ganglios afectados.


2. 2. Tamaño del tumor.
3. 3. Tipo histológico.
4. 4. Presencia de telorrea.
Gráfico de respuestas
Comentario

Pregunta bastante sencilla sobre los factores pronósticos del cáncer de mama. El número de
ganglios afectos es el más importante como elemento pronóstico, el tamaño tumoral mayor de 2
cm. También influyn en el pronóstico el grado de diferenciación histológica (G2- moderadamente
diferenciado, G3- pobremente diferenciado). La presencia de telorrea, en cambio, apenas nos
altera el pronóstico del tumor.(R4)

34. Una gestante de 34 semanas, sana, embarazo normo evolutivo hasta ahora, ingresa
por rotura de membranas desde 8 horas antes y amenaza de parto pretérmino. No tiene
fiebre ni signos de infección. El cuello uterino no está dilatado. Se detectan contracciones
uterinas. El feto está vivo, no tiene malformaciones por ecografía y el registro de la
frecuencia cardíaca fetal es normal. ¿Qué actitud tomaría?

1. 1. Tratamiento de uteroinhibición.
2. 2. Tratamiento de uteroinhibición sólo.
3. 3. Dejar evolucionar el parto.
4. 4. Cesárea.
Gráfico de respuestas
Comentario

Una pregunta importante sobre la conducta obstétrica ante el parto pretérmino.

Los mecanismos que desencadenan el parto no son todavía bien conocidos. Se sabe que la
oxitocina, tanto materna como fetal, influye en la provocación del parto. También influyen otras
hormonas, como las suprarrenales maternas y fetales, así como el tono simpático. El parto a
término aparece entre las semanas 37 y 42. Un 10-15% de los partos se producen entre las
semanas 28 y 37. Por tanto, la definición de parto pretérmino es puramente cronológica.

El tratamiento frente a la amenaza de parto prematuro se basa en la eliminación de las


contracciones (tocolisis) y la aceleración de la madurez pulmonar fetal (corticoides). Cuando se
produce una rotura prematura de membranas, se añade antibioterapiapara prevenir la infección
amniótica, normalmente con ampicilina. Sobre el manejo obstétrico de esta situación debemos
tener claros los siguientes principios:

 
 
 
 
1. A partir de la semana 34,el feto se considera maduro desde el punto de vista pulmonar. En estos
casos no está indicado utilizar tocolíticos. Se permite la evolución espontánea del parto, estén o no
rotas las membranas. La única excepción es cuando la bolsa está íntegra y el cérvix esté sin
modificar. En estos casos se comprueba el bienestar materno y fetal, indicando reposo domiciliario.

2. Antes de la semana 34, depende de cómo esté la bolsa.

•   - Bolsa íntegra.

1. Cérvix sin modificar: tocólisis + corticoides. Intentar frenar el parto.

2. Cérvix de parto: el parto ya no se puede detener, dejar que evolucione.

•   - Bolsa rota.

1. Cérvix sin modificar: tocólisis + corticoides + antibióticos.

2. Cérvix de parto: dejar que evolucione espontáneamente.

En fetos de menos de 34 semanas, si dejamos evolucionar el parto, debe intentarse acortar el


expulsivo todo lo posible. Se realiza episiotomía amplia y precoz, si la vía va a ser vaginal. Si la
presentación no es cefálica, sería de elección una cesárea.(R3)

35. Ante el diagnóstico de un Síndrome de Transfusión Feto-Fetal en semana 22, el


tratamiento de elección será:

Realizar un amniodrenaje en el feto transfundido con polihidramnios y una amnioinfusión


1. 1.
en el feto donante con oligoamnios.
Realizar ventanas amnióticas en la membrana de separación para equilibrar el volumen de
2. 2.
líquido.
3. 3. Realizar coagulación láser por fetoscopia de las anastomosis veno-venosas.
4. 4. Realizar coagulación láser por fetoscopia de las anastomosis arterio-venosas.
Gráfico de respuestas
Comentario
El tratamiento del síndrome de transfusión feto- fetal es la coagulación láser de las anastomosis
vasculares que provocan esta situación que son de tipo arterio- venosas. No es preciso esperar a
la semana 24. El amniodrenaje o la realización de ventanas amnióticas son opciones que no
resuelven etiológicamente la complicación y sólo neutralizan las alteraciones en el líquido
amniótico quedando relegadas a un segundo plano.(R4)

36. Mujer de 30 años sometida a esplenectomía por trombocitopenia inmune primaria que
comienza en el 5º día posoperatorio con picos febriles de 39º recurrentes. La causa más
probable de la fiebre en esta paciente es:

1. 1. Infección preexistente que pasó desapercibida.


2. 2. Reacción transfusional a los concentrados de plaquetas.
3. 3. Atelectasia pulmonar.
4. 4. Absceso intraabdominal.
Gráfico de respuestas

 
 
 
 
Comentario
Nos encontramos ante un caso de fiebre (en picos) durante el postoperatorio de una
esplenectomía (en el 5º día). Teniendo en cuenta el tiempo de postoperatorio en el que nos
encontramos (después del 3º día) debemos pensar como causa más frecuente en una causa
infecciosa (Herida quirúrgica, ITU, Neumonía, Abscesos intraabdominales o fugas o fístulas en el
caso de cirugía del tubo digestivo). Teniendo en cuenta que nos encontramos en el 5º día de
postoperatorio y que la fiebre es en agujas o picos la causa más frecuente de la misma es la
presencia de un absceso intraabdominal, además el resto de opciones son causa de fiebre en el
postoperatorio inmediato o en las primeras 24 horas.(R4)

37. A 32-year-old female with HIV infection comes to her physician with complaints of
dysphagia and pain on swallowing for the last 4 days. She says that she occasionally
feels substernal burning. No abnormal findings are seen on examination of her oral cavity.
Lungs are clear to auscultation. She is currently taking zidovudine, lamivudine, nelfinavir
and trimethoprim-sulfamethoxazole. His last CD4 count is 100 cells/microL. What is the
most appropriate next step in management?

1. 1. Oral fluconazole.
2. 2. Oral omeprazole.
3. 3. Oral ganciclovir.
4. 4. Oral acyclovir.
(R1)

38. Señale la respuesta FALSA referente al infarto intestinal:

1. 1. La trombosis venosa es una de las causas más frecuentes.


El choque hipovolémico puede ocasionar isquemia sin haber alteración de los vasos
2. 2.
intestinales.
3. 3. Los exámenes de laboratorio del infarto muestra leucocitosis con desviación a la izquierda.
La aparición de una acidosis metabólica hiperkaliémica tras la resección del segmento
4. 4.
afectado es una complicación frecuente de la terapéutica de este cuadro.
Gráfico de respuestas
Comentario
La isquemia mesentérica aguda se caracteriza por la aparición de dolor brusco con irritación
peritoneal. Por isquemia se produce lesión de la pared intestinal, cursando con dilatación de asas,
leucocitosis con desviación, acidosis y por último, perforación. Se produce con más frecuencia por
una embolia en pacientes con fibrilación auricular u otro proceso embólico, por tanto, el método
diagnóstico de elección será la arteriografía que detectará un stop brusco de la vascularización a
nivel de mesentérica superior fundamentalmente. Menos frecuente es por trombosis o por bajo
gasto sin necesidad de lesión vascular pero no por trombosis venosa. El tratamiento será la
embolectomía o tromboendarterectomía pero en caso de gangrena o perforación, resección del
segmento afecto, pudiendo producirse acidosis metabólica hiperkaliémica tras la misma por
compensación renal.(R1)

39. ¿Cuál es el factor pronóstico más importante en un cáncer de mama?:

1. 1. Tamaño tumoral.
2. 2. Afectación ganglionar.
3. 3. Multicentricidad.

 
 
 
 
4. 4. Receptores estrogénicos.
Gráfico de respuestas
Comentario

La diseminación metastásica a los ganglios axilares (N) es el factor pronóstico más significativo y
es utilizado como criterio para indicar un tratamiento sistémico adyuvante.(R2)

40. A 50-year-old man comes to the emergency department with symptoms of lethargy
and confusion. His past medical history is significant for alcoholic cirrhosis. Which of the
following is the most unlikely diagnosis?

1. 1. Central pontine myelinolysis.


2. 2. Hepatic encephalopathy.
3. 3. Hypoglycemic encephalopathy.
4. 4. Subdural hematoma.
Gráfico de respuestas
Comentario

La mielinolisis pontina es un cuadro que se produce en pacientes con hiponatremia severa a los
que se corrige de forma excesivamente rápida la hiponatremia con sueroterapia intravenosa. Esta
complicación, por tanto, no tiene por qué ocurrir en este paciente, ya que no ha estado
hospitalizado ni ha recibido ningún tratamiento que pueda justificarlo.

Por otra parte, la clínica que nos mencionan tampoco es demasiado sugestiva de este cuadro. Lo
verdaderamente característico de la mielinolisis central pontina es el síndrome del cautiverio, que
cursa con tetraplejia y afectación de la motilidad ocular horizontal (conserva solamente el parpadeo
y la motilidad ocular en plano vertical).

Dado que no encajan ni la clínica ni los antecedentes, la respuesta menos probable es la 1. El


resto de las alteraciones son posibles en un paciente cirrótico (encefalopatía alcohólica,
encefalopatía hipoglucémica) o pueden ser complicaciones del alcoholismo (hemorragia
subaracnoidea, hematoma subdural).(R1)

41. Todas estas entidades pueden aparecer durante el tratamiento de la artritis


reumatoide con D-penicilamina, EXCEPTO:

1. 1. Lupus inducido.
2. 2. Hipotiroidismo.
3. 3. Miastenia gravis.
4. 4. Síndrome de Goodpasture.
Gráfico de respuestas
Comentario

La D-penicilamina es uno de los fármacos utilizados como modificadores de la enfermedad en el


manejo de la AR. Comparte algunos efectos secundarios con las sales de oro administradas por
via parenteral como el rash cutáneo, la glomerulonefritis membranosa, la leucopenia y trombopenia
o las úlceras orales, pero tiene tambien otros propios como la ageusia o la inducción de
enfermedades autoimunes potencialmente graves como el lupus inducido, la
polimiosistis/dermatopolimiositis, miastenia gravis, síndrome de Goodpasture o pénfigo. El
hipotiridsmo no es uno de los efectos secundarios de este fármaco.(R2)

 
 
 
 
42. Señale entre las siguientes manifestaciones del lupus eritematoso sistémico la
MENOS frecuente:

1. 1. Cutáneas.
2. 2. Neurológicas.
3. 3. Gastrointestinales.
4. 4. Cardiopulmonares.
Gráfico de respuestas
Comentario
Para poder contestar a este tipo de preguntas debes fijarte en la tabla de manifestaciones clínicas
del LES que tienes en el manual y recordar las frecuencias globales de cada grupo de
manifestaciones. El resumen puede ser este: - Manifestaciones sistémicas 95% -
Musculoesqueléticas 95% - Cutáneas 80% - Hematológicas 85% - Neurológicas 60% -
Cardiopulmonares 60% - Renales 50% - Gastrointestinales 45%(R3)

43. Un niño de dos años de edad, con antecedentes de un hermano y un primo materno
muertos por neumonía en la infancia, ha presentado desde los 10 meses de vida tres
neumonías y seis episodios de otitis media, así como varias gastroenteritis agudas. Se
encuentra marcada hipogammaglobulinemia con recuento y fórmula leucocitarios
normales. Teniendo en cuenta el diagnóstico más probable, señale la respuesta
CORRECTA:

1. 1. El tratamiento de elección es el trasplante de médula ósea.


2. 2. Si el paciente tiene alguna hermana, sería insólito que padeciese esta enfermedad.
3. 3. La infección por Candida albicans es una de las más frecuentes en esta entidad.
4. 4. En estos pacientes está contraindicado el uso de inmunoglobulinas intravenosas.
Gráfico de respuestas
Comentario

Se nos presenta un cuadro clínico de inmunodeficiencia humoral; de entre las inmunodeficiencias


humorales que conocemos, el dato de debut clínico en el segundo semestre de vida nos debe de
hacer pensar en la agammaglobulinemia de Bruton. Como es una enfermedad genética ligada al X,
sólo se presenta en varones, por lo que si el paciente del caso tiene una hermana afecta, podemos
asegurar que no tiene la enfermedad.(R2)

44. Paciente en periodo expulsivo de 34 semanas de gestación, presentación en tercer


plano de Hodge, posición OIIT y aparición de una bradicardia fetal de 60-70 lat/min,
constatándose la existencia de una acidosis respiratoria (pH = 7.18, PCO 2 = 68 mg Hg,
EB = -2 mEq/l). ¿Cuál sería la actitud CORRECTA?

1. 1. Realización de una cesárea.


2. 2. Realización de un fórceps.
3. 3. Colocación de un pulsioxímetro para conocer la saturación de O2.
4. 4. Aplicación de O2 a la madre y repetición del pH en 10 minutos.
Gráfico de respuestas
Comentario

El caso que nos presentan tiene signos inequívocos de sufrimiento fetal agudo (pH <7.20
acompañado de bradicardia fetal), por lo que la necesidad de poner fin al parto es inmediata. En
este caso, habría que plantearse la 1 y la 2.

 
 
 
 
Entre la cesárea y el fórceps, en este caso debemos decantarnos por el último. Al haber alcanzado
el tercer plano, pondrá fin al parto de una forma más rápida que la cesárea, por lo que la respuesta
correcta es la 2.(R2)

45. Hombre de 37 años que acude a su consulta por presentar desde hace dos días fiebre
de 39ºC con escalofríos, malestar general, artromialgias y tos sin expectoración; estos
últimos días ha estado ayudando a su padre a almacenar el grano en los silos. Ha
presentado episodios similares que se resolvieron espontáneamente en pocos días, no
recuerda en qué circunstancias. En la placa de tórax aparecen infiltrados difusos mal
definidos.Hemograma: 20.000 leucocitos/mm3, con 17% de linfocitos, 3% de monocitos,
72% de neutrófilos segmentados, 7% de cayados, 0.5 de eosinófilos y 0.5% de basófilos;
los datos de la serie roja son normales. En la espirometría, se observa una disminución
de todos los volúmenes pulmonares y un índice de Tiffeneau de 70%. ¿Cuál sería su
diagnóstico de sospecha?:

1. 1. Alveolitis alérgica extrínseca (neumonitis por hipersensibilidad).


2. 2. Sarcoidosis.
3. 3. Asma, posiblemente de origen extrínseco.
4. 4. Fibrosis pulmonar idiopática.
Gráfico de respuestas
Comentario
El dato clave en este caso para sospechar fuertemente una neumonitis por hipersensibilidad es el
antecedente de exposición al grano y los silos. El actinomicetos termófilos que se encuentra en el
heno, granos y silos "mohosos ", produce una neumonitis por hipersensibilidad por exposición a
este antígeno que se denomina "pulmón del granjero ". Clínicamente este paciente presenta la
forma aguda de neumonitis por hipersensibilidad que consiste en fiebre, malestar general,
escalofríos y disnea y que generalmente aparace a las pocas horas de exposición al antígeno. Si
no se produce nueva exposición la sintomatología cede en pocos días. No hay alteraciones
radiológicas características pero es frecuente observar infiltrados mal definidos focales o difusos.
Funcionalmente podemos apreciar un patrón restrictivo. Como es lo habitual en esta enfermedad
NO HAY EOSINOFILIA periférica ( ni tampoco en el lavado broncoalveolar ). Lo fundamental en
cuanto al tratamiento es evitar en la medida de lo posible la exposición al antígeno. Generalmente
la forma aguda de neumonitis por hipersensibilidad desaparece sin necesidad de
glucocorticoides.(R1)

46. En un recién nacido con sospecha de anormal desarrollo del esófago(atresia), que
examen solicita Ud.

1. 1. Rx de esófago con bario muy diluido.


2. 2. Rx de esófago con bario espeso en papilla.
3. 3. Rx de esófago con contraste yodado hidrosoluble.
4. 4. Ninguno de ellos.
Gráfico de respuestas
Comentario

El diagnóstico de la atresia de esófago se realiza mediante:

- Sospecha clínica e imposibilidad para pasar una sonda nasogástrica.

- Rx simple de abdomen.

 
 
 
 
- Rx con contraste hidrosoluble.

- En las fístulas sin atresia (en H) puede ser necesario hacer una broncoscopia.
(R3)

47. Mujer de 30 años, sin hijos, que consulta por dismenorrea de varios años de
evolución, aunque el dolor se ha intensificado en los últimos meses. En la exploración se
detecta una tumoración en ovario derecho, con útero poco móvil y doloroso. Ovario
izquierdo normal y doloroso. Por ultrasonido se visualiza una formación quística,
homogénea con endometrioma en ovario derecho de 7 cm de diámetro. ¿Cuál es el
tratamiento indicado en esta paciente?

1. 1. Anticonceptivos orales.
2. 2. Quistectomía ovárica por laparoscopía.
3. 3. Análogos de la GnRH durante 3 meses.
4. 4. Ooforectomía por laparoscopía.
Gráfico de respuestas
Comentario

Caso clínico típico de endometriosis.

La endometriosis es la proliferación de tejido endometrial fuera de la cavidad uterina. La


localización más frecuente es el ovario, como en esta pregunta, donde se forman quistes rellenos
de sangre o "quistes de chocolate". Aparece en mujeres jóvenes y el síntoma fundamental es el
dolor, normalmente como una dismenorrea progresiva, aunque también puede producir
dispareunia (dolor con las relaciones sexuales). Otro síntoma frecuente es la esterilidad, dato que
no nos dicen directamente, pero que debemos sospechar en esta pregunta cuando nos dicen que
no tiene hijos. El diagnóstico se hace habitualmente por clínica y USG-Doppler, pero el diagnóstico
de certeza requiere una laparoscopia.

En cuanto al tratamiento, la técnica de elección es la laparoscopía con toma de biopsias, citología


de líquido peritoneal, quistectomía y extirpación de implantes peritoneales (respuesta 2 correcta).
La ooferectomía no sería necesaria en principio (respuesta 4 incorrecta), pero si no responde al
tratamiento de elección, podría ser precisa una histerectomía con doble anexectomía. El
tratamiento médico con análogos de la GnRH, danazol, gestágenos o anticonceptivos orales es
poco eficaz, aunque en ocasiones se usa en precirugía y poscirugía .(R2)

48. Señale la respuesta FALSA en relación a los trastornos del estado de ánimo:

1. 1. El trastorno depresivo mayor tiene el doble de prevalencia en mujeres que en varones.


2. 2. El trastorno bipolar I tiene doble prevalencia en varones que en mujeres.
3. 3. El trastorno bipolar I es menos frecuente que el trastorno depresivo mayor.
4. 4. El trastorno bipolar I comienza más precozmente que el trastorno depresivo mayor.
Gráfico de respuestas
Comentario

Una pregunta de dificultad media sobre la epidemiología de los trastornos del estado de ánimo.

Es cierto que la depresión mayor es mucho más frecuente en mujeres que en varones. Sin
embargo, el trastorno bipolar I tiene una frecuencia muy similar en ambos sexos (respuesta 2

 
 
 
 
falsa). Del trastorno bipolar debes recordar que existe una importante agregación familiar. También
existe en la depresión, pero de forma mucho menos intensa.(R2)
Prevalencia de los trastornos afectivos

49. Con la maniobra de Valsalva:

1. 1. Se hace más intenso el soplo de estenosis aórtica.


2. 2. Se hace más intenso el soplo de estenosis subaórtica hipertrófica.
3. 3. Se hace más intenso el soplo de estenosis mitral.
4. 4. Se hace más intenso el soplo de insuficiencia tricúspide.
Gráfico de respuestas
Comentario
Pregunta teórica de dificultad moderada de un tema que cada vez cobra más importancia en el
MIR como es la semiología y dentro de ella la auscultación de los soplos cardiacos. La maniobra
de Valsalva y la bipedestacion disminuyen la precarga, por lo que disminuyen la intensidad de la
mayoría de los soplos exceptuando la miocardiopatia hipertrófica y el prolapso mitral, cuya
intensidad aumenta con dichas maniobras. Por otro lado, la posición de cuclillas aumenta la
precarga e incrementa la intensidad de todos los soplos, exceptuando la MCH y el prolapso. Es
importante recordar que las maniobras que disminuyen la presión arterial, como el nitrito de amilo o
el ejercicio isotónico, disminuyen los soplos de las insuficiencia y empeoran los de las estenosis
(incluyendo la MCH y el prolapso). La estenosis subaórtica hipertrófica es el soplo de la
miocardiopatia hipertrófica.(R2)

50. En cuál de las siguientes situaciones se puede permitir el parto por vía vaginal:

1. 1. Gestante a término con presentación de frente.


2. 2. Feto cefálico en posición transversa.
3. 3. Nalgas incompletas con peso fetal estimado de 3200 g.

 
 
 
 
4. 4. Paciente con una cesárea previa de tipo corporal clásica.
Gráfico de respuestas
Comentario

No debemos confundir situación fetal con la posición fetal.

Situación es la relación entre el eje mayor del feto y la vertical uterina (longitudinal, transversa u
oblicua).

Presentación es la parte fetal que está en relación con la pelvis materna (cefálica o podálica).

Posición es la orientación de la presentación respecto a la pelvis (anterior, posterior y transversa,


bien sea derecha o izquierda.).

Actitud en las presentaciones cefálicas corresponde al grado de flexión de la cabeza fetal: vértice,
sincipucio, frente y cara.

Por ello, sería una cesárea una situación transversa, pero una posición transversa puede
evolucionar por vía vaginal. El resto sabemos que tienen contraindicado la vía vaginal: la
presentación de frente por su mayor diámetro de trabajo, los nalgas incompletas, la presencia de
una cesárea corporal y no segmentaria transversa contraindica el parto y los gemelares
monoamniótico al compartir bolsa.(R2)

51. ¿Cuál de las siguientes afirmaciones es FALSA en relación con la transmisión vertical
al feto/recién nacido por parte de la gestante con infección VIH?

1. 1. Aumenta el riesgo de malformaciones fetales.


2. 2. Tiene lugar sobre todo durante el parto.
3. 3. La lactancia aumenta el riesgo de transmisión entre un 10 y un 20%.
El tratamienteo con zidovudina no ha demostrado efectos adversos en el recién nacido y
4. 4.
hasta 5 años.
Gráfico de respuestas
Comentario

Cuidado con esta pregunta: la embriopatía por VIH no existe, es decir, no existe ningún tipo de
malformación fetal en la que se haya implicado directamente el virus (respuesta 1 falsa).

El momento de mayor probabilidad de transmisión es el del parto, puesto que es cuando se


presenta un contacto más directo con la sangre materna. Otra circunstancia que aumenta el riesgo
de transmisión es la lactancia materna, pero sólo lo hace ligeramente, como dice la respuesta 3.

Una estrategia que puede emplearse para reducir la transmisión intraparto es el tratamiento con
triple terapia (antiguamente se empleaba zidovudina). De hecho, el ser una gestante sería una
indicación para comenzar tratamiento antirretroviral.(R1)

52. Paciente de 25 años de edad que acude a Urgencias tras sufrir accidente esquiando.
Refiere focalidad traumática en su rodilla derecha. En la exploración presenta dolor en la
cara interna de la rodilla que se exacerba al forzar el valgo a 30 grados de flexión. Las
pruebas meniscales son negativas. En lo que respecta a la patología que sufre el
paciente, señale la respuesta correcta:

 
 
 
 
1. 1. Requiere una RMN para su diagnóstico.
2. 2. Es muy probable que tenga derrame articular.
3. 3. Su lesión es la más frecuente de las lesiones de los ligamentos de la rodilla.
4. 4. Cuando se identifica una verdadera lesión aislada es necesario tratamiento quirúrgico.
Gráfico de respuestas
Comentario
El paciente sufre una lesión del ligamento lateral interno de su rodilla. Su diagnóstico es clínico. Es
una estructura extra articular por lo que sus lesiones no producen derrame. En extensión completa
la cápsula posterior intacta impide la inestabilidad en valgo. En estas lesiones el tratamiento
quirúrgico no es necesario. Es la lesión ligamentosa más frecuente de la rodilla.(R3)

53. Recién nacido pretérmino, de peso adecuado para su edad, con grupo sanguíneo B
positivo y grupo materno O positivo, que debuta con ictericia a las 15 h de vida y con una
bilirrubina total de 31 mg/dl. ¿Qué medida terapéutica está indicada en primer lugar?:

1. 1. Iniciar fototerapia inmediatamente.


2. 2. Exanguinotransfusión.
3. 3. Pedir Coombs directo e indirecto para valorar la necesidad de tratamiento.
4. 4. Fototerapia doble y retirar la lactancia materna.
Gráfico de respuestas
Comentario

La isoimunización ABO es más frecuente que la antiD. No requiere sensibilización previa, y se


puede dar si la madre es O y el niño A o B. Puede dar lugar a ictericia precoz o hidrops fetal. En
nuestro enunciado nos están dando un caso de isoinmunización antiB grave (la bilirrubina es
elevadísima: 31 mg/dl), con riesgo de kernicterus, por lo cual hemos de hacer una
exanguinotransfusión.(R2)

54. En relación con el cáncer de endometrio, ¿Cuál de las siguientes respuestas es la


VERDADERA?

1. 1. El cáncer de endometrio es más frecuente en mujeres premenopáusicas.


2. 2. El uso de anticonceptivos orales durante más de tres años es un factor de riesgo.
3. 3. El tratamiento con tamoxifeno es un factor de riesgo.
4. 4. Las mujeres multíparas tienen más riesgo.
Gráfico de respuestas
Comentario

Concepto muy importante Los factores de riesgo asociados a cáncer endometrial son:

-Historia de terapia hormonal, estrogénica sustitutiva, sin oposición progestacional.

-Tratamiento con tamoxifeno.

-Menopausia tardía.

-Nuliparidad.

-Infertilidad o fallo terapeútico a inductores de la ovulación.

 
 
 
 
-Obesidad.

-La diabetes e hipertensión son consideradas también como factores de riesgo identificados para el
desarrollo de esta enfermedad.

Con esto, la respuesta correcta es tamoxifeno. (Raloxifeno no).(R3)

55. RN de 35 semanas de edad gestacional que, en las primeras 12 horas de vida, presenta taquipnea,
quejido audible y cianosis progresiva, refractaria a la administración de oxígeno. A la auscultación
encontramos crepitantes, sobre todo en bases pulmonares. En la gasometría aparece hipoxemia
progresiva, aumento de la PCO2 y acidosis metabólica. ¿Qué radiografía de tórax esperaría
encontrar en este momento?:

1. 1. Infiltrado de patrón reticulogranular con broncograma aéreo.


2. 2. Hiperinsuflación y aumento de las marcas vasculares, sin broncograma.
3. 3. Patrón de esponja.
4. 4. Borde de ambos pulmones colapsados.
Gráfico de respuestas
Comentario

56. Señale cuál de las siguientes es la causa más frecuente de hemorragia digestiva alta:

1. 1. Esofagitis.

 
 
 
 
2. 2. Síndrome de Mallory-Weiss.
3. 3. Ulcera duodenal.
4. 4. Ulcera gástrica.
Gráfico de respuestas
Comentario
A pesar de que las úlceras gástricas sangran más que las duodenales, estas últimas son mucho
más prevalentes que las gástricas. Por tanto, en números absolutos, la primera causa de
hemorragia digestiva alta es el ulcus duodenal. Le sigue el ulcus gástrico. En tercer lugar, se
encuentran las gastritis por estrés (en las que se incluye el síndrome de Mallory- Weiss); y en
cuarto lugar, las varices esofágicas (que serían la primera causa si se tratara de un paciente
cirrótico).(R3)

57. ¿Cuál de los siguientes factores NO se ha asociado con el síndrome de muerte súbita
del lactante?:

1. 1. Sexo femenino.
2. 2. Posición en decúbito prono.
3. 3. Tabaquismo materno.
4. 4. Bajo peso al nacimiento.
Gráfico de respuestas
Comentario

El principal factor de riesgo asociado es la posición en prono. Desde que se modificó este factor
años atrás, el siguiente en importancia sería el tabaquismo materno.

Es más preponderante en el sexo masculino (opción 1 falsa). Otros factores relacionados son el
bajo peso al nacimiento y la existencia de patología pulmonar que predisponga a apneas, como los
prematuros con displasia broncopulmonar.(R1)

58. Sobre el tratamiento de los pacientes con enfermedad coronaria aterosclerótica y


angina de esfuerzo crónica, señale la FALSA:

1. 1. Los antagonistas del calcio son especialmente útiles cuando se asocia vasoespasmo.
Los pacientes con lesión proximal de la arteria circunfleja y de la arteria coronaria derecha,
2. 2.
con buena función del ventrículo izquierdo, son subsidiarios de tratamiento quirúrgico.
La angina nocturna o de decúbito de algunos pacientes es mejorada con tratamiento con
3. 3.
diuréticos.
La actividad de los nitratos depende de su absorción, que es más rápida y completa a través
4. 4.
de las mucosas.
Gráfico de respuestas
Comentario

Un paciente con las características mencionadas en la respuesta 3 precisaría tratamiento con


angioplastia, no con cirugía. Si la has fallado, deberías revisar también el comentario de la 199,
donde se recoge en qué circunstancias habría que preferir la cirugía antes que la angioplastia.(R2)

59. ¿En qué grupo de edades es más frecuente la hemorragia uterina disfuncional (HUD)
anovulatoría?:

1. 1. Niñas y premenopáusicas.

 
 
 
 
2. 2. Adolescentes y perimenopáusicas.
3. 3. Adultas y menopáusicas.
4. 4. Niñas y adolescentes.
Gráfico de respuestas
Comentario

Las edades en que la HUD es más frecuente corresponden a los extremos de la vida fértil; esto es,
en la adolescencia y la perimenopausia; pues en estas edades son frecuentes los ciclos
anovulatorios.

Las niñas y las menopáusicas (respuestas 1 y 3) no tienen menstruación.(R2)

60. La radiografía torácica de un lactante de nueve meses de edad revela líquido pleural
y neumatocele. Estos hallazgos son causados posiblemente por:

1. 1. Staphylococcus aureus.
2. 2. Estreptococo beta-hemolítico.
3. 3. Haemophilus influenzae.
4. 4. Klebsiella pneumoniae.
Gráfico de respuestas
Comentario

Se llama neumatocele a la existencia en el interior del parénquima pulmonar de una cavidad de


paredes finas que se encuentra llena de aire, en ocasiones contiene también líquido que forma un
nivel. Suele producirse como consecuencia de una neumonía por Staphylococcus aureus u otros
microbios como Mycobacterium tuberculosis o Pneumocystis jirovecii. A veces está causado por un
traumatismo sobre el tórax. Generalmente se origina por un mecanismo valvular que favorece el
atrapamiento de aire y su acumulación en el intersticio pulmonar.

Los neumatoceles son especialmente frecuentes en niños y no deben confundirse con otras
acumulaciones de gas en el tórax, como el neumotorax, las bullas que se producen en el enfisema
pulmonar o el absceso de pulmón. Se diagnostican mediante una radiografía o TAC de tórax que
permiten ver un espacio lleno de aire que contrasta con el parénquima pulmonar que lo rodea.
Suelen desaparecer de manera espontánea, aunque en ocasiones es preciso realiza un drenaje
mediante la inserción de un cateter para lograr su resolución.(R1)

61. Lactante de 3 meses consulta por cuadro de vómitos y febrícula de 48 horas de


evolución, con progresiva pérdida del apetito. En la exploración no se detecta foco
infeccioso. BH con leucocitosis y desviación a la izquierda. EGO: leucocitos y nitritos
positivos. ¿Cuál sería la actitud más adecuada?:

1. 1. Tratamiento con antitérmicos y ver evolución.


2. 2. Diagnóstico de infección de orina y administrar antibióticos VO.
3. 3. Realizar punción suprapúbica e iniciar antibióticos.
4. 4. Descartar reflujo gastroesofágico.
Gráfico de respuestas
Comentario

Ante la sospecha de infección de orina por sedimento de orina con muestra procedente de una
bolsa (lactante de 3 meses no es continente, por lo tanto no puede ser de chorro medio), se debe
confirmar recogiendo la orina con un método más estéril. Inicialmente se haría con un sondaje ya

 
 
 
 
que es menos agresivo, pero como no nos dan esta opción lo correcto sería confirmación por
punción suprapúbica. Respuesta 3 correcta.(R3)

62. Paciente de 74 años que tras hemorragia uterina postmenopáusica es diagnosticada


de adenocarcinoma de endometrio. Se realizó tratamienteo quirúrgico consistente en
histerectomía abdominal total con salpingo-ooforectomía bilateral más toma de citología
peritoneal. Tras estudio anatomopatológico se determina que la paciente se encuentra
en un estadio I con grado de diferenciación histológico 1. ¿Cuál sería la actitud
CORRECTA?

1. 1. Administrar radioteraparia pélvica (4500- 5000 cGy dosis total).


2. 2. Administrar quimioterapia con progestágenos.
3. 3. Realizar controles periódicos sin tratamiento adicional.
4. 4. Completar cirugía con linfadenectomía pélvica.
Gráfico de respuestas
Comentario

Pregunta fácil. Aunque no conozca exactamente el estadiaje y tratamiento del cáncer de


endometrio, es fácil de intuir que no querrán que hagamos mucho más, por lo optimista de la
descripción. Nos están insistiendo en que está limitado al endometrio, que el grado de
diferenciación es el más favorable posible (G1), y no nos dan ningún dato de mal pronóstico.
Consecuentemente, el planteamiento más lógico sería el de no añadir ningún otro gesto
terapéutico.(R3)

63. Femenino de 21 años, gestante de 22 semanas. Se encuentra alerta y refiere dolor


abdominal; al examen: FC 90, PA 100/65; dolor leve en cuadrante superior izq. de
abdomen. El método de evaluación más apropiado es:

1. 1. Lavado peritoneal diagnóstico.


2. 2. Radiografías de abdomen y tórax bajo.
3. 3. Ultrasonido abdominal.
4. 4. Arteriografía.
Gráfico de respuestas
Comentario

Ante una mujer en edad fértil que presenta un dolor abdominal agudo siempre se debe tener en
cuenta una causa ginecológica y un posible embarazo. Por ese motivo no se puede realizar una
prueba que emita radiaciones y el ultrasonido es el procedmineto de elección.(R3)

64. Varón de 45 años de edad que tras sufrir traumatismo abdominopélvico presenta,
dificultad para la micción, hematuria con algún coágulo y dolor en hemiabdomen inferior
con defensa. La lesión que probablemente presenta este pacient

1. 1. Trau
2. 2. Rotu
3. 3. Lesi
4. 4. Lesi
Gráfico de respuestas
Comentario
Es una pregunta de dificultad media que no es habitual encontrar en el MIR. De todos modos
conviene conocer la clínica y la actuación ante una rotura vesical. Es básico reconocer los

 
 
 
 
antecedentes responsables de la rotura vesical. En el MIR siempre aparecerán antecedentes
quirúrgico o traumatismos muchas veces asociados a fractura pélvica. Reconocer este dato es
clave para la orientación de al pregunta. El segundo paso es recordar que la clínica de las lesiones
vesicales consiste en dificultad para la micción, hematuria, dolor pélvico e incluso abdomen agudo
(indica rotura intraperitoneal) como ocurre en este caso. Por último debes saber distinguir entre la
rotura extraperitoneal (producida por los fragmentos de una fractura pélvica) y rotura intraperitoneal
(por golpe directo cuando la vejiga está llena).(R2)

65. La estenosis aórtica se considera crítica cuando el área valvular es:

1. 1. Inferior a 1 cm2.
2. 2. Inferior a 0.8 cm2.
3. 3. Inferior a 0.7 cm2.
4. 4. Inferior a 0.4 cm2.
Gráfico de respuestas
Comentario

Esta pregunta tiene poca relevancia. Normalmente, el área valvular aórtica es de unos 3-5 cm2.
Cuando es menor de 0.75 cm2 (o menor de 0.5 cm2 por m2 de superficie corporal), o cuando el
gradiente medio es mayor de 50 mmHg, generalmente la estenosis es severa. Es entonces cuando
la EAo tiende a hacerse sintomática y empeora el pronóstico de los pacientes. Es importante que
recuerde que la indicación de cirugía en la EAo viene dada, sobre todo, por la aparición de
síntomas, y no por el área valvular.(R3)

66. Un enfermo con infección conocida por VIH, drogadicto, presenta un cuadro de
malestar general de 48 horas de evolución, acompañado de dolores torácicos pleuríticos,
tos y fiebre elevada. En la radiografía de tórax se objetivan infiltrados pulmonares
bilaterales cavitados. ¿Cuál de los siguientes es el diagnóstico más probable?

1. 1. Sarcoma de Kaposi metastásico.


2. 2. Endocarditis tricuspídea.
3. 3. Neumonía por Pneumocystis.
4. 4. Neumonía neumocócica.
Gráfico de respuestas
Comentario

La endocarditis tricuspídea es la más frecuente en el paciente ADVP, suele ser por S. aureus y se
caracteriza por los émbolos sépticos que alcanzan los pulmones a través de la arteria pulmonar
(respuesta 2 correcta). No es correcto, en este caso, pensar en una tuberculosis, porque se trata
de un cuadro muy agudo, mientras que la tuberculosis suele ser mucho más larvada.(R2)

67. Señale cuál sería el patrón radiológico que más frecuentemente encontraríamos en
un RN que ha sufrido un síndrome de aspiración meconial:

1. 1. Aumento de marcas vasculares con derrame.


2. 2. Infiltrados dispersos con atrapamiento aéreo.
3. 3. Patrón en esponja.
4. 4. Normalidad radiológica.
Gráfico de respuestas
Comentario

 
 
 
 
En el síndrome de aspiración meconial se produce una aspiración de meconio que obstruye la via
aerea. Radiologicamente se pueden ver enfisemas, e infiltrados algodonosos parcheados con
atrapamiento aéreo.

La respuesta se vería en el síndrome de taquipnea transitoria del RN.

La respuesta 3 se vería en la displasia broncopulmonar.(R2)

68. Which of the following is the most


frequent complication of the surgery shown in this picture?

1. 1. Infection.
2. 2. Amputation.
3. 3. Wear of components.
4. 4. Component oxidation.
Gráfico de respuestas
Comentario

Esta prótesis, como puede imaginar, trata de imitar los cóndilos femorales, que a su vez se
apoyarán sobre la meseta tibial. Se trata, por lo tanto, de una prótesis de rodilla. La complicación
más frecuente es la infección, que suele cursar de forma indolente, con síntomas muy leves y, por
consiguiente, con un retraso diagnóstico de varios meses. La causa más frecuente son S.
epidermidis y S. aureus, aunque en ocasiones también pueden verse implicados bacilos
gramnegativos y anaerobios.(R1)

69. Entre las contraindicaciones para la utilización de anticonceptivos orales, NO se


encontrarían:

1. 1. Antecedentes de enfermedad tromboembólica.


2. 2. Ictericia colestática previa.
3. 3. Diabetes insulinodependiente.
4. 4. Mastopatá fibroquística y antecedentes familiares de cáncer de mama.
Gráfico de respuestas
Comentario

 
 
 
 
Los preparados combinados de baja dosis de estrógenos y gestágenos poco androgénicos hacen
que el riesgo cardiocirculatorio sea casi nulo, salvo que se asocien otros factores de riesgo
cardiovascular como los que aparecen en las respuestas 1 y 3. La ictericia colostática gestacional
puede recidivar con nuevas gestaciones o con la toma de anticonceptivos por lo que aquellas
pacientes con un episodio previo tienen contraindicado su uso.

Los anticonceptivos orales disminuyen la incidencia de patología mamaria benigna como la


mastopatía fibroquística. Las pacientes con antecedente familiar de cáncer de mama no tienen un
riesgo adicional sobre el riesgo basal incrementado de cáncer de mama por ser usuarias de
anticonceptivos orales.(R4)

70. El eritema tóxico es:

1. 1. Más común entre los a término que en los pre términos.


2. 2. Usualmente asociado a fiebre y mal estado tóxico.
3. 3. Uualmente asociado con elevación de linfocitos en sangre.
4. 4. Se manifiesta como un rash maculo vesicular.
Gráfico de respuestas
Comentario

El eritema tóxico consiste en la presencia de vesiculopústulas (respuesta 4 incorrecta) sobre una


base eritematosa que suelen respetar palmas y plantas, formadas por un infiltrado de eosinófilos
(respuesta 3 incorrecta), con tendencia a desaparecer en la primera semana. Es más comun en los
RN a término, respuesta 1 correcta. Es una enfermedad que no causa alteraciones sistémicas,
respuesta 2 incorrecta.(R1)

71. ¿Cuál de las siguientes manifestaciones NO suele aparecer en relación con una
endocarditis infecciosa?:

1. 1. Artritis junto con factor reumatoide positivo.


2. 2. Glomerulonefritis proliferativa exudativa.
3. 3. Hepatitis colestásica.
4. 4. Anemia normocítica y normocrómica.
Gráfico de respuestas
Comentario

Esta es una pregunta complicada de acertar. Probablemente mucha gente haya pensado que es la
opción 1 porque lo del factor reumatoide suena sobre todo a la artritis reumatoide y otras
enfermedades reumatológicas. Pero no, este factor no es patognomónico de ninguna enfermedad
y es un reactante de fase aguda, además en esta enfermedad, en muchas ocasiones, hay una
alteración asociada del sistema inmunológico por lo que puede dar artritis con elevación de este
factor, además de vasculitis mucocutánea (de hecho se cree que los nódulos de Osler son una
vasculitis), glomerulonefritis (proliferativa exudativa u de otros tipos) - también puede da una
nefropatía por suelta de émbolos- , y anemia, en este caso sobre todo en las que son de tipo
subagudo o crónico, como anemia por enfermedad crónica (por eso es normocítica y
normocrómica). Pero lo que no es frecuente es la hepatitis colestática.(R3)

72. Respecto al carcinoma de mama “in situ”, ¿cuál de las siguientes afirmaciones es
CORRECTA?:

1. 1. El carcinoma lobulillar “in situ” tiene carácter bilateral y multicéntrico.

 
 
 
 
2. 2. El carcinoma intraductal no evoluciona a lesión palpable.
3. 3. Es muy frecuente la extensión de la enfermedad a los ganglios linfáticos.
4. 4. El tratamiento quirúrgico siempre debe ir acompañado de linfadenectomía axilar.
Gráfico de respuestas
Comentario

Típicamente el carcinoma lobulillar es bilateral con más frecuencia que el ductal.

-La respuesta 2 es falsa; el carcinoma intraductal tiende a convertirse en infiltrante con el tiempo, y
por tanto eventualmente da lugar a una masa palpable.

-La respuesta 3 es falsa, el carcinoma in situ, hasta que no se vuelve infiltrante, no puede dar
metástasis.

-La respuesta 4 es falsa, al quitar un carcinoma in situ no se suele hacer linfadenectomía, porque
en este estadio, el tumor no puede dar metástasis ganglionares.(R1)

73. Paciente de 41 años, fumador de 1 paquete/día durante 15 años, que consulta por
disnea de esfuerzo desde hace 3 años que se ha hecho progresivamente de pequeños
esfuerzos. En la radiografía de tórax se observa hiperinsuflación pulmonar con hilios
vasculares prominentes. Gasometría arterial basal: pH 7,46, pO2 57 mmHg, pCO2 36
mmHg. Pruebas de función respiratoria: FEV1 50% del teórico, FVC 85% del teórico,
FEV1/FVC 65%, TLC 131%, RV 207%, DLCO 60%. Su diagnóstico más probable es:

1. 1. Fibrosis pulmonar idiopática.


2. 2. EPOC tipo enfisema pulmonar.
3. 3. Tromboembolismo pulmonar.
4. 4. Neumonía.
Gráfico de respuestas
Comentario

Ante un paciente con obstrucción ventilatoria, deberíamos sospechar un EPOC, ya sea en forma
de bronquitis crónica o de enfisema. Si además nos dicen que la difusión de monóxido de carbono
está reducida de forma importante, debemos decantarnos por el enfisema pulmonar (respuesta 2
correcta). La hiperclaridad del parénquima pulmonar radiográfica también nos ayuda. Recuerda
que, en la bronquitis crónica, el dato clínico típico es la tos con expectoración abundante.(R2)

74. Un hombre obeso de 55 años, con historia de diabetes mellitus no


insulinodependiente, es conducido al departamento de urgencias porque su familia lo
nota confuso. La familia cree que está tomando "unas pastillas" para tratar su "azúcar".
Aunque la exploración neurológica no muestra ninguna focalidad, está claramente
desorientado. Todos los estudios de laboratorio son normales, excepto una glucemia de
30 mg/dl. ¿Cuál de las siguientes afirmaciones respecto a la situación actual es
CORRECTA?

Se debe tratar al paciente con glucosa intravenosa hasta que la concentración sérica de
1. 1.
glucosa se normalice, y después darle de alta.
2. 2. Es probable que esté elevada la concentración de insulina del paciente.
3. 3. Si el paciente está tomando clorpropamida, tiene riesgo de sufrir un infarto de miocardio.
4. 4. Se debe ingresar al paciente.
Gráfico de respuestas

 
 
 
 
Comentario

Este paciente presenta una hipoglucemia por hipoglucemiantes orales, probablemente por
sulfonilureas. Las hipoglucemias por sulfonilureas son menos frecuentes que con insulina, pero
cuando aparecen suelen ser graves y duraderas. Debido a la larga vida media de la mayoría de
estos fármacos, es necesario administrar grandes cantidades de glucosa durante 24-48 horas tras
la última dosis y por esto todos los pacientes con hipoglucemia inducida por sulfonilureas deben
ingresar en un hospital.(R4)

75. Mujer de 68 años que comienza con clínica de aumento del diámetro abdominal desde
hace 5 meses, que se acompaña de dolor en los últimos 15 días. Tras practicar las
exploraciones pertinentes se le diagnostica de cáncer de ovario de tipo
cistoadenocarcinoma seroso, en estadio IIIb. Se le realiza tratamiento quirúrgico sin dejar
ningún resto tumoral. ¿Cómo completaría el tratamiento?:

1. 1. No necesita más tratamiento.


2. 2. Radioterapia abdominal.
3. 3. Poliquimioterapia adyuvante.
4. 4. Radioterapia más quimioterapia con cisplatino.
Gráfico de respuestas
Comentario

El tratamiento inicial del cáncer de ovario es casi siempre quirúrgico (para poder realizar una
estadificación postquirúrgica del mismo). Recuerden que en caso de pacientes jóvenes con deseos
genésicos y estadio IA G1 se puede realizar una anexectomía unilateral y completar la cirugía una
vez cumplidos sus deseos reproductivos.

En este caso preguntan acerca del tratamiento postquirúrgico (es decir quimioterapia posterior).
Recuerden que en los estadios IA – IB (G1 – G2) no está indicado iniciar QT; en IA- IB G2 y IC está
indicado la monoterapia y en estadios > IIA está indicado la poliQT. Por ello, la respuesta correcta
es la número 3.(R3)

76. Uno de los siguientes agentes NO produce diarrea por invasión de mucosa intestinal:

1. 1. Salmonella.
2. 2. E. Coli.
3. 3. Yersinia.
4. 4. Vibrio cholerae.
Gráfico de respuestas
Comentario

El mecanismo fisiopatológico de producción de la diarrea ha sido preguntado en varias ocasiones


en el examen ENARM, y por lo tanto es un tema al que le debemos prestar especial atención;
Shigella, Salmonella, Yersinia y E. Coli enteroinvasivo son bacterias enteroinvasivas, las cuales no
actúan produciendo toxinas, sino invadiendo directamente la mucosa intestinal, provocando una
diarrea con productos patológicos ( moco, pus, sangre). Por otro lado, Vibrio cholerae es una
bacteria productora de enterotoxinas, las cuales actúan sobre la superficie de los enterocitos sin
destrucción de la mucosa, alterando el intercambio iónico y dando lugar a una diarrea acuosa sin
productos patológicos y sin leucocitos al analizar las heces por el microscopio. Así, opción 4
correcta.(R4)

 
 
 
 
77. ¿Cuál es el tratamiento del adenocarcinoma gástrico antral?

1. 1. Gastrectomía parcial.
2. 2. Gastrectomía subtotal radical.
3. 3. Antrectomia.
4. 4. Gastroentero anastomosis.
Gráfico de respuestas
Comentario

Dependiendo de la localización del tumor, la resección será:

-Tumores antrales y del cuerpo bajo: gastrectomia subtotal y gastroenteroanastomosis en Y de


Roux.

- Tumores de la mitad proximal del estómago, gastrectomía total y esofagoyeyunostomía.

-Tumores del cardias: gastrectomia total con esofaguectomía que depende de la afectación del
esófago y reconstrucción en Y de Roux. Si es necesaria la esofaguectomia se puede realizar una
coloplastía.(R2)

78. Niño de 8 años que presenta exantema eritematoso localizado en las mejillas,
asociado a exantema macular eritematos dispuesto en encaje principalmente en las
extremidades. Se debe sospechar en:

1. 1. Megaloeritema.
2. 2. Exantema súbito.
3. 3. Escarlatina.
4. 4. Rubéola.
Gráfico de respuestas
Comentario

No puede fallar las enfermedades exantemáticas.

El eritema infeccioso o quinta enfermedad o megaloeritema está causado por Parvovirus B19, es
una enfermedad propia de la edad escolar 5-15 años. Predomina durante los meses de primavera-
verano.

El paciente se encuentra afebril apareciendo un exantema que evoluciona en tres fases.

Fase inicial o del bofetón eritema de ambas mejillas de aparición brusca.

Fase intermedia: elementos maculopapulosos y eritematosos en tronco y extremidades,


generalmente en superficies de extensión

Tercera fase es la más característica: aclaramiento central de las lesiones, dándoles un aspecto de
encaje o reticulado sin descamación.

Desaparece y aparece en un periodo de entre una y cuatro semanas. Es posible que reaparezca
transitoriamente ante el ejercicio, exposición al sol o fricción de la piel.

 
 
 
 
Complicaciones: A'S: artritis y artralgias de grandes y pequeñas articulaciones, abortos o hydrops y
aplásicas graves.

Tratamiento es sintomático.(R1)

79. Un recién nacido comienza con vómitos desde las primeras tomas de alimento. En la
radiografía simple de abdomen se aprecia una única burbuja aérea gástrica. Dados estos
hallazgos, señale el diagnóstico que considere más probable:

1. 1. Atresia esofágica con fístula tráqueo-esofágica.


2. 2. Estenosis hipertrófica del píloro.
3. 3. Atresia pilórica.
4. 4. Páncreas anular.
Gráfico de respuestas
Comentario

Hay que recordar el diagnóstico diferencial entre estenosis hipertrófica del píloro, atresia pilórica y
atresia duodenal.

La EHP debuta en torno a los 20 días con vómitos no biliosos, la radiografía es inexpresiva y la
prueba diagnóstica es el ultrasonido.

La atresia pilórica cursa con vómitos no biliosos desde el nacimiento, y en la radiografía se aprecia
una burbuja única gástrica.

En la atresia duodenal, que también comienza a manifestarse desde la primera toma del alimento,
los vómitos sí son biliosos y en la radiografía la burbuja es doble, puesto que la obstrucción es
post- pilórica.

En el caso de la pregunta, se describe perfectamente la atresia pilórica.(R3)

80. Mujer de 35 años que acude por amenorrea secundaria. La reacción del embarazo es
negativa y en el estudio hormonal destacan niveles elevados de FSH. Esto es compatible
con:

1. 1. Síndrome adrenogenital.
2. 2. Menopausia precoz.
3. 3. Anorexia nerviosa.
4. 4. Tumor ovárico secretor de estrógenos.
Gráfico de respuestas
Comentario

Para el examen es importante conocer el diagnóstico diferencial de las amenorreas.

Lo primero en el es realizar una prueba de embarazo. También se debe recordar que en la


premenopausia el denominador común es el aumento de FSH probablemente debido al descenso
de la inhibina, consecuencia del escaso número folicular. Este incremento de FSH es la
manifestación endocrina más precoz del climaterio. La LH está normal o aumentada. La GnRH y
los estrógenos son normales.(R2)

 
 
 
 
81. A 60-year-old female is tested for BRCA1 gene after her sister is diagnosed with
BRCA1-positive breast cancer. The test is positive for BRCA1 in our patient too. Which of
the following antigens should we consider testing her for?

1. 1. CEA
2. 2. CA125
3. 3. CA19-9
4. 4. CA15.3
Gráfico de respuestas
Comentario
CA125. Positive BRCA 1 indicates a higher risk of both breast and ovarian cancer. Tests should be
performed to screen both. CA125 is known as a tumor marker for ovarian cancer.(R2)

82. Which of the following gastric neoplasms can be successfully treated by eradicating
H. pylori with antibiotics and proton pump inhibitors?

1. 1. Intestinal type gastric adenocarcinoma.


2. 2. Gastric MALT lymphoma.
3. 3. High grade gastric lymphoma.
4. 4. Gastric Hodgkin's lymphoma.
Gráfico de respuestas
Comentario

Pregunta importante sobre un tema incluido en los aspectos esenciales del Manual, ya que es bien
conocida la relación del H. pylori con el linfoma MALT gástrico (respuesta 2 correcta).

El tumor gástrico maligno más frecuente es el adenocarcinoma, seguido por el linfoma no Hodgkin
(LNH), prácticamente siempre de estirpe B. El estómago es la localización extranodal más
frecuente de los linfomas. La mayoría aparecen sobre lesiones de gastritis crónica y, con
frecuencia, sobre zonas de metaplasia intestinal que en un porcentaje variable están colonizadas
por H. pylori.

El linfoma gástrico relacionado con H. pylori suele ser un LNH de bajo grado (casi siempre tipo
MALT) recomendándose el tratamiento erradicador y con inhibidores de la bomba de protones,
antes de plantearse cirugía u otros tratamientos, obteniéndose remisiones completas en buena
parte de los casos, cuando está en estadios localizados.(R2)

83. Paciente de 3 semanas de edad, que inicia su enfermedad con vómitos lácteos
postprandiales, en buen estado general y con buena succión. El diagnóstico más
probable es:

1. 1. Páncreas anular
2. 2. Hernia diafragmática
3. 3. Estenosis hipertrófica pilórica
4. 4. Bridas de Ladd
Gráfico de respuestas
Comentario

La estenosis hipertrófica del piloro se da más frecuentemente en varones, de raza blanca y


primogénitos. Existe una incidéncia familiar del 15% y es de etiología multifactorial desconocida

 
 
 
 
(genética, ambiental grupos sanguíneos A y B). Se ha asociado además a la eritromicina para el
tratamiento y prevención de la tosferina en los primeros 15 días de vida, y macrólidos en la madre
durante embarazo y lactancia.

La clínica es de vómitos proyectivos inmediatos tras las tomas, alimenticios, no biliosos, con
hambre e irritabilidad. Más frecuentemente en torno a los 20 días de vida. Produce, en los
exámenes de laboratorio una alcalosis metabólica hipoclorhémica con tendencia a la
hipopotasemia.

En la exploración física se pueden ver grados variables de deshidratación y desnutrición y en


ocasiones puede palparse la oliva pilórica. A veces pueden observarse ondas peristálticas
gástricas.

Para el diagnóstico es útil el USG abdominal y se trata con cirugía: piloromiotomia de Ramsted
despues de correcta rehidratación y corrección de los electrolitos.(R3)

84. Con relación al tratamiento del hipotiroidismo congénito, indique la respuesta


CORRECTA:

Han de normalizarse los niveles de tiroxina de manera progresiva para evitar efectos
1. 1.
secundarios.
2. 2. El retraso en su inicio puede condicionar una lesión cerebral definitiva.
3. 3. Ha de retirarse a los 3 años a todos los pacientes para descartar que sea transitorio.
4. 4. Requiere un seguimiento de laboratorio semestral.
Gráfico de respuestas
Comentario

Tema muy importante en el examen hoy en día, es poco frecuente diagnosticar clínicamente un
hipotiroidismo congénito debido a que, en su mayor parte, son diagnosticados con el tamiz
neonatal, que se realiza determinando los niveles de TSH obtenida entre los dos y los cinco días
de vida (cuyos valores estarían elevados).

Las manifestaciones del hipotiroidismo congénito tardan varias semanas en presentarse


plenamente, estando totalmente establecidas hacia el 3º-6º mes de vida. Aparece una facies
peculiar (cara tosca, párpados y labios tumefactos, nariz corta con base deprimida, hipertelorismo,
boca semiabierta y macroglosia). Otras manifestaciones típicas son el retraso en la maduración
ósea, la hernia umbilical, ictericia prolongada, letargia y estreñimiento.

El principal problema del hipotiroidismo congénito es su repercusión sobre el desarrollo intelectual.


Éste sería más grave cuanto más se demore el diagnóstico (respuesta 2 correcta), por lo que es
importante instaurar el tratamiento cuanto antes. Aunque la causa más frecuente de hipotiroidismo
congénito es la disgenesia tiroidea, independientemente de cuál sea la causa exacta, ante
cualquier caso de hipotiroidismo debemos apresurarnos a establecer el tratamiento sustitutivo lo
antes posible.(R2)

85. Lactante de 9 meses de edad con historia de tres días de evolución que inicia con
rinorrea, posteriormente fiebre y tos. A las 36 horas presenta dificultad respiratoria e
hiporexia. Al examen se detecta subcrepitantes bilaterales, hígado a 2.5 cm debajo del
reborde costal, congestión timpánica bilateral y presenta albuminuria. Su diagnóstico
probable es:

 
 
 
 
1. 1. Bronconeumonía probablemente bacteriana.
2. 2. Shock séptico.
3. 3. Proceso infeccioso probablemente viral.
4. 4. Otitis media aguda probablemente bacteriana.
Gráfico de respuestas
Comentario

Ante una rinorrea con fiebre y tos en un lactante siempre debemos pensar en primer lugar en un
proceso viral. El hecho de que el lactante presente hepatomegalia durante el cuadro refuerza la
sospecha diagnóstica.(R3)

86. Sobre el riñón izquierdo, señalar la correcta:

1. 1. Mide 3 cm más que el derecho.


2. 2. Está localizado más bajo que el derecho.
3. 3. Mide aproximadamente 20 x 10 x 18 cm.
4. 4. Se localiza entre D12 y L2.
Gráfico de respuestas
Comentario
La anatomía renal no es un aspecto muy preguntado en la historia del MIR, aunque esta pregunta
no es de excesiva dificultad haciendo un ligero cálculo visual. Las dimensiones de un riñón son 3
cm de espesor, 6 cm de anchura y 12 cm de longitud (multiplica por dos la cifra precedente), muy
lejos del tamaño de la opción 3 que casi parece una pelota de balonmano. Ambos riñones tienen el
mismo tamaño, con lo cual la opción 1y 5 son falsas, de ahí que sólo podamos dudar de la
localización. En su desarrollo embriológico el riñón definitivo procede del metanefros, localizado en
la región pelviana, para más tarde experimentar un ascenso y una rotación medial, en este
ascenso el riñón derecho tiene el freno del parénquima hepático, con lo cual está situado más bajo
que el izquierdo (opción 2 incorrecta). Así, llegamos a la conclusión que la pregunta número 4 es la
correcta.(R4)

87. En el tratamiento de los quemados extensos, todo es cierto EXCEPTO:

1. 1. Lo primero en estos pacientes es mantener permeable la vía respiratoria.


Es importante una correcta reposición hidroelectrolítica, en las primeras 24 horas se
2. 2.
administrarán cristaloides.
3. 3. Es de poca utilidad el uso de coloides en estos pacientes.
4. 4. En caso de requerir el uso de diuréticos, es de elección el uso de osmóticos.
Gráfico de respuestas
Comentario

Es bueno tener presente algunas generalidades del tratamiento y manejo de los quemados. Lo
primero es mantener permeable la vía respiratoria. A continuación lo más importante es una
correcta reposición hidroelectrolítica. En las primeras 24h se administran cristaloides y en la
segundas 24h coloides. La necrosis tubular aguda es rara si se hace una correcta reposición
hidroelectrolítica; de todos modos en lo pacientes con extensas lesiones musculares está indicado
aumentar la diuresis como prevención de la NTA. Los diuréticos de elección son los diuréticos
osmóticos.(R3)

 
 
 
 
88. El problema más urgente en niños que presentan síndrome de Pierre Robin es:

1. 1. Falla cardíaca.
2. 2. Convulsiones.
3. 3. Obstrucción intestinal.
4. 4. Obstrucción de vía aérea.
Gráfico de respuestas
Comentario

Es una afección presente al nacer en la cual el bebé tiene la mandíbula más pequeña de lo normal,
la lengua replegada en la garganta y dificultad para respirar.

Se desconocen las causas exactas del síndrome de Pierre Robin, pero puede ser parte de muchos
síndromes genéticos.

La mandíbula inferior se desarrolla lentamente antes de nacer, pero se puede acelerar durante el
primer año de vida.

Los bebés que padecen esta afección NO se deben colocar boca arriba para evitar que la lengua
se retraiga hacia la vía respiratoria.

En casos moderados, el paciente necesitará que le coloquen una sonda por la nariz y las vías
respiratorias para evitar la obstrucción de ésta. En casos graves, se necesita cirugía para evitar
una obstrucción en las vías respiratorias altas. Algunos pacientes necesitan cirugía para realizarles
una traqueotomía.

La alimentación debe hacerse con mucho cuidado para evitar el ahogamiento y la broncoaspiración
de líquidos. Algunas veces, puede ser necesario alimentar al niño a través de una sonda para
prevenir ahogamiento.(R4)

89. Niña de 3 años de edad que acude a su pediatra por estancamiento ponderal desde
los 2 años y 3 meses de vida. Realiza 6-8 evacuaciones al día, abundantes, pastosas y
brillantes. Los padres refieren que es "mala comedora" y que siempre está enfadada y
llorando por todo. A la exploración llama la atención una importante distensión
abdominal, así como un aspecto distrófico de las extremidades. En relación a la
enfermedad que usted sospecha señale la afirmación CORRECTA:

El diagnóstico es de enfermedad celíaca y no es necesaria la realización de más pruebas


1. 1.
complementarias.
La actitud inicial más adecuada sería la determinación de los anticuerpos
2. 2.
antitransglutaminasa.
La gravedad del cuadro clínico expuesto hace necesaria la realización de una biopsia
3. 3.
intestinal urgente.
4. 4. La elevación de transaminasas descarta enfermedad celíaca.
Gráfico de respuestas
Comentario

La pregunta nos describe un caso típico de enfermedad celíaca. Ante esta sospecha, lo primero
que debemos hacer es una prueba poco invasiva y muy específica, como los anticuerpos anti-
transglutaminasa tisular. En caso de demostrar su positividad, haríamos la biopsia y posteriormente
instauraríamos dieta sin gluten, valorando la respuesta de los anticuerpos a este tratamiento.(R2)

 
 
 
 
90. Niño de 7 años presenta rash eritematoso en ambas mejillas, tronco y zona proximal
de extremidades, con tendencia a aclararse en la zona central. Con respecto a esta
enfermedad, señale la afirmación FALSA:

1. 1. Es producida por el parvovirus B19.


2. 2. El período prodrómico se caracteriza por fiebre alta.
3. 3. El exantema característico aparece en tres etapas.
4. 4. Como complicación, puede producirse una crisis aplásica grave.
Gráfico de respuestas
Comentario

Eritema infeccioso es causado por parvovirus B19, el exantema tiene 3 etapas y entre las
complicaciones se encuentra: artritis, aplasia, abortos. Repase la siguiente tabla.(R2)

 
 
 
 
91. El tratamiento con agonistas de la GnRH se utiliza en ginecología para el tratamiento
de:

1. 1. Síndrome de Swyer.
2. 2. Cáncer de mama.
3. 3. Liquen escleroso.
4. 4. Incontinencia urinaria.
Gráfico de respuestas
Comentario

Pregunta muy puntual sobre uno de los tratamientos hormonales en ginecología.

Los agonistas de GnRH producen estimulación ovárica si la administración es pulsátil, o inhibición


si la administración es mantenida. Así, entre los usos de estos fármacos están los miomas uterinos,
la inducción de la ovulación, endometriosis...

Recuerde que son uno de los tratamiento de segunda línea en el tratamiento hormonal del cáncer
de mama metastásico en mujeres premenopáusicas (uno de los más usados es la goserelina), con
lo que la opción correcta es la 2.

En el resto de opciones, no tiene sentido el uso de estos análogos, ya que ni en el Swyer ni en el


Rokitansky podrían inducir estimulación ovárica; ni en la incontinencia ni en el liquen escleroso
tampoco tienen utilidad. Por otra parte, estas patologías suelen acontecer en el climaterio, y si
usamos estos fármacos aumentarían los síntomas propios del mismo.(R2)

92. A patient who has been recently diagnosed with chronic HBV in its replicative phase
comes to your office complaining of fatigue and malaise for the past few days. She started
interferon alfa-2b therapy 2 months ago. Laboratory studies show a slight increase in
bilirubin levels and transaminases. Which of the following is the correct answer?

1. 1. Another cause of hepatitis must be sought, as she may have some additional etiology.
2. 2. These findings are probably due to IFN-related toxicity.
3. 3. Steroids should be added to her treatment.
The most likely explanation is that she is passing through the phase of anti-HBe sero-
4. 4.
conversion.
Gráfico de respuestas
Comentario

En la hepatitis crónica B en fase replicativa, el objetivo del tratamiento con IFN alfa no es la
negativización del virus (esto sucede en muy pocos casos), sino el paso de fase replicativa a no
replicativa. Cuando esto ocurre, se debe a que el IFN alfa ha conseguido potenciar la respuesta
inmune del paciente frente al virus, siendo ésta suficiente para pasar conseguir el objetivo
propuesto. Esto es lo que se llama seroconversión anti- HBe, y clínicamente suele acompañarse
de un aumento transitorio de la bilirrubina, transaminasas y ligero cansancio. Es como una hepatitis
aguda, pero más leve, traduciendo la potenciación de la respuesta inmunológica frente al VHB, con
la consiguiente inflamación transitoria a nivel hepático.(R4)

93. ¿Cuál de las siguientes afirmaciones es cierta respecto al nistagmo del vértigo
posicional paroxístico benigno?:

1. 1. Tiene latencia.

 
 
 
 
2. 2. Es de larga duración.
3. 3. No se acompaña de sensación de giro de objetos.
4. 4. El nistagmo es vertical.
Gráfico de respuestas
Comentario
El VPPB es un vértigo periférico y cumple las característica clínicas de estos vértigos: aparición
brusca, sensación de giro de objetos y síntomas vegetativos. La exploración fundamental es la
provocación del vértigo con las pruebas posicionales de Dix- Hallpike. Estas maniobras provocan
con los cambios de posición vértigo de características periféricas. El nistagmo es unidireccional, en
resorte, horizonto- rotatorio, con latencia, se agota a los minutos y se habitúa o fatiga con la
repetición de la prueba.(R1)

94. Todos los siguientes son rasgos clínicos característicos del chancroide EXCEPTO:

1. 1. Evolución a úlceras genitales de aspecto sucio.


2. 2. Linfadenopatías inflamatorias dolorosas.
3. 3. Incubación más prolongada que el chancro duro.
4. 4. Respuesta a la eritromicina.
Gráfico de respuestas
Comentario
Pregunta fácil perteneciente al tema de las ETS de dermatología, del que se pregunta con bastante
asiduidad, por lo que es aconsejable saberse bastante bien. El chancro blando o chancroide, es
una ETS producida por el Haemophilus ducreyi, que se caracteriza por un periodo de incubación
de 1 a 3 días (el más corto de todas las ETS), tras el cual aparece un chancro doloroso de aspecto
sucio con inflamación perilesional, que puede ser múltiple por autoinoculación, y se suele
acompañar de adenopatías unilaterales dolorosas con tendencia a la abscesificación. El
tratamiento de elección son las tetraciclinas, y en segundo lugar, los macrólidos. Bibliografía:
Manual dermatología CTO, 4ºEd. Pag 9(R3)

95. Un paciente joven le consulta por un episodio de pérdida de la visión por un ojo y
dolor en el mismo que se acentúa con los movimientos. La exploración del fondo de ojo
es normal. ¿Cuál de los diagnósticos siguientes será el más probable?

1. 1. Glaucoma.
2. 2. Neuritis retrobulbar.
3. 3. Hipertensión intracraneal.
4. 4. Iridociclitis.
Gráfico de respuestas
Comentario

Ante un cuadro de pérdida visual monocular con fondo de ojo normal, debe sospecharse que el
paciente sufre una neuritis retrobulbar. Dado que la afectación se encuentra en el nervio óptico, y
no en la retina, el fondo de ojo no tendrá alteraciones. La sospecha se confirmaría explorando las
pupilas, demostrando de este modo un defecto pupilar aferente relativo. La neuritis retrobulbar
afecta sobre todo a mujeres jóvenes, aunque en este caso se trata de un varón. Otro dato típico,
también presente en este caso, es la presencia de dolor al movilizar el globo ocular.(R2)

96. ¿Cuál de las siguientes combinaciones de antibióticos es un tratamiento apropiado


para EPI?

 
 
 
 
1. 1. Ampicilina + Metronidazol.
2. 2. Ofloxacina + Clindamicina.
3. 3. Ceftriaxona + Amoxicilina.
4. 4. Doxiciclina + Gentamicina.
Gráfico de respuestas
Comentario

Para reconocer la combinación adecuada, en primer lugar debe conocer la etiología de EIP. Los
agentes etiológicos más importantes y que debe cubrir son: Chlamydia y gonococo, además de un
anaerobicida.

La respuesta correcta es la 2, ya que con ofloxacino, una fluorquinolona cubre gonococo y


Chlamydia y con clindamicina los anaerobios.

La respuesta 1 no es del todo correcta ya que te haría falta cubrir Chlamydia.

La respuesta 3 no es del todo adecuada ya que únicamente estaría utilizando beta lactámicos y no
cubriría Chlamydia.

La respuesta 4 no es correcta, ya que de igual forma no tendría acción anaerobicida.(R2)

97. ¿Cuál de las siguientes bacterias es esporulada?:

1. 1. Escherichia coli.
2. 2. Clostridium tetani.
3. 3. Staphylococcus aureus.
4. 4. Enterococcus faecalis.
Gráfico de respuestas
Comentario
Clostridium tetani es un bacilo grampositivo, anaerobio, que forma esporas. Las esporas pueden
sobrevivir años, y cuando infectan una herida germinan; Cl. tetani produce una toxina que penetra
en el sistema nervioso por la placa motora, asciende, y a nivel de la neurona internuncial provoca
una inhibición en la liberación de neurotransmisores inhibidores.(R2)

98. Niña de 20 días de vida, que acude por presentar dificultad para la alimentación, con
inactividad, apatía y escaso llanto, así como respiración ruidosa, Entre sus antecedentes
destaca haber precisado ingreso hospitalario por ictericia. En la exploración presenta
apatía, abdomen globoso, con hernia umbilical, edema de miembros con piel fría y
moteada. La fontanela anterior mide 3x3 cm y la posterior 2x2 cm. Presenta baja
implantación del pelo y macroglosia. El recuento y fórmula sanguínea son normales, sin
alteraciones en los datos bioquímicos. El diagnostico mas probable es:

1. 1. Hipertiroidismo congénito.
2. 2. Fenilcetonuria.
3. 3. Hipotiroidismo congénito.
4. 4. Hiperplasia suprarrenal congénita por déficit de 21 hidroxilasa.
Gráfico de respuestas
Comentario

 
 
 
 
No puede fallar este caso. Le están hablando del típico caso de hipotiroidismo congénito... es más
y si tiene dudas con las demás opciones por algo el autor de esta pregunta, para que se confunda,
en la opción 1 menciona hipertiroidismo y en la 3 hipotiroidismo, por lo que la respuesta es una de
estas dos.

Recuerde los datos claves del hipotiroidismo congénito:

La clínica del hipotiroidismo congénito es de instauración progresiva. Aparece facies peculiar (cara
tosca con párpados y labios tumefactos, nariz corta con base deprimida, hipertelorismo con boca
abierta y macroglosia), estreñimiento, ictericia prolongada, letargía, hernia umbilical, fontanelas
amplias y retraso en la maduración ósea.(R3)

99. ¿Cuál de los siguientes tipos histológicos en el carcinoma de la mama presenta una
mayor frecuencia de fracasos terapéuticos?

1. 1. Carcinoma ductal infiltrante.


2. 2. Carcinoma medular.
3. 3. Carcinoma tubular.
4. 4. Carcinoma lobulillar no infiltrante.
Gráfico de respuestas
Comentario

Pregunta sencilla sobre aspectos muy básicos sobre el cáncer de mama.

El carcinoma ductal infiltrante es el tipo histológico con peor pronóstico, y también el más
frecuente, representando un 80% del total de cánceres de mama.

Recuerde que, en el cáncer de mama, también es muy importante el número de ganglios afectados
como elemento pronóstico.(R1)

100. Un paciente alcohólico ha presentado un cuadro de vómitos intensos, acompañado


posteriormente de dolor retroesternal y fiebre. En la radiografía de tórax presenta un
derrame pleural izquierdo moderado. Se realiza una toracocentesis y se obtiene un
líquido seroso con pH de 5.5. Señale cuál de los siguientes es su diagnóstico de
sospecha:

1. 1. Empiema.
2. 2. Pleuritis reumatoide.
3. 3. Rotura esofágica.
4. 4. Tuberculosis.
Gráfico de respuestas
Comentario

Se trata de una rotura esofágica o síndrome de Boerhave, casi siempre en relación con vómitos en
pacientes alcohólicos. Suele cursar con mediastinitis expresada por intenso dolor y, además,
derrame pleural izquierdo rico en amilasa.(R3)

 
 
 
 

101. Mujer de 28 años


que consulta por notarse un nódulo en la mama derecha de consistencia firme,
polilobulado, bien delimitado y no adherido, de 1 cm de tamaño. Usted realiza una
exploración y solicita una USG mamario cuyo resultado se muestra en la imagen. Ante la
patología que sospecha, señale la afirmación FALSA:

Parece no tener influencia hormonal, por lo que no varía de tamaño durante el embarazo, y
1. 1.
pueden aparecer durante la menopausia.
Su mayor incidencia se presenta entre los 15 y los 35 años, y son bilaterales en un 20% de
2. 2.
los casos.
3. 3. Puede cursar con calcificaciones groseras intratumorales.
4. 4. Presentan una recidiva del 15% tras la cirugía.
Gráfico de respuestas
Comentario

Teniendo en cuenta la edad de la paciente y las características de la palpación, habría que


sospechar un fibroadenoma. En mamas jóvenes, recuerde que el ultrasonido muchas veces resulta
más informativo que la mastografía, por ser más densas. De ahí que se haya recurrido a esta
técnica. Lo que apreciamos en esta imagen son lesiones hipoecoicas, nodulares y bien definidas,
lo que confirma el diagnóstico de sospecha casi con absoluta seguridad.(R1)

102. ¿Cuál sería su actitud en la paciente del caso clínico anterior?

1. 1. Solicitar biopsia para tener estudio histopatológico, ante el riesgo de malignización.


2. 2. Extirpación quirúrgica debido a su rápido crecimiento.
3. 3. Observación y controles con exploración y USG mamario y/o mastografía.
4. 4. No es necesaria la realización de una P.A.A.F.
Gráfico de respuestas
Comentario

Ante una sospecha de fibroadenoma con una ecografía de estas características, bastaría la
observación periódica, tal como se explica en la respuesta 3. Recuerde que el seguimiento
inicialmente sería por USG, por la edad de la paciente, y cuando pasen los años acabaremos

 
 
 
 
realizando las mamografías periódicas como en cualquier otra paciente.(R3)

103. Uno de los siguientes factores NO implica alto riesgo de suicidio:

1. 1. Menos de 40 años.
2. 2. Viudo.
3. 3. Problema físico grave.
4. 4. No creyente.
Gráfico de respuestas
Comentario
Ser creyente católico y judío protege del suicidio, y esto más que a la religión en sí se debe al
fuerte comunitarismo que fomentan estas religiones, lo que palia el aislamiento social, la cual sí es
un claro factor de riesgo. Ser varón, viudo o divorciado, enfermo, con problemas sociales y con
más de 65 años son claros factores de riesgo de suicidio. Ser joven ("tener menos de 40 años") no
es un factor de riesgo.(R1)

104. Mujer de 75 años de edad, con HTA controlada, insuficiencia cardíaca e


hipercolesterolemia, ingresa por descompensación de su insuficiencia cardíaca. El
segundo día del ingreso, comienza con dolor abdominal cólico, de inicio súbito,
acompañado de sudoración y rectorragia, sin vómitos acompañantes. En la exploración
física, destaca el abdomen doloroso, con leve distensión, y el tacto rectal muestra restos
hemáticos. Presenta Hto. 35%, Hb 11.2 g/dl, VCM 79 fl, urea 55, creatinina 1.1 mg/dl. Se
realiza una colonoscopía, que muestra inflamación y edema difuso a lo largo del colon,
especialmente en ángulo esplénico, respetándose el recto. Teniendo en cuenta el
diagnóstico más probable, señale la afirmación CORRECTA:

1. 1. Lo más probable es la oclusión de la arteria cólica media o de la mesentérica superior.


Si se encuentra una elevación de la amilasa, se confirmaría el diagnóstico de pancreatitis
2. 2.
aguda.
Aunque algunos casos ceden con medidas conservadoras, normalmente es necesario el
3. 3.
tratamiento quirúrgico.
4. 4. Si se realizase un enema opaco, sería probable encontrar imágenes en huella digital.
Gráfico de respuestas
Comentario

Un paciente anciano, con FRCV, dolor abdominal de tipo cólico y rectorragia debe hacerte pensar
en una colitis isquémica. Recuerde que es una enfermedad de la microvascularización colónica por
lo que la arteriografía no es útil. El diagnóstico se realiza mediante colonoscopía, donde
encontraremos que el recto está indemne, y en el enema opaco son típicas las imágenes en huella
digital (respuesta 4 correcta).(R4)

105. La hipertrofia tímica fisiológica de un lactante se localiza topográficamente por


medio de las radiografías frontal y lateral de tórax en:

1. 1. Mediastino postero-inferior.
2. 2. Mediastino antero-superior.
3. 3. Mediastino postero-superior.
4. 4. Retrotraqueal.
Gráfico de respuestas
Comentario

 
 
 
 
Pregunta muy sencilla sobre un subtema rentable para el MIR: la localización de las masas
mediastínicas. Para saber las masas que aparecen en el mediastino anterior, puedes utilizar la
regla de las 4 T: Timoma ((la más frecuente), Teratoma, Tiroides y el "Terrible" linfoma. En el
mediastino medio se encuentran los quistes del desarrollo (pericárdicos y broncogénicos), los más
frecuentes, y la hernia de Morgagni. En el posterior aparecen tumores neurógenos (los más
frecuentes), la hernia de Bochdaleck y quistes entéricos. Observa que el linfoma puede aparecer
en cualquiera de los compartimentos.(R2)

106. Señale la cierta con respecto al signo de Schwartz:

1. 1. Es la hiperemia del promontorio en la otosclerosis, vista a través del tímpano.


2. 2. Se ve en todos los casos de otosclerosis.
3. 3. Es la provocación de un nistagmo ante los ruidos fuertes.
4. 4. Es la producción de vértigo con cambios de presión en el CAE.
Gráfico de respuestas
Comentario
El signo de Schwartze es una mancha rojiza por hiperemia que aparece en el promontorio de
algunas otoesclerosis. Indica un foco otosclerótico activo y suele acompañarse a otosclerosis con
componente coclear. Además de no estar presente en todas las otoesclerosis es necesario que la
membrana timpánica presente algo de transparencia para poder observarla.(R1)

107. Un paciente de 17 años, varón, sin antecedentes de interés, acude al hospital por un
cuadro de 2 días de evolución de fiebre alta, vómitos y dolor torácico no irradiado. Se le
realiza un electrocardiograma que muestra ondas T discretamente negativas de V2 a V5
, DI, DII y aVF. Los marcadores de lesión miocárdica resultan positivos con una troponina
T en meseta elevada por 8 veces su valor normal. Usted establece el diagnóstico de
miocarditis. ¿Cuál de las siguientes considera ERRÓNEA de acuerdo con el diagnóstico
que ha establecido?:

1. 1. La biopsia endomiocárdica es imprescindible para el diagnóstico.


Los hallazgos en el ECG suelen ser muy variables, desde la inversión de la onda T al
2. 2.
ascenso del segmento ST o la aparición de un bloqueo de rama.
3. 3. Puede presentarse en forma de muerte súbita por fibrilación ventricular.
4. 4. Hay que realizar un ecocadiograma para valorar la función ventricular.
Gráfico de respuestas
Comentario

La biopsia endomiocárdica no es necesaria para el diagnóstico.


Las infecciones virales representan la etiología más común. Los virus implicados suelen ser
adenovirus, parvovirus B19 y enterovirus. Más raras son las causadas por sarcoidosis, enfermedad
de Crohn o toxinas entre otras. Las manifestaciones clínicas y los hallazgos en e ECG son muy
variables. Puede cursarde forma prácticamente inadvertida hasta la muerte súbita. Es necesario
valorar la función ventricular, porque aunque la mayoría de casos son leves y con buen pronóstico,
se han descrito casos de insauficiencia cardiaca grave por disfunción ventricular severa.(R1)

108. Un niño de 5 días de edad, nacido con 1,200 g de peso, muestra mal aspecto general,
gran distensión abdominal, retiene en el estómago las tomas administradas por sonda y
tiene algún vómito. En la radiografía simple hay distensión de asas abdominales, y se ve

 
 
 
 
aire en la pared del intestino, pero no en el peritoneo. ¿Qué diagnóstico le sugiere este
cuadro clínico?:

1. 1. Ileo meconial por fibrosis quística.


2. 2. Estenosis o atresia de duodeno.
3. 3. Enterocolitis necrotizante
4. 4. Atresia de ano.
Gráfico de respuestas
Comentario

El caso clínico es muy sugestivo de enterocolitis necrotizante. Es frecuente esta entidad en RNPT.
Además las radiografía muestra neumatosis intestinal (aire en la pared) lo que nos refuerza la
sospecha de que el neonato esté sufriendo una enterocolitis necrotizante.(R3)

109. Sobre la osteoporosis tipo I o menopáusica, marque lo falso:

1. 1. Se manifiesta principalmente en hueso trabecular


2. 2. Se manifiesta principalmente en los extremos distales de los brazos
3. 3. Presenta fracturas vertebrales silenciosas
4. 4. Presenta fracturas vertebrales por aplastamiento muy dolorosas
Gráfico de respuestas
Comentario

Las fracturas vertebrales en mujeres posmenopáusicas son una causa típica de dolor de espalda;
suelen ser muy dolorosas, y por tanto la respuesta 3 es falsa.(R3)

110. Los criterios de New York (1984) para el diagnóstico de espondilitis anquilosante
son los siguientes, EXCEPTO:

1. 1. Historia de dolor de espalda de tipo inflamatorio.


2. 2. Limitación de la movilidad lumbar en el plano frontal y sagital.
3. 3. Disminución de la lordosis lumbar y aumento de la cifosis dorsal.
4. 4. Limitación de la expansión torácica en relación a la edad y sexo.
Gráfico de respuestas
Comentario

Para el diagnóstico de espondilitis anquilosante se utilizan los criterios modificados de Nueva York.
Que comprende hallazgos radiológicos y clínicos.

CRITERIO RADIOLOGICO. - Presencia de sacroileitis radiológica (unilateral de grado III ó IV, o


bilateral de grado II).

CRITERIOS CLINICOS: - Dolor lumbar de características inflamatorias (insidioso, empeora con el


reposo, rigidez). - Limitación de la movilidad de la columna lumbar en el plano sagital y frontal (test
de Schöber). - Disminución de la expansión de la expansión torácica en relación con los valores
normales para el sexo y la edad del paciente. El diagnóstico se establece cuando al criterio
radiológico (ineludible) se une al menos uno de los criterios clínicos.(R3)

111. La positividad de los anticuerpos anti SSA/Ro se ha asociado a todas las


circunstancias siguientes EXCEPTO:

 
 
 
 
1. 1. Lupus eritematoso sistémico (LES).
2. 2. Lupus cutáneo subagudo (LCSA).
3. 3. Lupus inducido por fármacos.
4. 4. Lupus asociado a deficiencia del complemento.
Gráfico de respuestas
Comentario

Esta pregunta no debería plantearte dudas. Los anticuerpos Anti- Ro se han preguntado en el MIR
en numerosas ocasiones. Debes recordar que se asocian a todo lo siguiente:

Mayor frecuencia de daño renal, cuando aparecen en pacientes con LES.

Lupus neonatal.

Lupus del anciano.

Lupus cutáneo subagudo.

Los anticuerpos anti- Sm se caracterizan por ser los más específicos de la enfermedad. Los
anticuerpos anticentrómero son típicos de la forma circunscrita de la esclerodermia. Los anti- Jo1
aparecen en la polimiositis- dermatomiositis, asociando mayor frecuencia de afectación pulmonar.
Por último, los anti- histona son los típicos del lupus inducido por fármacos (respuesta 4
correcta).(R3)

112. A Urgencias llega un paciente que ha sufrido en la calle una agresión por arma
blanca. En la exploración, presenta herida inciso-contusa en cuadrante superior
izquierdo del abdomen, de unos 4 cm de longitud, por la que se eviscera parcialmente el
epiplón. Además, el paciente refiere fuerte dolor abdominal, con defensa importante a la
palpación y ausencia de ruidos intestinales. ¿Cómo se clasificaría esta herida
quirúrgica?:

1. 1. Limpia.
2. 2. Limpia-contaminada.
3. 3. Contaminada.
4. 4. Contaminada-sucia.
Gráfico de respuestas
Comentario

Tenga mucho cuidado con esta pregunta, aquí no sirve la clasificación habitual de la herida
quirúrgica… Porque no estamos ante una herida quirúrgica, sino traumática. No es muy probable
que el agresor se haya lavado las manos, puesto guantes ni utilizado un arma estéril... Por ello, las
heridas traumáticas se consideran contaminadas de entrada (respuesta 3 correcta). Si nos hablan
de la presencia de tejido desvitalizado, cuerpos extraños o contaminación fecal, entonces se diría
que es sucia.(R3)

113. ¿Cuál de las siguientes afirmaciones relativas al tratamiento de la hepatitis crónica


viral es FALSA?

Una respuesta viral mantenida (negativización del ADN) en la hepatitis crónica B se


1. 1.
considera curación de la infección viral.

 
 
 
 
La ausencia de respuesta viral temprana permite interrumpir el tratamiento en pacientes
2. 2.
con hepatitis crónica C, evitando efectos secundarios y costes económicos innecesarios.
3. 3. La duración del tratamiento antiviral en la hepatitis crónica C depende del genotipo viral.
El tratamiento antiviral indicado en la hepatitis crónica por virus C es la asociación de
4. 4.
interferón alfa pegilado y ribavirina.
Gráfico de respuestas
Comentario

Cuando veas dos enunciados muy parecidos pero distintos en algo, la respuesta correcta suele
corresponder a uno de ellos. Un claro ejemplo es lo que aquí sucede con las respuestas 1 y 2.

El tratamiento de una hepatitis B puede conllevar una respuesta viral sostenida, con negativización
del DNA, sin que ello implique la curación de la infección viral. De hecho, la curación, entendiendo
como tal la negativización del HBsAg, sólo se produce en un 10% de los casos. En la mayoría de
estos pacientes, lo que se consigue es un paso de fase replicativa a no replicativa, es decir, la
seroconversión anti-e. Por consiguiente, la respuesta falsa es la 2. Respecto al virus C, has de
saber que la duración del tratamiento es variable en función de la cepa: un año para el genotipo 1,
y seis meses en los genotipos 2 y 3 (en pacientes inmunocompetentes). Si se consigue una
respuesta viral mantenida (negativización del ARN), se considera curación de la infección viral.
Recuerda que el tratamiento se apoya en la combinación de IFN pegilado con ribavirina. Como
dice la respuesta 3, si no se consigue una negativización del RNA a los tres meses, está justificada
la retirada del tratamiento, puesto que es muy improbable que se produzca después.(R1)
Evolución natural de la infección por VHB

114. A 32-week pregnant woman, with a history of preeclampsia in her previous


pregnancy, presents uterine height and fetal ultrasound biometry smaller than
amenorrhea (offset -3 weeks). What diagnostic test would you order?

1. 1. Glucose-tolerance test
2. 2. Doppler ultrasound
3. 3. Nitrazine test
4. 4. Cervix sonographic assesment

 
 
 
 
Gráfico de respuestas
Comentario

Para descartar la opción 3 y 4 basta con saber para qué sirve cada test. El test de nitrazina se
utiliza en la sospecha de ruptura prematura de las membranas; utiliza el pH para distinguir el
líquido amniótico de la orina y secreciones vaginales. La valoración por USG cervical se utiliza en
la sospecha de parto prematuro, lo que tampoco tendría sentido ante la única clínica de feto
pequeño. La diabetes gestacional suele producir fetos macrosómicos (aunque también podrían ser
más pequeños si hubiera patología vascular). La clave aquí es el antecedente de preeclampsia,
una patología vascular placentaria que alteraría el crecimiento (CIR) en edades tardías de
gestación, y la prueba idónea para detectar ésta condición es la flujometría doppler
fetoplacentaria(R2)

115. Un hombre de 60 años acude a nuestra consulta por dolor lumbar. Refiere que nota
las piernas acorchadas y entumecidas. Conserva los reflejos osteotendinosos. Al
investigar nos dice que cuando se encuentra sentado le duele menos y le desaparecen
las otras molestias. ¿Cuál será su sospecha diagnóstica?:

1. 1. Hernia discal lumbar.


2. 2. Síndrome de la cola de caballo.
3. 3. Estenosis del canal raquídeo.
4. 4. Espondilolistesis.
Gráfico de respuestas
Comentario
Pregunta algo complicada sobre un tema poco importante de cara al MIR como es la estenosis del
canal raquídeo (opción 4 correcta). Recuerda la clínica típica: dolor lumbar, lumbociáticas (con
frecuencia bilaterales) y es la causa más frecuente de claudicación neurogénica de miembros
inferiores (dolor lumbar, en nalgas y en piernas al caminar o en bipedestación, que cede con el
reposo). El dolor aumenta con la hiperextensión de la columna y, a diferencia de la hernia discal,
cede al sentarse (con la flexión de la columna), único dato que necesitas para contestar a la
pregunta y que no debes olvidar.(R3)

116. Mujer de edad media con historia de asma infantil que, tras una infección respiratoria
de vías altas, es remitida a Urgencias por crisis de broncoespasmo severa. Se realiza una
radiografía de tórax que objetiva infiltrados pulmonares bilaterales y una analítica que
demuestra una proteinuria y anemia. La presencia de una intensa eosinofilia en sangre
periférica, junto con el carácter fugaz de los infiltrados en control radiológico posterior
evolutivo, nos orienta a la siguiente entidad diagnóstica:

1. 1. Angeítis alérgica o granulomatosa de Churg-Strauss.


2. 2. Síndrome de Löeffler.
3. 3. Aspergilosis broncopulmonar alérgica.
4. 4. Linfangitis carcinomatosa.
Gráfico de respuestas
Comentario

La eosinofilia podría hacernos pensar en las opciones 1, 2 y 3. Clínicamente, encajaría en una


enfermedad de Churg- Strauss, que es la única que explicaría la anemia y la proteinuria.(R1)

 
 
 
 
117. Recién nacido a término que presenta ictericia severa en las primeras 24 horas de
vida. La causa más probable es:

1. 1. Errores congénitos del metabolismo.


2. 2. Hipotiroidismo congénito.
3. 3. Cefalohematoma.
4. 4. Enfermedad hemolítica autoinmune.
Gráfico de respuestas
Comentario

Le preguntan la CAUSA MÁS PROBABLE de ictericia severa en el RN en las primeras 24 horas


por lo que debe de responder la opción 4.

La causa más frecuente de elevación de bilirrubina en RN por arriba del percentil 95% (severa), es
la incompatibilidad ABO.

En caso de hipotiroidismo se trata de una ictericia prolongada. Los erroes congénitos del
metabolismo podría ser una opción, pero vuelvo a repetir, le preguntan la CAUSA MÁS
PROBABLE. Por último cree que el cefalohematoma te produce con mucha frecuencia ictericias
severas?(R4)

118. Paciente de 30 años de edad con trastornos del ciclo menstrual, obesidad e
hirsutismo. Examen de laboratorio: aumento de resistencia a la insulina y de la relación
LH/FSH. ¿Cuál es el diagnóstico más probable?

1. 1. Insuficiencia ovárica secundaria


2. 2. Síndrome de ovarios poliquísticos
3. 3. Diabetes mellitus
4. 4. Hipogonadismo hipogonadotrópico
Gráfico de respuestas
Comentario

Para proceder al diagnóstico de SOP es preciso que se cumplan dos de los tres criterios de
Rotterdam:

- Oligoovulación o anovulación.

- Hiperandrogenismo (clinico o bioquimico).

- Ovarios poliquisticos demostrados por ultrasonido transvaginal. Con que la imagen sea en un solo
ovario es suficiente.(R2)

119. La aparición de un patrón de desaceleración variable o Dip III en la monitorización


tocográfica de una embarazada es indicativo de:

1. 1. Abruptio placentae.
2. 2. Compresión del cordón umbilical.
3. 3. Enfermedad hemolítica perinatal.
4. 4. Rotura uterina.
Gráfico de respuestas

 
 
 
 
Comentario

Una pregunta muy importante sobre los tipos de deceleraciones. Conceptualmente, una
deceleración es un desecenso de la línea de base de más de 15 lpm, durante más de 15- 20
segundos. Las deceleraciones NO deben estar presentes en condiciones normales. No obstante, el
significado de cada tipo de deceleración es absolutamente distinto. Debes conocer tres:

- Deceleraciones precoces (DIPS tipo I): Aparecen a la vez que la contracción uterina. Son las más
frecuentes. Suelen deberse a la estimulación vagal, por compresión de la cabeza fetal.

- Deceleraciones tardías (DIPS tipo II): Existe un decalaje de unos 20 segundos entre la
contracción uterina y la deceleración. Orientan a acidosis fetal (peor pronóstico). Si ves DIPS tipo
II, debes realizar una microtoma fetal para medir el pH.

- Deceleraciones variables (DIPS umbilicales): Son inconstantes en morfología y sincronía.


Sugieren patología de cordón, y el pronóstico es intermedio entre las I y II (respuesta 2
correcta).(R2)

120. De los siguientes ansiolíticos, señale cuál posee MENOS acción sedante:

1. 1. Buspirona.
2. 2. Diazepam.
3. 3. Alprazolam
4. 4. Clormetiazol.
Gráfico de respuestas
Comentario
El tema de psicofármacos es el más preguntado en Psiquiatría, aunque no constituye un capítulo
diferenciado, siempre debemos prestarle mucha atención. En este caso, una de las
particularidades que debemos conocer, es la existencia de la buspirona, un agonista parcial del
receptor serotoninérgico 5HT- 1a, propuesto como ansiolítico puro, que carece de efecto en dosis
única, sin sedación, sin presentar tolerancia cruzada con otros depresores del SNC, en el que no
se ha detectado síndrome de abstinencia y presenta latencia de respuesta (recuerda más a un
antidepresivo).(R1)

121. Indique en qué circunstancia de las que se enumeran a continuación NO sospecharía


la necesidad de una intubación inminente en un paciente que ha sufrido una quemadura:

1. 1. Nos encontramos al paciente inconsciente en el suelo.


2. 2. El paciente acude a urgencias con una quemadura en espalda y presenta ronquera.
3. 3. Tose profusamente y está muy nervioso.
4. 4. Presenta quemaduras en el abdomen y área genital.
Gráfico de respuestas
Comentario

Hay signos directos e indirectos en el quemado que sugieren lesiones significativas en la vía aérea
en relación a la inhalación de humo y que nos han de poner en alerta de una posible intubación
urgente, como son la tos, el estridor, las sibilancias, y la ronquera. En cuanto al tipo y localización
de la quemadura las circunferenciales en el tórax, las faciales que incluyen boca y nariz, y las del
cuello, así como cuando se encuentra hollín en boca o nariz o ampollas y edema en orofaringe,
deben hacer pensar en un posible compromiso de la vía aérea inminente y adelantarnos a sus

 
 
 
 
consecuencias mediante la intubación. Por supuesto siempre que el paciente esté inconsciente
presente insuficiencia respiratoria deberemos intubar.(R4)

122. Which of the following is not a causative mechanism of anemia in systemic


sclerosis?

1. 1. Iron deficit caused by gastrointestinal bleeding.


2. 2. Anemia of chronic disorders.
3. 3. Microangiopathic hemolytic anemia.
4. 4. Bone marrow T-lymphocyte infiltration.
Gráfico de respuestas
Comentario

Esta pregunta no refleja lo más importante del tema, pero puede ayudarle a introducirte en él. Debe
tener claro que la esclerosis sistémica progresiva es un proceso autoinmune en cierto sentido, pero
no existe expansión clonal desmesurada de células inmunes como los linfocitos, los fibroblastos
son normales pero producen colágeno excesivo. Si existe anemia, será por ferropenia como en la
población general; al tratarse de un trastorno crónico, puede darse esta anemia multifactorial; la
anemia microangiopática es propia de las crisis renales esclerodérmicas, que cursan con HTA
maligna; por último, el intestino delgado tiene un tránsito enlentecido, lo que facilita el íleo con
sobrecrecimiento bacteriano, y las bacterias así desconjugan la unión de B12 al factor intrínseco o
consumen factores fundamentales de la hematopoyesis.

La respuesta que no cuadra es la 4.(R4)

123. A full-term newborn is being attended at the delivery room presenting with a low
apgar score. His mother is a 17-year-old who denies drug consumption (she occasionally
smokes cigarettes). Prenatal screening was normal. The child has a respiratory rate of 30
rpm and a heart rate of 120 bpm. Physical examination shows a mild grimace with stimuli
and peripheral cyanosis. Which of the following statements is true?

1. 1. Cerebral palsy is highly probable


Observation is the best treatment. Antibiotic treatment shouldn't be started until blood
2. 2.
culture results are available
3. 3. Positive-pressure ventilation is required
4. 4. Teophylline may be useful
Gráfico de respuestas
Comentario
Positive-pressure ventilation is required. Apgar score summarizes the vital state of the newborn. It
is generally evaluated one and five minutes after birth, and can be repeated later. Low scores do
not necessarily mean bad prognosis, but medical assistance to the newborn must be given if the
score is below 3 in any case. The first measure is to provide an adequate respiratory support by
positive-pressure ventilation. Cerebral palsy incidence is only significantly increased if the Apgar
score remains low after 15 or 30 minutes.(R3)

124. A 48-hour full-term newborn with an uncomplicated vaginal delivery, is brought to


the emergency department by his mother presenting with green vomiting. He has not
passed his first stool yet and his abdomen is distended. Family history of cystic fibrosis
is present. Abdominal X ray findings are "air-fluid levels with "bubbly" appearance of the
distended intestinal loops". What is the most likely diagnosis?

 
 
 
 
1. 1. Intussusception
2. 2. Meconium ileus
3. 3. Necrotizing enterocolitis
4. 4. Pyloric atresia
Gráfico de respuestas
Comentario
Meconium ileus. Cystic fibrosis can start giving signs from birth with meconium ileus. Typical
radiological findings are the bubble sign, that represents the presence of impacted meconium
pellets inside the bowel, and air-fluid levels in the distended loops. Meconium plug may occur in
healthy children but in this condition meconium consistency is not altered.(R2)

125. La presencia de edema del paladar y úvula en un paciente con linfadenopatía y


esplenomegalia debe hacer pensar en:

1. 1. Linfoma de Hodgkin.
2. 2. Mieloma de Bence-Jones.
3. 3. Enfermedad de cadenas pesadas gamma.
4. 4. Enfermedad de cadenas pesadas mu.
Gráfico de respuestas
Comentario
La afección ORL es característica de dicha entidad.(R3)

126. La aparición de un patrón de desaceleración variable o Dip III en la monitorización


tocográfica de una embarazada es indicativo de:

1. 1. Abruptio placentae.
2. 2. Compresión del cordón umbilical.
3. 3. Enfermedad hemolítica perinatal.
4. 4. Sufrimiento fetal grave.
Gráfico de respuestas
Comentario

Una pregunta muy importante sobre los tipos de deceleraciones. Conceptualmente, una
deceleración es un desecenso de la línea de base de más de 15 lpm, durante más de 15-20
segundos. Las deceleraciones NO deben estar presentes en condiciones normales. No obstante, el
significado de cada tipo de deceleración es absolutamente distinto. Debe conocer tres:

-Deceleraciones precoces (DIPS tipo I): Aparecen a la vez que la contracción uterina. Son las más
frecuentes. Suelen deberse a la estimulación vagal, por compresión de la cabeza fetal.

-Deceleraciones tardías (DIPS tipo II): Existe un decalaje de unos 20 segundos entre la contracción
uterina y la deceleración (ver imagen). Orientan a acidosis fetal (peor pronóstico). Si ve DIPS tipo
II, debe realizar una microtoma fetal para medir el pH.

-Deceleraciones variables (DIPS umbilicales): Son inconstantes en morfología y sincronía.


Sugieren patología de cordón, y el pronóstico es intermedio entre las I y II (respuesta 2
correcta).(R2)

 
 
 
 
127. Una mujer de 58 años acude para una visita de seguimiento por diabetes mellitus e
hipertensión. Se siente bien pero afirma que ha dejado de tomar el verapamilo por
estreñimiento. Presenta intolerancia a los IECAs por tos. En la exploración, la presión
arterial es de 156/92 mmHg. En la analítica incluyen una creatinina de 1,6 mg/dl, excreción
de proteínas en orina de 24 horas de 1,5 g/día y un aclaramiento de la creatinina de 45
ml/min. Sobre esta base ¿cuál es el tratamiento más eficaz para enlentecer la progresión
de la nefropatía diabética tipo 2 de la paciente?

1. 1. Inhibidor de la enzima convertidora de la angiotensina.


2. 2. Bloqueador del receptor de la angiotensina.
3. 3. Antagonista del calcio.
4. 4. Alfa-bloqueante.
Gráfico de respuestas
Comentario

A la hora de seleccionar el tratamiento antihipertensivo de elección en un paciente concreto, hay 2


factores que deben evaluarse cuidadosamente: la comorbilidad y los efectos adversos conocidos.

Al tratarse de un paciente diabético con nefropatía, el tratamiento de elección sería un IECA, pues
estos fármacos en este contexto, han demostrado su eficacia para disminuir el riesgo y la velocidad
de progresión de la enfermedad renal por la diabetes. Sin embargo, nos refieren que la paciente no
tolera los IECA por tos. Este fenómeno, típico de mujeres de edad media o avanzada de raza
blanca, se sospecha que, al menos en parte, se produce por el acúmulo de sustancias como la
bradiquinina que se acumula por tener bloqueada la ECA (que es la enzima encargada de
degradarla). Para ese caso se ha demostrado que el bloqueo del eje renina-angiotensina-
aldosterona da un paso “más allá” con el empleo de un ARA-II que es también eficaz para prevenir
la progresión de la nefropatía y sería, por tanto, el tratamiento de elección.

En la tabla se recogen los principales fármacos indicados como tratamiento de elección para la hipertensión arterial en diversas
situaciones clínicas.

Parece que es un mito, curiosamente popularmente muy arraigado, que la hipertensión arterial produce habitualmente toda una
pléyade de síntomas a los pacientes afectados. La mayoría de los pacientes que hay en el mundo con hipertensión arterial no saben
que son hipertensos, y eso es así precisamente porque la hipertensión arterial no suele producir ningún síntoma. Por ello, la OMS
incluso ha declarado que la hipertensión arterial probablemente sea la principal causa de muerte natural tratable en el mundo.

En concreto, la cefalea ha sido investigada en varios estudios y la relación entre este síntoma y la hipertensión, en caso de existir,
es marginal.

Sin embargo, en casos concretos, son muchos los síntomas que pueden aparecer en función del desarrollo de las denominadas
lesiones en los “órganos diana” de la hipertensión arterial, como son el corazón (angina de pecho, infartos, disnea por fallo diastólico
o incluso sistólico...), el sistema nervioso central (cuadro de déficit neurológico global en las emergencia hipertensivas o focal por el
desarrollo de isquemia secundaria a la enfermedad aterosclerótica facilitada por la HTA), la retina (diferentes grados de deterioro de
agudeza visual), etc.

Situaciones que favorecen o hacen desaconsejable el empleo de diversos


fármacos para la hipertensión arterial

FÁRMACO ACONSEJABLE DESACONSEJABLE

 
 
 
 

b-bloqueantes ·Enfermedad coronaria· · Asma· Bradiarritmias·


Taquiarritmias· Insuficiencia Claudicación grave· Riesgo
cardíaca· Hipertiroidismo· de diabetes· Síndrome
Embarazo (segunda línea) metabólico

Tiazidas · Insuficiencia cardíaca· · Gota· Riesgo de diabetes·


Ancianos· Raza negra Síndrome metabólico

Ieca o ara II · Insuficiencia cardíaca· · Embarazo· Estenosis


Disfunción sistólica· Diabetes· bilateral de arteria renal·
Proteinuria· Insuficiencia renal Hiperpotasemia
crónica· HTA vasculorrenal·
Raza negra· Síndrome
metabólico· Hipertrofia
ventricular

Calcioantagonistas · Angina de pecho· Ancianos· · Dihidropiridinas en


Claudicación intermitente· monoterapia en la isquemia
Síndrome metabólico· miocárdica· Estreñimiento
Hipertrofia ventricular·
Embarazo (segunda línea)

a-bloqueantes · Hipertrofia benigna de · Insuficiencia cardíaca·


próstata· Feocromocitoma Hipotensión ortostática

 
 
 
 

128. Niño de 4 años con fiebre


alta de 6 días de evolución acompañada de irritabilidad. La madre refiere además haberle
notado las manos y los pies hinchados y dolorosos. A la exploración llama la atención
hiperemia conjuntival intensa sin exudado, un exantema de predominio en tronco y labios
enrojecidos y resecos. Es ingresado con sospecha de enfermedad infecciosa y tratado
con antibióticos tras la toma de cultivos. Al 10º día de evolución, la fiebre no ha
desaparecido y comienza a presentar alteraciones en manos y pies como los que se
muestran en la imagen. En relación al cuadro clínico citado, señale la opción FALSA:

1. 1. El diagnóstico más probable es enfermedad de Kawasaki.


2. 2. La imagen muestra la descamación en dedo de guante característica de la enfermedad.
Para confirmar el diagnóstico es necesario realizar biopsia de algún ganglio linfático
3. 3.
accesible.
4. 4. La base patogénica de la enfermedad es una vasculitis.
Gráfico de respuestas
Comentario

La imagen, en esta pregunta, es sólo un dato más, y nos muestra lo que ya nos podríamos haber
imaginado por el contexto de la pregunta: descamación plantar.

Se trata de una enfermedad de Kawasaki, en cuya patogenia parece estar implicado un


superantígeno que produciría una sobreactivación policlonal de los linfocitos T, que producirían
citoquinas responsables de las manifestaciones de la enfermedad.

En esta patología, el problema fundamental es que a veces se producen dilataciones


aneurismáticas en las coronarias, por lo que estos niños estarían predispuestos a infartos de
miocardio a edades muy precoces.

El diagnóstico de enfermedad de Kawasaki se realiza en base a criterios clínicos y no precisaría


confirmación histológica (respuesta 3 falsa).(R3)

 
 
 
 
129. El niño de la pregunta anterior fue ingresado y tratado con gammaglobulina
intravenosa además de aspirina. Sobre el manejo de este paciente, señale la opción
FALSA:

Si la fiebre no cede con gammaglobulina el fármaco de segunda elección son los


1. 1.
corticoides.
La gammaglobulina es eficaz en la prevención de la aparición de aneurismas coronarios en
2. 2.
la mayoría de los casos.
La complicación más temida en la enfermedad de Kawasaki es la aparición de aneurismas
3. 3.
coronarios.
La descamación en dedo de guante es característica de la fase aguda de la enfermedad y
4. 4.
por tanto muy útil para el diagnóstico precoz y el inicio del tratamiento.
Gráfico de respuestas
Comentario

La descamación es habitual cuando la enfermedad ya lleva cierta evolución, por lo que no tendrá
aplicación alguna para llegar a un diagnóstico precoz (respuesta 4 falsa).

El problema principal que puede complicar este cuadro son los aneurismas coronarios, que
aparecen con menor frecuencia cuando se utiliza gammaglobulina intravenosa, como se comenta
en las opciones 1 a 3.(R4)

130. Una mujer de 64 años acude a su consulta por dolor en la rodilla izquierda e
imposibilidad para extender completamente la pierna desde hace tres días, cuando, al
entrar a un coche en una posición forzada, sintió un dolor agudo y un chasquido. ¿Cuál
de las siguientes respuestas es cierta respecto a los hallazgos que espera encontrar en
la exploración?

La prueba del cajón anterior es positiva y produce un desplazamiento mayor si se realiza


1. 1.
con la tibia en rotación externa.
Se produce dolor en la interlínea articular al rotar a un lado y otro con las rodillas en
2. 2.
flexión de 90º.
A la palpación de la interlínea articular se produce dolor, agravándose cuando se hace con
3. 3.
la rodilla en extensión completa.
Es muy probable que haya un derrame articular intenso, pero mucho menor que el que se
4. 4.
instauraría en las primeras horas tras el movimiento desencadenante de la lesión.
Gráfico de respuestas
Comentario

Al entrar en un coche, es muy probable que la pierna estuviese en semiflexión y hubiese cierto
grado de rotación en la rodilla, lo que resulta sugestivo de una lesión meniscal. Por este motivo,
cabría esperar el hallazgo descrito en la respuesta 3: dolor en la interlínea articular cuando se
produce la rotación (maniobra de McMurray).(R2)

131. Pre escolar masculino, de 2 años 9 meses de edad, pesa 14 kg; hermano mayor está
en tratamiento hace 2 meses por TB pulmonar (baciloscopia ++); el pre escolar
asintomático examen físico normal, PPD 13mm; ¿Cuál es su diagnóstico y conducta,
marque lo CORRECTO:

1. 1. Infección TB: INH 210 mg/ día.


2. 2. TB pulmonar: tomar radiografía de tórax.

 
 
 
 
3. 3. Infección por TB: INH 70 mg/día.
4. 4. TB pulmonar, baciloscopia por sonsa orogástrica y esperar frotis.
Gráfico de respuestas
Comentario

Tema que debes dominar a la perfeción, generalmente se pregunta en forma de caso clínico.
Indicaciones tratamiento enfermedad tuberculosa latente:

- Conversores recientes (positivización de la prueba de la tuberculina a lo largo de los últimos dos


años), independientemente de la edad del paciente.

- Infectados por el VIH con prueba de la tuberculina positiva, así como aquellos con prueba de la
tuberculina negativa que hayan estado en contacto con pacientes tuberculosos.

- Portadores de lesiones fibróticas estables en la radiografía de tórax con una prueba de la


tuberculina positiva (excluyendo calcificaciones y paquipleuritis). En esta categoría se incluyen
igualmente los pacientes con silicosis y los UDVP, incluso si presentan una prueba de la
tuberculina negativa.

- Pacientes en lista de espera de trasplante de órgano sólido, o que vayan a ser sometidos a
tratamiento inmunosupresor prolongado (particularmente fármacos anti-TNF alfa), con prueba de la
tuberculina positiva.

- Contactos íntimos con un paciente bacilífero que tenga la prueba de tuberculina positiva, con
independencia de su edad. En los niños y los pacientes menores de 20 años se iniciará tratamiento
durante dos meses, aunque la primera prueba de la tuberculina hubiera sido negativa. A los dos
meses debe ser repetida y, si se ha hecho positiva, se continuará tratamiento durante siete meses
más; si siguie siendo negativa, se suspenderá.

La pauta consistirá en la administración de isoniacida durante un período entre 6-12 meses


dependiendo del caso en el que nos encontremos.(R3)

132. Paciente de 29 años de edad con antecedentes de hepatitis B fulminante que en el


10ª día postrasplante de hígado presenta fiebre, dolor en hipocondrio derecho y
disminución del volumen y color de la bilis. En la analítica presenta un aumento de
bilirrubina y de transaminasas y en la biopsia hepática se objetiva infiltración portal,
colangitis no destructiva y endotelitis. El tratamiento para la complicación que Ud
sospecha en este paciente es:

1. 1. Ganciclovir.
2. 2. Interferón.
3. 3. Interferón más lamivudina.
4. 4. Metilprednisolona en bolos.
Gráfico de respuestas
Comentario
Sobre los transplantes debes conocer los mecanismos de rechazo. No ha sido un tema muy
preguntado pero si han existido algunas preguntas sobre él y conviene tener en cuenta ciertas
ideas claves. El caso clínico nos describe un rechazo agudo o reversible que es el más frecuente y
se produce entre el 4º y el 14º día postoperatorio. La clínica y la analítica es inespecífica y difícil de
valorar. Debemos sospecharlo en los pacientes jóvenes (menores de 30 años) y cuando la
indicación de transplante fue por fallo hepático fulminante. La biopsia hepática revela endotelitis,

 
 
 
 
colangitis no destructiva o infiltrado portal y arteritis. Inmunológicamente, cuando los CD4 son
mayores que los CD8 es reversible con corticoides (20 % de los casos).(R4)

133. Una mujer de 20 años presenta desde hace una semana fiebre, dolor pleurítico en
hemitórax derecho y tos seca. En la Rx de tórax se evidencia un derrame basal derecho.
Tras realizar una toracocentesis se obtiene un exudado con predominio de monocitos y
glucosa disminuida. ¿Cuál de las siguientes afirmaciones respecto al caso de la paciente
le parece FALSA?:

La realización del Mantoux aportará poca información para el diagnóstico final de la


1. 1.
paciente.
2. 2. La determinación de la enzima ADA es útil ante la sospecha de tuberculosis.
3. 3. De ser una tuberculosis probablemente sea secundaria a una tuberculosis pulmonar previa.
4. 4. Para establecer el diagnóstico final puede ser necesario la realización de biopsia pleural.
Gráfico de respuestas
Comentario

Una pregunta sencilla sobre el derrame pleural tuberculoso. Vamos a enumerar algunas
características típicas del mismo:

- Predominio de linfocitos entre las células.

- Ausencia o pobreza en células mesoteliales.

- Glucosa disminuida.

- Adenosindeaminasa aumentada (ADA +).

- Características de exudado.

De las opciones que nos plantean, la incorrecta es la 3, puesto que el derrame suele ser una
manifestación de primoinfección. Recuerda que el diagnóstico puede ser muy difícil, ya que tanto la
baciloscopia como el cultivo pueden ser negativos incluso padeciendo la enfermedad (son pruebas
poco sensibles), lo que a veces obliga a proceder a una biopsia pleural. En cuanto al Mantoux, es
cierto que nos aporta poca información, por dos razones:

- Si es positivo, no significa que el derrame tenga que ser forzosamente tuberculoso.

- Si es negativo, no descarta un origen tuberculoso, ya que el derrame podría estar en relación con
una primoinfección, con lo que el Mantoux podría no haber positivizado todavía.(R3)

134. Una niña de 9 años, diagnosticada de coartación de aorta consulta por talla baja. En
la exploración física se observa talla en percentil 3 para su edad y pterigium coli. ¿Cuál
de los siguientes es el diagnostico más probable y qué exploración realizaría para
confirmarlo?

1. 1. Déficit de GH y determinación de IGF-1 sérica.

 
 
 
 
2. 2. Síndrome de Turner y cariotipo.
3. 3. Hipotiroidismo y determinación de TSH y T4 séricas.
4. 4. Disgenesia gonadal pura y cariotipo.
Gráfico de respuestas
Comentario

Pregunta importante y fácil de deducir, se ha preguntado ocasiolmente sobre el síndrome deTurner


en el ENARM. En las preguntas tipo caso clínico debe de tomar los datos más improtantes que le
dan: niña, coartación de aorta, talla baja, pterigium coli. Al darle estos datos inmediatamente debe
pensar en Síndrome de Turner y como bien sabe buscará la monosomía 45,X0 en el cariotipo.(R2)

135. Embarazada de 34 semanas que comienza con dolor uterino, de aparición brusca,
que se acompaña de ligero sangrado genital de color oscuro. En la exploración, el útero
es doloroso y no se relaja entre contracciones. Los tonos cardíacos fetales se escuchan
con dificultad. Su diagnóstico de sospecha será:

1. 1. Placenta previa.
2. 2. Desprendimiento de placenta normalmente inserta.
3. 3. Corioamnionitis.
4. 4. Aborto tardío inminente.
Gráfico de respuestas
Comentario

Las preguntas sobre las hemorragias del tercer trimestre son un clásico en el ENARM así que
debes dominar este tema.

Dentro de las causas de hemorragia del 3º trimestre, solamente hay dos que cursan con dolor, con
lo que el caso se simplifica bastante. Son dolorosas la rotura uterina y el desprendimiento
prematuro de placenta normalmente inserta (DPPNI, o “abruptio placentae”).

En la rotura uterina, el dolor suele ser brusco y localizado en la zona de rotura. El tono uterino
disminuye y el estado fetal es malo.

En el abruptio, se produce una hipertonía uterina. El dolor es, por ello, más generalizado, ya que
no es en un punto concreto. También existe mal estado fetal.

Cuando nos dicen que el útero “no se relaja” entre las contracciones, nos están dando el dato
clave. Por otra parte, nos hablan de una hemorragia color oscuro, que también es bastante típico
del DPPNI.(R2)

136. Ante un paciente con gota tofácea crónica, insuficiencia renal crónica y una
excreción renal de ácido úrico de 600 mgr/día ¿cuál de las siguientes pautas terapéuticas
le parece más adecuada?:

1. 1. Uricosúricos.
2. 2. Uricosúricos y colchicina.
3. 3. Alopurinol.
4. 4. Alopurinol y colchicina.
Gráfico de respuestas
Comentario

 
 
 
 
Una de las preguntas más habituales que se realizan sobre la hiperuricemia es la actitud
terapéutica que debes tomar en las distintas formas clínicas de la enfermedad. Existe un amplio
consenso sobre la necesidad de tratar a los pacientes con hiperuricemia que presetan episodios
recidivantes de artritis aguda (gota intercrítica) y a los que desarrollan manifestaciones crónicas de
la enfermedad (gota tofácea crónica). Dentro del arsenal terapéutico disponemos de dos tipos de
fármacos que disminuyen los niveles de ácido úrico: el alopurinol que disminuye su producción y
que está indicado en aquellos pacientes con aumento de producción de ácido úrico y los
uricosúricos que aumentan su eliminación renal y que está indicado en los que tieen disminuida la
excreción renal de ácido úrico. Para saber que tipo de paciente tienes se realiza el balance de
ácido úrico determinando la uricosuria de 24 horas. Lo normal es eliminar al menos 800 mgr de
ácido úrico en orina de 24 horas. Si la eliminación esta por encima de esta cifra ya sabes que la
causa de la hiperuricemia será el aumento de formación de ácido úrico y por lo tanto el tratamiento
será la admisnitración de alopurinol. En caso de que la cifra sea menor la causa de la
hiperuricemia es la disminución de la excreción renal de ácido úrico y debes adminístrale un
uricosúrico. Sin embargo no debes olvidar que los uricosúricos están contraindicados en pacientes
con nefrolitiasis o insuficiencia renal. En estos casos la alternativa es el alopurinol Como ves en el
caso que nos plantean el paciente tiene hiposecreción de ácido úrico (600 mgr), pero no puede
recibir uricosúrico porque tiene insuficiencia renal. El tratamiento correcto será el alopurinol
asociado a colchicina para prevenir la aparición de nuevos episodios de artritis aguda.(R4)

137. Una mujer de 59 años presenta una imagen espiculada de 8 mm de diámetro en la


mama derecha en la mastografía. Una biopsia con aguja gruesa evidencia la presencia de
un carcinoma ductal infiltrante grado 2, con receptores para estrógenos positivos en un
80% de las células tumorales, receptores para progesterona positivos del 60%, y un
oncogén Her-2 neu negativo. ¿Cuál sería el tratamiento primario de elección?

1. 1. Quimioterapia primaria en combinación con trastuzumab.


2. 2. Tumorectomía con exéresis del ganglio centinela con radioterapia posterior.
3. 3. Hormonoterapia con tamoxifeno.
4. 4. Mastectomía con linfadectomía axilar.
Gráfico de respuestas
Comentario

Con respecto al tratamiento del cáncer de mama que nos presentan en el enunciado se puede
optar por una tumorectomía, puesto que el estadio es un IIa (es decir, <5 cm, N1). En la segunda
parte del tratamiento se debe asociar radioterapia. Está indicada tras la cirugía conservadora.
También se emplea después de la mastectomía en pacientes de alto riesgo de recidiva local.

Con respecto a la linfadenectomía, que también ha de hacerse siempre en la cirugía del cáncer de
mama, optaremos por la exéresis del ganglio centinela (ganglio o grupo de ganglios que reciben en
primer lugar el drenaje linfático del tumor).(R2)

138. ¿Cuál de los siguientes signos radiológicos es específico de infarto intestinal?:

1. 1. Asas de intestino delgado dilatadas.


2. 2. Gas intramural o en el territorio portal.
3. 3. Neumoperitoneo.
4. 4. Líquido libre intraperitoneal.
Gráfico de respuestas
Comentario

 
 
 
 
El infarto intestinal se produce como consecuencia de una isquemia mesentérica aguda. Por ésta
se produce lesión de la pared intestinal, cursando con dilatación de asas, leucocitosis con
desviación, acidosis y por último, perforación. El signo específico de infarto intestinal es el gas
intramural como signo previo a perforación y gas en la porta como signo ominoso.(R2)

139. Una de las entidades que se enumeran a continuación NO está presente en el


síndrome de Gardner. Señale cuál es:

1. 1. Fibromas.
2. 2. Osteomas.
3. 3. Pólipos adenomatosos.
4. 4. Tumores malignos del sistema nervioso central.
Gráfico de respuestas
Comentario

Otra pregunta sobre síndromes de poliposis intestinal, tema poco preguntado en el nacional. La
forma más rentable de enfocar el estudio de estas enfermedades es conocer el dato típico. La
respuesta falsa es la 4, ya que los tumores cerebrales aparecen en el síndrome de Turcot, no en el
de Gardner.(R4)

140. ¿Qué prueba diagnóstica se recomienda para el "screening" anual del cáncer de
endometrio en las mujeres postmenopáusicas asintomáticas?

1. 1. Ninguna.
2. 2. Citología endometrial.
3. 3. Ecografía transvaginal.
4. 4. Determinación en plasma del marcador tumoral CA 12.5.
Gráfico de respuestas
Comentario

Pregunta fácil acerca de un tema importante en el ENARM y que se ha preguntado con bastante
frecuencia en las últimas convocatorias: cáncer de endometrio. No obstante, el detalle de esta
convocatoria no había aparecido con anterioridad.

Típica pregunta en la que el enunciado puede confundir y es muy importante fijarse bien en el
mismo. En un principio podemos pensar que nos piden la prueba diagnóstica del cáncer de
endometrio, pero incluye un detalle que cambia completamente el sentido: PRUEBA
DIAGNÓSTICA PARA EL CRIBADO ANUAL (SCREENING). Por tanto, la cuestión se reduce a si
se hace cribado del mismo o no, siendo la respuesta correcta la 1, puesto que en el cáncer de
endometrio NO se hace cribado.

Hay que recordar que en otros cánceres en los que sí se hace cribado, y que ya se han
preguntado, son el de cérvix, mediante citología (Papanicolaou) y el de mama, mediante
mastografía. Por otra parte, en el cáncer de colon se hace mediante colonoscopia en mayores de
50 años, así como en familiares de personas con antecedentes a edades más tempranas. Además,
conviene recordar que en alguna ocasión se ha preguntado por el cribado de cáncer de pulmón,
mediante radiografía de tórax, que no ha demostrado ser útil.

En cuanto al diagnóstico de cáncer de endometrio en mujeres con sintomatología sugestiva


(metrorragia posmenopáusica sobre todo), conviene señalar que actualmente la prueba de

 
 
 
 
elección es la histeroscopia con biopsia endometrial dirigida. Antiguamente se hacía legrado
fraccionado, pero en estos momentos ha pasado a ser de segunda elección.(R1)

141. En una campimetría, para que el defecto encontrado sea más congruente, la lesión
anatómica deberá encontrarse en:

1. 1. Ambos nervios ópticos.


2. 2. El quiasma óptico.
3. 3. El cuerpo geniculado externo.
4. 4. El córtex occipital.
Gráfico de respuestas
Comentario

Se dice que un defecto es más congruente cuanta mayor similitud existe entre ambos campos
visuales. La máxima incongruencia sería una lesión unilateral de un nervio óptico, ya que
solamente habría afectación en un campo visual. Sin embargo, las lesiones occipitales son muy
parecidas entre ambos campos (respuesta 5 correcta). En general, las lesiones más congruentes
son las que se producen en las regiones más posteriores de la vía óptica.(R4)

142. En cuanto a las lesiones nerviosas indique la respuesta INCORRECTA.

1. 1. El Tinel no es positivo ni de forma precoz ni de forma tardía.


2. 2. La compresión crónica de un nervio causa dolor intenso.
Los signos que se recuperan de forma más temprana al descomprimir un nervio son el
3. 3.
entumecimiento y el hormigueo.
4. 4. Tras una descompresión nerviosa el tiempo de inmovilización debe ser el menor posible.
Gráfico de respuestas
Comentario
El signo de Tinel es positivo si el paciente nota hormigueo que irradia distalmente a lo largo de la
distribución del nervio alterado al percutirlo (Respuesta 1 correcta). Este signo aparece tanto en
lesiones nerviosas como en la regeneración nerviosa. La compresión de un nervio origina cambios
que acaban produciendo una degeneración axónica, al inicio de la compresión el Tinel es negativo,
por ser la lesión muy leve y en estadios avanzados también es negativo ya que se produce una
necrosis de los axones sin regeneración ulterior (Respuesta 2 correcta). La compresión nerviosa si
es aguda causa dolor, por isquemia y necrosis, pero la compresión crónica no lo causa (Respuesta
3 INCORRECTA). La primera sintomatología que se recupera tras la descompresión es el
entumecimiento y el hormigueo (Respuesta 4 correcta). En cuanto a la exploración se recupera en
el siguiente orden: 1. toque de una punta en movimiento. 2. toque estático. 3. discriminación de dos
puntas en movimiento. 4. discriminación estática de dos puntas. Por último, tras la descompresión
nerviosa, la inmovilización se debe mantener el menor tiempo posible (generalmente 1 semana) ya
que más tiempo favorece la formación de adherencias(R2)

143. Mujer de 32 años con historia de tres partos eutócicos, refiere perdida de orina al
toser. Al examen cistouretrocele de grado I, el tratamiento recomendado es:

1. 1. Observación.
2. 2. Ejercicios de Kegel.
3. 3. Colporrafia anterior.
4. 4. Uretropexia.

 
 
 
 
Gráfico de respuestas
Comentario

Al tratarse de incontinencia de esfuerzo leve, se sugiere como tratamiento ejercicios para


rehabilitar el suelo pélvico... en este caso la respuesta 2 los ejercicios de Kegel.

Para grados más avanzados, el manejo suele ser quirúrgico, mediante la colocación de bandas
suburetrales libres de tensión.(R2)

144. Un paciente pediátrico con una otitis media aguda derecha con mala evolución y
mastoidismo desarrolla en las últimas horas imposibilidad para mover el ojo derecho a
la derecha. ¿qué complicación sospecha?

1. 1. Síndrome de Gradenigo
2. 2. Signo de Griesinger
3. 3. Trombosis de la vena emisaria mastoidea.
4. 4. Absceso de Bezold
Gráfico de respuestas
Comentario

Lo que se nos presenta es una complicación de las otits medias denominada Síndrome de
Gradenigo.

El sindrome de Gradenigo es un cuadro desencadenado por la formación de absceso a nivel de la


punta del peñasco.

Consiste en parálisis ipsilateral del sexto par craneal con dolor severo en el área inervada por la
rama oftálmica asociado a un proceso otítico.

El resto de opciones son también complicaciones de las otitis. El signo de Griesinger se presenta
con la trombosis de la vena emisaria mastoidea, y consiste en dolor e hinchazón a nivel
retromastoideo por trombosis de la vena emisaria mastoidea.

El absceso de bezold es un absceso formado en la punta de la mastoides.(R1)

145. Para medir el filtrado glomerular, el aclaramiento de creatinina endógena:

1. 1. Es una prueba útil y sencilla de realizar.


2. 2. Es más exacto que el aclaramiento de inulina.
3. 3. En presencia de insuficiencia renal avanzada infravalora el FG renal.
4. 4. No es útil en personas ancianas.
Gráfico de respuestas
Comentario

La definición de aclaramiento de una sustancia es: "el volumen de plasma que queda totalmente
libre de dicha sustancia a su paso por el riñón en la unidad de tiempo". Esto es el resultado de la
filtración glomerular menos la reabsorción o más la secreción, según la sustancia que se trate. La
inulina es solamente filtrada, ni se reabsorbe ni se secreta, así su medida es el equivalente exacto
del filtrado glomerular, pero la inulina no está presente en el organismo. El aclaramiento de
creatinina es lo que se utiliza en la práctica, pues no es necesario inyectar una sustancia exógena

 
 
 
 
(con los posibles peligros que esto puede acarrear), la creatinina se filtra, y en casos de muy bajo
filtrado tiende a reabsorberse, pero esto es tan escaso que lo solemos despreciar en la práctica,
por eso quizás su aclaramiento es menos exacto que el de inulina para medir el filtrado glomerular,
pero sí que es más útil que el de urea.(R1)

146. Lactante de 6 meses que acude después de 2-3 días con estornudos y rinorrea clara
por dificultad respiratoria y rechazo de tomas. Presenta sibilancias espiratorias
diseminadas. Afebril, retracción subcostal moderada y hepatomegalia de 2 cm. Usted
sospecha una bronquiolitis vírica, conteste la FALSA:

1. 1. Está comprobada la eficacia del tratamiento con corticoides.


2. 2. Están contraindicados los sedantes.
3. 3. Plantea diagnóstico diferencial con cuerpo extraño en tráquea y con insuficiencia cardíaca.
4. 4. La biometría hemática posiblemente será normal.
Gráfico de respuestas
Comentario

Esta pregunta de bronquiolitis es muy importante. No debemos tener problemas en reconocer un


caso típico de bronquiolitis.

La mayoría de los casos de bronquiolitis están causados por el virus sincitial respiratorio, pero el
cuadro es indistinguible de los producidos por otros virus: parainfluenza, adenovirus, rinovirus,
influenza o el recientemente implicado metaneumovirus.

En un niño menor de 2 años con dificultad respiratoria y tos seca acompañado de sibilancias hay
que plantearse el diagnostico diferencial con asma, tos ferina, aspiración por reflujo, neumonía por
Chlamydia, anomalías pulmonares congénitas, ICC, cuerpo extraño, FQ.

El tratamiento se centra en medidas de nutrición e hidratación adecuadas, oxigeno y


broncodilatadores en aerosol (salbutamol y adrenalina).

No existen evidencias que apoyen uso de corticoides. Se deben evitar los sedantes.

El uso de ribavirina sólo debe ser considerada en casos graves en pacientes de alto riesgo.

El recuento celular y fórmula son inespecíficos.(R1)

147. A 14-year-old patient sees her gynecologist for the first time. Physical examination
shows absent pubic hair, hypoplastic breasts and olfactory deficit. She has no menses.
Ultrasonography shows the presence of uterus and ovaries. FSH levels are 1 (normal
range 3-28). What is the most likely karyotype in this case?

1. 1. 47 XXY
2. 2. 45 XO
3. 3. 48 XXXY
4. 4. 46 XX
Gráfico de respuestas
Comentario
46 XX. Kallman's syndrome is a disorder that causes hypothalamic sexual infantilism due to absent
GnRH production. It is typically associated to anosmia secondary to disorders of the olfactory bulb.

 
 
 
 
Karyotype is normal in these patients, immature internal genitalia is due to lack of central hormonal
stimulation.(R4)

148. Femenino de 40 años de edad, con una tumoración mamaria derecha, poco móvil,
localizada en el cuadrante superoexterno. ¿Cuál de las siguientes pruebas confirma el
diagnóstico?

1. 1. Marcadores tumorales.
2. 2. Ultrasonido mamario 2D.
3. 3. Biopsia del tumor.
4. 4. Ultrasonido Doppler 3D.
Gráfico de respuestas
Comentario

Detecte lo que le están preguntando, ¡¡¡CONFIRME EL TUMOR!!! Debe tener mucho cuidado al
resolver el ENARM, ya que una lectura rápida hará que responda erroneamente.

Obviamente el diagnóstico de neoplasia es histológico, por lo que la única opción que permitiría
tener una muestra de tejido es la biopsia.(R3)

149. ¿Cuál de las siguientes no es una manifestación clínica característica de la


endometriosis?

1. 1. Hipermenorrea.
2. 2. Dismenorrea.
3. 3. Esterilidad primaria.
4. 4. Dispareunia.
Gráfico de respuestas
Comentario

Pregunta controvertida sobre la endometriosis, no es preocupante si se ha fallado.

Dismenorrea, esterilidad y dispareunia forman la tríada típica de la endometriosis. Como


consecuencia de la irritación crónica que se puede producir, es lógico pensar que la endometriosis
puede ser causa de dolor pélvico crónico, por lo que debe incluirse esta entidad en el diagnóstico
diferencial ante todo dolor pélvico recidivante de causa no filiada.

A pesar de que en el Manual y en otros tratados de ginecología mencionan como característica


clínica de la endometriosis la hipermenorrea, ésta no es tan frecuente como lo son las otras
opciones. De todos modos, aunque todas las opciones aparecen asociadas a la endometriosis, si
tuviéramos que elegir “la más errónea”, tendría que ser la respuesta 1.(R1)

150. En un paciente en la cincuentena, debe establecerse una indicación quirúrgica


electiva para el tratamiento de una úlcera gástrica, en las siguientes circunstancias,
EXCEPTO cuando hay:

1. 1. Falta de respuesta positiva al tratamiento médico en una úlcera gástrica recidivada.


Nueva recidiva de la úlcera gástrica después de dos respuestas positivas a sendos
2. 2.
tratamientos médicos.

 
 
 
 
3. 3. Imposibilidad de descartar que se trate de un adenocarcinoma gástrico.
4. 4. Una úlcera gástrica de tipo III (prepilórica).
Gráfico de respuestas
Comentario

Las cuatro primeras respuestas están dando razones de peso para operar una úlcera: mala
respuesta al tratamiento médico (o recidiva) o que pueda haber un tumor maligno. Sin embargo, la
posición de la úlcera no debería ser, al menos en principio, un criterio para intervenirla
quirúrgicamente (respuesta 5 falsa). El tratamiento médico sería el mismo y, antes de decidir la
intervención, habría que darle una oportunidad.(R4)

151. Señale qué patología NO asocia fontanela grande en el recién nacido:

1. 1. Acondroplasia.
2. 2. Síndrome de Down.
3. 3. Hidrocefalia.
4. 4. Craneosinostosis.
Gráfico de respuestas
Comentario

La craneosinostosis consiste en el cierre precoz de alguna fontanela. Por ende, en esta patología
las fontanelas estarán disminuidas de tamaño, y no aumentadas.(R4)

152. Un neonato prematuro recibe teofilina oral por presentar pausas de apnea. ¿Cuál de
los siguientes antibióticos puede provocar disminución de su metabolismo e
intoxicación?:

1. 1. Ampicilina.
2. 2. Anfotericina.
3. 3. Eritromicina.
4. 4. Vancomicina.
Gráfico de respuestas
Comentario
La verdad es que, hoy por hoy, la cafeína ha desplazado de una forma prácticamente total a la
teofilina en el tratamiento de las pausas de apnea. La razón es que la teofilina tiene un difícil
manejo, hay que tener cautela con las posibles interacciones que genera (por ejemplo, con la
eritromicina), y hay que hacer periódicamente evaluaciones de su nivel en sangre.(R3)

153. Una alteración ventilatoria obstructiva en un paciente joven no fumador obliga a


descartar todas las siguientes, EXCEPTO:

1. 1. Asma bronquial.
2. 2. Bronquiectasias.
3. 3. Neumoconiosis.
4. 4. Bronquiolitis obliterante.
Gráfico de respuestas
Comentario
Esta pregunta es de dificultad media y debes saber bien interpretar lo que nos están preguntando
para facilitar la respuesta. Recuerda que una de las cosas más importantes en la fisiopatología del
bloque de neumología es clasificar las enfermedades como principalmente obstructivas o

 
 
 
 
principalmente restrictivas, ya que así podrás responder sin problemas muchas cuestiones y casos
clínicos. Las enfermedades principalmente obstructivas se caracterizan por un Tiffeneau menor del
70% y un aumento del VR; principalmente son: asma, EPOC, bronquiolitis, bronquiectasias,
linfangioleiomiomatosis, histiocitosis X,... . Por el contrario las enfermedades principalmente
restrictivas tendrán un Tiffeneau mayor de 80% y un VR normal o disminuido; entre ellas
encontramos: fibrosis pulmonar, enfermedades intersticiales, sarcoidosis, neumoconiosis,... .
Recuerda que hay una serie de enfermedades que dan patrones mixtos (Tiffenau > 80% y aumento
del VR) como son el Guillain- Barré, distrofias musculares, miastenia gravis y espondilitis
anquilosante.(R3)

154. A propósito del tratamiento del tromboembolismo pulmonar, uno de los siguientes
enunciados NO es correcto. Indique cuál:

1. 1. La heparina produce una inhibición inmediata del crecimiento del trombo.


El control de la dosis de heparina se realiza mediante la medición del tiempo de
2. 2.
protrombina, que debe duplicarse.
Si el tratamiento se va a hacer a largo plazo, se puede utilizar la warfarina o heparina
3. 3.
subcutánea.
4. 4. El tratamiento prolongado suele mantenerse un mínimo de 3 meses.
Gráfico de respuestas
Comentario
El tratamiento de TEP se ha preguntado con mucha frecuencia y no debería ofrecerte ninguna
duda. La heparina produce una inhibición del crecimiento del trombo por acelerar la función de la
antitrombina III, lo que previene la formación de trombo adicional. Lo que hay que monitorizar es el
tiempo de tromboplastina parcial activado, que debe duplicarse (opción 2 falsa). Si el tratamiento
va a emplearse a largo plazo, se utilizarían anticoagulantes orales (warfarina, dicumarínicos) o
heparina subcutánea en caso de que éstos estuviesen contraindicados, como sucede en el
embarazo. El tratamiento se prolongaría por lo menos 3 meses, aunque ante ciertas situaciones,
como TEP de repetición, puede estar indicado un tratamiento más prolongado e incluso de por
vida. Por otra parte, el uso de agentes trombolíticos no han demostrado disminuir las recurrencias.
El uso fundamental de los trombolíticos es el TEP masivo. Su objetivo es la rápida lisis del trombo,
acortando la fase de alto riesgo y disminuyendo la mortalidad.(R2)

155. Mujer de 28 años, primípara; parto eutócico hace 10 días. Puerperio afebril. Lactancia
materna. Acude a urgencias refiriendo cuadro de dolor en mama derecha de 48 horas de
evolución, Tª de 38º C, enrojecimiento y calor local. Dice tener algunas dificultades para
vaciarla al amamantar. Exploración: mamas asimétricas (derecha de mayor tamaño); no
alteraciones a nivel de areola-pezón; secreción láctea espontánea. Mama derecha
ingurgitada, eritematosa a nivel de CSE y caliente; no alteraciones tróficas en piel. A la
palpación se detecta una zona indurada, dolorosa, pero sin zonas fluctuantes evidentes
en CSE. Señale la respuesta INCORRECTA:

Los gérmenes más frecuentemente implicados en este proceso son estafilococos,


1. 1.
estreptococos y colibacilos.
2. 2. Es necesario retirar la lactancia.
3. 3. El tratamiento incluye la administración de antibióticos y analgésicos antiinflamatorios.
4. 4. Es posible la formación de un absceso que requiere drenaje quirúrgico.
Gráfico de respuestas
Comentario

 
 
 
 
El caso clínico que nos presentan es el de una mastitis. Se trata de una infección muy frecuente en
mujeres con lactancia materna. La producen gérmenes que pasan de la piel a la glándula mamaria
a través de grietas y heridas que se forman secundariamente a la lactancia. Ante una mastitis,
debe pensar en microorganismos grampositivos, comoS. aureus.

El tratamiento de la mastitis se realiza con antibióticos que cubra S. aureus y analgésicos,


evacuación periódica del pecho y aplicación de calor local. En el caso de que se forme un absceso,
será necesario completar el tratamiento anterior con el drenaje quirúrgico. No es necesario inhibir
la lactancia, salvo que sea una mastitis bilateral.(R2)

156. A worried first-time mother comes to the pediatrician's office very worried. She is
breastfeeding her baby, and he is developing adequately, but she has heard that cow milk
is more beneficial. She wants to know the differences between both types of milk:

The proteins in human milk are absorbed more easily than cow's milk proteins and they
1. 1.
contribute to a better gastric transit
2. 2. Human's milk is richer in saturated fats
3. 3. Cow's milk is richer in essential amino acids
4. 4. Human's milk has a lower concentration of immunoglobulin A
Gráfico de respuestas
Comentario
The proteins in human milk are absorbed more easily than cow's milk proteins and they contribute
to a better gastric transit. Breastfeeding is the most beneficial diet for the child and provides all the
essential nutrients during the early months. Cow milk preparations try to copy these properties, but
the perfect formula does not exist. Proteins present in human’s milk are absorbed betetr by the
newborn’s gut. Iron concentration is low in human milk. Therefore, after six months, iron
supplements are required.(R1)

157. Una mujer de 27 años recibe sales de litio por un trastorno bipolar. Ante la posibilidad
de embarazo, debe suspenderse por riesgo de malformaciones congénitas,
especialmente:

1. 1. Tiroideas.
2. 2. Cardíacas.
3. 3. Labio leporino.
No debe suspenderse bajo ningún concepto si ha presentado al menos 2 episodios
4. 4.
maníacos.
Gráfico de respuestas
Comentario

El litio es un fármaco importante para el examen MIR. En alguna ocasión, incluso han preguntado
cuál es el rango óptimo de litemia en los pacientes que lo utilizan, por lo que debes dominar todos
los aspectos relacionados con él.

El litio está contraindicado durante el embarazo, puesto que se han descrito malformaciones
cardiovasculares (anomalía de Ebstein) en aproximadamente el 3% de los fetos expuestos. Dado
que, además, se excreta por leche materna, las madres en tratamiento con litio no deben dar el
pecho a sus hijos.(R2)

158. En niños, las causas más frecuente de Insuficiencia Renal Aguda son de origen:

 
 
 
 
1. 1. Pre-renal.
2. 2. Post-renal.
3. 3. Mixtas.
4. 4. Intrínsecas.
Gráfico de respuestas
Comentario

El síndrome urémico hemolítico es más frecuente en niños y es la causa más común de


insuficiencia renal aguda en este grupo, por lo que la respuesta correcta es la 1.(R1)

159. En una enferma diagnosticada de LES hace 1 año, con biopsia renal compatible con
glomerulonefritis lúpica membranosa, que en el momento actual no presenta
manifestaciones extrarrenales y tiene la siguiente analítica: creatinina 1 mg/dl, urea 45
mg/dl, proteínas totales 7 g/dl, proteinuria 10 mg/kg/día, hematuria 9.500 hem/minuto, sin
cilindros hemáticos, ANA positivo, anti-ADNds negativo, ¿qué tratamiento emplearía?:

1. 1. Prednisona 1 mg/Kg/día vo.


2. 2. Azatioprina 1,5 mg/Kg/día v.o.
3. 3. Clorambucil 1 mg/Kg/día/v.o.
4. 4. No precisa tratamiento.
Gráfico de respuestas
Comentario

Esta pregunta es de dificultad media- baja. Recuerda que el lupus cursa con afectación renal en un
50% de los casos y su presencia ensombrece el pronóstico siendo una de las causas de muerte.
En nuestro caso, se trata de una paciente sin manifestaciones extrarrenales, sin insuficiencia renal
pese a tener en la biopsia datos compatibles con una GN membranosa con proteinuria y
hematuria, por lo que se trata de un caso LEVE cuyo pronóstico suele ser favorable con una
supervivencia del 85% a los 10 años. Vemos que todas las opciones dadas (salvo la 4) son
tratamientos que serían usados en casos más graves por lo que es fácil deducir cuál es la
correcta.(R4)

160. Un recién nacido, hijo de madre diabética, no ha expulsado meconio tras 72 horas
de vida. ¿Cuál es el cuadro que con más frecuencia se asocia a esta situación?

1. 1. Fibrosis quística.
2. 2. Síndrome de colon izquierdo hipoplásico.
3. 3. Atresia anal.
4. 4. Atresia duodenal.
Gráfico de respuestas
Comentario

El hijo de madre diabética es con frecuencia macrosómico, por la acción de la insulina sobre los
tejidos fetales, ya que actúa como factor de crecimiento. Aparte de la macrosomía, pueden
asociarse otras anomalías más o menos frecuentes, como son:

-Colon izquierdo hipoplásico.

- Estenosis subaórtica hipertrófica.

- Hipoglucemia.

 
 
 
 
- Policitemia.

- Mayor frecuencia de ictericia.

- Trombosis de la vena renal.

- Hipocalcemia.

- Hipomagnesemia.

- Hidramnios.

La clave de la pregunta, por tanto, es el antecedente de diabetes materna. Las otras enfermedades
también podrían justificar una lenta eliminación del meconio, pero no se relacionarían con la
diabetes.(R2)

161. La primera prueba a realizar ante la sospecha de una masa renal:

1. 1. Ultrasonido.
2. 2. TAC helicoidal con reconstrucción 3D
3. 3. Uro TAC.
4. 4. Angiografía.
Gráfico de respuestas
Comentario

Ante la sospecha de una masa renal la 1a prueba a realizar es una ECOGRAFIA (Ultrasonido) ya
que nos permitirá diferenciar si se trata de un quiste simple que sólo requiere observación o bien
nos encontramos ante una masa sólida o quiste complejo que requiere más pruebas.(R1)

162. Uno de los siguientes síntomas o signos NO corresponde al síndrome de Heerfordt:

1. 1. Eritema nodoso.
2. 2. Agrandamiento parotídeo.
3. 3. Uveítis anterior.
4. 4. Parálisis facial.
Gráfico de respuestas
Comentario

El síndrome de Heerfordt (fiebre uveoparotídea) es una forma de sarcoidosis de inicio agudo o


subagudo

que se presenta con fiebre, aumento del tamaño de las parótidas, uveítis anterior y parálisis del
nervio facial. El eritema nodoso es una afectación cutánea típica de la sarcoidosis, aunque poco
específica, que consiste en nódulos subcutáneos rojos y sensibles a la palpación, localizados en la
cara anterior de las piernas, y que es mucho más frecuente en el síndrome de Löfgren, una forma
aguda de sarcoidosis.(R1)

163. Un compañero de la Facultad de Derecho, sano, le comenta que va a someterse a la


extracción de la muela del juicio y que se dentista le ha comentado que, para evitar una

 
 
 
 
infección denominada endocarditis, debería tomar antibiótico. Al pedirle consejo, usted
le diría:

Efectivamente, las guías recomiendan la profilaxis a los pacientes sometidos a


1. 1.
manipulación de la mucosa oral.
2. 2. Debe iniciar tratamiento con amoxicilina en monodosis previa a la extracción.
3. 3. Como es alérgico a las penicilinas, una buena alternativa es la azitromicina.
La ausencia de cardiopatía o factores de riesgo predisponentes no hace necesario que
4. 4.
reciba ninguna profilaxis antibiótica.
Gráfico de respuestas
Comentario
Las guías más recientes sobre la profilaxis de la endocarditis bacteriana sólo recomiendan el
tratamiento antibiótico en aquellos casos en que existan circunstancias de riesgo predisponentes
tales como cardiopatía congénita, endocarditis previa, prótesis, valvulopatía… En estas situaciones
y ante maniobras invasoras sobre la mucosa orofaríngea debe realizarse profilaxis antes del
procedimiento con amoxicilina o clindamicina o un macrólido en el caso de alergia a las
penicilinas.(R4)

164. Paciente con prurito vulvar, que al examen ginecológico se encuentra secreción
blanquecina maloliente. Cuello uterino con punteado rojizo como fresas. La infección
vaginal seria causada por:

1. 1. Candida albicans.
2. 2. Trichomona vaginalis.
3. 3. Flora mixta.
4. 4. Chlamydia trachomatis.
Gráfico de respuestas
Comentario

Hay tres grandes causas de infecciones vaginales:

-Vaginosis bacteriana: Producida por un desequilibrio en la flora bacteriana. Produce leucorrea


maloliente, pero no suele ocasionar prurito ni inflamación. Por tanto, no corresponde a este caso.

-Candidiasis: Produce prurito y secreción blanquecina, pero el punteado como fresas que nos
describen no es típico. La candidiasis produce una inflamación difusa, con manchas blancas como
de "requesón", parecido a lo que ocurre en la candidiasis oral.

-Tricomoniasis: Típicamente produce una leucorrea maloliente, que puede ser blanquecina, o
también se describe clásicamente como verdosa o amarillenta, y espumosa. La presencia de
petequias en la pared vaginal, dándole un aspecto afresado, es muy típica de esta infección. Por
tanto, la leucorrea maloliente y la presencia de punteado como fresas, apuntan en la dirección de
una infección por Trichomonas: la respuesta 2 es correcta.(R2)

165. Un paciente de 37 años, sin antecedentes patológicos de interés, consulta por fiebre
y molestias abdominales de 3 semanas de duración. En el análisis de sangre, destaca la
presencia de leucocitosis con neutrofilia y ligera elevación de las enzimas hepáticas. Se
realiza un USG que muestra la existencia de una colección en el parénquima hepático y
la presencia de aire en el sistema venoso portal. La causa más probable que desencadenó
este proceso es:

 
 
 
 
1. 1. Apendicitis aguda.
2. 2. Fiebre reumática.
3. 3. Absceso hepático amebiano.
4. 4. Isquemia mesentérica.
Gráfico de respuestas
Comentario

Pregunta difícil. Se trata de un paciente con afectación sistémica infecciosa en el que se detecta un
absceso hepático y una imagen por USG de aire en el sistema venoso portal. Todo ello es
sugestivo de una peliflebitis en estadio avanzado (tromboflebitis séptica de la vena porta) que ya
ha emitido trombos sépticos al parénquima hepático. Entre las causas más frecuentes de esta rara
patología destaca la apendicitis aguda y la diverticulitis aguda, pero teniendo en cuenta lo expuesto
en el enunciado hay que decantarse por la opción 1.(R1)

166. Un paciente con pólipos hamartomatosos en intestino delgado y que presenta


manchas melánicas alrededor de la boca, los ojos y en región perianal, es diagnosticado
de síndrome de Peutz-Jeghers. En este caso, ¿cuál de las siguientes afirmaciones es
FALSA?:

1. 1. La poliposis de Peutz-Jeghers se hereda con carácter autosómico dominante.


Los pólipos hamartomatosos pueden aparecer también en el estómago y en el intestino
2. 2.
grueso.
En estos pacientes está indicada la colectomía profiláctica debido al elevado riesgo de
3. 3.
padecer carcinoma de intestino grueso.
Los niños con síndrome de Peutz-Jeghers tienen mayor riesgo que la población general de
4. 4.
padecer tumores del sistema reproductor y de la vejiga.
Gráfico de respuestas
Comentario

Tema poco preguntado en el ENARM, pero si debes repasarlo. El síndrome de Peutz- Jeghers se
trasmite con herencia autosómica recesiva. El signo cutáneo típico es la pigmentación
mucocutánea. Durante la infancia, aparece una pigmentación melanótica alrededor de la nariz,
labios, mucosa bucal, manos y pies. Ocasionalmente, se acompaña de hiperpigmentación genital y
perianal. En la pubertad, pueden desaparecer las pigmentaciones, excepto las de la mucosa oral.

Los pólipos pueden aparecer en estómago, intestino delgado y colon, siendo sobre todo frecuentes
en intestino delgado, y rara vez malignizan, pues son de tipo hamartomatoso. Por ello, no está
justificada la colectomía profiláctica. Se han descrito cánceres en duodeno, yeyuno, íleon y colon.
En un estudio, casi el 50% de los pacientes tuvieron cánceres intestinales o extraintestinales, con
una media de edad de 50 años al diagnóstico. También pueden aparecer pólipos benignos en
localizaciones extraintestinales, incluyendo nariz, bronquios, vejiga, vesícula biliar y conductos
biliares. En el 5- 12% de mujeres, aparecen quistes o tumores ováricos. En hombres jóvenes,
pueden aparecer tumores testiculares de células de Sertoli con signos de feminización. Otros
tumores que pueden aparecer son: mama (a menudo bilateral), páncreas, colangiocarcinoma y
cáncer de vesícula biliar.(R3)

167. Señale qué afirmación, de las siguientes, relativa al meconio fetal normal, es
CORRECTA:

1. 1. Contiene abundantes E. coli.


2. 2. Contiene escasos estafilococos.
3. 3. Puede contener Proteus en casos de prematuridad.

 
 
 
 
4. 4. No contiene ningún germen.
Gráfico de respuestas
Comentario

El meconio es estéril, por lo que la respuesta correcta es la 4. Esta pregunta no puede fallarla.(R4)

168. Un hombre de 40 años sin enfermedades concomitantes, acude a Urgencias por


melenas de 24 horas de evolución sin repercusión hemodinámica. Refiere consumo de
antiinflamatorios no esteroideos los días previos por cefalea. La biometría hemática es
normal salvo el hematocrito de 33%. La endoscopía digestiva alta realizada de forma
urgente a las 6 horas del ingreso muestra una úlcera antral excavada de 8 mm de diámetro
con base de fibrina limpia y bordes lisos y regulares. Las biopsias de la úlcera resultan
benignas y muestran infección por Helicobacter pylori. ¿Cuál de las siguientes
afirmaciones es CIERTA?

Dada la ausencia de malignidad en las biopsias iniciales y el aspecto endoscópico de la


1. 1. úlcera, se puede descartar de manera fiable el cáncer gástrico y es innecesario el
seguimiento endoscópico.
Sólo está indicado el tratamiento erradicador del Helicobacter pylori si se constata recidiva
2. 2.
de la úlcera en ausencia de antiinflamatorios no esteroideos.
Dado que se trata de una úlcera complicada (hemorragia) la mejor opción terapéutica es
3. 3.
una vagotomía y piloroplastia.
4. 4. El paciente podría ser dado de alta hospitalaria de forma precoz.
Gráfico de respuestas
Comentario

Dependiendo del aspecto endoscópico de la úlcera, el riesgo de resangrado es variable. La


clasificación de Forrest permite predecirlo con bastante precisión.

Integrando los datos de la pregunta (estabilidad hemodinámica e imagen de base de fibrina), el


riesgo de resangrado es mínimo y, por tanto, no cabría esperar un ingreso prolongado, y todavía
menos una intervención quirúrgica inmediata (respuesta 4 correcta). No obstante, sí se precisaría
un seguimiento endoscópico posterior, a diferencia de lo que dice la respuesta 2.(R4)

169. Paciente de 15 años que consulta por haber presentado en el último año 3-4
episodios de pérdida brusca de conocimiento con caída al suelo, movimientos tónico-
clónicos en las cuatro extremidades de 4-5 minutos de duración, mordedura de lengua e
incontinencia urinaria. A continuación presentaba somnolencia y cefalea durante 30
minutos. Además, refería frecuentes sacudidas musculares bruscas y momentáneas,
aisladas, en ambos brazos y, a veces, en las piernas; esto le había provocado algunas
caídas y que se le cayeran objetos de las manos. Tanto unos como otros ocurrían en las
primeras horas de la mañana, generalmente en el desayuno. El paciente no tenía
antecedentes personales de interés, pero sí historia familiar positiva de crisis
convulsivas. La exploración general y neurológica fue normal. La TC no mostraba
alteraciones. El EEG intercrítico mostraba descargas de punta-onda o polipunta-onda
lenta generalizadas. ¿Qué síndrome epiléptico padece el paciente?

1. 1. Epilepsia mioclónica benigna de la infancia.


2. 2. Epilepsia mioclónica juvenil (Síndrome de Janz).
3. 3. Síndrome de Lennox-Gastaut.
4. 4. Epilepsia mioclono-astática.

 
 
 
 
Gráfico de respuestas
Comentario

La epilepsia mioclónica juvenil es el prototipo de epilepsia generalizada idiopática. Supone el 10%


de todas las epilepsias y es la epilepsia mioclónica más frecuente. La edad de inicio es entre los 8
y 25 años de edad. La mayoría de los pacientes presentan distintos tipos de crisis, además de las
mioclónicas: 90 % asocian crisis tónico-clónicas y el 30% ausencias típicas.

Las crisis se presentan como sacudidas musculares breves, habitualmente en miembros


superiores, característicamente al despertar, y favorecidas por la privación previa de sueño y el
consumo de alcohol. Se mantiene el nivel de conciencia, excepto en las crisis severas.

El EEG muestra actividad paroxística punta-onda en la mayoría de los casos, y siempre es


patológico durante el sueño. Un tercio de los pacientes presenta actividad paroxística fotosensible,
aunque no es frecuente que tengan un correlato clínico.

El tratamiento de elección de este tipo de crisis es el ácido valproico. Del resto de las opciones, no
olvide que la carbamacepina se emplea sobre todo para crisis parciales (también en el dolor
neuropático, como la neuralgia del trigémino), y la etosuximida se reserva para las crisis de
ausencia típicas.(R2)

170. ¿Qué tipo de patología sospecharía en un recién nacido que para respirar necesita
de la colocación de un chupón en el biberón, con dos grandes orificios fijada a la boca,
con lo que el niño respira por movimientos de chupeteo?:

1. 1. Paladar hendido.
2. 2. Microrretrognatia.
3. 3. Atresia de coanas.
4. 4. Membrana subglótica congénita.
Gráfico de respuestas
Comentario

Los neonatos respiran por el sitio ideal (las fosas nasales), siempre y cuando no haya factores
mecánico-obstructivos que lo impidan. Sospecha la presencia de éstos cuando te describan un
neonato con dificultad para respirar en reposo y al mamar que mejora con el llanto. Tenga en
cuenta que, cuando el neonato llora, coge aire por la nariz y por la boca.(R3)

171. ¿Cuál de los siguientes NO produce una disminución de la afinidad de la


hemoglobina por el oxígeno?:

1. 1. Disminución del pH.


2. 2. Exposición al monóxido de carbono.
3. 3. Disminución de la temperatura.
4. 4. Aumento del dióxido de carbono.
Gráfico de respuestas
Comentario
La disminución de la temperatura no disminuye la afinidad de la hemoglobina, sino un aumento. Es
el aumento de la temperatura, junto a la disminución del pH, el aumento de dióxido de carbono y el
aumento del 2- 3 difosfoglicerato lo que va a aumentar la cesión de O2 o lo que es lo mismo va a
disminuir la afinidad de la hemoglobina en condiciones normales. Además en casos de intoxicación

 
 
 
 
por el CO, este desplaza al O2 de su lugar de union a la Hb, y por tanto también disminuiria la
afinidad de la hemoglobina por el oxigeno.(R3)

172. Varón de 36 años presenta unas pápulas amarillentas en las flexuras antecubital y
cervical, que confluyen para formar placas reticuladas de aspecto similar a la piel de pollo
desplumada. La biopsia descubre fibras elásticas hinchadas y agrupadas irregularmente
con depósitos de calcio. Refiere antecedentes familiares similares en varios miembros
de su familia. Además, el paciente es hipertenso, anginoso y ha tenido dos episodios
previos de hematoquecia. ¿Cuál de las siguientes patologías es más probable que
presente?:

1. 1. Síndrome de Rendu-Osler.
2. 2. Hiperlipidemia familiar combinada.
3. 3. Fibroelastosis endomiocárdica extracardíaca.
4. 4. Pseudoxantoma elástico.
Gráfico de respuestas
Comentario
Pregunta de importancia media ya que esta patología la han preguntado varias veces en el MIR.Se
trata de un cuadro de pseudoxantoma elástico, alteración genética del tejido conectivo,
caracterizada por la calcificación y degeneración progresiva de las fibras elásticas, que lleva a la
aparición de lesiones cutáneas, manifestaciones oculares y cardiovasculares. La forma más
frecuente es la autosómica recesiva tipo 1 que cursa con lesiones a modo de pápulas amarillentas
confluentes con aspecto típico en ?piel de pollo desplumado? localizadas en las caras laterales de
cuello y flexuras. Junto a esto, existe vasculopatía que se manifiesta por claudicación intermitente,
hemorragias digestivas, hipertensión y oclusiones coronarias. Es importante recordar que la
afectación ocular más característica es la aparición de estrías angioides en la retina (50%), aunque
son más precoces las alteraciones del pigmento retiniano.(R4)

173. Concerning the embryology of genital ducts, which is the INCORRECT statement:

1. 1. Muller ducts develop to form the fallopian tubes and uterus.


2. 2. The upper one-third of the vagina derives from the Muller ducts.
3. 3. Wolf and Muller ducts appear after the genital or gonadal ridges.
4. 4. Mullerian inhibiting factor is produced by Leydig cells.
Gráfico de respuestas
Comentario

Tema de ciclos básicos pero poco rentable para el ENARM. Son las células de Sertoli las que
producen el FIM. Todo lo demás es cierto.(R4)

174. Todos los microorganismos siguientes son agentes etiológicos de la otitis externa,
¿cuál de ellos es el más frecuente?:

1. 1. Staphylococcus epidermidis.
2. 2. Proteus.
3. 3. Pseudomonas.
4. 4. Candida albicans.
Gráfico de respuestas
Comentario

 
 
 
 
La Pseudomonas, responsable de la llamada otitis del nadador, es el germen más frecuentemente
implicado en las otitis externas. A recordar las características principales de estas otitis, muy
frecuentes en niños, pues son felices usuarios de las piscinas: dolor intenso a la movilización del
pabellón auricular, signo del trago positivo y otoscopia patológica (CAE eritematoso y
tumefacto).(R3)

175. De entre los siguientes agentes, ¿Cuál es el que causa con más frecuencia
bacteriemia?

1. 1. Legionella.
2. 2. Staphylococcus aureus.
3. 3. Salmonella typhimurium.
4. 4. Streptococcus pyogenes.
Gráfico de respuestas
Comentario

Una pregunta relativamente sencilla. Aunque cualquiera de estas bacterias puede ser causa de
bacteriemia, está claro que S. aureus es, con mucho, la más probable. Recuerda su implicación en
patologías como la endocarditis aguda (de la cual es la causa más frecuente). Otras bacterias
aisladas con mucha frecuencia son S. epidermidis y E. coli, con las que podrías haber dudado si
apareciesen entre las opciones.(R2)

176. Una paciente joven consulta por la reciente aparición en


las piernas de las lesiones que pueden observarse en la Imagen 8. Por las características
de las mismas, una de las siguientes opciones diagnósticas es muy IMPROBABLE:

1. 1. Paniculitis de Weber-Christian.
2. 2. Vasculitis leucocitoclástica.
3. 3. Eritema indurado o enfermedad de Bazin.
4. 4. Linfangitis nodular.
Gráfico de respuestas
Comentario

 
 
 
 
La imagen que nos presentan resulta muy típica y, a primera vista, el diagnóstico más probable es
el de eritema nodoso. Como puedes ver, se trata de lesiones nodulares, eritematosas, en la cara
anterior de las piernas de una paciente joven.

Lo primero que tienes que hacer es comprender muy bien lo que se nos pregunta. Nos piden el
diagnóstico más improbable. Desde luego, el más probable ya sabemos cuál es… Ahora tenemos
que marcar el que menos se le parece. Tienes que considerar que, en esta imagen, sea o no un
eritema nodoso, lo que tenemos claro es que se trata de nódulos. El nódulo, como lesión básica,
debes relacionarlo siempre con las paniculitis.

Entre las opciones que nos presentan, cuatro de ellas tienen al nódulo como lesión elemental
(respuestas 1, 3, 4 y 5). Sin embargo, la respuesta 2 produce lesiones purpúricas (a veces
palpables), necrosis con úlceras cutáneas… Pero resultaría extraordinario que esa entidad
produjese nódulos. Por lo tanto, la respuesta correcta es la 2.

Es interesante señalar que, en esta pregunta, muchos opositores se confundieron y marcaron la


opción 3, basados en el razonamiento de que el eritema indurado de Bazin afecta a la cara
posterior de las piernas. Les influyó que, en el Examen MIR, el eritema nodoso y la vasculitis
nodular presentan algunas manifestaciones contrarias, como se explica en el Manual CTO.

Todo esto es verdad, qué duda cabe… Pero la pregunta no es ésta. Lo que nos piden es: ¿qué
diagnóstico es más IMPROBABLE?, y esto cambia mucho las cosas. Eritema indurado de Bazin es
una paniculitis, lo mismo que el eritema nodoso, afectando también a las piernas, y en algunos
casos se precisa biopsia para poder distinguirlos. En definitiva, aunque tengan algunas
características opuestas, también tienen una en común: las dos son paniculitis y ambas producen
nódulos (en otras palabras, se parecen más). En cambio, la respuesta 2 no tiene nada que ver y
podría distinguirse a simple vista de cualquiera de las otras cuatro entidades.(R2)
Clasificación de la paniculitis

 
 
 
 

Imagen oficial 8 del MIR. Nódulos inflamatorios en miembros inferiores, clínicamente sugestivos de eritema nodoso

Púrpura palpable en una vasculitis por hipersensibilidad

177. Which of the following findings is not characteristic of gestational trophoblastic


disease?

1. 1. First-trimester metrorrhagia.
2. 2. Increased pregnancy symptoms.
3. 3. Poor uterine growth, not consistent with the time of amenorrhea.
4. 4. Bunch-of-grapes or snow-storm appearance in the ultrasound study.
Gráfico de respuestas
Comentario
 
 
 
 
Tema que se ha preguntado en diversas ocasionaes en el ENARM. Los síntomas y signos de la
enfermedad trofoblástica son todos los que tienes enunciados en las respuestas menos la opción
3. Recuerda que el útero está ocupado por un tejido trofoblástico que prolifera de forma anormal
por lo que suele ser mayor que lo que correspondería para el tiempo de amenorrea. Otro dato
clínico que podría sugerirnos también la presencia de esta patología es la aparición de signos y
síntomas de hipertiroidismo (nerviosismo, palpitaciones, sudoración, temblores) por el parecido
hormonal entre HCG y TSH.(R3)

178. Severe sepsis is defined as sepsis with one or more signs of organ dysfunction.
Which of the following is INCORRECT?

1. 1. Hypotension.
2. 2. Oliguria (diuresis < 0.5 ml/kg/h).
3. 3. Elevated plasma levels of C-reactive protein or procalcitonin.
4. 4. Hypoxemia.
Gráfico de respuestas
Comentario

Es una pregunta fácil. Tanto la hipotensión arterial, oliguria, hipoxemia e hiperlactoacidemia son
criterios de sepsis grave. Los niveles elevados de proteína C reactiva o procalcitonina no son datos
que impliquen disfunción de órgano. Ni siquiera se requiere tener niveles altos de proteína C
reactiva o procalcitonina para hacer el diagnóstico de sepsis.

Por otro lado, la mayoría de personas con una infección bacteriana presentarán niveles
incrementados de proteína C reactiva o procalcitonina sin que ello suponga necesariamente una
mayor gravedad del cuadro infeccioso.(R3)

179. Mujer de 33 años de edad,


trabajadora de limpieza en un kinder, con antecedentes de frecuentes episodios de
amigdalitis. 48 horas antes de la consulta, inicia con fiebre de 38°C con odinofagia. Por
este motivo se automedica con amoxicilina-clavulánico. A las 12 horas aparece un
exantema pruriginoso generalizado, motivo por el que consulta a su médico de familia,
que cambia el antibiótico a azitromicina y añade corticoides al tratamiento por sospecha
de toxicodermia medicamentosa. 24 horas más tarde, la fiebre persiste, se añade cierta
sensación disneica y tos, motivo por el que consulta al servicio de urgencias hospitalario.
La exploración física muestra a una paciente con fiebre de 39°C, con estado general
afectado, exantema en cara, tronco y espalda, pruriginoso, confluente, con lesiones

 
 
 
 
papulares, habonosas, diseminadas que afectan palmas y plantas. La exploración de la
cavidad oral no muestra exudados purulentos en las amigdalas, pero sí unas lesiones
eritematosas con centro blanquecino frente a ambas arcadas dentarias superiores
(imagen n° 11). ¿Cuál sería la mejor manera de definir estas lesiones teniendo en cuenta
el contexto de la paciente?:

1. 1. Muguet.
2. 2. Manchas de Koplik.
3. 3. Aftas.
4. 4. Leucoplasia.
Gráfico de respuestas
Comentario

Aunque se trate de una mujer de 33 años, a la hora de responder esta pregunta no se puede
obviar que trabaja en un kinder.

Ante una enfermedad exantemática el diagnóstico diferencial es amplio, pero en este caso nos dan
un dato patognomónico como es la presencia de lesiones eritematosas con centro blanquecino
frente a las arcadas dentarias superiores, vinculadas a una imagen característica que corresponde
con Manchas de Koplik.

El resto de datos (fiebre, afectación del estado general, cuadro respiratorio, exantema que afecta a
palmas y plantas) no hacen más que corroborar el diagnóstico de sarampión, enfermedad de
aparición típicamente infantil y que ha aumentado de forma importante su incidencia en los últimos
años gracias al auge del “movimiento antivacunas”.

El contexto clínico descarta las cuatro opciones restantes.(R2)

180. La paciente presentaba crepitantes bibasales y una saturación de 02 del 90%. La


radiografía de tórax mostró un aumento tenue de densidad bibasal de aspecto intersticial.
La serología para VIH de urgencia fue negativa. Se cambió el tratamiento antibiótico a
levofloxacino, se ingresó a la paciente y se solicitaron diversas pruebas
complementarias. Al cuarto día, la paciente quedó afebril, se recuperó de la disnea y las
lesiones cutáneas se fueron aclarando progresivamente en sentido inverso a su
aparición. ¿Cuál de las siguientes pruebas cree Ud. que permitió el diagnóstico
definitivo?:

1. 1. Serología virus influenza H1N1.


2. 2. Serología IgM contra sarampión.
3. 3. Serología IgM contra Parvovirus B-19.
4. 4. Pruebas treponémicas y reagínicas para la sífilis.
Gráfico de respuestas
Comentario

Nos preguntan qué prueba nos permitirá llegar al diagnóstico definitivo en relación con el caso de
la pregunta anterior. De entre todas las opciones solo la respuesta 2 nos permitiría aclarar la
etiología del cuadro de la paciente, a través de pruebas serológicas que determinen anticuerpos
IgM frente a sarampión.

 
 
 
 
Hay que recordar que la neumonía es una de las complicaciones más frecuentes del sarampión,
secundario a la sobreinfección bacteriana y en un menor porcentaje al propio virus (neumonía de
células gigantes de Hecht) de curso grave y que afecta principalmente a inmunodeprimidos.(R2)

181. Mujer de 65 años que acude a Urgencias por un cuadro de disnea progresiva en la
última semana hasta hacerse de reposo, con ortopnea de dos almohadas en los últimos
días. A la exploración física está taquipneica, presentando un soplo holosistólico en ápex
III/VI irradiado a la base. En el ecocardiograma presenta un prolapso del velo posterior de
la válvula mitral con datos de regurgitación severa, conservando una buena función
ventricular. ¿Cuál sería la mejor opción terapéutica en esta paciente?

1. 1. Tratamiento médico exclusivamente.


2. 2. Implantación de prótesis biológica.
3. 3. Implantación de prótesis mecánica.
4. 4. Reconstrucción de la válvula mitral.
Gráfico de respuestas
Comentario

El prolapso mitral es la causa más frecuente de insuficiencia mitral aislada grave. Cuando es
severa y se produce sintomatología importante, la mejor opción es la cirugía. La técnica de
elección sería la reconstrucción valvular mitral (mientras sea posible). Es preferible esto al
recambio protésico, porque se evita la anticoagulación crónica y la supervivencia es mejor con la
reparación valvular.(R4)

182. Paciente que presenta celulitis en la pierna izquierda tras recibir una mordedura de
gato. ¿Cuál es el agente etiológico más frecuentemente implicado?:

1. 1. Francisella tularensis.
2. 2. S. epidermidis.
3. 3. Bacteroides fragilis.
4. 4. Pasteurella multocida.
Gráfico de respuestas
Comentario

Pregunta directa fácil que nos sirve para repasar etiologías. La Francisella turalensis, produce la
turalemia, típicamente en cazadores y veterinarios, mediante vector o por contacto directo con
animales. El S. epidermidis es típico de las lesiones superficiales en piel y en la forma protésica
precoz de la endocarditis infecciosa. El Bacteroides fragilis, podría producir entre otros absceso
peritoneal. La P. multocida es muy frecuente en la mordedura de gato.(R4)

183. Recién nacido de 32 semanas de edad gestacional, que nació con líquido amniótico
claro, no fétido, Apgar 9-9. Presenta dificultad respiratoria progresiva con quejido audible
y retracción intercostal. ¿Cuál es el diagnóstico más probable?

1. 1. Neumonitis inflamatoria.
2. 2. Taquipnea transitoria del recién nacido.
3. 3. Síndrome de aspiración meconial.
4. 4. Enfermedad de membrana hialina.
Gráfico de respuestas
Comentario

 
 
 
 
Tema fundamental en el ENARM. La clave en esta pregunta es la edad gestacional. Por lo que un
RN de 32 SDG con datos clínicos de dificultad respiratoria, la primera sospecha diagnóstica es la
enfermedad de membrana hialina. Respuesta 4 correcta(R4)

184. En la incontinencia urinaria de esfuerzo se produce el siguiente proceso:

1. 1. Hipermovilidad uretral.
2. 2. Fístula vesicovaginal.
3. 3. Rebosamiento vesical.
4. 4. Hiposensibilidad vesical.
Gráfico de respuestas
Comentario

En el caso de incontinencia urinaria de esfuerzo la causa generalmente se debe a hipermovilidad


uretral, respuesta 1 correcta, mientras que en la de urgencia se debe a hiperreactivdad del
detrusor.(R1)

185. A 66-year-old male is admitted because of abdominal discomfort. His past medical
history reveals chronic liver disease with ascites, managed with furosemide and
spironolactone. He is on beta-blockers because of a past episode of esophageal bleeding.
Temperature is 37.8 C and paracentesis shows neutrophils 325/mm3 with albumin 0,8
mg/dL. Serum albumin is 2.0 mg/dL and creatinine is 1.1 mg/dL. What is the cause of this
patient's current condition?

1. 1. Community acquired pneumonia


2. 2. Spontaneous bacterial peritonitis
3. 3. Hepatocellular carcinoma
4. 4. Congestive heart failure
Gráfico de respuestas
Comentario
Spontaneous bacterial peritonitis. In a patient with ascites, the presence of new onset fever,
abdominal pain, hepatic encephalopathy, leukocytosis or other symptom of infection should urge a
diagnostic paracentesis for ascitic fluid analysis and culture. The diagnosis of confirmed
spontaneous bacterial peritonitis requires an elevated ascitic fluid absolute polymorphonuclear
leukocyte (PMN) count of at least 250 cells/mm3 (0.25 x 109/L) and a positive ascitic fluid bacterial
culture without an obvious intra-abdominal source of infection. Ascitic fluid diagnostic testing should
be performed before treatment is initiated.(R2)

186. El misoprostol está indicado en:

1. 1. Prevención de lesiones agudas de la mucosa gástrica por AINE.


2. 2. Prevención de cualquier tipo de úlcera duodenal.
3. 3. El tratamiento de la gastropatía hemorrágica.
4. 4. En la profilaxis de úlceras de Curling.
Gráfico de respuestas
Comentario

El misoprostol es un fármaco análogo de prostaglandinas. Por tanto, se puede emplear en el


tratamiento de la gastropatía por AINEs, e incluso del ulcus provocado por los AINEs (que inhiben
la acción protectora de la mucosa gástrica que tienen las prostaglandinas). Sin embargo, aunque

 
 
 
 
este fármaco es eficaz, son necesarias 3 ó 4 dosis al día, por lo que actualmente su uso es
bastante escaso. Por ello, son mucho más utilizados los inhibidores de la bomba de protones.(R1)

187. Paciente de 53
años de edad, hipertenso sin otros antecedentes patológicos de interés. A partir del
estudio por su hipertensión se realiza un estudio con TC abdominal cuyas imágenes
corresponden a la imagen 12. El paciente no presenta ninguna sintomatología. Ante este
hallazgo ¿cuál es la opción adecuada a seguir?

Remitir al paciente a Urgencias por riesgo de sangrado agudo y realizar una embolización
1. 1.
selectiva del riñón derecho.
2. 2. Solicitar un estudio con renograma con diurético para valorar la funcionalidad renal.
Realizar una punción-biopsia de la lesión para poder diagnosticar al paciente y decidir el
3. 3.
tratamiento adecuado.
4. 4. Programar una nefrectomía radical tras realizar un estudio de extensión.
Comentario

Se trata de una pregunta sencilla sobre tumores renales.

Ante el caso clínico descrito, es esencial realizar el correcto diagnóstico de tumor renal, del que se
ha preguntado en numerosas ocasiones el manejo diagnóstico-terapéutico. Ante una masa renal,
debe realizarse, en primer lugar, una ecografía (para valorar si es un quiste simple -actitud
conservadora- o un quiste complicado/masa sólida).

En el caso de hallarnos ante un quiste complejo o una masa sólida, debe realizarse una tomografía
computarizada (TC), que es el mejor método para evaluar una masa renal (y no la RM, que sólo es
de elección cuando se sospecha afectación vascular).

Una vez realizada la TC, pueden plantearse exploraciones complementarias para descartar
afectación metastásica (radiografía de tórax, analítica hepática y, en ocasiones, gammagrafía
ósea). En cambio, no son necesarios estudios funcionales, ni histológicos, pues su resultado no
modifica la actitud terapéutica a adoptar. Una vez descartada la afectación metastásica, el
tratamiento de elección de un tumor renal es la nefrectomía radical (que incluye la fascia de Gerota
y la glándula suprarrenal).

 
 
 
 
No se considera adecuado, un manejo expectante, ni la embolización selectiva del riñón afectado,
pues el tratamiento de una masa renal sólida debe ser quirúrgico.(R4)

Algoritmo diagnóstico de las masas renales

188. ¿Cuál es el diagnóstico más probable del paciente del caso anterior?

1. 1. Oncocitoma.
2. 2. Carcinoma de células claras.
3. 3. Angiomiolipoma.
4. 4. Pielonefritis xantogranulomatosa.
Gráfico de respuestas
Comentario

Esta pregunta podría contestarse de dos formas: la primera de ellas, más sencilla, es sabiendo que
el tumor renal más frecuente es el carcinoma de células claras (respuesta 2), también llamado
hipernefroma, adenocarcinoma renal o tumor de Grawitz. La segunda de ellas requiere conocer
rasgos diferenciales (tanto clínicos como epidemiológicos y de radiodiagnóstico) de cada uno de
los tipos histológicos citados.

•   Oncocitoma (respuesta 1): tumor benigno muy infrecuente que no suele causar clínica
(hallazgo ecográfico) y que no presenta hemorragia ni necrosis intratumoral, mostrándose
homogéneo en la TC o con una característica cicatriz central.
•   Angiomiolipoma (respuesta 3): tumor benigno de forma redonda u ovalada con presencia
característica de grasa intratumoral, ofreciendo una imagen típica en la TC (densidad grasa
en el seno del tumor).
•   Pielonefritis xantogranulomatosa (respuesta 4): forma infrecuente de una pielonefritis
bacteriana crónica que puede confundirse con un carcinoma renal. La mayoría de los

 
 
 
 
pacientes refieren infecciones urinarias de repetición (frecuentemente complicadas con
litiasis) y tienen un patrón ecográfico con zonas hipoecoicas e hiperecoicas. La TC
demuestra aumento difuso (y no focal) del tamaño renal, hallándose múltiples imágenes
redondeadas hipodensas sustituyendo el parénquima renal (corresponden a abscesos y
cálices dilatados).
•   Carcinoma de células claras (respuesta 2): es la que mejor se ajusta al cuadro clínico
(varón hipertenso de mediana edad) y a la imagen tomográfica (masa renal de bordes
irregulares y densidad heterogénea).

(R2)

189. Uno de los siguientes datos NO se encuentra habitualmente entre los hallazgos de
laboratorio del síndrome de Sjögren:

1. 1. Hipergammaglobulinemia.
2. 2. Trombopenia.
3. 3. Células LE.
4. 4. Anti-DNA.
Gráfico de respuestas
Comentario
El síndrome de Sjögren ha sido preguntado tan sólo una vez en los últimos 10 años. Debes saber
que puede ser secundario, y en tal caso asociarse a diversas enfermedades del tejido conectivo,
como la AR (con FR positivo), el LES (con presencia de células LE y anticuerpos anti- DNA
positivos); el ESP,? al ser una enfermedad autoinmune crónica, puede presentar
hipergammaglobulinemia (reflejo de la alteración inmunitaria con infiltración de linfocitos CD4+), y
es muy posible que presente anemia de trastornos crónicos (la inflamación provoca un mal
metabolismo del hierro). Lo que no se produce sin embargo, es trombopenia, porque no existe
infiltración de la médula ósea y no hay otra causa que la origine. Lo que sí puede existir es
trombocitosis, puesto que es frecuente en la inflamación crónica intensa.(R2)

190. Una paciente de 60 años, con el diagnóstico de bocio multinodular y en tratamiento


con Levotiroxina, inicia síntomas de hipertiroidismo, ¿Cuál será su actitud?:

1. 1. Suprimir el tratamiento.
2. 2. Añadir al tratamiento fármacos antitiroideos.
3. 3. Solicitar hormonas tiroideas y modificar la dosis en función de los resultados.
4. 4. Utilizar tratamiento con radioyodo.
Gráfico de respuestas
Comentario
Nos presentan a un paciente con bocio multinodular que recibiendo tratamiento con hormonas
tiroideas, comienza a presentar clínica de hipertiroidismo. La actitud más correcta con estos
pacientes es en primer lugar confirmar el hipertiroidismo mediante la determinación de hormonas
tiroideas, y si se confirma el diagnóstico disminuir la dosis de levotiroxina. Si posteriormente
persistiera el hipertiroidismo, se suspendería el tratamiento y se realizaría una gammagrafía para
descartar la existencia de un nódulo tóxico, con lo que el tratamiento de lección sería el radioyodo.
Suspender el tratamiento de entrada no está indicado, al igual que añadir tratamiento con
antitiroideos. La cirugía en el bocio multinodular sólo se encuentra indicada en los casos en los que
exista sintomatología compresiva.(R3)

191. Con respecto a la ergometría o prueba de esfuerzo, señale la afirmación CORRECTA:

 
 
 
 
Durante la prueba, la frecuencia cardíaca y la presión arterial pueden subir, mantenerse sin
1. 1. variaciones importantes, o descender, no teniendo ningún valor pronóstico sean cuales
sean los cambios de estas dos constantes.
La insuficiencia aórtica sintomática es una contraindicación para la realización de una
2. 2.
ergometría.
Es la prueba ideal para detectar isquemia cuando el ECG basal es patológico con descenso
3. 3.
o elevación del ST mayor o igual a 1 mm (BCRI, ritmo de MP, cubeta digitálica...).
La hipotensión arterial progresiva durante su realización con datos de isquemia (dolor y
4. 4.
cambios eléctricos) es un dato de mal pronóstico y sugiere enfermedad coronaria severa.
Gráfico de respuestas
Comentario

La TA tiene valor pronóstico (respuesta correcta la 4) y la FC nos sirve para saber si la prueba es o
no concluyente. Una ergometría positiva precoz indica mal pronóstico y por tanto sugiere
enfermedad coronaria severa. Es la estenosis aórtica severa sintomática y no la insuficiencia
aórtica la que contraindicaría la realización de una prueba de esfuerzo. Si el ECG basal es
patológico lo ideal es recurrir a técnicas de detección de isquemia con imagen (eco de estrés o
perfusión con isótopos) por ser más fiables al ser más sensibles y específicas.(R4)

192. Which of the following statements is INCORRECT regarding severe aortic stenosis?

1. 1. The development of symptoms provides a clear indication for sugical intervention.


A significant increase in mean aortic valve gradient estimation may be an indication for
2. 2.
replacement surgery in asymptomatic patients.
3. 3. Exercise treadmill testing is absolutely contraindicated in this cardiac valvulopathy.
Stress echocardiogram with dobutamine may be useful to assess the indication for valve
4. 4.
replacement.
Gráfico de respuestas
Comentario

La estenosis aórtica severa debe intervenirse quirúrgicamente cuando el paciente esté sintomático
o bien exista disfunción sistólica ventricular izquierda. También serían candidatos a cirugía
aquellos pacientes que, pese a estar asintomáticos y mantener fracción de eyección normal,
muestran intensa calcificación de la válvula o incremento rápido del gradiente transvalvular. Existe
un grupo de pacientes con disfunción ventricular en los que existe discrepancia en cuanto a
severidad de la estenosis aórtica por mostrar área valvular aórtica menor (AVA) de 0.6 cm2/m2 de
superficie corporal (criterio de severidad) y sin embargo el gradiente medio está por debajo de 40
mmHg (propio de estenosis aórtica ligera o moderada). En esta circunstancia, la realización de una
ecocardiografía con dobutamina puede ayudar a discernir entre una verdadera estenosis aórtica
severa (incremento del gradiente sin modificar el AVA) de una estenosis pseudosevera, en la que
existirá aumento del AVA con un gradiente final transaórtico menor de 50 mmHg. Recientemente
se ha desarrollado el implante de una prótesis aórtica por vía transapical o transarterial femoral,
con resultados prometedores.(R3)

193. Con respecto a la epidemiología de la polimiositis, señalar lo INCORRECTO:

1. 1. Es más frecuente en hombres con una relación 2:1.


2. 2. La edad de aparición en adultos es alrededor de los 50 años.
Los formas asociadas a neoplasias se detectan con mayor frecuencia en mayores de 60
3. 3.
años.
4. 4. La forma clínica más frecuente es la polimiositis idiopática del adulto.
Gráfico de respuestas

 
 
 
 
Comentario
La epidemiología de la polimiositis no es relevante para el MIR. Si recuerdas que la mayoría de las
enfermedades autoinmunes se dan en mujeres de mediana edad, habrás contestado con éxito.
Sólo la miositis por cuerpos de inclusión es más frecuente en varones (es la excepción a la regla, al
igual que la espondilitis anquilopoyética; la policondritis recidivante no muestra predominio sexual).
De esta pregunta destacar que hay formas de dermatomiositis asociadas a neoplasia, las cuales
pueden aparecer antes o después de la conectivopatía; lo más frecuente es que se trate de
pacientes ancianos en este caso, y debemos enfocar el estudio hacia la localización de un tumor
oculto. Sin embargo, lo más frecuente es que no exista neoplasia, y que sea polimiositis o
dermatomiositis idiopática del adulto. Los casos infantiles son poco frecuentes.(R1)

194. Con respecto a los cuadros denominados "encefalitis postinfecciosa", señale la


opción FALSA:

1. 1. Aparecen típicamente tras una infección vírica o tras la administración de algunas vacunas.
2. 2. El hallazgo anatomopatológico característico es una desmielinización perivenosa.
El virus causante del proceso al que sigue la encefalitis se encuentra casi siempre presente
3. 3.
en el SNC.
La encefalomielitis diseminada aguda y la encefalomielitis necrotizante aguda son dos
4. 4.
encefalitis postinfecciosas.
Gráfico de respuestas
Comentario

El nombre de la enfermedad nos lo revela todo, o al menos lo esencial para resolver esta pregunta.
Si se trata de una encefalitis POST- infecciosa, no será necesaria la presencia del virus (respuesta
3 falsa), sino una reacción de hipersensibilidad inducida por la infección, que ya habría sido
controlada por el sistema inmune y que, por tanto, ya habría desaparecido.(R3)

195. Señale la afirmación INCORRECTA:

1. 1. La hipoxia alveolar provoca vasoconstricción local.


La hipoxemia en las fístulas arteriovenosas pulmonares se corrige administrando oxígeno
2. 2.
al 100%.
3. 3. En los pacientes con fístulas arteriovenosas puede observarse acropaquia.
4. 4. La hemorragia pulmonar puede ser una complicación de las fístulas arteriovenosas.
Gráfico de respuestas
Comentario

Pregunta muy importante sobre fisiopatología del aparato respiratorio. No debe tener ninguna duda
al respecto. Si la ha fallado, estudie cuidadosamente lo que a continuación diremos.

1.- La hipoxia alveolar produce vasoconstricción de los vasos relacionados con ese alvéolo. Por
este motivo, los pacientes con EPOC tipo bronquitis crónica acaban teniendo hipertensión
pulmonar a largo plazo, ya que la consecuencia final es la hiperplasia e hipertrofia de la
musculatura lisa de estos vasos, por la vasoconstricción sostenida en el tiempo. Y por el mismo
motivo, la oxigenoterapia hace que mejore esta hipertensión pulmonar, ya que el oxígeno es
vasodilatador para los vasos pulmonares.

2.- La vasoconstricción pulmonar ante la hipoxia es, en realidad, un “mecanismo de defensa” del
aparato respiratorio. Lógicamente, si el aire del alvéolo tiene poco oxígeno, no interesa que los

 
 
 
 
vasos de este alvéolo participen en el intercambio gaseoso, porque la sangre que pase por ellos no
se oxigenaría. En cambio, si le damos oxígeno, estos vasos se abren, oxigenándose la sangre que
los recorre. Si utilizáramos un vasodilatador pulmonar que no fuera el oxígeno, pasaría a través de
ellos más sangre, pero no se oxigenaría, y esto podría producir hipoxemia.

3.- Una fístula arteriovenosa produce hipoxemia por un mecanismo tipo shunt. Esto consiste en
que la sangre que pasa a través de ella no está en contacto con el espacio aéreo, por lo que no
podría oxigenarse aunque respirásemos oxígeno al 100%. Recuerde que la hipoxemia por shunt
NO mejora con oxígeno.

4.- Las fístulas arteriovenosas son una posible causa de acropaquias. Recuerde que existen otras
muchas, como la fibrosis pulmonar idiopática, otras enfermedades intersticiales, la fibrosis quística,
el cáncer de pulmón…

5.- Las fístulas arteriovenosas, además de no mejorar con oxígeno, en ocasiones pueden
romperse y producir hemorragias pulmonares, con o sin hemoptisis.(R2)

196. Sobre el síncope cardiovascular, ¿cuál de las siguientes afirmaciones es


INCORRECTA?:

Alrededor del 20% de la población adulta ha sufrido alguna vez en su vida un cuadro
1. 1.
sincopal.
Se debe a la alteración de los componentes sanguíneos que produce un déficit transitorio
2. 2.
del metabolismo cerebral.
El nivel crítico de descenso del flujo cerebral para la aparición de disfunción cerebral es de
3. 3.
75 ml/100 g/min.
La fase de lesiones neurológicas irreversibles se produce a partrir del 3-5º minuto de
4. 4.
ausencia de flujo cerebral.
Gráfico de respuestas
Comentario

Pregunta de dificultad moderada. No solo relevante en cardiología, puede ser preguntada en


neurología el flujo cerebral necesario. Es la típica pregunta que se saca descartando opciones. La
causa más frecuente de síncope es la cardiovascular, más concretamente el síncope vasovagal.
En torno al 20% de la población lo ha sufrido alguna vez. El periodo de tiempo que aguanta el
cerebro en ausencia de flujo cerebral es en torno a 4 minutos, más allá se producen lesiones
neurológicas irreversibles (opción 4 cierta). La opción 2 tambien es cierta, el síncope se produce
por déficit transitorio del metabolismo cerebral. Descartando nos queda la opción 3 que es la
falsa(R3)

197. La parálisis de la neurona motora inferior se caracteriza por todo lo siguiente,


EXCEPTO:

1. 1. Atrofia muscular intensa.


2. 2. Flaccidez e hipotonía de los músculos afectados.
3. 3. Signo de Babinski.
4. 4. Fasciculaciones.
Gráfico de respuestas
Comentario

 
 
 
 
Se trata de una pregunta que no te puedes permitir fallar el día del MIR. Si la has fallado, es
importante que repases los signos de 1ª y 2ª motoneurona.(R3)

1ª motoneurona 2ª motoneurona

Hiperreflexia Hipo/Arreflexia

Respuesta cutaneoplantar extensora (Babinski) Respuesta cutaneoplantar flexora

Amplios grupos musculares Músculos aislados

Atrofia por desuso Amiotrofia precoz, fasciculaciones

198. Mujer de 44 años que acude a consulta de ginecología por menorragias, sin otra
sintomatología acompañante. La exploración ginecológica muestra vagina y cérvix
normales, mamas normales y tacto vagino-abdominal con útero aumentado de volumen,
de consistencia dura y superficie irregular. El ultrasonido confirma el diagnóstico de tres
miomas, de entre 3-4 cm de diámetro, de localización submucosa. En la BH se detecta
una anemia ferropénica moderada. ¿Qué tipo de tratamiento propondría a esta paciente?

1. 1. Miomectomía abdominal.
2. 2. Tratamiento médico de la anemia ferropénica y observación de los miomas.
3. 3. Histerectomía abdominal.
4. 4. Tratamiento de la anemia ferropénica e histerectomía abdominal.
Gráfico de respuestas
Comentario

La paciente del caso clínico presenta una patología orgánica (miomas uterinos) con repercusión
clínica (menorragias o sangrado cíclico abundante) y de laboratorio (anemia ferropénica).

Requiere, por tanto, un tratamiento causal además de sintomático. Los análogos de la GnRH sólo
los usamos antes de la cirugía, pues una vez interrumpido el tratamiento, los miomas regresan a su
tamaño anterior. Se nos presenta además una paciente de cierta edad, a la que se puede realizar
una cirugía más completa que evite las menorragias, como la histerectomía, dejando la
miomectomía para pacientes más jóvenes con deseos genésicos no cumplidos. Por último, habría
que añadir el tratamiento necesario para corregir la anemia ferropénica, por lo que la opción
correcta es la 4.(R4)

 
 
 
 
199. Una niña de 10 años de edad acude para valoración de un posible hipocrecimiento.
Su talla actual se encuentra en el percentil 10 de la población general. Su talla genética
está situada en el percentil 15 de la población de referencia. Su desarrollo sexual
corresponde a una pubarquia 1 y a una telarquia 1 de Tanner. Su edad ósea es de 9 años.
La velocidad de crecimiento del último año ha sido de 5 cm/año. ¿Qué situación cree
usted que presenta la niña?

1. 1. Un crecimiento normal.
2. 2. Un hipocrecimiento por deficiencia de la hormona del crecimiento.
3. 3. Un hipocrecimiento por una enfermedad celíaca.
4. 4. Un hipocrecimiento por un síndrome de Turner.
Gráfico de respuestas
Comentario

Aunque sea aparentemente compleja, esta pregunta se puede contestar fácilmente conociendo la
definición de talla baja o hipocrecimiento.

Se habla de hipocrecimiento cuando la talla se encuentra por debajo del percentil 3 para la edad y
sexo. La paciente del caso tiene una talla en el P10, con lo que en principio debemos asumir que
su crecimiento es normal (opción 1 correcta).

Los demás datos del enunciado no hacen sino apoyar este diagnóstico. La paciente es una niña de
diez años que todavía no ha iniciado el desarrollo puberal (estadio 1 de Tanner). Se considera que
en el período prepuberal inmediato la velocidad de crecimiento debe ser al menos de 4 cm/año (la
de la paciente es de 5 y, por consiguiente, normal).

Por otro lado, la talla genética de la paciente (determinada por la talla media de los progenitores)
está en el P15, bastante próxima a su talla real. Finalmente, se asume que la edad ósea de un
paciente es normal cuando se encuentra en un intervalo de +/– 1 año en torno a la edad
cronológica.(R1)

200. NO es típico de la leucemia linfática crónica:

1. 1. Presencia de inmunoglobulinas en la membrana linfocitaria.


2. 2. Hipergammaglobulinemia.
3. 3. Ausencia de síntomas en estadios iniciales.
4. 4. Trombopenia autoinmune.
Gráfico de respuestas
Comentario

Todos los datos son característicos de la LLC, excepto la hipergammaglobulinemia. Recuerda que
en esta patología, a pesar de que es frecuente la patología autoinmune, existe
hipogammaglobulinemia progresiva, responsable de la inmunodepresión humoral que sufren estos
pacientes.(R2)

201. Mujer de 41 años que presenta cuadro de varios meses de evolución de astenia,
disnea de esfuerzos, palidez progresiva, molestias orales con la ingestión de alimentos
ácidos y diarrea. En el hemograma se detecta Hb 10 g/dL, VCM 112, leucocitos 3000/mm3,
plaquetas 100000mm3. En el frotis se objetivan macroovalocitos, poiquilocitos, algún
eritroblasto e hipersegmentación de neutrófilos. La paciente le cuenta que toma muchos

 
 
 
 
fármacos, ¿Cuál de los siguientes cree que puede ser el causante del cuadro
hematológico de esta paciente? :

1. 1. Ciclosporina.
2. 2. Vincristina.
3. 3. Placitaxel.
4. 4. Metotrexate.
Gráfico de respuestas
Comentario
Para contestar las preguntas de Hematología, es fundamental , tener en cuenta tres cosas: cuadro
clínico, patrón hematológico y posibles factores de riesgo. CLINICA: compatible con anemia, sobre
todo por la palidez.La afectación de la mucosa oral nos orienta a un tipo en concreto de anemia.
HEMATOLOGICO: siempre buscar datos de Reticulocitos,frotis y VCM. Se tata de anemia
MACROCORPUSCULAR,con neut#rofilos hipersegmentados en frotis, datos que siempre te deben
hacer pensar en MEGALOBLASTOSIS. FACTOR DE RIESGO: en este caso es lo que te
preguntan. Aunque la causa más frecuente de déficit de folatos es nutricional, los fármacos son
también frecuentes, y sobre todo el METROTEXATE,por producir un bloqueo en laactivación de
folatos.(R4)

202. ¿Cuál de las siguientes relaciones medio de cultivo-microorganismo NO es la


correcta?:

1. 1. Thayer-Martin-Streptococcus pneumoniae.
2. 2. TCBS- Vibrio cholerae.
3. 3. Lowestein-M. tuberculosis.
4. 4. Sabouraud-Candida albicans.
Gráfico de respuestas
Comentario

El medio de Thayer-Martin se utiliza para cultivo de gonococos a partir de secreciones genitales o


respiratorias. V. cholerae crece en agar TCBS (Tiosulfato-Citrato-sales Biliares-Sucrosa). El medio
de Lowenstein es un medio sólido utilizado para cultivar micobacterias. El medio de Sabouraud es
el medio habitualmente utilizado para recuperar hongos.(R1)

203. Un niño de 2 años, durante una gastroenteritis aguda presenta diarrea líquida
intensa, vómitos continuos, sed intensa, diuresis escasa y somnolencia. Pesaba 17.5 kg
antes del episodio y ahora pesa 15 kg. ¿Qué grado de deshidratación tiene y cuál es la
vía de elección para rehidratarle?:

1. 1. Deshidratación leve - rehidratación oral.


2. 2. Deshidratación moderada - rehidratación oral.
3. 3. Deshidratación severa - rehidratación oral.
4. 4. Deshidratación severa - rehidratación intravenosa.
Gráfico de respuestas
Comentario

Tema muy importante para el ENARM. En un niño de esta edad, una pérdida superior al 10% de su
peso se considera una deshidratación severa. En este caso, hemos sobrepasado el límite
ampliamente, llegando casi al 15%

 
 
 
 
Observe que, por otra parte, está teniendo una importante repercusión clínica (escasa diuresis y
somnolencia). La respuesta correcta sería, por tanto, la 4, ya que en este caso se trata de un caso
lo suficientemente grave como para plantearse la rehidratación intravenosa.(R4)

204. ¿Cuál de los siguientes trastornos del curso del pensamiento NO se presenta en los
trastornos mentales funcionales y sí en los orgánicos?:

1. 1. Fuga de ideas.
2. 2. Perseveración del pensamiento.
3. 3. Incoherencia del pensamiento.
4. 4. Disgregación del pensamiento.
Gráfico de respuestas
Comentario
Una pregunta extraordinariamente difícil, no te preocupes si la has fallado.

El pensamiento incoherente ha sido considerado de varias maneras: como un concepto similar a la


disgregación o como un grado por encima de ésta. Sin embargo, y en general se considera lo más
acertado, se entiende por pensamiento incoherente la disgregación asociada al deterioro del nivel
de conciencia (obnubilación) que se produce en el delirium. Como el delirium también es conocido
como síndrome orgánico cerebral (obedece a una causa orgánica, tal como dice el enunciado), la
respuesta correcta sería la 4.

Este tipo de preguntas son muy raras en el examen MIR. Cuando aparecen, casi nadie es capaz
de acertarlas, si no es por efecto del puro azar, con lo que no representan un problema (el
porcentaje de fallos es tan alto que no resultan discriminativas).(R3)

205. En el hemograma de un paciente con astenia se informa sobre la existencia de


blastos. ¿Cuál es el diagnóstico MENOS probable?

1. 1. Leucemia mieloide aguda.


2. 2. Leucemia mieloide crónica.
3. 3. Leucemia linfática crónica.

 
 
 
 
4. 4. Mielodisplasia.
Gráfico de respuestas
Comentario

A nadie le extrañaría encontrarse blastos en una leucemia aguda, con lo que las respuestas 1 y 4
se descartan con facilidad. También puede haberlos en algunos síndromes mielodisplásicos (por
ejemplo, en la anemia refractaria con exceso de blastos), por lo que el propio nombre de la
enfermedad nos permite descartar la respuesta 5.

La duda razonable estaría entre las opciones 2 y 3. La correcta es la 3, ya que en la leucemia


linfática crónica es absolutamente extraordinaria la evolución hacia formas más agresivas,
falleciendo los pacientes por infecciones bacterianas debidas a su déficit de anticuerpos
(neumococo, Moraxella, Haemophilus…). Sin embargo, la historia natural de la leucemia mieloide
crónica termina con frecuencia en una fase acelerada, a la que sigue una crisis blástica, donde
lógicamente abundan este tipo de células.(R3)

206. A 3-year-old child is brought to the emergency room by his mother because of
persistent cough. His mother reports very frequent respiratory infections (almost every
week) with continuous sneezing and a runny nose. Lung auscultation shows bilateral,
diffuse sibilant rhonchi. Chest X-ray shows the heart silhouette located on the right side.
Which of the following is the most likely diagnosis?

1. 1. Bruton's agammaglobulinemia
2. 2. Ataxia-telangiectasia syndrome
3. 3. DiGeorge syndrome
4. 4. Kartagener's syndrome
Gráfico de respuestas
Comentario
Kartagener's syndrome. Dextrocardia + frequent respiratory infections should make us think of a
Kartagener syndrome, also called primary ciliary dysfunction. 50% of patients present situs inversus
which can occur with or without dextrocardia.(R4)

207. La edad de mayor frecuencia de las convulsiones febriles corresponde a niños:

1. 1. De los 7 a 12 meses.
2. 2. De los 13 a 24 meses.
3. 3. De los 25 a 36 meses.
4. 4. Mayores de 4 años.
Gráfico de respuestas
Comentario

De los 7 a los 12 meses es cuando más riesgo existe de convulsiones febriles.(R1)

208. Recién nacido de 42 semanas de edad gestacional y antecedentes de sufrimiento


fetal y eliminación intraútero de meconio, que presenta, a las pocas horas de vida,
cianosis importante, quejido, aleteo, tiraje sub e intercostal, hipotensión y taquicardia.
En la gasometría se detecta una marcada hipoxia, que no mejora con la administración
de oxígeno con casco cefálico al 100%. Señale cuál es su diagnóstico de presunción:

 
 
 
 
1. 1. Sepsis por Streptococo del grupo B.
2. 2. Transposición de grandes arterias.
3. 3. Persistencia de circulación fetal.
4. 4. Hernia diafragmática.
Gráfico de respuestas
Comentario

Las complicaciones del síndrome de aspración meconial són: extravasación extraalveolar de aire,
persistencia de la circulación fetal, complicaciones de la instrumentalización e hipertensión
pulmonar.

Las cuatro opciones són posibles causas de distrés respiratorio pero sólo dos no mejoran con el
oxígeno, la 2 y la 3. Puesto que la 3 es una complicación del SAM y no mejora con el oxígeno sería
la opción correcta.(R3)

209. Durante la revisión médica anual en una fábrica, a un trabajador de 35 años se le


detecta una cifra de transaminasas elevada. Remitido a su médico de cabecera, éste le
pide marcadores de hepatitis en suero, obteniéndose los siguientes resultados: HBsAg
(+), anti-HBs (-), anti-HBc IgG (+), anti-HBc IgM (-), HBeAg (+), Ac anti-VHC (-), Ac IgM anti-
VHD (+), IgG anti-VHA positivo. ¿Cuál cree que es el diagnóstico correcto?

1. 1. Tiene una infección crónica por VHB y VHD.


2. 2. Tiene una infección simultánea por VHB y VHD (coinfección).
Tiene una infección aguda por VHD sobre una infección crónica por VHB
3. 3.
(sobreinfección).
4. 4. Es una infección aguda por VHB por una cepa mutante precore.
Gráfico de respuestas
Comentario

Es un caso clínico típico del MIR sobre serología de hepatitis virales. La presencia de IgG anti-
VHA demuestra la existencia de una infección antigua por dicho virus, no es su problema actual.
Respecto al virus B, está presente es su organismo (AgHBs +) y ha tenido contacto con el mismo
hace tiempo (IgG anti- HBc +) por lo que se trata de una hepatitis crónica por VHB. Respecto al
VHD, presenta anticuerpos positivos por lo que si carece de VHC, la única opción correcta es la
4.(R3)

210. ¿En cuál de los siguientes casos es más probable que la ictericia de un lactante a
término sea "fisiológica"?

1. 1. Cuando la bilirrubina sérica se eleva 5 mg/dl/24 horas o menos en los 2 a 4 primeros días.
2. 2. Cuando la bilirrubina sérica directa (conjugada) es mayor de 1 mg/dl.
3. 3. Ictericia que aparece a los 12 días de nacer.
4. 4. Máxima concentración de bilirrubina en el 7º día.
Gráfico de respuestas
Comentario

La ictericia fisiológica parece ser un mecanismo de “autodefensa” del organismo frente al estrés
por radicales libres que acecha al neonato. La ictericia fisiológica nunca comienza en las primeras
24 horas de vida, sino el 2º o tercer día de vida. La elevación de la bilirrubina nunca será mayor de
5 mg/dl/día (opción 2, correcta). Además, cabe reseñar que es una hiperbilirrubinemia no

 
 
 
 
colestática. Por ello, la fracción directa debe ser menor de 1 (opción 3 falsa). La máxima
concentración se obtiene alrededor del tercer día de vida (opción 4 falsa).(R1)

211. Varón de 43 años, sin factores de riesgo cardiovascular, con adenocarcinoma de


colon en tratamiento con capecitabina. Consulta por llevar 20 minutos con dolor
centrotorácico opresivo, iniciado durante el sueño. Se realiza un ECG que demuestra
supradesnivel de 5 mm del ST de V2 a V6. Se le administra un comprimido de
nitroglicerina sublingual con lo que cede el dolor en tres minutos, normalizándose
inmediatamente el electrocardiograma. ¿Cuál es el diagnóstico más probable?

1. 1. Pericarditis aguda.
2. 2. Embolia de pulmón.
3. 3. Vasoespasmo coronario.
4. 4. Disección de aorta ascendente.
Gráfico de respuestas
Comentario
El contexto clínico (varón, joven, ausencia de factores de riesgo), la presentación (dolor anginoso,
inicio nocturno en reposo), las características ECG (supradesnivel del ST) y la respuesta a NTG
(resolución del cuadro) apuntan a un vasoespasmo coronario. Además, el paciente está bajo
tratamiento oncológico con capecitabina, que es un análogo oral del 5- fluoruracilo, fármaco que
tiene como efecto adverso característico el desarrollo de vasoespasmo coronario.(R3)

212. El uraco persistente se debe al fallo de cierre de:

1. 1. Conducto alantoideo.
2. 2. Conducto onfalomesentérico.
3. 3. Divertículo de Meckel.
4. 4. Rectos abdominales.
Gráfico de respuestas
Comentario

La presencia de un uraco persistente se debe a la ausencia de obliteración del conducto


alantoideo. Puede dar lugar a un simple tracto fistuloso (por el que sale un líquido amarillo
transparente de pH ácido: orina), o a la asociación de éste con un pólipo o un quiste.(R1)

213. Con respecto a la nefropatía diabética, señale la respuesta FALSA:

La diabetes tipo 2 es la etiología más frecuente de insuficiencia renal terminal en el mundo


1. 1.
occidental.
Más del 90% de los diabéticos tipo 1 desarrollan nefropatía a los 30 años del diagnóstico
2. 2.
de diabetes.
3. 3. La alteración renal más temprana es la hiperfiltración.
4. 4. La existencia de microalbuminuria predice el desarrollo de nefropatía clínica.
Gráfico de respuestas
Comentario

La causa más habitual de insuficiencia renal crónica es la diabetes mellitus tipo 2 (respuesta 1
correcta). Hasta hace algunos años, eran más frecuentes las glomerulonefritis, pero han sido
superadas por esta enfermedad endocrina. En las primeras fases de la nefropatía diabética, existe
una situación de hiperfiltración glomerular, que poco a poco deteriora el glomérulo, apareciendo

 
 
 
 
microalbuminuria y después proteinuria franca (respuestas 3 y 4 ciertas). La presencia de
nefropatía es una manifestación microangiopática, igual que la retinopatía, que suele aparecer a la
vez o, incluso, un poco antes (respuesta 5 correcta). De hecho, existe un adagio nefrológico que
dice, en este contexto, no hay nefropatía sin retinopatía.

La respuesta 2 es falsa porque la aparición de la nefropatía diabética es más precoz (suele


diagnosticarse a los diez años del comienzo de la enfermedad) y porque el porcentaje de
diabéticos tipo 1 que desarrolla nefropatía a lo largo de su vida no alcanza un valor tan alto como el
90%.(R2)

214. Se recibe un recién nacido a término de madre diabética con peso de 4,000 grs y que
presenta a la hora de vida un dextrostix de 30 mg. Indique el tratamiento más adecuado:

1. 1. Suero glucosado 20 cc y control en tres horas.


2. 2. Infusión de suero glucosado IV a 4 mg/Kg/minuto.
3. 3. Bolo de dextrosa al 10% 200 mg/kg, seguido de infusión de glucosa a 6-8 mg/kg/min.
4. 4. Glucosa al 50% 10 mL.
Gráfico de respuestas
Comentario

Respuesta sencilla. La correcta es la 3.

El tratamiento de una hipoglicemia severa es con un bolo inicial de 200 mg/kg de glucosa (2 ml/kg
de dextrosa al 10% en agua) en un minuto, seguido de una infusión de glucosa a 6-8 mg/kg/min. El
pequeño bolo minimiza la hiperglicemia que puede provocar secreción de insulina y una posible
hipoglicemia prolongada.

Se revisa la concentración sérica de glucosa a los 20 minutos después del bolo y se ajusta la
velocidad de infusión o la concentración de dextrosa para mantener una concentración de glucosa
plasmática superior a 50 mg/dL.

En caso que no sepa la respuesta correcta, generalmente la respuesta más completa o más
extensa es la respuesta correcta, por lo que hubiera elegido la número 3 y hubiera acertado.(R3)

215. Niña de 3 años de edad que presenta antecedentes familiares de rinitis alérgica y
personales de alopecia areata. Tras comenzar el colegio hace dos meses presenta
lesiones pruriginosas, polimorfas y localizadas en las flexuras antecubital y poplítea.
¿Que NO sería frecuente encontrar dentro del contexto de su patología?

1. 1. Ictiosis vulgar.
2. 2. Pulpitis digital crónica.
3. 3. Pitiriasis alba.
4. 4. Granuloma tricofítico de Majocchi.
Gráfico de respuestas
Comentario
La información que nos aporta el enunciado (la clínica y los antecedentes familiares y personales)
nos debe llevar al diagnóstico de dermatitis atópica. La dermatitis atópica es una enfermedad
frecuente de la piel que puede aparecer a cualquier edad. Se piensa que aparece por una
interacción de factores ambientales y genéticos. Es una dermatitis crónica, recidivante, asociada a
un intenso prurito. Es frecuente su asociación con xerosis y con una historia personal o familiar de
atopia. Clínicamente se distinguen tres fases; la del lactante, la infantil y la del adolescente- adulto.

 
 
 
 
La forma infantil, que es la que padece esta paciente, comprende el periodo de los 2- 10 años. Se
manifiesta como lesiones subagudas clásicamente localizadas en los huecos poplíteos y cubitales,
zonas de flexión de las muñecas, párpados, cara y cuello. Existen formas menores de atopia, entre
las que se incluyen el prúrigo estrófulo, la pulpitis digital crónica y la pitiriasis alba. El prúrigo
estrófulo es una dermatosis papulovesiculosa, muy pruriginosa, que evoluciona en brotes y tiende
a localizarse en extremidades y cara. La pulpitis digital crónica se presenta como eritema y fisuras
de los pulpejos de los dedos. La pitiriasis alba se manifiesta como placas hipopigmentadas con
descamación fina, más frecuentes en la cara. Además, la dermatitis atópica puede asociarse a
otras enfermedades cutáneas como la alopecia areata, la ictiosis vulgar y el vitíligo. El granuloma
tricofítico de Majocchi es una tiña inflamatoria del folículo piloso que no forma parte del complejo
asociado a la dermatitis atópica.(R4)

216. Femenino de 30 años de


edad, con amenorrea de 8 semanas que acude a urgencias por sangrado menor que regla
de horas de evolución. Se realiza un control por USG con el resultado que se muestra en
la imagen. Usted diagnosticaría:

1. 1. Gestación gemelar bicorial biamniótica.


2. 2. Gestación gemelar monocorial biamniótica.
3. 3. Gestación interrumpida.
4. 4. Gestación monocorial monoamniótica.
Gráfico de respuestas
Comentario

La imagen representa un USG del primer trimestre donde se observan dos embriones en una única
bolsa, por lo tanto sólo puede corresponder a una gestación gemelar monocorial y monoamniótica.
No existe membrana de separación entre ambos fetos, estando los dos en el mismo saco. Cuanta
mas temprana es el USG, el diagnóstico de corionicidad es más acertado.(R4)

217. La paciente del caso clínico anterior, en la semana 32 de gestación, acude a


urgencias por sensación de dinámica uterina esporádica y metrorragia escasa cuantía.
Al realizar monitorización fetal se obtiene un trazado de ambos gemelos con una FCF 170
lpm, ondulatoria baja, sin ascensos pero sin presentar desaceleraciones. Dinámica
uterina irregular 0-1 contracción cada 10 minutos. En la ecografía se visualizan ambos
fetos en presentación cefálica, biometría acorde con amenorrea y volumen de líquido
amniótico normal. ¿Cuál sería la conducta obstétrica más adecuada?:

 
 
 
 
1. 1. Ingreso de la paciente para observación y ver evolución de la dinámica uterina.
2. 2. Realizar amnioscopia para evaluación bienestar fetal.
3. 3. Finalizar la gestación mediante cesárea.
4. 4. Finalizar la gestación mediante parto vaginal.
Gráfico de respuestas
Comentario
En este caso, la monitorización fetal que presentan ambos gemelos corresponde a un trazado poco
reactivo con una taquicardia asociada. Podríamos realizar alguna prueba de valoración del
bienestar fetal para complementar el registro, pero las dos opciones que nos dan (amnioscopia y
prueba de Pose) no podemos elegirlas por tratarse de una gestación de 32 semanas. En los casos
de una gestación monocorial, monoamniótica se recomienda finalización de la gestación de
manera precoz dada la alta tasa de complicaciones asociadas en este tipo de gestación gemelar.
En todos los casos se debe realizar una cesárea con independencia de la estática fetal.(R3)

218. RN presenta al nacimiento hepatomegalia, ictericia, lesiones ampollosas en manos


y pies y rinorrea profusa, respecto a este cuadro todo es verdadero, EXCEPTO.

1. 1. En un 40% provoca muerte fetal o perinatal.


2. 2. La alteración más frecuente en la forma precoz es la hepatomegalia.
3. 3. La FTA-Abs se utiliza como marcador de actividad de la enfermedad.
En el liquido sinovial de la articulación de Clutton no pueden detectarse treponemas
4. 4.
viables.
Gráfico de respuestas
Comentario

Tema muy importante debes dominarlo. En este caso nos presentan un neonato con sífilis
congénita. La manifestación más frecuente de la sífilis congénita precoz (dos primeros años de
vida) es la hepatomegalia. Otros síntomas son lesiones cutaneomucosas, polimorfias con
afectación palmoplantar, rinorrea y afectaciones óseas con lesión perióstica (que producen dolor y
dan lugar a una falsa parálisis llamada paralisis de Parrot).

Se debe diagnosticar y tratar la enfermedad con penicilina antes de la semana 16 para evitar la
sífilis congénita.

No sirven los anticuerpos como marcadores de actividad, sino de presencia de enfermedad, ya


habíamos comnetado este aspecto en el simulacro previo.(R3)

219. Una mujer posmenopáusica fue tratada de un carcinoma mamario infiltrante de 1 cm


mediante tumorectomía y linfadenectomía. Esta última fue negativa. ¿Qué tratamiento
añadiría?

1. 1. Ovariectomía bilateral.
2. 2. Monoquimioterapia con taxol.
3. 3. Radioterapia.
4. 4. Radioterapia y tamoxifeno.
Gráfico de respuestas
Comentario

Sobre esta pregunta, debemos hacer una serie de observaciones. Durante los últimos años, han
cambiado algunos conceptos sobre el cáncer de mama, por lo que podría ser anulada.

 
 
 
 
Ante un tumor tan pequeño, puede plantearse una cirugía conservadora en vez de una
mastectomía radical, con iguales resultados en términos de supervivencia. Pero debe quedar clara
una cosa: para que los resultados sean los mismos, es preciso complementar la tumorectomía con
radioterapia.

No está indicada la quimioterapia, puesto que los ganglios axilares son negativos y no existe
ningún tipo de factor de mal pronóstico. Recuerda que el factor pronóstico más importante en el
cáncer de mama es el número de ganglios afectados, y por ello la indicación más típica de
quimioterapia.

La hormonoterapia, actualmente, se reserva para las pacientes con receptores estrogénicos


positivos. Como en esta paciente no nos dicen nada de esto, la pregunta no está bien construida.

El problema de esta pregunta es que es antigua. Hasta hace poco, se trataba con hormonoterapia
a todas las pacientes postmenopáusicas, independientemente de la positividad de los receptores
estrogénicos, siendo necesario hoy día determinarlos para iniciar el tratamiento con
hormonoterapia.(R4)

220. Los siguientes trastornos se relacionan con la resección de la porción terminal del
íleon, EXCEPTO:

1. 1. Diarrea.
2. 2. Bajas concentraciones séricas de hierro.
3. 3. Depleción de sales biliares.
4. 4. Hipovitaminosis B12.
Gráfico de respuestas
Comentario
La resección del íleon distal condiciona déficit de la absorción de vitamina B12 y, por tanto,
incrementa el riesgo de padecer una anemia megaloblástica. Al mismo tiempo se produce
deplección de sales biliares, ya que se reabsorven a ese nivel. Esta reducción de las sales biliares
puede condicionar cierto grado de esteatorrea, ya que se impide la absorción de la grasa. Además
puede provocar diarrea por otro mecanismo, ya que los ácidos biliares que se pierden por el colon
provocan en éste una auténtica diarrea secretora. Este último tipo de diarrea se trata con
colestiramina, que impide el efecto lesivo de los ácidos biliares en colon. El hierro se absorve en
intestino delgado a nivel de duodeno o yeyuno proximal y no se afecta por las resecciones
ileales.(R2)

221. Mujer embarazada de 36 semanas de gestación, primigesta, es trasladada al hospital


para valoración tras accidente automovilístico, presentando dolor cervicodorsal. Durante
la exploración, la paciente inicia con dolor abdominal intenso, leve sangrado vaginal
oscuro y aumento mantenido del tono uterino. ¿Qué diagnóstico le parece el más
probable?:

1. 1. Desprendimiento prematuro de placenta normoinserta.


2. 2. Amenaza de parto prematuro.
3. 3. Rotura uterina.
4. 4. Rotura de vasa previa.
Gráfico de respuestas
Comentario

 
 
 
 
En esta pregunta nos piden el diagnóstico más probable, teniendo en cuenta el dolor con aumento
del tono uterino y el sangrado vaginal oscuro la opción correcta sería el desprendimiento de
placenta.

Una duda razonable sería la rotura uterina, pero tiene menor probabilidad, ya que no hay ningún
antecedente de cirugía uterina, el sangrado debería ser rojo intenso y no oscuro como en nuestra
paciente, y en la exploración no hay ningún dato que nos lo sugiera.(R1)

222. ¿Cuál de los siguientes antibióticos se considera más eficaz para tratar la vaginosis
bacteriana?

1. 1. Tetraciclina o eritromicina.
2. 2. Metronidazol.
3. 3. Cefalosporinas.
4. 4. Ciprofloxacino.
Gráfico de respuestas
Comentario

Concepto básico que no puede fallar. El tratamiento de elección para la vaginosis bacteriana es el
metronidazol.(R2)

223. El tratamiento de elección del Carbunco es:

1. 1. Eritromicina.
2. 2. Cloxacilina.
3. 3. Aztreonam.
4. 4. Penicilina.
Gráfico de respuestas
Comentario

El carbunco es una infección bacteriana causada por el B. anthracis que se observa con frecuencia
en los herbívoros. En el hombre la forma más frecuente de afectación es cutánea (95%),
provocando una lesión ulcerada necrótica con intenso edema.

El carbunco adquirido por inhalación (enfermedad de los cardadores de lana) presenta como
complicación típica una mediastinitis hemorrágica y una infección generalizada con rápida
agravación y una tasa de mortalidad muy alta.

El carbunco digestivo es muy infrecuente y produce cuadros de gravedad.

El tratamiento de elección para todas sus formas es la penicilina, en forma de bencilpenicilina. Otra
medida en enfermos muy graves es la inmunización pasiva con antitoxina carbuncosa, pero no se
halla disponible en el mercado.(R4)

224. La enteroparasitosis que origina cuadros de mal-absorción y afecta con más


frecuencia en inmunodeficiencias, especialmente en los déficit aislados de IgA, debe
tratarse con:

1. 1. Praziquantel.
2. 2. Bithionol.

 
 
 
 
3. 3. Metronidazol.
4. 4. Niclosamina.
Gráfico de respuestas
Comentario

La infección por la que se nos pregunta es la giardiasis. Su tratamiento de elección es el


metronidazol. Recuerdemos que, en esta infección, no existe fiebre ni sangre en heces (ni siquiera
sangre oculta), al no ser un microorganismo enteroinvasor.(R3)

225. Un varón de 72 años consulta por un cuadro de tres meses de astenia, anorexia y
dolor y rigidez matutina en hombros y caderas que mejora con la actividad. En la
exploración presenta limitación funcional en hombros y caderas con fuerza conservada.
En la analítica destaca anemia normocítica, VSG de 60 mm en la primera hora. El
electromiograma es normal. ¿Cuál es el diagnóstico más probable?:

1. 1. Fibromialgia.
2. 2. Polimialgia reumática.
3. 3. Polimiositis.
4. 4. Artritis reumatoide.
Gráfico de respuestas
Comentario
Es importante que sepas el cuadro clínico de la polimialgia reumática y sobre todo que no lo
confundas con la artritis reumatoide, de la que es uno de los síntomas. La polimialgia reumática
consiste en dolor, rigidez e impotencia funcional de cintura escapular y pelviana (hombros y
caderas). Se ve en pacientes muy ancianos, sobre todo mujeres, y pueden ser muy invalidantes.
Lo típico es la VSG muy elevada y la anemia normocítica. Si además presentara ceguera o
cefalea, entraríamos en el contexto de la arteritis de la arteria temporal, que a diferencia de la mera
polimialgia reumática, requiere altas dosis de corticoides. Recuerda que en la fibromialgia el dolor
es general y se desencadena por presión en los puntos gatillo; que en la polimiositis el dolor
también es proximal (de cinturas) pero el EMG es anormal, propio de miopatía; en la AR las
articulaciones más afectas son las muñecas, metacarpofalángicas, rodillas,? con gran componente
inflamatorio; en el lupus las artralgias y mialgias son lo más frecuente, pero el paciente suele ser
joven y la manifestación más típica es la artropatía de Jaccoud.(R2)

226. Varón de 35 años que padece una ileítis terminal desde hace diez años. En los
últimos 3 años ha sido tratado con mesalamina sin precisar corticoides. No ha sido
intervenido quirúrgicamente por su enfermedad. En el último año presenta episodios
frecuentes de espasmos y ruidos abdominales, náuseas y estreñimiento. No refiere fiebre
ni rectorragia, pero ha perdido 5 kg de peso en ese período. ¿Cuál sería su actitud
diagnóstica?:

Ninguna exploración, sino ensayo terapéutico con prednisona durante un mes y valorar en
1. 1.
función de la respuesta.
2. 2. Endoscopia alta para descartar enfermedad de Crohn gastroduodenal.
3. 3. TAC abdominal en busca de posibles fístulas o abscesos.
4. 4. Tránsito intestinal en busca de signos de estenosis del intestino delgado.
Gráfico de respuestas
Comentario

 
 
 
 
La afectación más frecuente en la enfermedad de Crohn suele ocurrir a nivel del íleon terminal. Si,
por otra parte, se trata de casos de larga evolución (10 años) y nunca ha precisado cirugía, nos
están dando una razón para que pueda haberse formado una estenosis como consecuencia de los
sucesivos brotes que se producen en esta enfermedad. Las manifestaciones clínicas del paciente
son, desde luego, muy compatibles con este diagnóstico: espasmos, ruidos de tripas y
estreñimiento, en ausencia de signos de actividad, como fiebre o malestar general. Por este
motivo, debemos plantearnos una posible estenosis en íleon, que habrá que investigar, como se
explica en la respuesta 4.(R4)

227. Si tenemos un niño con 10% de eosinófilos, que parasitosis puede ser:

1. 1. Strongiloides.
2. 2. Toxocara.
3. 3. Fasciola.
4. 4. Todos los anteriores.
Gráfico de respuestas
Comentario

De la lista que dan, los 3 pueden producir eosinofilia.

Los parásitos que pueden causarla son: Angiostrongyliasis, Ascariasis, Strongyloidiasis,


Trichinosis, larva migrans visceral, Gnathostomiasis, filarias, Schistosoma, Fasciola, Clonorchis,
Paragonimus, Fasciolopsiasis.(R4)

228. Una mujer de 65 años consulta por metrorragias escasas desde hace tres meses. La
exploración ginecológica es normal. En una ecografía transvaginal se observa un útero
de 7 x 3 x 4 cm con un endometrio de 14 mm de espesor y unos ovarios atróficos. La
citología cervicovaginal informa de un frotis atrófico sin otras alteraciones celulares. En
una biopsia endometrial realizada por aspiración con una cánula flexible de tipo Cornier
o Pipelle (microlegrado) se informa de escaso material endometrial de tipo atrófico,
insuficiente para un diagnostico endometrial adecuado. ¿Cuál de las siguientes
indicaciones es la más adecuada?

Tratamiento hemostático con estrógenos + progestágenos, seguido de progestágenos


1. 1.
cíclicos cada mes durante 6 meses.
2. 2. Histeroscopía
Repetición de la biopsia si vuelva a sangrar y en caso contrario repetición de la ecografía a
3. 3.
los 4 – 6 meses.
Completar el estudio con determinación de marcador tumoral CA 125 y otras pruebas de
4. 4.
imagen como RMN o TAC.
Gráfico de respuestas
Comentario

Cuando se sospecha una hiperplasia endometrial / carcinoma endometrial es muy importante llegar
a un diagnóstico preciso. Ya que la biopsia endometrial no fué diagnóstica, el siguiente paso sería
realizar una histeroscopía, Respuesta 2 correcta.(R2)

229. Which is the treatment of choice for nephrogenic diabetes insipidus?

1. 1. Desmopressin.
2. 2. Chlorpropamide.

 
 
 
 
3. 3. Clofibrate.
4. 4. Hydrochlorothiazide.
Gráfico de respuestas
Comentario

El tratamiento de elección de la diabetes insípida nefrogénica es la restricción salina y el uso de


diuréticos tiazídicos. Este tipo de diuréticos producen un aumento de la natriuresis a nivel distal en
la nefrona, por lo que el túbulo contorneado proximal responde reteniendo sodio y
secundariamente agua. Si no existe respuesta a estas medidas, como tratamiento coadyuvante se
puede administrar indometacina que bloquea la síntesis de prostaglandina E2. En los casos de
diabetes insípida, existe un estado relativo de hipovolemia que activa la producción de esta
protaglandina, que a su vez activa la producción de renina y consecuentemente el SRAA,
promoviendo una mayor retención de sodio contraproducente en este tipo de sujetos. Es
importante recordar que la ingesta de agua se debe limitar en los pacientes tratados ante el riesgo
de sobrecarga hídrica. El tratamiento del proceso de base causante de la diabetes insípida
nefrogénica (hipopotasemia, hipercalcemia, hiperaldosteronismo, nefropatía intersticial, tratamiento
con litio,...) mejora también la clínica poliúrica.(R4)

230. Mujer de 62 años que acude a la consulta por presentar dolor en el borde interno del
antebrazo y a nivel del dedo anular y meñique de la mano derecha. También explica
hormigueos, frialdad y pérdida de sensibilidad. ¿Qué tipo de neuropatía por compresión
tiene la paciente?

1. 1. Compresión próximal nervio cubital.


2. 2. Síndrome canal carpiano.
3. 3. Atrapamiento nervio mediano y cubital a nivel de la muñeca.
4. 4. Compresión del nervio radial.
Gráfico de respuestas
Comentario

La afectación del nervio cubital ocasiona parestesias y dolor en mitad cubital de dorso y palma de
la mano, así como en el 5º dedo y mitad cubital del 4º, tal como nos describen en este caso. Sin
embargo, cabe destacar la ausencia de atrofia muscular y signos de afectación motora, lo que
orientaría más a un síndrome compresivo incipiente que a una sección completa del nervio
(respuesta correcta 1).

Desde el punto de vista motor, cuando el cuadro es más avanzado, se aprecia dificultad para
separar y juntar los dedos (paresia de los interóseos), dificultad para la aproximación del pulgar
que se intenta suplir con la con-tracción del flexor largo del pulgar (signo de Froment), separación
permanente del 5º dedo (signo de Wartemberg) e hiperextensión de las articulaciones
metacarpofalángicas con flexión de las interfalángicas de 4º y 5º dedos y abducción del pulgar y
depresión de los espacios interóseos (“garra cubital”). Se ha descrito el fenómeno de la “paradoja
cubital”: cuanto más proximal es la lesión, menos afectada está la mano. Sin embargo, en las
lesiones más distales, existe mayor afectación y deformidad.(R1)

231. Una mujer de 25 años presenta en una citología cervicovaginal de rutina la presencia
de coilocitos. La biopsia cervical demuestra la presencia de NIC de alto grado. Mediante
técnicas de hibridación se pone de manifiesto la existencia de VPH tipos 16-18. ¿Cuál es
el significado y el pronóstico de este hallazgo?

 
 
 
 
Que el genoma vírico se halla integrado en el ADN celular del huésped y que el riesgo de
1. 1.
desarrollo de un carcinoma invasivo es alto.
Que el ADN viral permanece en forma episómica libre en la célula huésped y que el riesgo
2. 2.
de desarrollo de carcinoma infiltrante es muy alto.
Que el ADN viral permanece en forma episómica libre en la célula huésped y que el riesgo
3. 3.
de desarrollo de carcinoma infiltrante es bajo.
4. 4. Que existe ya un carcinoma microinvasor.
Gráfico de respuestas
Comentario

Una pregunta muy difícil, que requiere de conocimientos bastante profundos sobre la biopatología
del VPH.

La mayoría de los virus de bajo riesgo permanecerían en el núcleo de la célula infectada en


situación episómica, es decir, no integrados en el genoma. Sin embargo, aquí nos hablan de los
serotipos 16 y 18, en el contexto de una displasia de alto grado. En estos casos, el VPH ejerce su
actividad oncogénica integrándose en el genoma celular (respuesta 1 correcta).

El mecanismo mejor conocido de inducción neoplásica se produciría a partir de la síntesis de las


proteínas virales E6 y E7. Estas proteínas se ligan a las producidas por dos genes supresores
celulares, que son p53 y Rb, respectivamente, y hacen que se degraden. Éste es uno de los
mecanismos que justifican su potencial oncogénico.(R1)

232. Una paciente de 27 años es intervenida de urgencia por un cuadro de dolor


abdominal en fosa ilíaca derecha. En la cirugía se encuentra una tumoración en ciego y
se practica una hemicolectomía derecha reglada. El estudio histopatológico revela la
existencia de un adenocarcinoma estadio B de Dukes. Su padre está diagnosticado de
un síndrome de Muir Torre. La paciente tiene un hijo varón de 2 años de edad. La
conducta diagnóstico-terapéutica con el hijo debe ser:

1. 1. Colonoscopia a los 10 años de edad y cada 5 años.


2. 2. Colonoscopia a los 23 años de edad y cada 2 - 3 años.
3. 3. Ofrecer la colectomía abdominal total a partir de la mayoría de edad.
Screening genético, analizando los genes hMSH2, hMLH1, hPMS1 y hPMH2 en sangre
4. 4.
periférica.
Gráfico de respuestas
Comentario

El diagnóstico clínico de esta enfermedad se basa en los criterios de Amsterdam: 3 ó más


familiares afectos de cáncer ce colon (al menos uno que sea familiar de primer grado de los otros
2), uno o más casos en la familia de cáncer colorrectal antes de los 50 años y afectar al menos a
dos generaciones consecutivas.

Repasando, esta enfermedad se hereda de forma autosómica dominante, con alta penetrancia. El
gen mutante aparece con más frecuencia en los cromosomas 2 (gen hMSH2) y 3 (hMLH1),
afectando a la reparación del ADN. Sin embargo, se han descrito otras mutaciones, por lo que un
test genético negativo no descartaría la enfermedad. Por ello, se recomiendan colonoscopias
periódicas a los familiares en primer grado de estos pacientes. Comenzarán a realizarse a partir de
los 25 años, o 5 años antes que la del familiar más joven afecto de cáncer de colon. A los
familiares de los pacientes con síndrome de Lynch tipo II, también deben realizárseles
mamografías y exploraciones genitales periódicas, dada su asociación al cáncer de mama y
endometrio, entre otros tumores.(R2)

 
 
 
 
233. What is the most common type of seizure in children?

1. 1. Benign partial epilepsy with fronto-temporal paroxysms.


2. 2. West's syndrome.
3. 3. Juvenile myoclonic epilepsy.
4. 4. Typical absence seizures.
Gráfico de respuestas
Comentario

La epilepsia más frecuente en la infancia, afortunadamente es la epilepsia benigna de la infancia.


El síndrome de West y el síndrome de Lennox-Gastaut son síndromes que aparecen en niños con
patología subyacente cerebral grave (lo más típico es la patología anóxica perinatal). La epilepsia
mioclónica juvenil no se da en la infancia, si no en adultos jóvenes. Las ausencias típicas sí son
frecuentes en la infancia, pero menos que la EBI. La EBI se caracteriza por su carácter benigno y
por no precisar tratamiento.(R1)

234. Mujer de 23 años, sin


antecedentes de interés, nulípara, que acude a revisión ginecológica rutinaria. Refiere
encontrarse bien, aunque presenta leves “molestias” en hipogastrio de forma ocasional
de dos meses de evolución, por las que no ha consultado previamente. La última revisión
ginecológica hace un año fue normal. En la exploración física se observa útero de tamaño
y consistencia normales, en anteversión. El anejo derecho es normal al tacto bimanual y
el anejo izquierdo parece aumentado de tamaño. Se realiza USG transvaginal, donde se
observa útero regular con endometrio lineal, anejo derecho de aspecto normal de 32 mm
y en el anejo izquierdo se observan los hallazgos de la imagen. No hay líquido libre en
Douglas. En relación con el diagnóstico de sospecha, señale la respuesta INCORRECTA:

1. 1. Es el tumor maligno más frecuente dentro de los tumores germinales.


Con frecuencia presenta tejido de las tres hojas embrionarias, siendo posible la presencia
2. 2.
de pelo, cartílago o hueso en su interior.
3. 3. Puede producir alfa-fetoproteína.
4. 4. Por lo general, está constituido por tejidos bien diferenciados.
Gráfico de respuestas
Comentario

 
 
 
 
Esta pregunta sirve como repaso de los tumores germinales del ovario, centrándose principalmente
en las características del teratoma. La respuesta incorrecta es la primera opción, ya que el
teratoma maligno es infrecuente. El tumor germinal maligno más frecuente es el disgerminoma. En
cuanto al teratoma, es cierto que es el tumor ovárico más frecuente en la mujer joven, presenta
tejidos de las tres hojas embrionarias, siendo estos bien diferenciados y dando lugar a la formación
de pelo, cartílago, hueso,… Al igual que el resto de los tumores germinales, puede producir alfa-
fetoproteína.(R1)

235. ¿Cuál de las siguientes afirmaciones respecto al linfoma gástrico es FALSA?:

1. 1. Su pronóstico es peor que el del adenocarcinoma gástrico.


2. 2. La mayor parte de los linfomas gástricos son de tipo no Hodgkinianos.
3. 3. El dato endoscópico más característico es su frecuente extensión al duodeno.
4. 4. La palpación de una masa en abdomen no es un signo clínico infrecuente.
Gráfico de respuestas
Comentario
En los linfomas gástricos, la histología puede ser de bajo grado (normalmente linfomas MALT),
asociados a infección por H.pylori. En estos casos, está indicada la terapia erradicadora previo a
cualquier otro tratamiento, ya que puede ser curativa. Además, su quimiosensibilidad es alta, lo
que les confiere un mejor pronóstico a los linfomas gástricos que a los adenocarcinomas.(R1)

236. Which of the following is the most common reason for removing intrauterine
dispositive?

1. 1. Uterine perforation.
2. 2. Bleeding.
3. 3. Leucorrhea.
4. 4. Pain.
Gráfico de respuestas
Comentario

Según la OMS, la causa más frecuente de la retirada del DIU es la perforación. En el resto de
complicaciones, no suele ser necesario retirarlo. Pero si debes repasar las complicaciones que
pueden tener las portadoras de DIU.(R1)

237. Recién nacido de 33 semanas, con antecedente de ruptura de membranas de 27


horas, presenta a las 6 horas de vida, hipotermia, distress, mala perfusión periférica y
letargia, En la BH destaca la presencia de 2970 leucocitos, con índice infeccioso de 0.32
Señale lo FALSO sobre la sepsis del recién nacido:

1. 1. Las manifestaciones focales predominan en la sepsis de inicio precoz.


2. 2. Los agentes causales mas frecuentes son el estreptococo del grupo B y E coli.
3. 3. La mortalidad es mayor en aquella que aparece antes de la semana de vida.
4. 4. El tratamiento empírico adecuado es ampicilina y gentamicina, durante al menos 7 días.
Gráfico de respuestas
Comentario

Esta es una pregunta difícil. De entrada debemos identificar el caso como una sepsis neonatal y
posteriormente, saber que es una sepsis de inicio precoz (antes de los 3 días).

 
 
 
 
Los microorganismos más comúnmente implicados en la sepsis neonatal difieren entre
instituciones, sin embargo los gram negativos como Klebsiella pneumoniae, Escherichia coli ,
Pseudomonas aeruginosa y Salmonella han sido reportados como importantes agentes etiológicos
de sepsis, sobre todo en la sepsis de presentación temprana.

De los microorganismos gram positivos, el estreptococo del grupo B, Staphylococcus aureus,


estafilococos coagulasa negativo y Listeria monocytogenes son los más comunmente aislados.

Se estima que la mortalidad es mayor en la primera semana de vida, aunque los resultados de
algunos estudios muestran una tendencia similar en la mortalidad.

El tratamiento inicial recomendado de acuerdo a la etiología, se basa en la asociación de


gentamicina/ampicilina.

Las manifestaciones en la sepsis precoz suelen ser inespecíficas, tal y como lo decribe el cuadro
clínico, con lo cual la respuesta falsa es la 1.(R1)

238. Durante el embarazo, NO es cierto que:

1. 1. Suele aumentar el tamaño de la glándula tiroides.


2. 2. La cifra de tiroxina total aumenta.
3. 3. El uso de hormonas tiroideas está contraindicado.
4. 4. Está contraindicado el iodo radioactivo.
Gráfico de respuestas
Comentario

Pregunta sobre las modificaciones gravídicas maternas, muy explicado en las clases. Durante la
gestación, hay un ligero aumento del tamaño del tiroides por hiperplasia e hiperemia (opción 1). El
aumento de estrógenos provoca un aumento en la síntesis de la proteína transportadora de
hormonas tiroideas y aumenta la T4 unida a proteínas y la fracción libre, aumentando así la cifra
global (opción 2). Es obvio que todos los tratamientos con radiaciones ionizantes (opción 4) están
contraindicados en el embarazo. Así, la opción falsa es la 3 porque el tratamiento con hormonas
tiroideas sí está permitido en el embarazo para el tratamiento del hipotiroidismo (patología muy
frecuente en el embarazo).(R3)

239. A 7-day-old fullterm newborn is being evaluated at a routine visit. The boy is receiving
a breastmilk exclusive feeding. The child had a weight of 3600 gr at birth. He makes
around 7 diapers a day and passed meconium at the second day. His weight at the day of
consultation is only 3580 and his mother is worried. What should be done in this case?

1. 1. Reassure the mother and continue breastmilk feeding


2. 2. Withold breastmilk and start with a cow's milk formula
3. 3. Encourage the mother to supplement with water between feedings
4. 4. Intravenous fluid administration and discharge after propperly hydration
Gráfico de respuestas
Comentario
Reassure the mother and continue breastmilk feeding. The first days of life, newborns have a mild
physiological weight loss, as in the case presented. Once we rule out signs of dehydration or other
situation requiring treatment (vomiting, diarrhea), the best approach is to continue to feed the child
with breastfeeding. Physiological weight loss is quickly resolved and children usually exceed their
birthweight around the 10th day after birth.(R1)

 
 
 
 

240. Una mujer de 47 años de edad acude a la consulta refiriendo sequedad vaginal de
varios meses de evolución, con relaciones sexuales dolorosas, a lo que se suma la
reciente aparición de sofocos, así como un estado de irritabilidad que, según ella, se ve
empeorado por las dificultades que presenta para conciliar el sueño. En relación a las
modificaciones endocrinas que caracterizan este período, señale la respuesta
INCORRECTA:

1. 1. Durante la menopausia existen niveles elevados de FSH.


2. 2. Durante la menopausia existen niveles elevados de LH.
Durante la menopausia, los niveles de testosterona de origen ovárico disminuyen
3. 3.
drásticamente.
4. 4. Durante la menopausia, las obesas tienen mayor producción de estrógenos periféricos.
Gráfico de respuestas
Comentario

Pregunta fácil sobre menopausia que podemos sacar por descarte si no sabemos la correcta.

Se sabe que las gonadotropinas aumentan en la menopausia, con mayor aumento de FSH
(opciones 1 y 2). Durante la menopausia, las mujeres obesas tienen mayor producción de
estrógenos periféricos ya que éstos se sintetizan en la grasa, y el estrógeno periférico más
importante es la estrona, que en la menopausia es el estrógeno principal (opción 4).

La opción a elegir por descarte será la 3, pero lo explicaremos a continuación. Durante la


menopausia la testosterona plasmática global disminuye muy poco, habiéndo demostrado un
aumento de testosterona de origen ovárico, ya que la LH estimularía la síntesis de andrógenos en
el estroma ovárico. Esto justifica que, en algunas mujeres ancianas, aparezca mayor vello facial
que en sus años jóvenes.(R3)

241. El flujo de la vena ovárica derecha drena hacia:

1. 1. El plexo pampiniforme.
2. 2. La vena cava inferior.
3. 3. La vena renal derecha.
4. 4. La vena hipogástrica.
Gráfico de respuestas
Comentario

Pregunta sumamente memorística.

El vena ovárica derecha desemboca en la vena cava inferior, mientras que la izquierda lo hace en
la vena renal izquierda. Respuesta correcta 2.(R2)

242. Secundigesta con antecedente de un parto anterior en la semana 36, que consulta
en la semana 32 por percibir contracciones. En la exploración se comprueba un cérvix
permeable al dedo, borrado en 30% y con 3 cm de longitud por USG, la monitorización
cardiotocográfica fetal revela una frecuencia cardíaca de 140 lat/min y una contracción
uterina cada 5 minutos. El test de fibronectina exocervical es negativo. ¿Cuál de las
siguientes afirmaciones es INCORRECTA?

 
 
 
 
Se trata de una amenaza de parto pretérmino establecida que obliga a realizar tocolisis y
1. 1.
maduración pulmonar fetal con betametasona.
El test de fibronectina negativo se asocia a un riesgo de parto pretérmino inferior al 1% en
2. 2.
las 2 semanas siguientes.
Una longitud cervical de 3 cm supone un elevado valor predictivo negativo para el parto
3. 3.
pretérmino.
Se trata de una amenaza de parto pretérmino incierta que requiere nueva evaluación clínica
4. 4.
a los 60-120 minutos.
Gráfico de respuestas
Comentario

Conceptos fundamentales en el ENARM. La amenaza de parto pretérmino se define como la


aparición de una o más contracciones uterinas en diez minutos, con nulas o escasas
modificaciones cervicales, y con rotura o no de membranas, en una gestación de menos de 37
semanas. En el caso clínico que nos ofrecen, la paciente presenta un cérvix de 3 cm de longitud y
fibronectina negativa. Esto indica que, a pesar del antecedente de parto pretérmino y las
contracciones uterinas frecuentes, no es obligada la presencia de amenaza de parto (opción 1
incorrecta). De hecho, si te fijas las opciones 2 y 3 respaldan esta idea. En cuanto a la longitud
cervical, entre las semanas 12-36 son valores normales entre 4+/-1,2 cm. Son las longitudes
inferiores a 3 cm las que suponen riesgo de parto pretérmino (opción 3 verdadera). No obstante, y
dado el antecedente de parto pretérmino en un embarazo anterior y como nos hallamos en una
situación incierta (4 verdadera), se aconseja la administración de esteroides para la maduración
pulmonar fetal por encontrarnos antes de la semana 35 de gestación.(R1)

243. Paciente de 60 años, menopáusica desde los 51, que consulta por metrorragias de 4
meses de evolución y tensión mamaria. No presenta clínica urinaria o digestiva. La
palpación mamaria es algo dolorosa, pero no se aprecia ninguna tumoración ni
adenopatías. El examen ginecológico muestra unos genitales externos normales, cuello
sano macroscópicamente con abundante cantidad de moco a su través, útero normal,
anejo derecho de tamaño normal, mientras que en lado izquierdo se palpa una
tumoración de 5x6 cm. Los laboratorios son normales. El ultrasonido confirma el
diagnóstico de tumoración sólida de ovario izquierdo. El resultado del legrado
endometrial es el de hiperplasia endometrial. ¿Cuál es el diagnóstico más probable?:

1. 1. Tumoración ovárica benigna.


2. 2. Cistoadenocarcinoma seroso.
3. 3. Cistoadenocarcinoma endometrioide.
4. 4. Tumor de la granulosa.
Gráfico de respuestas
Comentario

Fijese en los datos importantes: Mujer menopáusica de 60 años con metrorragias, dolor mamario,
sin tumoración, con genitales externos normales y cuello sano con abundante moco y en ovario
izquierdo una tumoración. ¿Cree que es normal que a esa edad y menopáusica tenga los genitales
normales y moco abundante? La respuesta correcta es un ovario productor de estrógeno como es
el de la granulosa.(R4)

244. A 6-year-old girl is brough to clinic for presenting fever and a diseminated
erythematous edematous rash that began behind the ears and the back of the neck and
then spread all over her body. Blood laboratory test results shows HB 9 g/dl, Leukocytes
2000/mm3, Platelets

 
 
 
 
150,000/mm3. Which of the following has shown to reduce complications in patients with
this kind of infection?

1. 1. Vitamin C.
2. 2. Vitamin D.
3. 3. Vitamin A.
4. 4. Vitamin B12.
Gráfico de respuestas
Comentario
Vitamin A. Measles is a viral infection common in children under eight. The vaccination programs
have lowered the incidence of this disease but it remains very frequent in developing countries.
Vitamin A is important for the proper growing of skin and conjunctival epithelium. Recent clinical
trials have shown that administration of oral high dose of vitamin A daily to children with measles in
areas where vitamin A is deficient reduce the risk of mortality.(R3)

245. Acude a nuestra consulta de ginecología una mujer de 34 años para revisión. No
refiere nada de interés salvo que en los últimos años ha tenido varios compañeros
sexuales, pero que no ha padecido ninguna enfermedad de transmisión sexual. Como
hace tres años de su última citología, se decide repetirla. El resultado de la misma es
informado como presencia de células con displasia leve. ¿Cuál es el siguiente paso para
el despistaje de un posible cáncer de cérvix?:

1. 1. Repetición de la citología tras dos meses.


2. 2. Conización para el estudio anatomo-patológico adecuado.
3. 3. Legrado endocervical.
4. 4. Realización de colposcopía con tinción de Schiller.
Gráfico de respuestas
Comentario

El screening del cáncer de cérvix se hace mediante las citologías cervicales. Cuando una citología
resulta positiva, el siguiente paso es la realización de una colposcopía para confirmar el origen de
las células alteradas. El test de Schiller consiste en la tinción con una solución yodada de lugol
para delimitar las zonas sospechochas ya que no se tiñen y así podemos hacer biopsias dirigidas.
El tratamiento definitivo dependerá del diagnóstico que obtengamos en esas zonas biopsiadas.(R4)

 
 
 
 

246. Acude a su consulta una chica de 24 años, diabética


insulinodependiente, con las placas atróficas en las áreas pretibiales que se muestran en
la imagen. Respecto a esta dermatosis, señale la que considere INCORRECTA:

Las placas rosadas-amarillentas, atróficas, translúcidas y con telangiectasias en el interior


1. 1.
son compatibles con el diagnóstico clínico de necrobiosis lipoídica.
El mantenimiento de la hemoglobina glicosilada dentro de los límites asegura que las
2. 2.
lesiones de necrobiosis lipoídica se reduzcan progresivamente.
En pacientes diabéticos es de especial importancia la exploración de las extremidades
3. 3.
inferiores, ya que gran parte de las dermatosis asociadas a ésta pueden afectarlas.
4. 4. Es frecuente que las placas de necrobiosis lipoídica se ulceren.
Gráfico de respuestas
Comentario

Tal vez no te resulte familiar a primera vista, pero basta la siguiente conjunción de términos:

- Diabetes mellitus.

- Cara anterior de las tibias.

- Placas atróficas con los bordes bien definidos.

Aunque no reconozcas la imagen con total seguridad, los datos son suficientes para intuir que se
trata de una necrobiosis lipoídica, lesión muy vinculada a la diabetes mellitus. La necrobiosis
lipoídica no es la manifestación cutánea más frecuente de la diabetes, sino la dermopatía diabética
(ojo, que es un motivo de fallo habitual).

La respuesta incorrecta es la 2. Por supuesto, en Medicina nada es “nunca” o “siempre”, ni


tampoco “asegura”. Son términos muy radicales que hacen muy difícil sostener cualquier
afirmación. Existe poca o ninguna asociación entre el control glucémico y la aparición de este tipo
de lesiones.(R2)

247. ¿Cuál de los siguientes síntomas EXCLUYE el diagnóstico del síndrome de colon
irritable?

 
 
 
 
1. 1. Estreñimiento.
2. 2. Diarrea.
3. 3. Sangre oculta en heces.
4. 4. Presencia de moco en las heces.
Gráfico de respuestas
Comentario

Pregunta básica sobre el colon irritable.

La característica esencial del síndrome de intestino irritable es el dolor que alivia al defecar, con
alteración en la morfología de las heces y estreñimiento. En ocasiones, puede haber diarrea sin
aumento del volumen líquido como consecuencia del aumento de la frecuencia defecatoria. No
obstante, en general coexiste con estreñimiento. La abundancia de moco en heces también es
típica. Sin embargo, la presencia de sangre en heces, pérdida de peso, anemia o diarrea nocturna
son signos que sugieren organicidad y excluyen este diagnóstico.(R3)

248. ¿En cuál de las siguientes enfermedades es necesario que una bacteria esté
infectada por un bacteriófago para producir una exotoxina patógena para la especie
humana?:

1. 1. Diarrea del viajero.


2. 2. Colitis pseudomembranosa.
3. 3. Difteria.
4. 4. Botulismo.
Gráfico de respuestas
Comentario

La toxina diftérica solo aparece en cepas infectadas por un fago. Es una curiosidad microbiológica
poco importante desde el punto de vista clínico, pero llamativa.(R3)

249. La manifestación más típica en una lesión del tálamo es:

1. 1. Hiperpatía.
2. 2. Tetraplejia.
3. 3. Corea.
4. 4. Trastornos de la memoria.
Gráfico de respuestas
Comentario
Esta pregunta es relevante para el MIR, pues la semiologia en la asignatura de neurología es
importante, ya que conocerla te ayudará a responder tanto preguntas directas como formuladas en
forma de caso clínico. A cerca del tálamo lo que debes saber es que sus lesiones afectan a todas
las sensibilidades del hemicuerpo contralateral, incluidas las de la cara. En ocasiones estas
lesiones talámicas evolucionan para producir un cuadro de dolor o hiperpatía en el hemicuerpo
afecto ( Síndrome de Déjérine- Roussy).(R1)

250. Paciente de 24 años que acude a consulta por ánimo deprimido secundario a su
incapacidad de mantener relaciones afectivas estables. Ha cambiado continuamente de
trabajo por despido, ya que le cuesta aceptar las normas sociales y acatar la autoridad.
Se encuentra irritable y agresivo y constantemente se está metiendo en peleas, robos,
etc. Todas estas conductas se remontan a la infancia, en la que ya presentaba frecuente

 
 
 
 
absentismo escolar, agresividad con los compañeros, crueldad con animales, robos a
sus padres... ¿Cuál es el diagnóstico?

1. 1. Esquizofrenia simple.
2. 2. Psicosis orgánica.
3. 3. Personalidad antisocial.
4. 4. Personalidad esquizoide.
Gráfico de respuestas
Comentario

Una pregunta de cierta dificultad. Analicemos las distintas respuestas:

1.- El cuadro corresponde a un trastorno hipocondríaco. En la hipocondría obsesiva, el paciente es


capaz de criticar su conducta y reconocer lo absurdo de sus temores.

2.- Este cuadro lo ven con más frecuencia los médicos de atención primaria o los médicos de los
servicios de urgencias. El psiquiatra suele ser interconsultado por éstos.

3.- Efectivamente, se precisa el criterio temporal: más de 6 meses de duración.

4.- La prevalencia es igual en ambos sexos.

5.- Lo que debe hacerse es evitar la yatrogenia. La negatividad de las pruebas no convencerá a
este tipo de enfermos.
(R3)

251. En un paciente con metástasis pulmonares y sin afectación hepática por un


carcinoma de origen intestinal. ¿Cuál es la localización más probable del primario?

1. 1. Recto.
2. 2. Sigma.
3. 3. Ángulo esplénico del colon.
4. 4. Colon transverso.
Gráfico de respuestas
Comentario

La clave es: metástasis pulmonares sin afectación hepática. La mayoría de los cánceres digestivos
tienen un drenaje venoso que pertenece al sistema porta, es decir, toda la sangre procedente de
ellos llegará al hígado y después al pulmón. Si se afecta directamente el pulmón y el hígado está
íntegro, habrá que pensar en un órgano con un drenaje venoso distinto. En el caso del recto,
recuerda que las venas hemorroidales inferiores drenan directamente al sistema cava, por lo que
esta sangre evita el paso por el hígado, pudiendo dar metástasis directamente en el pulmón
(respuesta 1 correcta).(R1)

 
 
 
 

252. Supongamos que la biopsia cutánea del paciente


correspondiente a la imagen muestra un granuloma no caseificante y se establece el
diagnóstico de sarcoidosis. ¿Cuál de las siguientes sería la respuesta más apropiada?

Con las características de la lesión cutánea y la radiografía de tórax, el diagnóstico puede


1. 1.
aceptarse.
La elevación del enzima convertidora de angiotensina se considera una prueba poco
2. 2.
sensible pero muy específica para el diagnóstico.
3. 3. La prueba de la tuberculina es positiva en la mayoría de pacientes con sarcoidosis.
4. 4. La eventual hipercalcemia se atribuye a un aumento de actividad de la parathormona.
Gráfico de respuestas
Comentario

El diagnóstico de sarcoidosis se basa en criterios histológicos. Aunque el pulmón es el órgano


biopsiado con mayor frecuencia, en este caso han encontrado granulomas no caseificantes en las
lesiones cutáneas, que también resulta suficiente. Dado que estos granulomas no son
patognomónicos, para hablar de sarcoidosis necesitamos un contexto clínico-radiológico
compatible con este diagnóstico, que en este caso tenemos (eritema nodoso y adenopatías hiliares
bilaterales). Por lo tanto, los datos que nos ofrecen son suficientes como para establecer el
diagnóstico (respuesta 1 correcta) .(R1)

 
 
 
 
253. En el hemograma de un paciente que consulta por debilidad y orinas oscuras se
objetiva una hemoglobina de 7gr/dl. ¿Cúal considera que es el diagnóstico MENOS
probable?.

1. 1. Cirrosis hepática.
2. 2. Hemoglobinuria paraxística nocturna.
3. 3. Deficiencia de glucosa 6-P.deshidrogenasa.
4. 4. Rasgo talasémico beta.
Gráfico de respuestas
Comentario
No os asustéis con esta pregunta, es más fácil de lo que pensáis. Simplemente recordando que los
pacientes con rasgos talasemicos beta (opción 5 correcta) son asintomático, luego es poco
probable que presenten orinas oscuras o debilidad. No olvidéis que tanto la hemoglobinuria
paroxística nocturna como el déficit de G6PD y la esferocitosis hereditaria son anemias hemolíticas
y claramente pueden presentar la clínica de este paciente.(R4)

254. Para establecer una vía aérea permeable se requiere:

1. 1. Cuello hiperextendido.
2. 2. Cuello ligeramente extendido.
3. 3. No es importante posicionarlo.
4. 4. Ninguna de las anteriores.
Gráfico de respuestas
Comentario

Para poder mantener una vía aérea permeable e deberá tener una posición con el cuello
ligeramente extendido o de olfateo. Respuesta 2 correcta.(R2)

255. Un paciente de 38 años acude a Urgencias con dolor centrotorácico de


características anginosas. Por estudios realizados se concluye que padece angina de
Prinzmetal. ¿Qué tratamiento estaría CONTRAINDICADO?

1. 1. Nifedipina.
2. 2. Betabloqueantes.
3. 3. Mononitrato de isosorbide.
4. 4. Antiacoagulantes orales.
Gráfico de respuestas
Comentario

Aunque los betabloqueantes son excelentes fármacos antianginosos, también tienen cierto
carácter vasoconstrictor. Por este motivo, no deben emplearse en la angina de Prinzmetal, ya que
el fenómeno de base es un espasmo coronario, que podría agravarse en caso de utilizar este
tratamiento. Recuerda que, por el mismo motivo, no debemos emplear betabloqueantes en
pacientes con Raynaud, claudicación intermitente o disfunción eréctil.(R2)

256. Indique lo que considera FALSO respecto al prolapso de la válvula mitral:

En la etiología desconocida no es rara la agrupación familiar (herencia autosómica


1. 1.
dominante).

 
 
 
 
2. 2. Puede asociarse a CIA y síndrome de preexcitación.
3. 3. Debe hacerse un seguimiento estrecho por la alta frecuencia de muerte súbita.
4. 4. La "amaurosis fugaz" es una forma característica de presentar embolias periféricas.
Gráfico de respuestas
Comentario
Esta pregunta del Prolapso mitral carece de relevancia. Sobre todo debes saber que la mayoría de
pacientes están asintomáticos, y que suele ser una enfermedad benigna (opción 4 falsa).
Raramente puede asociar complicaciones, como embolismos procedentes de la válvula, y por ello
causar amaurosis fugaz. La etiología parece deberse a una alteración del colágeno tipo III, con
degeneración mixomatosa y acumulación de mucopolisacáridos. Puede tener herencia autonómica
dominante, y se asocia a muchas otras enfermedades, como la CIA o el síndrome de preexcitación
(Wolff- Parkinson- White).(R3)

257. Lactante masculino de 7 meses de vida que acude a urgencias por presentar restos
hemáticos mezclados con las heces en las 5 últimas evacuaciones, sin cambios en la
consistencia ni la frecuencia de las mismas. A la exploración el paciente está activo, bien
hidratado y perfundido y presenta una temperatura de 36.3º C con frecuencia cardíaca de
107 lpm y tensión arterial de 105/65 mmHg. Usted aprecia leve palidez facial, la
auscultación cardiopulmonar es normal, el abdomen es blando y depresible, sin masas
ni megalias. En el tacto rectal sólo se aprecian restos hemáticos y a la inspección anal
no se observa nada anormal. Decide dejarlo en observación, donde el paciente mantiene
buen estado general y hace una toma de lactancia artificial sin incidencias, pero una hora
después de su llegada a urgencias presenta de nuevo otro episodio de rectorragia.
Respecto a la patología que sospecha, señale la afirmación CORRECTA:

Probablemente se trate de una diarrea invasiva por lo que recogería coprocultivo y dejaría
1. 1.
ingresado para observación y sueroterapia intravenosa.
Probablemente se trate de una invaginación intestinal por lo que solicitaría un ultrasonido
2. 2. abdominal urgente y llevaría a cabo una reducción hidrostática si se confirma el
diagnóstico.
Probablemente se trate de una fisura anal por lo que recomendaría una pomada
3. 3.
antiinflamatoria.
Probablemente de trate de una malformación congénita del tubo digestivo cuyo tratamiento
4. 4.
es quirúrgico.
Gráfico de respuestas
Comentario

Este caso clínico describe una rectorragia en un lactante sano sin alteraciones en la exploración
física y sin antecedentes personales de interés. En un caso como este lo primero que debemos
pensar es que el paciente presenta un divertículo de Meckel. Éste constituye la malformación
congénita más frecuente del tubo digestivo. En adultos es asintomático pero en los casos en los
que debuta en la infancia suele hacerlo antes de los 2 años de edad como hemorragia rectal
indolora e intermitente, aunque en algunos casos da lugar a hemorragia oculta en heces y anemia
ferropénica o incluso puede debutar con dolor abdominal. El tratamiento en los casos sintomáticos
es la extirpación quirúrgica. El resto de opciones se pueden descartar. En primer lugar, una diarrea
invasiva alteraría el ritmo y la consistencia de las heces y produciría fiebre. En segundo lugar, una
invaginación intestinal se manifiesta con dolor abdominal de instauración brusca, con crisis de
llanto y encogimiento de piernas además de la rectorragia (“heces en jalea de grosellas”) este dato
es clave y nunca falta. Por último, una fisura anal presentaría alguna lesión en la exploración anal,
la sangre no estaría mezclada con las heces y los padres contarían antecedentes de
estreñimiento.(R4)

 
 
 
 
258. ¿Cuál de las siguientes relaciones entre microorganismo y enfermedad es
INCORRECTA?

1. 1. Arbovirus-dengue.
2. 2. Rickettsias-tifus.
3. 3. Protozoos-esquistosomiasis.
4. 4. Hongos-histoplasmosis.
Gráfico de respuestas
Comentario

Una pregunta de dificultad media, que exige conocer pequeños detalles sobre la clasificación de
ciertos microorganismos.

•   R1: el dengue es un virus perteneciente a los Flaviviridae, como el de la fiebre amarilla,


que a su vez se incluyen dentro de los arbovirus (ARthropod BOrne VIRUS), que son
transmitidos por artrópodos vectores (en su caso, por el mosquito Aedes).
•   R2: la palabra tifus procede del sánscrito y significa estupor. Existen varias enfermedades
que incluyen esta palabra: tifus exantemático, tifus endémico, fiebre tifoidea, tifus de las
malezas, etc. Existen varios tipos de tifus producidos por el género Rickettsia, que son el
tifus endémico o murino (R. typhi) y el tifus epidémico (R. prowazekii). El tifus de las
malezas lo produce Orientia tsutsugamushi, antiguamente conocida como Rickettsia
orientalis.
•   R3: el esquistosoma no es un protozoo, sino un helminto (es decir, un gusano). Por ello, la
respuesta 3 es falsa.
•   R4: Histoplasma capsulatum es un hongo capaz de ocasionar afectación sistémica, con
lesiones pulmonares similares a las que produce la tuberculosis. De hecho,
histológicamente también puede encontrarse necrosis caseosa.

(R3)

259. Una paciente de 65 años, con antecedentes de cáncer de mama, después de varios
años en tratamiento con tamoxifeno, presenta metrorragias. Es estudiada y se
diagnostica un adenocarcinoma de endometrio. La pieza quirúrgica de histerectomía
muestra, tras la estadificación completa y como única focalidad neoplásica, un
adenocarcinoma endometrioide, limitado al endometrio, con aproximadamente un 70%
de patrón de crecimiento sólido. Indíquese el estadio y grado:

1. 1. Ia G2.
2. 2. Ia G3.
3. 3. Ib G3.
4. 4. Ia G1.
Gráfico de respuestas
Comentario

Una masa tumoral única que no afecta al miometrio sólo puede ser estadio Ia. El hecho de que
tenga un 70% de patrón de crecimiento sólido nos dice que el tumor está en su mayoría
indiferenciado, por lo que corresponde clasificarlo como G3 (respuesta 2 correcta).(R2)

260. La presencia de células claves en secreción vaginal es patognomónico de:

1. 1. Vaginitis viral.

 
 
 
 
2. 2. Vaginitis crónica.
3. 3. Vaginosis bacteriana.
4. 4. Vaginosis crónica.
Gráfico de respuestas
Comentario

Tema muy preguntado en el nacional, y no debe olvidar este concepto. Las células clave o "clue-
cells" son típicas de la vaginosis bacteriana causada por Gadnerella Vaginalis.(R3)

261. Neonato de 20 días de vida que presenta, en su domicilio, un episodio de cese de


respiración con cianosis perioral, se encuentra afebril, sin otra clínica acompañante. La
hermana de 20 meses de edad, recibió tratamiento con azitromicina la semana previa, por
un cuadro que la madre refiere de “tos intensa”. Respecto al manejo de este paciente
señale la opción correcta.

1. 1. Alta por ser probablemente un atragantamiento, ya que no tiene clínica de tos.


2. 2. Alta con azitromicina al niño y a toda la familia.
Ingreso con monitorización cardiorrespiratoria, test de VRS y PCR para Bordetella y
3. 3.
tratamiento empírico con azitromicina.
Alta con monitorización al domicilio en espera de resultado del test VRS y PCR para
4. 4.
Bordetella.
Gráfico de respuestas
Comentario

Exponen el caso de un neonato que ha presentado una apnea en su domicilio, sin otra
sintomatología, pero con una hermana que probablemente haya cursado con un cuadro de tos
ferina la semana anterior. Ante la gravedad de una pausa de apnea se debe ingresar a este
paciente con monitorización y realizar exámenes de laboratorio, ante ante la sospecha de tos ferina
o bronquiolitis (aunque no haya comenzado con clínica respiratoria, la primera manifestación
puede ser la apnea) y ya que la PCR para Bordetella tarda unos días, se iniciará tratamiento con
azitromicina hasta llegar los resultados o completar 5 días. Respuesta 3 correcta.(R3)

262. Respecto a la etiología de la enuresis el factor más importante es un:

1. 1. Trastorno congénito del sistema urinario.


2. 2. Trastorno del sueño.
3. 3. Trastorno genético.
4. 4. Alteraciones en el proceso educacional sobre el control de esfínteres.
Gráfico de respuestas
Comentario
Es una pregunta complicada sobre la etiología de la enuresis. Recuerda que es una alteración en
el aprendizaje del control de esfínteres durante el día o noche de forma involuntaria y repetida. Se
diagnostica a partir de los 5 años ( 4 de edad mental). De los actores en la pregunta mencionados
las opciones 1 y 2 pueden ser factores predisponentes para la diuresis diurna sobre todo, lo mismo
que hay parasomnias sobre todo e sonambulismo pero no es factor etiológico. El factor más
importante es el genético ( opción 4) con agrupación familiar y mayor concordancia en
monocigotos que en dicigotos. La opción 5 es la definición de enuresis. Manual CTO 4ª Edición,
Psiquiatría, Tema 9.3.1 Págs. 46(R3)

263. Una de las siguientes expresiones es INCORRECTA para Clostridium:

 
 
 
 
1. 1. Son microorganismos de hábitat endógeno y exógeno.
2. 2. Algunas especies están involucradas en toxiinfecciones alimentarias.
3. 3. Son sensibles a Penicilina G.
4. 4. La cromatografía gaseosa de la muestra biológica no es útil para el diagnóstico.
Gráfico de respuestas
Comentario

Pregunta que se contesta por descarte.

Las bacterias de género Clostridium son responsables de cuadros como el tétanos (C. tetanii), el
botulismo (C. botulinum), colitis pseudomembranosa (C. difficile), etc. Aunque son anaerobios, su
capacidad de formar esporas les permite sobrevivir en medios muy variados y hostiles. Es ese
metabolismo anaerobio el que justifica su olor característico, tanto en cultivos como en heridas
infectadas. En general, son sensibles a penicilina. Es decir, las características que debes conocer
te sirven para descartar la respuesta 4, que es un dato irrelevante para el ENARM.(R4)

264. Puede ser causa de Insuficiencia Renal Aguda en el lactante:

1. 1. Síndrome Urémico Hemolítico.


2. 2. Deshidratación severa.
3. 3. Vómitos.
4. 4. Todos los anteriores.
Gráfico de respuestas
Comentario

Existen múltiples causas de insuficiencia renal aguda... Recuerde que se dividen en prerenales,
intrínsecas y postrenales. En el caso de deshidratación severa y vómitos podrían causar una IRA
prerenal y el SUH una intrínseca, por lo que la respuesta correcta es la 4.

En el ENARM ya no aparecen preguntad de de todas las anteriores y exepto pero aquí funciona de
repaso.(R4)

265. What would be the best treatment approach for this patient?

1. 1. Radical prostatectomy
2. 2. Radical prostatectomy and cystectomy
3. 3. Radiation therapy
4. 4. Chemotherapy
Gráfico de respuestas
Comentario
Radical prostatectomy. Initial treatment for stage T2 prostate cancer is surgical, especially in young
males who are otherwise healthy. Radiation therapy should be reserved for patients with high risk
for surgery. Up to 40% of stages T2b and T2c prove to be stage T3 in final pathological study(R1)

266. ¿Que actitud tomaría ante un paciente ingresado por presentar un infarto de
miocardio y que presenta una taquicardia ventricular sostenida, aunque sin deterioro
hemodinámico?:

1. 1. Cardioversión eléctrica.
2. 2. Lidocaína (1 o 2 dosis de 50 a 100 mg).

 
 
 
 
3. 3. Bretilio.
4. 4. Verapamil iv.
Gráfico de respuestas
Comentario
Las arritmias que pueden ocurrir en la fase aguda del infarto son variadas. La más importante por
su repercusión es la fibrilación ventricular primaria que es la causa más frecuente de mortalidad
antes de llegar al hospital. La fibrilación ventricular secundarias (que es la que aparece pasadas
las primeras 48- 72 horas) es muy grave y a diferencia de la FV primaria afecta gravemente al
pronóstico a largo plazo del paciente. Las taquicardias ventriculares monomórficas sostenidas no
suelen ser muy frecuentes en la fase aguda del IAM. El tratamiento es similar al de las TVMS fuera
de la fase aguda del IAM. Si el paciente está hemodinámicamente estable se puede hacer un
intento de cardioversión farmacológica y en ese caso el antiarrítmico de elección es la lidocaína o
la amiodarona iv. Si no cede, o si el paciente presenta inestabilidad hemodinámica en cualquier
momento, se realizaría cardioversión eléctrica.(R2)

267. ¿Por qué los pacientes en edad infantil diagnosticados de artritis idiopática juvenil
deben someterse a revisiones periódicas oftalmológicas, aunque no presenten signos o
síntomas clínicos de enfermedad ocular?

Porque la artritis idiopática juvenil se asocia con mucha frecuencia a alteraciones


1. 1.
congénitas retinianas.
2. 2. Porque la artritis idiopática juvenil se asocia con frecuencia a uveítis anterior crónica.
Porque los niños con artritis idiopática juvenil deben ser revisados con la misma frecuencia
3. 3.
que un niño con otro tipo de enfermedad infantil.
4. 4. Porque la artritis idiopática juvenil se asocia a subluxación del cristalino.
Gráfico de respuestas
Comentario

La artritis crónica juvenil (ACJ) no es una enfermedad muy importante, pero si se ha preguntado en
el nacioanl. Sobre ella debes aprender únicamente los detalles más rentables. La ACJ,
especialmente en su forma oligoarticular precoz, puede producir uveítis crónica. Recuerda que esta
variante suele asociarse a anticuerpos antinucleares positivos (ANA) y se relaciona con una mayor
frecuencia de DR5. Es importante conocer esta asociación, ya que el pronóstico visual depende de
que el diagnóstico y tratamiento sean lo más precoces posible.

Formas clínicas en la artritis crónica juvenil

 
 
 
 

268. A 33-year-old male is admitted into the psychiatric unit due to agitated behavior. He
was found in the streets assaulting pedestrians and telling them that he is on a special
mission to save the world. His past medical history is unremarkable. He is started on
haloperidol, and 5 days later he is found in his room lying in bed, confused, sweating and
stiff. Vital signs are temperature 39,3ºC, BP 163/95 mm Hg, pulse 130/min and respiratory
rate 33/min. What would be the most appropriate treatment?

1. 1. Biperiden
2. 2. Life support and dantrolene
3. 3. Dantrolene
4. 4. Amantadine
Gráfico de respuestas
Comentario
Life support and dantrolene. Treatment of neuroleptic malignant syndrome (NMS) is based on life
support and it is directed towards controlling the rigidity and hyperthermia and preventing
complications (eg, respiratory failure, rhabdomyolysis, renal failure). Dantrolene, a postsynaptic
muscle relaxant, can also be used. Monitoring and management in an ICU is recommended.(R2)

269. La espongiosis es el hallazgo histológico habitual en una de las siguientes


dermatosis:

1. 1. Eccema.
2. 2. Psoriasis.
3. 3. Herpes Gestationis.
4. 4. Micosis Fungoide.
Gráfico de respuestas
Comentario
Se trata de una pregunta de conocimientos básicos, que no se debe fallar. Existen hallazgos
histológicos típicos que debes conocer. El Herpes simple es un ejemplo típico de Balonización
(respuesta 1 incorrecta), es decir, edema intracelular. La Espongiosis o edema intercelular
intraepidérmico es característico del Eccema (respuesta 2 correcta). El Psoriasis se caracteriza por
una abigarrada histología, en la que predominan la hiperqueratosis (aumento de la capa córnea)
con zonas en las que la longitud de las papilas dérmicas está aumentada (papilomatosis) y zonas

 
 
 
 
con aumento del estrato espinoso (acantosis) y disminución de la granulosa (hipogranulosis focal).
También se puede ver la presencia de núcleos en las células córneas (paraqueratosis). El Herpes
Gestationis es una enfermedad ampollosa autoinmune, por lo que histológicamente veremos una
ampolla subepidérmica debida a la IgG antimembrana basal circulante. Por último, la Micosis
fungoide es un linfoma cutáneo T caracterizado por tener tres fases diferenciadas. En la fase
eccematosa existen lesiones maculosas en tronco, de histología inespecífica; en la fase infiltrativa
aparecen placas eritematosas, de histología diagnóstica (infiltrado dérmico de linfocitos T atípicos
en banda, con núcleos de morfología cerebriforme); en la fase tumoral la clínica se generaliza,
volviéndose de nuevo inespecífica la histología (respuesta 5 falsa).(R1)

270. En un ciclo normal, de 34 días de duración, la ovulación ocurre:

1. 1. Dos semanas antes del flujo menstrual


2. 2. A mitad del ciclo.
3. 3. En el momento de la producción máxima de progesterona.
4. 4. Antes de la producción máxima de estrógenos.
Gráfico de respuestas
Comentario

Pregunta sencilla sobre el ciclo menstrual. La repsuesta correcta es la 1, ya que la única fase que
no varía es la secretora con duración de 14 días.(R1)

271. ¿Cuál es la manifestación clínica más relevante de la fisura anal?:

1. 1. Dolor.
2. 2. Esfuerzo intenso al defecar.
3. 3. Sangre mezclada con heces.
4. 4. Eliminación de pus durante la defecación.
Gráfico de respuestas
Comentario

El síntoma más relevante, e incapacintante de las fisuras anales es el dolor, opción 1 correcta.(R1)

272. Ante la presencia de una masa hepática asintomática descubierta de forma casual
es FALSO:

1. 1. El diagnóstico diferencial entre masa sólida y quística se realizará mediante ultrasonido.


Se debe establecer el diagnóstico diferencial entre hiperplasia nodular focal y el adenoma
2. 2.
hepático.
El tratamiento de elección de la hiperplasia nodular focal es quirúrgico por la posibilidad
3. 3.
de malignidad.
4. 4. La biopsia hepática de la lesión será de utilidad para instaurar una terapia no quirúrgica.
Gráfico de respuestas
Comentario

La hiperplasia nodular focal no es preneoplásica (respuesta 3 falsa), sino un hamartoma. Los


hamartomas son neoplasias benignas compuestas por una mezcla de elementos de tejidos
normales o una proporción anormal de un elemento simple presente en un órgano. Consta de una
masa única, asintomática y estable en su tamaño. El patrón arteriográfico es típico, con aspecto
estrellado.

 
 
 
 
Es imprescindible distinguirlo del adenoma, del hepatocarcinoma bien diferenciado y con la
transformación nodular parcial del hígado sometido a hipertensión portal. Lógicamente, dado que
no es preneoplásico, sólo precisa seguimiento radiológico, extirpando únicamente cuando se
detecta crecimiento o cuando no puede realizarse un diagnóstico diferencial adecuado.(R3)

273. Es típico de la enfermedad de membrana hialina EXCEPTO:

1. 1. Es más frecuente en hijos de madre diabética.


2. 2. Aparece a los pocos minutos de vida.
3. 3. Se caracteriza por dificultad respiratoria que no responde al oxígeno.
4. 4. Rx de tórax: infiltrados granulares con broncograma aéreo e hiperinsuflación pulmonar.
Gráfico de respuestas
Comentario

La enfermedad de membrna hialina es muy importante y la repasaremos constantemente, es más


frecuente en los prematuros y en los hijos de madre diabética. Aparece a los pocos minutos de
vida: desde el punto de vista clínico, destaca el componente de retracción y la tendencia a la
aparición de pausas, con hipoxia importante refractaria al oxígeno indirecto. Su expresión
radiológica engloba: infiltrado retículogranular con broncograma aéreo y escasa aireación del
pulmón (dada la rigidez resultante de la falta de surfactante).(R4)

274. A 5-year-old child is brought to consultation presenting with several crusting lesions
around her mouth and nose. The lesions have a yellowish colour and exudate a thick and
creamy substance. Which is the preferred treatment in this case?

1. 1. Oral ampicillin
2. 2. Intravenous penicillin
3. 3. Topical mupirocin
4. 4. Topical griseofulvine
Gráfico de respuestas
Comentario
Topical mupirocin. The skin lesions described here are typical of impetigo, a common condition in
childhood. It may be due to a staph or strep infection. The treatment may be performed topically if
the skin lesions are not very extensive. The antibiotic of choice is mupirocin, which is active against
gram-positive skin bacteria.(R3)

275. En relación con la sífilis en el embarazo sólo una de las siguientes afirmaciones es
CORRECTA:

1. 1. En embarazadas alérgicas a penicilina el tratamiento de elección son las tetraciclinas.


Las lesiones de sífilis congénita se desarrollan en general antes del cuarto mes de
2. 2.
gestación.
3. 3. La reacción de Jarisch-Herxheimer en el embarazo desencadena el parto prematuro.
Todas las embarazadas deben someterse a una prueba no treponémica en el primer control
4. 4.
del embarazo, incluso en las zonas con escasa prevalencia de sífilis prenatal.
Gráfico de respuestas
Comentario

 
 
 
 
Concepto muy importante del bloque de Ginecología. La transmisión materno-fetal es por vía
transplacentaria, siendo lo más frecuente que se dé en el tercer trimestre. El riesgo de infección
fetal a partir de una madre con sífilis precoz no tratada es del 50-80%.

En cuanto a la clínica, los fetos infectados antes de las 18 semanas no desarrollan signos debido a
su inmunoincompetencia (respuesta 2 incorrecta). Podemos distinguir entre dos tipos de
manifestaciones:

•   Precoces (en los dos primeros años de vida): Pénfigo sifilítico, coriza (rinitis),
hematológicas (trombopenia, anemia hemolítica), Cutáneas maculopapulosas y
condilomas, Óseas (osteocondritis) (palabra mnemotécnica: PRECOz).

•   Tardías (en las dos primeras décadas si no es tratada): óseas (periostitis con
engrosamiento de óseo), dentales (dientes de Hutchinson), alteración del cartílago nasal
(nariz en silla de montar), alteraciones del SNC, queratitis y sordera por afectación del VIII
par.

Para el diagnóstico disponemos de la serología principalmente y el examen en campo oscuro. Ha


de realizarse una prueba no treponémica a todas las embarazadas en el primer control del
embarazo (respuesta 4 correcta).

El tratamiento de elección es la penicilina. En pacientes alérgicos a penicilina se deben realizar


pruebas cutáneas para confirmarla y proceden a una desensibilización, debido a que no existe
alternativa a la penicilina durante el embarazo. Recuerde que a una embarazada no se le deben
dar tetraciclinas (tampoco a niños menores de ocho años) (respuesta 1 incorrecta). Otros
antibióticos que no se le han de dar son los aminoglucósidos, las quinolonas, las sulfamidas (en
tercer trimestre) y el metronidazol.

La reacción de Jarisch-Herxheimer aparece en el 40% de los pacientes tratados por la destrucción


masiva de treponemas, y el 30% desarrollan contracciones regulares que generalmente ceden a
las 24 horas de tratamiento (respuesta 3 incorrecta).(R4)

Infecciones congénitas

 
 
 
 
276. Paciente de 32 años, que hace 7 días tuvo un parto eutócico. Acude por fiebre de 39°
con escalofríos, mastalgia que comenzó en cuadrante superoexterno de la mama derecha
y actualmente es generalizado, eritema, calor local, aumento de la turgencia, palpación
de masa intramamaria y adenopatías axilares derechas. ¿Cuál de las siguientes medidas
le parece menos correcta?:

1. 1. Antiinflamatorio.
2. 2. Antibiótico.
3. 3. Aplicación de calor local.
4. 4. Mantener la lactancia materna.
Gráfico de respuestas
Comentario

La paciente presenta un claro caso de mastitis puerperal en la que el tratamiento lo constituye:


antiinflamatorios y antitérmicos, aplicación de frío local. En mastitis que se encuentren en fase
flemonosa (que no es el caso), previo a un absceso mamario, puede ser útil la aplicación de calor,
con la intención es conseguir que evolucione a absceso. En mastitis unilaterales, en principio se
puede mantener la lactancia materna, pero en las mastitis bilaterales o recidivantes hay que
considerar la supresión de la lactancia. Antibióticos: cloxacilina sódica, amoxicilina + clavulánico,
eritromicina o clindamicina son válidos. En caso de absceso es preciso el drenaje quirúrgico con
incisión en la zona de máxima fluctuación, desbridamiento y colocación de drenaje con toma de
cultivo. Es necesario realizar una biopsia para descartar un carcinoma mamario en caso de curso
tórpido o en caso de duda.(R3)

277. Paciente diabético de 60 años de edad que ha sufrido un infarto de miocardio hace
unos 3 meses. El paciente está asintomático. Un estudio ecocardiográfico reveló una
FEVI del 35%. El colesterol total es de 220 mg/dl y el LDL de 130 mg/dl. ¿Cuál de los
siguientes tratamientos es el MENOS beneficioso para este paciente?:

1. 1. Un betabloqueante.
2. 2. Un IECA.
3. 3. Un antagonista del calcio bradicardizante tipo verapamilo o diltiacem.
4. 4. Un antialdosterónico.
Gráfico de respuestas
Comentario
Dada la disfunción ventricular y cardiopatía isquémica asociada es obligado un betabloqueante,
IECA, antialdosterónico tipo eplerrenona, estatina y antiagregación. Los calcioantagonistas tipo
verapamilo o diltiacem se contraindican cuando existe disfunción ventricular ya que son inotropos
negativos.(R3)

278. A 60-year-old male patient with a history significant for smoking more than 30
cigarettes per day, hypertension and angina pectoris presents to his physician
complaining of periumbilical colicky pain after meals. On further questioning, it is known
that he has lost 7 kg due to reduced food intake. Mark the suspected diagnosis:

1. 1. Acute mesenteric ischemia.


2. 2. Intestinal angina.
3. 3. Ischemic colitis.
4. 4. Angiodysplasia of the colon.
Gráfico de respuestas

 
 
 
 
Comentario
El paciente que previamente ya presenta factores de riesgo cardiovascular presenta el dolor típico
de un cuadro de angina intestinal que aparece en el período postpandrial cuando se deriva gran
cantidad de flujo sanguíneo al sistema circulatorio intestinal. Estos pacientes presentan
disminución de peso como consecuencia de que el paciente tiene miedo de comer ya que ello le
ocasiona dolor. Un cuadro parecido es la isquemia mesentérica aguda que se caracteriza por un
cuadro brusco de dolor abdominal de curso agudo y alta mortalidad.(R2)

279. A 70-year-old woman is admitted to a coronary unit for anterior myocardial infarction.
She receives treatment with tissue plasminogen activator (t-PA) 2 hours after the
aforementioned treatment, the patient reports severe retrosternal pain. Elevated ST-
segment is seen in leads V2, V3 and V4. Which of the following is the most appropriate
next step in management?

1. 1. Troponin levels.
2. 2. Coronary angiography.
3. 3. Ventilation/perfusion scintigraphy.
4. 4. Chest X-ray.
Gráfico de respuestas
Comentario

Tema muy importante, que seguramente aparecera una pregunta en el ENARM. Se trata de un
infarto agudo de miocardio tratado con trombolíticos. A las dos horas, sufre otro episodio, con
reascenso del ST en V2, V3 y V4. Esto sugiere que se ha producido una reoclusión.

En este caso, lo que habría que hacer para diagnosticarlo es una angiografía coronaria, pudiendo
solucionar al mismo tiempo el problema mediante una angioplastia del vaso, ya que los
trombolíticos no han sido eficaces (recuerda que la angioplastia se considera más eficaz que la
fibrinolisis).(R2)

280. Una mujer de 32 años acude por presentar ictericia cutaneomucosa, astenia y
anorexia en las últimas dos semanas. En los exámenes de laboratorio se evidenció
elevación importante de las transaminasas, GOT 2.390 y GPT 2.543. Hace seis meses
realizó un viaje a la India en primavera. No refiere haber recibido transfusiones ni
consumo de drogas por vía parenteral. A la exploración se objetiva un tatuaje en el glúteo
derecho que le realizó hace 9 semanas un amigo mientras bebían grandes cantidades de
Martini con limón en una fiesta. La serología viral fue la siguiente: Hbs Ag negativo, anti-
Hbc positivo, IgM anti-Hbc negativo, ELISA de VHC negativo, IgG de VHA positivo. ¿Qué
afirmación de las siguientes es correcta?

1. 1. La paciente presenta una infección por VHB.


2. 2. Nos encontramos ante una hepatitis alcohólica.
3. 3. Presenta una hepatitis aguda VHA.
4. 4. No se ha descartado la infección por VHC.
Gráfico de respuestas
Comentario

Aunque la serología para VHC sea negativa, eso no significa que la infección esté descartada, ya
que el período ventana puede prolongarse durante meses, y esta mujer tuvo un factor de riesgo

 
 
 
 
para la transmisión de esta entidad hace pocas semanas (tatuaje por aficionado). Sin embargo, el
resto de las respuestas tienen algunos datos en contra.

R1: La paciente NO presenta infección por VHB. Date cuenta de que el HBsAg es negativo.

R2: La hepatitis alcohólica se caracteriza por GOT>GPT y cifras de transaminasas muy inferiores.

R3: La paciente es inmune al VHA, puesto que presenta IgG positiva frente a este virus. En caso
de tratarse de IgM, entonces sí habría que pensar en una hepatitis A.(R4)

281. Señale lo FALSO en relación con las crisis epilépticas de aparición posterior a un
TCE:

Las crisis inmediatas al TCE (primeras horas) se relacionan por lo general con mal
1. 1.
pronóstico.
2. 2. Las crisis precoces postraumáticas (primera semana) se deben tratar.
La aparición de crisis durante las primeras semanas tras el TCE implican un mayor riesgo
3. 3.
de crisis tardías.
Las crisis tardías (a partir de tres meses) son más frecuentes en pacientes adultos que en
4. 4.
niños.
Gráfico de respuestas
Comentario

Las crisis que aparecen en las primeras horas tras el TCE (inmediatas) no se asocian con mayor
riesgo de presentar epilepsia posterior (respuesta 1 falsa).

Las crisis que aparecen en la primera semana (precoces) sí conllevan un mayor riesgo de epilepsia
a largo plazo, y por tanto se debe utilizar tratamiento anticomicial. Además del momento de
aparición de las crisis, el riesgo de epilepsia se correlaciona con la intensidad del TCE y sus
características (heridas abiertas, hundimiento craneal...)(R1)

282. Recién nacido de 24 h de vida con peso de 1,700 gr y que presenta irritabilidad,
apnea, arritmias cardíacas y convulsiones. ¿Cuál de los transtornos metabólicos sería el
causal de dicha sintomatología?

1. 1. Hipercalcemia.
2. 2. Hipocalcemia.
3. 3. Hiperglicemia.
4. 4. Hipoglicemia.
Gráfico de respuestas
Comentario

La respuesta correcta es la 2. Podría confundirse con la 4, pero sería muy extraño que una
hipoglucemia causara arritmias cardiacas.(R2)

 
 
 
 

283. Mujer de 82 años con melenas de 2


días de evolución. Como únicos antecedentes refería artrosis, hipertensión arterial y
fibrilación auricular crónica, estando en tratamiento con enalapril y acenocumarol. En la
exploración destaca cierta palidez de piel y mucosas, taquicardia a 105 lpm y TA de 90/35
mmHg. En los exámenes de laboratorio destacaba una Hb de 5.6 g/dl, leucocitos 15,300
con fórmula normal, plaquetas 300,000, glucosa 88, urea 158 mg/dl, creatinina 1.3 mg/dl,
AST 12, ALT 17, GGT 14, bilirrubina 0.6 mg/dl, fosfatasa alcalina 68, sodio 136, potasio
4.1. Se realizó una gastroscopía informada como normal y una colonoscopía con restos
hemáticos frescos desde íleon terminal hasta recto, pero sin aparentes lesiones en
mucosa. Con la sospecha de hemorragia de origen oscuro, se realizó cápsula
endoscópica con la imagen. ¿Cuál sería la actitud más adecuada en este momento de
ingreso en Urgencias?:

1. 1. Perfusión de líquidos presores.


2. 2. Realización de resonancia magnética.
3. 3. Dieta líquida.
4. 4. Avisar a cirugía para evaluación.
Gráfico de respuestas
Comentario

Una sospecha de “hemorragia de origen oscuro”, con una foto también bastante oscura. No debe
preocuparle si en la imagen no ve prácticamente nada, porque en realidad no la necesita. Tienes
que aprender a distinguir qué fotografías son imprescindibles y cuáles son accesorias (o meros
distractores, como en este caso).

Para la primera pregunta de esta serie, sea cual sea el diagnóstico, lo más importante es
garantizar la estabilidad hemodinámica de la paciente. Observe que la presión arterial está en
90/35 mmHg, por lo que lo prioritario será mejorar estos parámetros, previamente a la realización
de cualquier prueba diagnóstica (respuesta 1 correcta).(R1)

284. En relación al caso anterior, ¿cuál es su principal diagnóstico de sospecha?:

1. 1. Úlcera duodenal asociada a consumo de acenocumarol.


2. 2. Enfermedad celíaca.

 
 
 
 
3. 3. Angiodisplasia.
4. 4. Síndrome de Boerhaave.
Gráfico de respuestas
Comentario

El diagnóstico de sospecha, a pesar de que en la foto se ve muy poco, es bastante más simple de
lo que puede pensar a primera vista:

- La colonoscopía demuestra restos hemáticos frescos desde íleon hasta recto, mientras que en la
gastroscopía no se ha visto nada. Por lo tanto, se descarta la respuesta 1.

- La enfermedad celíaca puede producir un síndrome anémico crónico (por ejemplo, si afecta a la
absorción del hierro), pero difícilmente comienza por encima de los 80 años, y tampoco justifica la
presencia de sangre fresca a nivel cólico.

- Las respuesta 4 es una complicación del vómito en pacientes alcohólicos. El síndrome de


Boerhaave es una rotura esofágica y el de Mallory-Weiss consiste en hematemesis secundaria a
laceraciones longitudinales en la mucosa del esófago, lo que tiene muy poco que ver con la clínica
descrita.

Nos queda la angiodisplasia, que es una enfermedad muy propia de ancianos. Recuerde que es la
causa más frecuente de hemorragia digestiva baja recidivante en el anciano, y la segunda más
frecuente de hemorragia (globalmente considerada) después del sangrado diverticular. Teniendo
en cuenta la ubicación del sangrado (sangre fresca desde la zona del íleon hasta el recto) y la
edad de la paciente, se trata de la respuesta más lógica. Posiblemente, no se ve en la
colonoscopia, puesto que la sangre, si es abundante, puede ocultar los vasos
angiodisplásicos.(R3)

285. La acidosis tubular renal (ATR) proximal o tipo II consiste en un defecto en la


reabsorción proximal del bicarbonato filtrado, lo que causa un aumento del bicarbonato
que llega a la nefrona distal y finalmente a la orina (bicarbonaturia). En relación con este
proceso, señale la afirmación FALSA:

1. 1. La bicarbonaturia está presente sin importar la concentración sérica de HCO3-.


2. 2. La bicarbonaturia disminuye a medida que los hace la concentración sérica de HCO3-.
La bicarbonaturia desaparece cuando la concentración sérica de HCO3- es inferior a 16
3. 3.
mmol/L.
4. 4. La bicarbonaturia se acompaña de aumento de la excreción urinaria de potasio.
Gráfico de respuestas
Comentario

Se trata de una pregunta difícil, pues debe dominar la fisiología renal para responderla con
seguridad.

En la ATR tipo II o proximal, existe una alteración en la reabsorción de bicarbonato a nivel del
túbulo contorneado proximal, produciéndose su pérdida por la orina incluso con niveles normales
de bicarbonato en sangre. A medida que disminuye el bicarbonato plasmático, la carga filtrada cae
a un nivel en el que el túbulo defectuoso puede reabsorber. Por ello, si desciende por debajo de
determinada concentración en plasma, desaparecería la bicarbonaturia.(R1)

 
 
 
 
286. Un paciente con infección por VIH y 100 linfocitos T CD4/mm3 consulta por disfagia
y fiebre de tres días de evolución. En la exploración únicamente destaca la presencia de
leucoplasia oral peluda y muguet oral. ¿Cuál de estos tratamientos empíricos sería más
adecuado para esta paciente?

1. 1. Aciclovir.
2. 2. Ganciclovir.
3. 3. Metronidazol.
4. 4. Fluconazol.
Gráfico de respuestas
Comentario

Se trata de un caso de disfagia secundaria a candidiasis esofágica. Debes sospechar esta entidad
en pacientes con SIDA evolucionado que presentan odinofagia y disfagia de corta evolución,
acompañada o no de fiebre, sobre todo si además presentan muguet oral. Su tratamiento de
primera elección es una monodosis de 750 mg de fluconazol.(R4)

287. Un paciente con carcinoma de pulmón de células escamosas inoperable desarrolla


una hipercalcemia de 14 mg/dl. Se inicia una perfusión con suero salino y, una vez bien
hidratado, furosemida, con lo que horas más tarde la calcemia es de 11,5 mg/dl. ¿Cuál es
el siguiente paso terapéutico más apropiado?

1. 1. Corticoides.
2. 2. Nitrato de galio.
3. 3. Plicamicina.
4. 4. Zoledronato intravenoso.
Gráfico de respuestas
Comentario

La hipercalcemia es la urgencia metabólica más común en oncología, con una incidencia global del
15-20%, dándose con mayor frecuencia en el mieloma, cáncer de mama y carcinoma epidermoide
de pulmón, como en este caso.

El principal factor causante es la mayor liberación de calcio por el hueso. El riñón también tiene un
papel importante al aumentar la reabsorción tubular de calcio. Otro mediador esencial es la PrPTH
(proteína relacionada con la parathormona, que es lo que principalmente explica la hipercalcemia
en este paciente).

En este paciente, las medidas iniciales (hidratación, furosemida) han sido suficientes para disminuir
considerablemente la calcemia, de 14 a 11,5 mg/dl. No obstante, dado que la causa persiste
(péptidos PTH-like), la resorción ósea continuará siendo muy elevada, lo que nos obliga a hacer
algo que antagonice a este factor.

De las opciones que nos presentan, lo lógico sería optar por un bifosfonato, cuyo mecanismo de
acción consiste precisamente en disminuir la resorción ósea. Entre las opciones 3 y 5, lo lógico es
optar por el zoledronato intravenoso, no por el alendronato, que se administra por vía oral y, por
consiguiente, tardaría más en alcanzar el efecto deseado.(R4)

 
 
 
 

288. Posteriormente,
tras el posible diagnóstico, se convence al paciente para ingreso y realización de
endoscopia y ecoendoscopia (ver imagen). ¿Qué sospecharía en este enfermo y qué
técnica se indicaría después?

Carcinoma epidermoide con metástasis. Ya que la ecoendoscopia me permite ver la


1. 1. infiltración del tumor y adenopatías, solicitaría un TAC para descartar metástasis a
distancia.
2. 2. Realizaría un PET-TAC de inicio.
3. 3. Seguramente sea una recidiva del divertículo de Zenker.
4. 4. Adenocarcinoma esófago. Realizaría un TAC de extensión.
Gráfico de respuestas
Comentario

Siendo coherentes con lo que hemos dicho de la pregunta anterior, la sospecha diagnóstica sería
la misma: carcinoma epidermoide con metástasis (de ahí el dolor óseo). Para realizar un estadiaje
correcto, debemos realizar una TC, que pondrá de manifiesto las adenopatías tumorales y la
afectación a distancia, en caso de existir.

Observe que, mediante ecoendoscopía, podemos intuir la infiltración de la pared esofágica. No


importa si no es capaz de interpretar correctamente la mitad derecha de la imagen. Esta técnica
nos permite valorar el grado de infiltración (aunque, lógicamente, no de forma tan precisa como el
estudio anatomopatológico).(R1)

289. Ante una mujer de 23 años que consulta por ciclos largos con baches amenorreicos
de hasta 3 meses, acné facial y aumento del vello, la prueba complementaria que mejor
nos orientará el diagnóstico será:

1. 1. Ultrasonido transvaginal.
2. 2. Test de estrógeno-progestágenos.
3. 3. Determinación plasmática de FSH y LH.
4. 4. Determinación plasmática de estradiol y progesterona.
Gráfico de respuestas
Comentario

La duda se encuentra entre la respuesta 1 (ultrasonido transvaginal) y la 3 (FSH y LH). Si bien


ambas son utilizadas para el diagnóstico, junto con la clínica, debemos fijarnos en que se nos pide
aquella prueba que mejor lo orienta.

 
 
 
 
El hallazgo de quistes en los ovarios no es sinónimo de anovulación, en cambio, un cociente
LH/FSH > 2.5 es muy característico del SOP. Además, en los exámenes previos se ha hecho
especial hincapié en el aspecto hormonal. El ultrasonido tiene un papel confirmatorio una vez que
se ha solicitado un estudio hormonal (respuesta 3 correcta).

El resto de opciones tampoco serían ciertas porque pueden aportar datos para el diagnóstico, pero
tampoco son las mejores para orientarlo.

La prolactina debe ser solicitada para descartar una hiperprolactinemia, así como tendríamos que
descartar otras causas secundarias (hiperplasia suprarrenal, Cushing).

El test de estrógenos-progestágenos alterado nos hablaría de una alteración genital (estenosis,


sinequias) que cursaría con amenorrea secundaria, pero sin hiperandrogenismo; y la
determinación del estradiol y la progesterona pueden aparecer alteradas (especialmente útil la PG
en segunda fase del ciclo, que está ausente), pero no son diagnósticas.(R3)

290. En un neonato a término sin antecedentes pre y perinatales de importancia, que a


las 6 horas de nacido tiene un flujo urinario de 0.8 mL/kg/h. Usted pensaría que:

1. 1. El flujo urinario está aumentado.


2. 2. El flujo urinario está normal para el tiempo de nacido.
3. 3. El flujo urinario está disminuido.
4. 4. El flujo urinario revela insuficiencia renal.
Gráfico de respuestas
Comentario

El 90% de los niños orinan en las primeras 24 horas de vida y el 99% en los primeros 2 días de
vida. Si a las 72 horas de vida no se ha presentado diuresis, esta indicado iniciar con estudios
nefrourológicos. Las causas mas frecuentes a considerar son deshidratación, falla renal intrínseca
y obstrucción del tracto urinario. La osmolalidad urinaria máxima en el RNPT es de 600 mOsm/kg.
La diuresis mínima sería de 22ml/kg/día (más o menos l ml/kg/hr).(R2)

291. Un paciente de 65 años acude a su revisión anual al servicio de Neumología.


Presenta una historia prolongada de disnea de esfuerzo con tos mínima. De hábito
asténico, no presenta cianosis, su frecuencia respiratoria es de 19 rpm y, a la
auscultación, lo único destacado es una espiración prolongada y una disminución de los
ruidos pulmonares. En la placa de tórax existe un aumento de la transparencia
retroesternal, aplanamiento de los diafragmas, ausencia de vascularización periférica y
un típico "corazón en gota". ¿Qué resultado NO esperaría encontrar en las pruebas
complementarias de esta revisión?:

1. 1. PaCO2 de 40 mmHg.
2. 2. Hematocrito de 53%.
3. 3. Aumento del volumen residual en la espirometría.
4. 4. Disminución de la capacidad de difusión.
Gráfico de respuestas
Comentario
La diferenciación entre las características de un paciente EPOC tipo enfisema y tipo bronquitis
crónica es un tema que debes dominar para tu examen MIR ya que las preguntas que a él hacen
referencia son muy frecuentes. Lo primero es identificar el caso clínico, y ver en él como hay
muchas características que nos apuntan a un paciente enfisematoso: predominio de la disnea

 
 
 
 
sobre la tos, hábito asténico, ausencia de cianosis, ruidos pulmonares disminuidos y RX con
hiperinsuflación y corazón pequeño. Ahora tenemos que ver entre las opciones cuál es menos
propia del enfisema. Sabemos que sus alteraciones gasométricas son menos graves que en la
bronquitis crónica por lo que no debemos dudar de las opciones 1 y 2. Tampoco deben admitir
dudas la opción 4, ya que el aumento del VR es típico de las enfermedades obstructivas, y la 5, ya
que es el descenso del DLCO una característica típica del enfisema y que lo diferencia dela
bronquitis. La opción falsa es la 3, ya que un aumento del hematocrito se da mas en pacientes tipo
bronquitis crónica, con una pO2 crónica menor a la dada y con cianosis.(R2)

292. Un paciente de 62 años acude a su consulta porque ha notado últimamente


disminución de la potencia del chorro de la orina con goteo postmiccional y para el inicio
de la micción. Refiere cierta urgencia miccional que no se acompaña de incontinencia y
que se levanta una vez todas las noches a orinar. El paciente realiza un cuestionario IPSS
en la consulta y obtiene una calificación de 9, lo que califica su sintomatología de
moderada. En una ecografía abdominal se objetiva un volumen prostático en torno a
21cc. Usted le propondrá como tratamiento:

1. 1. Iniciar un alfa-bloqueante.
2. 2. Iniciar terapia combinada de alfa-bloqueante más finasteride.
3. 3. Adenomectomía abierta.
4. 4. Conducta expectante.
Gráfico de respuestas
Comentario
Nos encontramos ante un paciente con sintomatología moderada y próstata de pequeño tamaño,
por lo que no hay duda respecto al inicio del tratamiento con un alfa- bloqueante.(R1)

293. La angiodisplasia del colon es:

Una malformación congénita que afecta a los plexos venosos de todo el colon y que es una
1. 1.
causa infrecuente de hemorragia digestiva en lactantes.
Una malformación congénita de la arteria colónica derecha que origina una hipoplasia del
2. 2.
hemicolon derecho y cuadros obstructivos intestinales en la infancia.
Una alteración de la muscular de las arterias mesentéricas y colónicas en mujeres jóvenes
3. 3.
que origina cuadros de colitis isquémica.
Una lesión degenerativa que consiste en dilatación y distorsión de vasos de pared fina en
4. 4.
colon derecho y que producen hemorragia digestiva baja en ancianos.
Gráfico de respuestas
Comentario

La angiodisplasia de colon es una lesión degenerativa que consiste en dilatación y distorsión de


vasos de pared fina en colon derecho y que producen hemorragia digestiva baja en ancianos.
Debe sospecharse con antecedentes de estenosis aórtica. El diagnóstico se realiza con mediante
una colonoscopia completa, pudiendo realizarse tratamiento con laser-argón para "quemar" las
lesiones y evitar los episodios de sangrado. Otra opción de tratamiento es la administración de
estrógenos- progestágenos que interviene en la reparación vascular. En caso de mala respuesta a
tratamiento, se indica hemicolectomía derecha.(R4)

294. El coriocarcinoma es un tumor constituido por:

 
 
 
 
1. 1. Elementos del sincitio y citotrofoblasto.
2. 2. Degeneración hidrópica de la vellosidad corial.
3. 3. Hiperplasia del sincitiotrofoblasto.
4. 4. Trofoblasto de origen materno.
Gráfico de respuestas
Comentario

Esta pregunta sobre el coriocarcinoma, tumor muy raro, no tiene especial importancia.
Simplemente debe recordar que se desarrolla a partir de las células germinales. La mayoría
aparecen combinados con otros tipos tumores germinales. Son idénticos a los placentarios pero
mucho menos frecuentes, y al igual que éste elabora gonadotropina corionica. Son muy malignos y
mucho más resistentes a la quimioterapia que sus homólogos placentarios. Histológicamente
consiste en proliferación anormal de elementos trofoblásticos: sincitio y citotrofoblasto, que invade
vasos y metastatiza. La degeneración hidrópica de la vellosidad corial se da en la enfermedad
trofoblástica porque la ausencia de linfáticos hace que se acumule liquido en el espacio
extracelular.(R1)

295. Se define policitemia en el RN, cuando el Hto central es igual o superior a:

1. 1. 35%.
2. 2. 60%.
3. 3. 65%.
4. 4. 70%.
Gráfico de respuestas
Comentario

La policitemia neonatal se define a través de la presencia de un hematocrito central mayor del


65%. No sirve para definirlo por ser un parámetro menos preciso el hematocrito capilar. Desde el
punto de vista clínico, recuerde que su expresión engloba: irritabilidad, ictericia, hipocalcemia,
acrocianosis, insuficiencia cardíaca y predisposición al desarrollo de trombosis.(R3)

296. Lactante de 6 meses de edad, que es llevada a urgencias por presentar desde hace
2 días fiebre de 38.2 °C. Antecedentes no importantes. Exploración física normal. ¿Qué
haría?

BH, EGO, hemocultivo, si son negativos dar de alta con ceftriaxona IM y cita al día
1. 1.
siguiente.
2. 2. BH, EGO y sin son negativos alta y cita al dia siguiente.
BH, EGO, hemocultivos, punción lumbar y si son negativos dar de alta sin antibioticos y
3. 3.
cita al dia siguiente.
4. 4. Antipireticos, signos de alarma y observación en 24 a 48 horas.
Gráfico de respuestas
Comentario

Ante la ausencia de antecedentes personales de interés o de algún signo de alarma y con una E.F.
normal se opta por una actitud conservadora (antipiréticos) y observación unos días. Respuesta 4
correcta.(R4)

297. Marque la respuesta CORRECTA respecto al síndrome nefrótico en el niño:

 
 
 
 
Es característica la presencia de fusión pedicelar, que podemos ver con la técnica de
1. 1.
inmunofluorescencia indirecta.
Los cuerpos de Cruz de Malta corresponden a la presencia de lípidos en orina vistos a
2. 2.
través de microscopio de luz polarizada.
Se trata de un síndrome nefrótico florido, con elevado riesgo de complicaciones, entre ellas
3. 3.
el riesgo de sufrir una trombosis de vena renal.
Uno de los factores de riesgo típico de sufrir un síndrome nefrótico por GN de cambios
4. 4.
mínimos es haber padecido sarampión.
Gráfico de respuestas
Comentario

Es una pregunta sencilla si se lee con atención. Aunque la fusión pedicelar es característica del SN
por GNCM, sólo puede verse utilizando la microscopía electrónica (respuesta 1 incorrecta).

El SN del niño, 85% en relación con GNCM, es a expensas de proteinuria selectiva y por tanto es
raro ver complicaciones como la TVR (más frecuente en la membranosa, respuesta 3 incorrecta).

La repuesta a corticoides es excelente en monoterapia con buena respuesta inicial en más del 90%
de los pacientes.

En ocasiones el SN en el niño remite tras sufrir una infección intercurrente por sarampión.

Es típico en la GNCM el encontrar cuerpos de Cruz de Malta. Respuesta 2 correcta.(R2)

298. ¿Cuál de las siguientes situaciones NO es contraindicación absoluta para la


anticoncepción hormonal combinada?

1. 1. Mujer de 30 años con antecedente de colestasis intrahepática gestacional.


2. 2. Mujer de 32 años, diabética con nefropatía.
3. 3. Mujer de 27 años con litiasis biliar.
4. 4. Mujer de 41 años, fumadora.
Gráfico de respuestas
Comentario

Las contraindicaciones absolutas para el uso de anticonceptivos orales son:

- Fumardoras > 35 años o no fumadoras > 40años; TVP o TEP; Cirugía mayor; Alteraciones de la
coagulación, HTA MAL controlada; DM CON afectación vascular; Vasculopatía inflamatoria;
Cardiopatías graves; Alteración hepática importante (hepatitis A no es contraindicación absoluta),
Porfiria aguda intermitente, Clestasis intrahepática, Cáncer de mama u otros tumores
hormonodependientes; Sangrado genital no filiado.(R3)

299. A 20-month-old previously healthy child presents with a 3-day-history of fever,


headache, vomiting, and sore throat. Neurological examination reveals diminished
consciousness and nuchal rigidity. An erythematous and purpuric rash is present at
physical examination. The cerebrospinal fluid (CSF) is turbid with10,000 leukocytes/mm3.
What is the most likely causative agent related to this case report?

1. 1. Neisseria meningitidis
2. 2. Echovirus
3. 3. M. Tuberculosis

 
 
 
 
4. 4. Listeria monocytogenes
Gráfico de respuestas
Comentario
Neisseria Meningitidis. Markedly increased leukocyte count in CSF indicates a bacterial meningitis,
the presence of a petechial rash suggests Neisseria Meningitidis as the causative agent.(R1)

300. En el uso de corticoides en la rotura prematura de membranas, antes de las 34


semanas de gestación, las siguientes afirmaciones son ciertas menos una. Indique cuál:

1. 1. Aumenta la mortalidad perinatal por infección.


2. 2. Disminuye la aparición del distrés respiratorio del RN.
3. 3. Disminuye el riesgo de enterocolitis necrosante.
4. 4. Acelera la maduración pulmonar.
Gráfico de respuestas
Comentario

Pregunta difícil sobre los efectos de los corticoides. La que no debe plantear ningún tipo de duda
es la respuesta 4, ya que se indican para acelerar la síntesis de surfactante pulmonar. Además,
reducen la incidencia de hemorragia de la matriz germinal, de enterocolitis necrosante, de
persistencia de conducto arterioso y de neumotórax.

El resto de las opciones no deben preocuparle.

La opción 1 es falsa, porque los corticoides únicamente aumentan el riesgo de infección cuando se
emplean crónicamente, produciendo en tal caso inmunodepresión celular. Pero el uso puntual de
corticoides, aunque sea una dosis relativamente alta, carece de este efecto.(R1)

301. Un hombre de 16 años tiene cólico abdominal, inapetencia y evacuaciones


reblandecidas, presentes durante los últimos dos días. El examen abdominal presenta
hipersensibilidad leve en el cuadrante inferior derecho. La temperatura, el dia de ayer
37.2ºC, y 37.8ºC el día de hoy. Los signos radiográficos siguientes son compatibles con
el diagnóstico de apendicitis, EXCEPTO:

Presencia de un fecalito en el cuadrante inferior derecho de la radiografía abdominal


1. 1.
simple.
2. 2. Apéndice lleno de gas en la radiografía abdominal sencilla.
Engrosamiento de la porción terminal de íleon y un fino hilillo de bario en el enema de
3. 3.
bario.
4. 4. Efecto de masa en el borde inferior del ciego, con el enema de bario.
Gráfico de respuestas
Comentario

Las manifestaciones que afecten al apéndice o al ciego, lógicamente, podrán estar en relación con
una apendicitis aguda. Lo mismo sucede con la respuesta 1, ya que un fecalito podría ser el
causante de este cuadro. Sin embargo, la respuesta 4 no es coherente con este diagnóstico, sino
con una ileítis. Podríamos encontrar una imagen como ésta en la enfermedad de Crohn.

Recuerde que, a pesar de las alteraciones radiológicas mostradas en esta pregunta, la radiografía
abdominal más frecuente en la apendicitis aguda es una placa de abdomen normal.(R3)

 
 
 
 
302. Mujer de 30 años, gestante de 32 semanas que acude a su consulta refiriendo prurito
generalizado, más intenso por la noche, y que no presenta otras lesiones dermatológicas
salvo las producidas por el rascado. Señale lo FALSO en cuanto a la patología que
sospecha:

1. 1. No es recurrente en futuras gestaciones.


2. 2. El tratamiento consiste en administrar ácido ursodesoxicólico y colestiramina.
3. 3. El prurito es generalizado, incluyendo la distribución palmoplantar.
El pronóstico materno es favorable, pero el fetal puede estar comprometido, con
4. 4.
mortalidad del 5%.
Gráfico de respuestas
Comentario

La patología sospechada es la colestasis intrahepática gestacional, que típicamente puede recurrir


en futuros embarazos, incluso con tratamiento anticonceptivo de tipo hormonal, por lo que es de
hecho una contraindicación de dicho tratamiento.(R1)

303. La articulación más afectada por la artropatía por depósito de pirofosfato cálcico es:

1. 1. Rodilla.
2. 2. Tobillo.
3. 3. Sacroiliaca.
4. 4. Codo.
Gráfico de respuestas
Comentario
Las artritis por microcristales de pirofosfato cálcico son las segundas en importancia en este
capítulo. Se ha preguntado en el MIR su localización predilecta, y esta es la rodilla. Le siguen la
muñeca (ligamento triangular del carpo) y el tobillo. Así, pues, el caso clínico típico es "mujer
anciana con inflamación de rodilla, líquido articular inflamatorio y condrocalcinosis radiológica", o
"joven con hemocromatosis,? con condrocalcinosis de rodilla". La mayoría se asocian a edad
avanzada, pero recordar los factores que predisponen en la juventud: HPP, hipotiroidismo,
hemocromatosis, hipofosfatasia, hipomagnesemia, gota tofácea crónica y ocronosis. Los cristales
son romboidales con birrefringencia débilmente +.(R1)

304. Un niño de 5 años es llevado a consulta por fiebre y tos de 10 días de evolución, Dos
semanas antes había tenido sarampión. En el examen físico se le encuentra en regular
estado general, febril (39°C), polipnea, escasos subcrepitantes en base derecha y
disminución del murmullo vesicular en dicha base, además de verse en la piel del tórax
y del abdomen unas máculas hiperpigmentadas de bordes imprecisos, La forma de
acercarse al diagnóstico incluye básicamente:

1. 1. BH y hemocultivo.
2. 2. Radiografía de tórax y BH.
3. 3. Antecedentes patológicos familiares.
4. 4. Ninguna de las anteriores.
Gráfico de respuestas
Comentario

Exsten distintas complicaciones del sarampión entre las que destacan:

 
 
 
 
Pulmonars: broncoenumonía, laringotraqueobronquitis, o bronquiolitis, otitis media.

Neurológicas: encefalitis, encefalomielitis diseminada aguda, panencefalitis esclerosante


subaguda, oculares.

Gastrointestinales y cardiacas.

Por lo que la forma de llegar al diagnóstico es con una rx de tórax y estudios de laboratorio. La
respuesta correcta es la número 2.(R2)

305. Mujer de 25 años que consulta por dolor intenso en fosa ilíaca izquierda. En la
anamnesis destaca una amenorrea de 9 semanas. A la exploración destaca un útero
aumentado de tamaño, aunque menor que amenorrea, dolor a la movilización del cérvix,
sangrado vaginal escaso y oscuro y anexo izquierdo discretamente aumentado de
tamaño. El ultrasonido demuestra un útero vacío. La TA es de 80/50 mmHg y la paciente
da impresión de gravedad. La actitud más CORRECTA sería:

1. 1. Salpingostomía.
2. 2. Laparotomía urgente.
3. 3. Culdocentesis.
4. 4. Metotrexate vía oral.
Gráfico de respuestas
Comentario

Este tipo de preguntas generalemte no falla su presencia cada año en el ENARM. En el tratamiento
del embarazo ectópico tenemos tres opciones:

- Observación. Se podrá utilizar cuando la paciente cumpla criterios de tratamiento conservador y


además la localización del ectópico sea tubárico.

- Metotrexate. La paciente deberá reunir los siguientes criterios: tamaño inferior a 4 cm, cifras
bajas de HCG y ausencia de sangre o líquido en la cavidad abdominal.

- Cirugía. Se realizará cuando no cumpla criterios de tratamiento conservador o si éste ha


fracasado. Dentro de las opciones quirúrgicas, en el caso de ectópico roto con choque
hipovolémico, como el que nos presentan en la pregunta, se debe realizar directamente una
laparotomía (respuesta2).(R2)

306. En la exploración clínica de un síndrome compartimental puro encontraremos todos


estos datos EXCEPTO uno:

1. 1. Dolores muy intensos.


2. 2. Edema importante del miembro.
3. 3. Trastornos motores.
4. 4. Ausencia de pulsos distales.
Gráfico de respuestas
Comentario
El síndrome compartimental es un aumento de la presión osteofascial por encima de la presión de
perfusión capilar, de forma que se produce un cuadro de isquemia, con las 5 "p" características
(pain,- dolor, pailor- palidez, parestesias, paralisis y "púlseless" que mas bien e el caso del

 
 
 
 
compartimental debe denominarse "pulselessness" que quiere decir que el pulso puede ser débil,
pero no desaparece, puesto que la presión compartimental aumenta, pero no hasta los 70- 80
mmHg, presión del pulso (Presión arterial diastólica).(R4)

307. Un pastor de 48 años de edad presenta una sintomatología consistente en astenia,


anorexia, dolor de espalda, pérdida de peso. En la exploración presenta esplenomegalia,
algunas adenopatías y dolor a la palpación en la región lumbosacra. Ha comenzado hace
pocos días con dolor testicular y molestias al orinar. Sobre la epidemiología de la
enfermedad que más probablemente padece este individuo, señale la cierta:

1. 1. El contagio interhumano es relativamente frecuente.


2. 2. La vía digestiva es la más frecuente de transmisión.
3. 3. En el litoral, la incidencia es más frecuente que en el interior.
La mucosa genital es importante como vía de transmisión en animales, pero no en el
4. 4.
hombre.
Gráfico de respuestas
Comentario

La fiebre de Malta o brucelosis produce un cuadro muy variable, por eso es conocida como “la
enfermedad de mil caras”. En cualquier caso, lo típico es que comience con un cuadro febril (fiebre
ondulante), acompañado de síntomas sistémicos, artralgias y hepatoesplenomegalia. Más
adelante, puede manifestarse como osteomielitis vertebral, con predilección por la región lumbar, o
como sacroileítis. Vamos a repasar algunas ideas sobre la brucelosis:

- Es típica la transmisión al ser humano a través de lácteos y derivado, o bien por contacto
directo.

- El medio de cultivo para Brucella es el de Ruiz- Castañeda. No obstante, el diagnóstico se


alcanza casi siempre por serología (Rosa de Bengala, entre otras pruebas posibles).

- El tratamiento se realiza con doxiciclina + gentamicina, y debe ser bastante prolongado.

- La mejor profilaxis es la vacunación del ganado y la pasteurización de la leche y sus derivados.

Esta pregunta es bastante difícil. Incluso sabiendo esto, puedes fallarla con bastante probabilidad.
Veamos por qué las diferentes opciones son falsas:

- R1. El contagio interhumano es posible, pero muy raro.

- R2. La vía más frecuente de transmisión es el contacto directo, ya que la mayor parte de los
casos se ven en veterinarios, ganaderos o gente expuesta de alguna manera a esta bacteria.

- R3. Es más frecuente en poblaciones interiores y más rara en zonas costeras.

- R4. El queso manchego es un queso curado, por lo que sería muy difícil que transmitiese esta
infección. Es el queso fresco el que implica más riesgo.
(R4)

 
 
 
 
308. El tapón de meconio puede ser un signo precoz de:

1. 1. Estenosis pilórica.
2. 2. Megacolon agangliónico congénito.
3. 3. Atresia esofágica.
4. 4. Galactosemia.
Gráfico de respuestas
Comentario

Concepto importante. Los trastornos meconiales se deben a diversas entidades cuya expresión
común es la no eliminación de meconio en las primeras 48 horas de vida extrauterina. Entre las
entidades que predisponen a estos trastornos tenemos: fibrosis quística, prematuridad, hijos de
madres consumidoras de drogas por vía parenteral, tratamiento de preeclampasia materna con
sulfato de magnesio, megacolon agangliónico (respuesta 2) y colon izquierdo hipoplásico.(R2)

309. Un paciente de 65 años, sin antecedentes de interés, ha sido operado hace 3 días de
un melanoma localizado en la pierna izquierda, habiéndosele realizado extirpación de la
lesión tumoral con márgenes adecuados, tanto en superficie como en profundidad, y
linfadenectomía inguinal izquierda. Actualmente presenta tumefacción indolora de la
herida inguinal, sin signos inflamatorios. Señale cuál, de entre los siguientes, le parece
el diagnóstico más probable:

1. 1. Hernia inguinal postoperatoria.


2. 2. Infección de la herida.
3. 3. Seroma infectado.
4. 4. Seroma.
Gráfico de respuestas
Comentario

Pregunta repetida, que no puede fallar.

Una colección de líquido de aspecto seroso en el interior de una herida quirúrgica es lo que se
conoce como seroma. En principio, esta colección es estéril. No obstante, cuando drena al exterior
debemos considerar que esta colección (o mejor dicho, la cavidad residual) estará contaminada.
Es una complicación quirírgica relativamente común. Es más frecuente cirugías con disecciones
subcutáneas extensas, linfadenectomías axilares o inguinales, como en el caso que nos ocupa.

Si se tratase de una herida o seroma infectados, habría dolor y signos inflamatorios. Lo


característico del seroma es la tumefacción indolora sin inflamación circundante, como dicen en el
enunciado.(R4)

310. ¿Cuál es la etiología más frecuente de convulsiones en un recién nacido a termino?

1. 1. Hipocalcemia.
2. 2. Meningitis.
3. 3. Encefalopatía hipóxico isquémica.
4. 4. Ninguna de las anteriores.
Gráfico de respuestas
Comentario

 
 
 
 
Existen múltiples causas de convulsiones, pero la más importante es la encefalopatía hipóxico-
isquémica, por lo que la respuesta correcta es la número 3.

Recuerde las causas de convulsiones en RN ordenadas de mayor a menor frecuencia:

1. Encefalopatía hipóxico-isquémica.

2. Hemorragia intracraneal: intraventircular, intracerebral, subdural, subaracnoidea.

3. Infección SNC: meningitis, encefalitis, intrauterina.

4. Metabólico: hipoglicemia, hipocalcemia, hipomagnesemia.

5. Anormalidades cromosómicas.

6. Anormalidades congénitas cerebrales.

7. Desordenes neurodegenerativos.

8. Errores innatos del metabolismo.

9. Convulsiones neonatales benignas.

10. Convulsiones neonatales familiares benignas.

11. Abstinencia o intoxicación.

(R3)

311. A 28-year-old male presents to his physician with a painful ulcerative lesion on his
penis for the past 3 days. He also complains of dysuria and mild fever. He admist to recent
sexual activity with a prostitute. Tzanck preparation of his lesion reveals multi-nucleated
giant cells. He undergoes testing for other STDs and the HIV serology by ELISA is
positive. Which of the following is the most appropriate confirmatory test for HIV
infection?

1. 1. HIV viral load.


2. 2. Absolute CD4 count.
3. 3. P 24 antigen assay.
4. 4. HIV serology by western blot.
Gráfico de respuestas
Comentario
HIV serology by western blot. The preferred screening test for HIV infection is ELISA. Its sensitivity
is greater than 99.9%. Remember that, given a positive ELISA test can be a false-positive, a
confirmatory test is needed. Usually, WESTERN BLOT is the chosen test as a confirmatory test for
HIV infection.(R4)

 
 
 
 
312. ¿Cuál es, de los siguientes, el más útil y eficaz factor de pronóstico en el cáncer de
mama?

1. 1. Negatividad de los receptores de estrógenos en las células tumorales.


2. 2. Presencia de focos de componente intraductal en puntos distantes al tumor.
3. 3. Invasión de ganglios linfáticos.
4. 4. Presencia de mutación en el gen BRCA1.
Gráfico de respuestas
Comentario

Pregunta muy importante que debe recordar para simpre.

El factor pronóstico más importante en el cáncer de mama es la invasión linfática regional


(respuesta 3 correcta). Por este motivo, ante un cáncer de mama infiltrante, se realiza
linfadenectomía en todos los casos. Si se encuentra afectación metastásica de los ganglios, sería
indicación de añadir quimioterapia.

El resto de las opciones pueden tener más o menos importancia, pero nunca tanta como el número
de ganglios afectados.(R3)

313. Which of the following statements is false regarding Chagas disease?

1. 1. It is endemic in certain areas of Central and South America.


2. 2. Trypanosoma cruzi is the causative agent.
3. 3. Arrhythmias and conduction disturbances are common findings.
4. 4. Antiparasitic drugs are effective in chronic cardiac damage due to this disease.
Gráfico de respuestas
Comentario

La afectación cardíaca en forma crónica de la enfermedad de Chagas se caracteriza por trastornos


del ritmo cardíaco (tanto bradiarritmias como taquicardias) y afectación miocárdica, que suele
remedar en sus signos y síntomas a la miocardiopatía dilatada, con un tratamiento farmacológico
similar a esta situación, siendo poco eficaz el uso de anti-parasitarios.(R4)

314. En la enfermedad de membrana hialina o síndrome de distrés respiratorio neonatal,


existe un déficit de surfactante que se ha relacionado con todas, EXCEPTO una de las
siguientes situaciones:

1. 1. Prematuridad.
2. 2. Hijo de madre diabética.
3. 3. Hipoxemia.
4. 4. Hijos de madres adictas a heroína.
Gráfico de respuestas
Comentario

Es muy importante que domine el diagnóstico diferencial entre las distintas entidades responsables
de distress respiratorio en el recién nacido en el ENARM es de lo más preguntado cada año.

 
 
 
 
La enfermedad de membrana hialina se debe a un déficit de surfactante, que no alcanza
plenamente la superficie pulmonar hasta la semana 34-35. Este déficit produce un aumento de la
tensión superficial y una tendencia de los pulmones al colapso.

Hay distintas situaciones que pueden variar la síntesis de surfactante:

- Aumenta con las situaciones de estrés: desprendimiento placentario, la rotura precoz de


membrana, consumo de opiáceos HTA y vasculopatía renal materna.

- Disminuye en el hydrops fetal, trastorno metabólico de la síntesis de lecitina y diabetes


materna.

Afecta sobre todo a prematuros. Su frecuencia también es mayor en los hijos de madre diabética y
en los embarazos múltiples.(R4)

315. The image


shows the preoperative chest X-ray films of a 45-year-old woman who is currently visiting
the anesthesiologist for a routine preoperative evaluation. She is asymptomatic and will
undergo surgery for Morton's neuroma. On questioning, it is learned that the patient used
to smoke 25 cigarettes per day until 3 years ago. Which of the following options best
describes the imaging findings?

1. 1. Central bronchopulmonary tumor. Affected lymph nodes.


2. 2. Loculated interlobar pleural effusion.
3. 3. Anterior mediastinal mass.
4. 4. Central bronchiectasis.
Gráfico de respuestas
Comentario
En la radiografía presentada se observa una masa pulmonar hiliar derecha, con adenopatías
mediastínicas, por lo que la respuesta correcta es la 2.(R1)

 
 
 
 
316. Which of the following is the most likely diagnosis?

1. 1. Allergic bronchopulmonary aspergillosis.


2. 2. Invasive thymoma.
3. 3. Small-cell lung cancer.
4. 4. Pleural mesothelioma.
Gráfico de respuestas
Comentario
El carcinoma broncogénico indiferenciado de células pequeñas, o microcítico, se caracteriza por
ser preferentemente central (junto con el carcinoma epidermoide) y por presentar, con mucha
frecuencia, adenopatías mediastínicas e hiliares (a diferencia del epidermoide).(R3)

317. Maniobra para diagnosticar la luxación congénita de cadera:

1. 1. Barlow y Ortolani.
2. 2. Lennoy y Bernoulli.
3. 3. Lasege y Macshin.
4. 4. Kouer y Candre.
Gráfico de respuestas
Comentario

Pregunta básica y bastante preguntada en el ENARM.

Barlow: cadere dentro del acetábulo pero al presionar muslos en ángulo de 90 grados se luxa,
cadera luxable

Ortolani: al hacer abducción de la cadera se nota resalte de entrada de la cabeza del fémur dentro
del acetábulo, cadera luxada reducible.(R1)

318. Paciente femenino de 17 años acude por amenorrea primaria. Tras el tratamiento con
progesterona no se obtiene menstruación, y tampoco tras el tratamiento con
anovulatorios. La etiología debe localizarse en:

1. 1. Hipotálamo.
2. 2. Hipófisis.
3. 3. Utero, cérvix o vagina.
4. 4. Eje hipotálamo-hipófiso-gonadal.
Gráfico de respuestas
Comentario

Es muy importante para el ENARM que conozcan el diagnóstico diferencial de las amenorreas.

Ante una amenorrea lo primero a realizar es un test de embarazo. Después se determinará TSH y
PRL. Si estas son normales, se procederá a administrar progesterona durante 5 días. Si existiera
regla el diagnóstico sería de anovulación; si por el contrario, ésta no existiese, se darán estrógenos
y progestágenos combinados.

Si tras estos últimos persistiera la inexistencia de menstruación, la alteración indicaría un origen


genital: útero (síndrome de Asherman) en el cervix (estenosis) o en la vagina (estenosis).(R3)

 
 
 
 
319. ¿Cuál es el diagnóstico más probable en una mujer de 64 años que presenta un
déficit visual que mejora al mirar por un agujero estenopeico y transiluminación pupilar
normal?:

1. 1. Catarata.
2. 2. Defecto de refracción.
3. 3. Maculopatía.
4. 4. Ambliopía.
Gráfico de respuestas
Comentario
Pregunta sencilla al referirse a un tema que ya ha sido motivo de pregunta MIR. Debemos saber
que típicamente son los defectos de refracción los que mejoran al estrechar el haz de rayos
procedentes del objeto, ya que así se disminuyen las posibles interferencias; este es el efecto que
se consigue con un agujero estenopeico. Del resto de opciones sabemos que ni la maculopatía ni
la lesión de las vías ópticas ni la ambliopía mejoran con la visión a través del agujero estenopeico.
Debemos recordar que la catarata mejora la visión con el agujero estenopeico, en zonas poco
iluminadas o al usar midriáticos, pero no tendría una transiluminación pupilar normal.(R2)

320. A su consulta acude un niño de 6 años con fiebre y exantema muy pruriginoso, con
elementos vesiculosos y pustulosos de predominio en tronco, que le ha aparecido el día
anterior. Cuando usted aconseja a la madre aislar al niño del resto de los niños y de
embarazadas, ella asegura estar embarazada de 2 meses y no sabe si ha pasado la
varicela. ¿Qué actitud es la más apropiada?

No hacer nada, dado que lo más probable es que la madre haya pasado la varicela en la
1. 1.
infancia.
2. 2. Ante la duda, aislar al niño de su madre hasta que todas las lesiones estén en fase de costra.
3. 3. Realizar serología inmediata a la madre, y administrar Ig anti-varicela-zoster.
4. 4. Tratar a la madre con aciclovir.
Gráfico de respuestas
Comentario

Pregunta sumamente importante para el ENARM. Ante un caso clínico como éste, debemos hacer
las siguientes consideraciones.

- La gestante está en elprimer trimestre y no tenemos evidencia de que haya padecido la varicela.
En caso de que, efectivamente, nunca la haya pasado, podría contagiarse en este momento.

- El máximo riesgo de afectación fetal es el primer trimestre de la gestación, por ser el período
durante el que se produce la organogénesis fetal. Realmente, no se trata de un riesgo muy alto, un
1-2%, pero es el período donde es más probable.

Aunque la eficacia de la gammaglobulina para la prevención de la varicela congénita es dudosa


hasta cierto punto, la respuesta más adecuada es la opción 3. El balance riesgo-beneficio aconseja
su uso y, por otra parte, el haber tomado esta medida, también servirá para protegernos ante
posibles reclamaciones legales en caso de que la madre se contagie y el recién nacido tuviese
algún tipo de malformación atribuible a la varicela.(R3)

321. A 4-year-old child is brought to the emergency department presenting with bloody
diapers and high fever. The child seems sick and sleepy. Blood laboratory test results

 
 
 
 
are: HB 5 g/dL, Platelets 30000/mm3, Creatinine 3 mg/dL, Total bilirubin 6 mg/dL. What is
the most common complication of this condition?

1. 1. Renal failure
2. 2. Liver failure
3. 3. Aseptic meningitis
4. 4. Respiratory infections
Gráfico de respuestas
Comentario
Renal failure. Hemolytic-uremic syndrome is characterized by a triad that includes hemolytic
anemia, kidney failure and thrombocytopenia. Cases during childhood are begin with an invasive
diarrhea caused by Shigella, Campylobacter or most importantly E. coli O157:H7. Renal failure is
often severe and may require renal replacement therapy.(R1)

322. Niño de 10 meses de edad, presenta un cuadro de un día de evolución, caracterizado


por fiebre alta, irritabilidad y vómitos. Antecedentes de otitis media aguda tratada hace
dos semanas. Al examen: mal estado general, fontanela anterior pulsátil, rigidez de nuca.
El LCR es purulento, con presencia de cocos grampositivos, con una sensibilidad
intermedia a penicilina. ¿Cuál es el fármaco más adecuado para el tratamiento?

1. 1. Ampicilina.
2. 2. Amikacina.
3. 3. Vancomicina.
4. 4. Ceftriaxona.
Gráfico de respuestas
Comentario

Con los datos clínicos que le dan es obvio que se trate de una meningitis aguda. Si se detectan
diplococos grampositivos pensará inmediatamente en Streptococcus pneumoniae. Es importante
que se de cuenta que mencionan una sensibilidad intermedia.

En neumococos sensibles a penicilina podría utilizar penicilina o cefalosporina de tercera


generación como cefuroxima o ceftriaxona, pero al reportarse sensibilidad intermedia debería
administrar ceftriaxona en este caso. Cuando le mencionan resistente a cefalosporinas entonces el
fármaco a elegir es la vancomicina. En pacientes en los que sospeche infección por Lysteria es
importante utilizar ampicilina.

Otro punto importante es el siguiente: no le están pidiendo la terapia empírica que iniciaría, sino el
tratamiento ajustado al patógeno aislado. Por cierto no debe olvidar la administración de
esteroide!!.(R4)

323. La mejor prueba en la detección selectiva (screening) en casos de hipercortisolismo


es la determinación de cortisol tras:

1. 1. Supresión nocturna con 1 mg de dexametasona (test de Nuggent).


2. 2. Supresión con dexametasona 0,5 mg/6 horas durante dos días.
3. 3. Test de estimulación prolongada con ACTH.
4. 4. Supresión con dexametasona 2 mg/6 horas durante dos días.
Gráfico de respuestas
Comentario

 
 
 
 
Pregunta muy sencilla conociendo los distintos test empleados en el Síndrome de Cushing. Dentro
de los test de screenig podemos realizar la determinación del cortisol libre urinario o bien el test de
1 mg DXT (test de Nuggent). El test de elección para el diagnóstico bioquímico es el de 2 mg DXT
(0.5/6h/48h o test de Liddle débil). Para el diagnóstico diferencial se emplea el test de 8 mg DXT
(2mg/6h/48h o test de Liddle fuerte). Por último, señalar que el test de estimulación rápida con
ACTH es la prueba diagnóstica de elección en la insuficiencia suprarrenal primaria.(R1)

324. El daño precoz de quiasma óptico por tumor hipofisario causa:

1. 1. Dolor ocular.
2. 2. Pérdida de visión en campos temporales.
3. 3. Papiledema.
4. 4. Atrofia de pupila.
Gráfico de respuestas
Comentario

Una de las preguntas más clásicas en el examen MIR. La afectación de la porción central del
quiasma, normalmente debida a los adenomas de hipófisis, produce una lesión de las fibras
procedentes de la mitad nasal de ambas retinas, donde se proyectan las imágenes procedentes de
los campos temporales. El defecto resultante es, por lo tanto, una hemianopsia heterónima
bitemporal (respuesta 2 correcta).(R2)

325. Niña de 5 años,


diagnosticada de asma y síndrome de lóbulo medio. Presenta como antecedentes distrés
respiratorio neonatal y rinorrea. Tos crónica húmeda, infiltrados radiológicos
cambiantes, pobre respuesta al tratamiento antiasmático. Rinitis crónica. Otitis de
repetición con otorrea persistente a pesar de los drenajes. Se le realiza la siguiente
radiografía. ¿Qué le sugiere?:

1. 1. Neumonía en lóbulo superior izquierdo.


2. 2. Atrapamiento aéreo sugestivo de episodio de asma agudo.
3. 3. Discinesia ciliar primaria.
4. 4. Neumonía intersticial.
Gráfico de respuestas

 
 
 
 
Comentario

La historia de rinitis, otitis y neumonías podría ser sugestiva de una inmunodeficiencia humoral,
pero esta posibilidad ni siquiera se plantea entre las opciones.

Sin embargo, nos ofrecen una respuesta que podría justificar todas estas infecciones respiratorias
persistentes: la discinesia ciliar primaria, que alteraría la capacidad del epitelio respiratorio para
evacuar mecánicamente las secreciones mucosas impregnadas en ciertos microorganismos. De
ahí que la respuesta correcta sea la 3.

Ojo con la radiografía, porque no está al revés... Se trata de un caso de dextrocardia. ¿Adivine
ahora el diagnóstico exacto del paciente? Se trata de un síndrome de Kartagener.(R3)

326. ¿Cuál es el pronóstico de la enfermedad que padece?:

1. 1. Tras tratamiento antibiótico, rápida resolución, sin secuelas.


2. 2. Infecciones respiratorias recurrentes y probable evolución a bronquiectasias.
3. 3. Con tratamiento escalonado de asma crónico, evolución favorable.
4. 4. Según el número de otitis, definirá el pronóstico.
Gráfico de respuestas
Comentario

Como se puede imaginar, se trata de una probable enfermedad de Kartagener, con situs inversus y
afección de los cilios del epitelio respiratorio. El paciente presentará infecciones de repetición. Por
otra parte, la retención mucosa justificaría, a largo plazo, la aparición de bronquiectasias,
complicación frecuente en esta enfermedad.(R2)

327. La lepra lepromatosa puede afectar progresivamente la mayoría de órganos y


aparatos. El pronóstico grave está relacionado sobre todo con:

1. 1. La profesión.
2. 2. El clima.
3. 3. El estadio inmunológico.
4. 4. El Mitsuda positivo.
Gráfico de respuestas
Comentario

Esta es una pregunta relativamente sencilla, que puede responderse basándose en puro sentido
común. La lepra es producida por una micobacteria al igual que la tuberculosis. Estas bacterias se
caracterizan por su habilidad para sobrevivir al interior de las células, siendo por lo tanto clave el
papel de la inmunidad celular para controlar la infección (opción 3 correcta). Las tres primeras
opciones están bastante fuera de lugar y deberíamos ser capaces de descartarlas sin mayores
dudas. El Mitsuda no es otra cosa que el equivalente al PPD de la tuberculosis que se usa en la
lepra: un test cutáneo de hipersensibilidad retardada. Si el paciente ha estado en contacto con la
micobacteria y tiene una BUENA respuesta inmune celular el test será POSITIVO. Por lo tanto,
esto tendrá en cualquier caso un mejor pronóstico que si fuera negativo.(R3)

 
 
 
 

328. Paciente de 59 años que acude a


consulta de ginecología para revisión. Entre sus antecedentes destaca la existencia de
mastodinia cíclica con aumento de densidad mamaria actualmente asintomática. En la
exploración no se detectan tumoraciones a nivel de las mamas ni adenopatías axilares.
Se realiza mastografía de la mama derecha en la que aparece lo que se muestra en la
imagen. ¿Cuál sería la conducta terapéutica más apropiada?

La mastografía es normal, por lo que continuaremos con el screening normal con nuevo
1. 1.
control en 2 años.
2. 2. Complementar el estudio con ecografía, y si esta es normal, nuevo control en 1 año.
3. 3. Punción de la lesión en consulta con citología de la misma.
4. 4. Biopsia diferida previo marcaje con arpón.
Gráfico de respuestas
Comentario

Aunque en esta paciente no encontramos ningún nódulo en la palpación, la ecografía nos revela un
aumento de densidad de bordes imprecisos, de localización focal. Este hallazgo debe considerarse
sospechoso de malignidad, por lo que es imperativo tomar una muestra para estudio histológico.

Teniendo esto en cuenta, la duda razonable estaría entre las opciones 3 y 4. La respuesta 3 no
puede ser correcta: no puede realizarse una PAAF, porque no existe ninguna zona identificable al
tacto (no hay nódulo). Realizar la punción sería como pinchar a ciegas. Por lo tanto, necesitaremos
marcar la zona con arpón, para obtener la biopsia de la región que nos interesa (respuesta 4
correcta).(R4)

329. Indique cuál de los siguientes hallazgos NO es sospechoso de malignidad en una


mastografía de screening:

1. 1. Nódulo denso, espiculado, de bordes imprecisos.


2. 2. Edema alrededor de la areola mamaria.

 
 
 
 
3. 3. Calcificaciones groseras.
4. 4. Densidad focal asimétrica.
Gráfico de respuestas
Comentario

Las calcificaciones sugestivas de malignidad son finas y con tendencia a la agrupación. Una
calcificación grosera sería más propia de una mastopatía fibroquística o de otras entidades que, en
cualquier caso, no se consideran malignas.(R3)

330. Respecto al carcinoma de mama, ¿cuál de las siguientes afirmaciones NO es


correcta?

1. 1. El tipo anatomopatológico más frecuente es el carcinoma ductal infiltrante.


2. 2. El carcinoma lobulillar tiene carácter bilateral y multicéntrico.
3. 3. El carcinoma intraductal no evoluciona a lesión palpable.
4. 4. El principal factor pronóstico es la afectación ganglionar.
Gráfico de respuestas
Comentario

De las opciones que nos ofrecen, es importante que conozca lo que dicen las respuestas 1 y 4,
aspectos preguntados en el ENARM

La respuesta incorrecta es la 3. El carcinoma intraductal, en caso de no extirparse a tiempo, crece


y evoluciona hacia una lesión palpable, como cualquier otra neoplasia de carácter maligno. Otra
cuestión es que esto sea más o menos frecuente (cada vez menos, gracias a la detección precoz
mediante mastografía), pero no puede decirse que NO evolucione, porque sí puede hacerlo.(R3)

331. El órgano de Corti está situado sobre:

1. 1. La membrana de Reissner.
2. 2. La membrana basilar.
3. 3. La lámina espiral.
4. 4. La estría vascular.
Gráfico de respuestas
Comentario

Tema poco preguntado en el ENARM, yo no me estresaría por eso, puede preguntar cualquier
cosa de anatomía no sabes que, no es rentable leer el Moore a estas alturas. La cóclea está
formada por dos vueltas y media de espira arrolladas alrededor de un eje óseo o modiolo.

La lámina espiral ósea forma un saliente óseo que se extiende desde la base hasta la punta en
forma espiral. Desde ella salen la membrana de Reissner y la membrana basilar que dividen el
conducto en tres partes: la rampa vestibular, el conducto coclear y la escala timpánica.

Sobre la membrana basilar se encuentra el órgano de Corti. En la parte lateral del conducto coclear
se encuentra la estría vascular, profusamente irrigada, donde se produce la endolinfa.(R2)

332. Usted atiende en urgencias a un neonato procedente de provincia, de 4 días de vida,


que ha nacido en su domicilio, atendido por una partera. La madre no ha seguido ningún

 
 
 
 
tipo de control durante el embarazo y ahora acude porque el niño ha comenzado hace 3
horas con una hemorragia umbilical y una deposición melénica. El niño tiene buen estado
general, buen color y buen estado de hidratación; la fontanela está a nivel; la exploración
ORL es normal, así como la auscultación cardio-pulmonar y la palpación abdominal. Los
reflejos primitivos son normales. El niño recibe lactancia materna exclusivamente. ¿Cuál
de las siguientes opciones acerca de la patología que sospecha NO es correcta?:

Suelen existir hemorragias leves como signo de alarma antes de que aparezcan
1. 1.
hemorragias severas, como la intracraneal.
Las complicaciones hemorrágicas severas aparecen con mayor frecuencia en los niños con
2. 2.
lactancia artificial.
Si aparecen después de la primera semana de vida pueden sugerirnos la presencia de
3. 3.
hepatitis neonatal.
El inicio en las primeras 24 horas de vida implica la aparición de una clínica más severa, y
4. 4. se suele asociar al tratamiento con fármacos como fenobarbital o fenitoína durante el
embarazo.
Gráfico de respuestas
Comentario

La enfermedad hemorrágica del recién nacido puede ser prevenida a través de una maniobra que
se hace al poco de nacer el niño: la administración de 1 mg de vitamina K, que en este caso, no
fue administrada al haberse atendido fuera del sistema sanitario. Este proceso es especialmente
grave, frecuente y precoz en hijos de consumidoras de anticomiciales, así como en niños
alimentados al pecho (pues la leche de madre tienen escasa cantidad de vitamina K).(R2)

333. Paciente de 24 años de edad que consulta por opsoamenorrea, hirsutismo


importante e hipertrofia de clítoris. En el estudio hormonal se observan cifras elevadas
de 17-hidroxiprogesterona (> de 10 ng/ml) y del sulfato de dehidroepiandrostenediona
(DHEA-S). ¿Ante qué cuadro clínico nos encontramos?:

1. 1. Síndrome adrenogenital por déficit de 21-hidroxilasa.


2. 2. Síndrome de Morris o de feminización testicular.
3. 3. Arrenoblastoma.
4. 4. Síndrome de ovario poliquístico.
Gráfico de respuestas
Comentario

Lo mas importante de las amenorreas primarias es conocer su diagnóstico diferencial, pero


también es interesante que se fije en las causas más frecuentes y en los datos típicos de algunos
síndromes.

La hiperplasia suprarrenal congénita, síndrome adrenogenital o pseudohermafroditismo femenino,


es una patología en que el cariotipo es normal femenino (46,XX). Hay una elevación de
andrógenos por hiperproducción suprarrenal, lo cual produce virilización de los genitales externos
(observe que este síndrome es en cierto modo, lo contrario del síndrome de Morris). La clínica
varía según el déficit enzimático (HTA e hipocaliemia, en el déficit de la 17 alfa hidroxilasa,
síndrome pierde sal...).El déficit más frecuente es el de la 21 hidroxilasa.(R1)

334. En relación a la reparación de las hernias inguinales señale la afirmación


INCORRECTA:

 
 
 
 
1. 1. La técnica de Bassini aproxima la zona conjunta con el ligamento inguinal.
2. 2. La técnica de Bassini es adecuada para las hernias femorales.
El uso de material protésico promueve la formación de mayor cicatrización para reforzar el
3. 3.
cierre del defecto de pared.
Una indicación precisa para la hernioplastía laparoscópica es la recidiva repetida del
4. 4.
abordaje anterior.
Gráfico de respuestas
Comentario

En algunas oasiones pueden preguntar algún tipo de técnica quirúrgica no esta demás repasar.

La técnica de Bassini es un tipo de reparación de la hernia inguinal (respuesta 2 correcta).


Consiste en la aproximación y cierre mediante sutura del tendón conjunto formado por los extremos
distales de los músculos transversos del abdomen y los oblicuos internos, al ligamento
inguinal.(R2)

335. ¿Cuál es el método de detección (screening) de la diabetes gestacional que se


recomienda en todas las embarazadas?

1. 1. Determinación de glucosa en orina con tiras reactivas.


2. 2. Determinación de glucemia en ayunas.
3. 3. Determinación de glucemia después de una sobrecarga oral de 50 gramos de glucosa.
4. 4. Determinación de glucemia en dos días diferentes.
Gráfico de respuestas
Comentario

Pregunta my omportante y muy fácil sobre el diagnóstico de la diabetes gestacional.

El test que se utiliza para el screening de la diabetes gestacional es el test de O´Sullivan, que se
apoya en la administración de 50 gramos de glucosa, determinándose la glucemia basal a los 60
minutos de la ingesta Se ha de realizar a todas las gestantes entre la semana 24 y 28, y en
aquellas gestantes con factores de riesgo de diabetes gestacional se puede realizar en el primer
trimestre del embarazo.

Factores de riesgo de diabetes gestacional:

•   Obesidad (IMC >80).


•   Historia familiar de DM.
•   Antecedentes personales de diabetes gestacional o intolerancia glucídica.
•   HTA crónica o preeclampsia.
•   Antecedentes obstétricos: abortos de repetición, fetos muertos, muerte neonatal no
explicada, prematuridad, malformaciones congénitas, hidramnios, macrosomía fetal,
historia de parto traumático.

(R3)

336. En relación con el liquen escleroso vulvar (LEV), señale lo FALSO:

Se caracteriza principalmente por prurito de intensidad variable, intermitente o


1. 1.
permanente. Aunque no es infrecuente el ardor vulvar y la dispareunia.
2. 2. El riesgo de desarrollar un carcinoma epidermoide se estima en un 25-30% de los casos.

 
 
 
 
3. 3. La confirmación diagnóstica se realiza mediante el examen histológico.
El tratamiento del LEV es médico, orientado principalmente a calmar los síntomas y evitar
4. 4.
la evolución atrófica.
Gráfico de respuestas
Comentario

El liquen escleroso vulvar no es una lesión premaligna, por tanto tiene poco riesgo de desarrollar
en carcinoma epidermoide de vulva. Es clásico de mujeres en la postmenopausia por el déficit
estrogénico si bien puede aparecer en otras edades. El tratamiento suele realizarse con corticoides
locales o testosterona en pomada.

Fíjese quede algún modo la opción 2 es contraria a la 4, ya que si realmente existe un potencial
oncogénico tan elevado, el tratamiento se orientaría a la prevención de este riesgo, no al
tratamiento sintomático.(R2)

337. Recién nacido de 39 semanas por cesárea, presenta dificultad respiratoria, quejido
inconstante, tiraje subcostal, aleteo nasal leve y retracción xifoidea. Ruidos cardíacos
normales. ¿Cuál es el diagnóstico más probable?

1. 1. Cardiopatía congénita.
2. 2. Aspiración meconial.
3. 3. Neumonía neonatal.
4. 4. Taquipnea transitoria.
Gráfico de respuestas
Comentario

La taquipnea transitoria del recién nacido es la causa más frecuente de distrés respiratorio
neonatal. Se debe a un retraso en la absoricón del líquido de los pulmones fetales por el sistema
linfático, dando lugar a un colapso pulmonar que conlleva un retraso en el proceso de adaptación a
la vida extrauterina. El paciente prototipo es un recién nacido a termino nacido por cesárea o por
parto vaginal rápido. También es más frecuente en el sexo masculino y en recién nacidos
macrosómicos e hijos de madre diabética. No requiere tratamiento.(R4)

338. Paciente en su quinto día posoperatorio, debido a perforación traumática de colon y


peritonitis generalizada. La herida operatoria presenta vesículas hemorrágicas en la piel,
con aponeurosis edematizada de color gris y tejido celular subcutáneo esfacelado. ¿Cuál
es el diagnóstico más probable?

1. 1. Fascitis necrotizante.
2. 2. Absceso por grampositivos.
3. 3. Gangrena gaseosa.
4. 4. Celulitis postestreptocosica.
Gráfico de respuestas
Comentario

Pregunta fácil. En un paciente con peritonitis secundaria, con datos de infección cutánea, tejido
celular subcutáneo y en aponeurosis, el diagnóstico es fascitis necrotizante. Respuesta 1.(R1)

 
 
 
 

339. Recién nacido a término


(38+5) de peso adecuado para la edad gestacional. Presenta al nacimiento dificultad
respiratoria severa con cianosis y baja saturación periférica de oxígeno. A la exploración
destaca abdomen excavado y latido cardíaco rítmico sin soplos en hemitórax derecho.
Tras reanimación neonatal con intubación orotraqueal y ventilación mecánica, el paciente
es trasladado a la unidad de cuidados intensivos neonatales donde se obtiene la
siguiente radiografía. Señale la afirmación CORRECTA respecto a lo observado en la
prueba de imagen:

1. 1. La presencia de la dextrocardia evidenciada se suele asociar con anomalías esplénicas.


2. 2. Lo más probable es que se trate de una malformación adenomatoidea quística.
3. 3. El paciente no está intubado.
4. 4. Se evidencia una marcada hipoplasia pulmonar.
Gráfico de respuestas
Comentario
Se comenta en la pregunta siguiente.(R4)

340. Respecto a esta patología, señala la opción CORRECTA:

1. 1. En una mayoría de los casos hay participación hepática y esto empeora el pronóstico.
El tratamiento es una emergencia quirúrgica, debiendo ser intervenido en las primeras 24
2. 2.
horas de vida.
La localización más frecuente es un defecto a nivel de los pilares diafragmáticos
3. 3.
posteriores, sobretodo en el lado derecho.
4. 4. En su manejo el uso del óxido nítrico inhalado no tiene ningún papel.
Gráfico de respuestas
Comentario

Hay distintos tipos de hernias (defectos) a nivel diafragmático.

 
 
 
 
La hernia de Morgagni es de localización anteromedial (retroesternal), más frecuente en el lado
derecho, generalmente asintomática al nacimiento se suele diagnosticar como un hallazgo
incidental en una radiografía de tórax realizada por otro motivo.

En la hernia de hiato el contenido abdominal asciende a su través.

En esta pregunta hacen referencia a la hernia diafragmática congénita o de Bochdaleck. Se


produce por un defecto a nivel posterior, generalmente izquierdo (85%), aunque puede ser incluso
bilateral (<5%). La malrotación intestinal y la hipoplasia pulmonar forman parte de la malformación,
siendo esta última la causa de muerte en los peores casos. En muchos de ellos a la dificultad para
una adecuada función de intercambio gaseoso se añade una hipertensión pulmonar. La
hipertensión pulmonar se puede tratar con óxido nítrico inhalado al ser este un potente
vasodilatador pulmonar.

El cuadro clínico es el de un recién nacido con dificultad respiratoria que aparece en las primeras
48 horas de vida, frecuentemente con cianosis. El abdomen aparecerá excavado y es posible
escuchar ruidos hidroaereos, correspondientes a borborigmos intestinales. En el caso más
corriente (herniación por el lado izquierdo) el corazón se ve desplazado a la derecha. La
auscultación sugerirá dextrocardia aunque no es una dextrocardia real, las cuales efectivamente se
asocian a anomalías a nivel del bazo (asplenia, poliesplenia). Con mucha frecuencia se hernian
también vísceras sólidas, sobretodo el hígado, lo cual empeora el pronóstico. El tratamiento inicial
consiste en garantizar una adecuada ventilación, oxigenación y circulación, siendo la cirugía un
paso posterior (como mínimo se suele retrasar 48 horas).(R1)

341. A 4-year-old boy is referred to the ophthalmologist by his pediatrician. Visual acuity
is 0.4 in his right eye and 0.6 in his left eye. +1 diopters hyperopia is observed in each eye
and high astigmatism, (especially higher in the right eye). Which of the following
statements is FALSE?

1. 1. His right eye is probably amblyopic.


2. 2. The healthy eye should be occluded to restore vision in the affected eye.
3. 3. The patient graduation should be performed with cycloplegic.
4. 4. Refractive errors in children should be corrected as soon as possible.
Gráfico de respuestas
Comentario

Cuando llega un niño por primera vez a la consulta de oftalmología con pérdida de agudeza visual,
lo primero que debemos hacer es graduarle para ver cuál es su mejor agudeza visual corregida
con cada ojo. Para una correcta graduación en un niño siempre tendremos que basarnos en el
defecto de refracción obtenido con cicloplejia, que no permite saber cuál es la hipermetropía total y
explorarle el fondo de ojo para descartar anomalías retinianas o coriodeas que le impidan una
correcta fijación.

Si el niño presenta diferente graduación en cada ojo, puede que el ojo con mayor graduación esté
desarrollando una ambliopía. Eso lo sabremos porque el ojo ambliope no alcanza la misma visión
que el otro ojo, a pesar de la corrección óptica. En este caso, habría que pautar oclusiones del ojo
sano para favorecer el desarrollo de la visión del ambliope. Sin embargo, lo más importante ante
un niño con defectos de refracción es pautarle la corrección óptica adecuada para que su ojo cree
una imagen en la retina lo más clara posible.(R2)

 
 
 
 
342. Hombre de 30 años que después de un resfriado comienza con náuseas, vómitos y
oliguria. En la los exámenes de laboratorio encontramos una creatinina de 4 mg/dl y en
la orina abundantes hematíes y cilindros hemáticos junto con una proteinuria de 2,5
mg/dl. Al día siguiente la creatinina es de 8 mg/dl. Se decide hacer una biopsia renal. Al
microscopio óptico encontramos abundante proliferación extracapilar en forma de
semilunas en más del 80% de los glomérulos. Diagnóstico:

1. 1. GN rápidamente progresiva.
2. 2. GN aguda postestreptocócica.
3. 3. GN focal y segmentaria.
4. 4. GN cambios mínimos.
Gráfico de respuestas
Comentario
Sencillo caso clínico típico de la GN rápidamente progresiva (OPCION 1 CORRECTA). El dato
fundamental que resume el caso es una insuficiencia renal (=elevación de creatinina) rápidamente
progresiva (se duplica en 1 día), que ya nos debe orientar a la GNRP. Además existe hematuria y
proteinuria. El diagnóstico se confirma mediante la biopsia renal que muestra el dato típico de la
proliferación extracapilar con semilunas. En estas semilunas se diferencian dos componentes: uno
celular, constituido por monocitos que han migrado al espacio de Bowman, células propias del
epitelio glomerular, células del intersticio renal y fibroblastos, y otro componente formado por una
matriz de polímeros de fibrina, fibras colágenas, que terminan organizando la semiluna en una
cicatriz fibrosa. El pronóstico de esta GNRP está estrechamente ligado al porcentaje de semilunas
y a su tamaño.(R1)

343. A 24-year-old girl presents to the primary care doctor complaining of a three-day
history of right upper abdominal pain, malaise, unusually dark urine, anorexia and
aversion to cigarette smoking. She returned from a trip from Mexico weeks ago. She has
jaundice and total bilirubin is 6.8 mg/dL. The most likely cause is:

1. 1. Hepatitis A
2. 2. Hepatitis B
3. 3. Hepatitis C
4. 4. Hepatitis D
Gráfico de respuestas
Comentario
Hepatitis A. The clinical picture is typical for an acute liver inflammation. Acute hepatitis can be
caused by several agents, including virus and other insults. The epidemiologic context, with a
recent trip to Mexico, helps us choose HAV as the most likely cause. HBV acute infection might also
show joint pains or skin eruption. Smoking aversion, although potentially present in every acute
hepatitis case, has been classically linked to HAV acute hepatitis.(R1)

344. Which of the following options is not a risk factor for ovarian cancer?

1. 1. Ovulation induction drugs.


2. 2. Polycystic ovary syndrome.
3. 3. Talcum powder.
4. 4. Early menarche and late menopause.
Gráfico de respuestas
Comentario

 
 
 
 
Esta pregunta de cáncer de ovario es útil para recordar que el factor de riesgo más importante es
la ovulación incesante.

Así, toda situación que produzca anovulación como la toma de anticonceptivos orales, la
multiparidad o el SOP reducen el riego de cáncer de ovario.

El talco es factor de riesgo pero no tiene por qué saberlo.

El otro factor de riesgo para el cáncer de ovario es el genético. Recuerda su asociación con el
carcinoma colorrectal no polipósico o síndrome de Lynch tipo 2(R2)

345. Señale la afirmación INCORRECTA respecto a la afectación respiratoria que aparece


en el contexto de las enfermedades neuromusculares:

1. 1. El índice de Tiffenau probablemente esté alterado.


Aunque no es frecuente, la constante de difusión (KCO) puede estar aumentada si la
2. 2.
hipoxia alveolar es importante.
3. 3. El gradiente alveolo-arterial de oxígeno será, con gran probabilidad, normal.
El volumen residual probablemente estará disminuido, al contrario de lo que sucede en las
4. 4.
alteraciones restrictivas parenquimatosas.
Gráfico de respuestas
Comentario

En cuanto a la afectación respiratoria que aparece en las enfermedades neuromusculares hay


varios datos importantes:

- hay dificultad para el llenado de aire pulmonar, es decir tienen un patrón restrictivo, la capacidad
pulmonar total y la capacidad vital están disminuidas.

- al pulmón le cuesta tanto llenarse como vaciarse, por lo que el volumen residual estará
aumentado (respuesta 5 incorrecta).(R4)

346. Un prematuro de 900 g de peso y 27 semanas de gestación sufre distrés respiratorio


desde el nacimiento. A las 36 horas de vida presenta hipotensión, bradicardia, cianosis y
tensión aumentada de la fontanela. ¿Qué exploración diagnóstica le parece más oportuna
entre las que se muestran?:

1. 1. Análisis de sangre con serie roja y plaquetas.


2. 2. Exploración cerebral con ultrasonido.
3. 3. Estudio del tiempo de protrombina y del tiempo parcial de tromboplastina.
4. 4. Punción lumbar y cultivo de LCR.
Gráfico de respuestas
Comentario

En un RNPT con alta probabilidad de enfermedad de membrana hialina, otra de las complicaciones
que debería pensar es la hemorragia intraventricular.

Al contarte que presenta hipotensión, bradicardia, cianosis y aumento de la tensión de la fontanela


deberá pensar en dicho diagnóstico. La exploración diagnóstica que debe realizar es el ultrasonido
cerebral. Respuesta 2 correcta.(R2)

 
 
 
 
347. A 12-year-old chid is brought to her pediatrician's office by her worried mother, who
reports unusual behavior, as well as permanent unquenchable thirst. She drinks almost
four liters of water a day. Her school performance has worsened, and she complains of
headaches almost every day. Vital signs are: 120/70 mmHg (blood pressure), 70 bpm
(pulse rate) and 36ºC (body temperature). Neurological examination shows bilateral
peripheral loss of vision. What is the most likely diagnosis?

1. 1. Craniopharyngioma
2. 2. Neuroblastoma
3. 3. Glioma
4. 4. Pituitary adenoma
Gráfico de respuestas
Comentario
Craniopharyngioma. Craniopharyngioma is a tumor that grows in the vicinity of the hypothalamus.
Hypothalamic involvement or invasion of the posterior pituitary produces deficit in the release of
antidiuretic hormone. The growth of this tumor can compress the optical chiasm causing visual loss.
Pituitary adenomas are not common at this age, neither are meningiomas. Neuroblastoma occurs in
younger children.(R1)

348. Es correcto respecto a las infecciones post-transplante de médula ósea:

1. 1. La vacunación antineumocócica en estos enfermos está contraindicada.


2. 2. La infección por CMV es característica del primer mes.
La infección por VEB ocurre preferentemente por reactivación del virus en linfocitos B del
3. 3.
receptor.
4. 4. El virus del herpes simple se aisla en los primeras semanas postransplante.
Gráfico de respuestas
Comentario

Lo más importante de este tema es que según el tiempo trascurrido desde el TMO, las infecciones
son por determinados microorganismos:

- 0- 4 semanas postrasplante. Se trata de pacientes hospitalizados con supresión profunda de la


médula ósea, predominando las infecciones nosocomiales y del neutropénico: bacilos gram
negativos (incluida P. aeruginosa), S. aureus y hongos. Además, es característica la reactivación
de la infección por VHS (RC- 4).

- 2- 6 meses. En esta etapa, es más frecuente la infección por CMV como causa de fiebre.
También se producen infecciones por otros microorganismos oportunistas como Micobacterias,
Nocardia, Listeria, Cryptococcus, Toxoplasma o Pneumocystis jiroveci.

- Más de 6 meses postrasplante se producen infecciones adquiridas en la comunidad, por ejemplo,


neumococo.
(R4)

349. Un fibroma puede llegar a ser:

1. 1. Pedunculado

 
 
 
 
2. 2. Liquenificado
3. 3. Incarcerado
4. 4. Eccematoso
Gráfico de respuestas
Comentario

Asumiendo que se está hablando de fibromas uterinos. Entre los diferentes tipos de tumores
fibroides, que incluyen los fibromas submucosos, intramurales y subserosos, se encuentran los
tumores fibroides pedunculados.

Los fibromas uterinos pedunculados se producen cuando un tumor fibroide crece en un tallo, lo que
da como resultado fibromas submucosos pedunculados o fibromas subserosos pedunculados.
Estos fibromas pueden crecer en el útero y también tienden a crecer en la parte exterior de la
pared uterina.

El resto de características no son aplicables, entonces la respuesta correcta sería la 1.(R1)

350. Mujer de 21 años de edad, que ha comenzado hace tres meses con “pitos” en el
pecho y tos especialmente por la noche. Se ha hecho más frecuente y los últimos días
no le deja descansar. La exploración muestra una discreta disminución del murmullo
vesicular y algunos sibilantes, de forma poco intensa en ambos hemitórax, frecuencia
cardíaca 86 pulsaciones por minuto. ¿Cuál será la primera decisión terapéutica?

Prescribir broncodilatadores de larga acción como formoterol asociado a corticoides


1. 1.
inhalados.
2. 2. Indicar antibióticos asociados a broncodilatadores anticolinérgicos.
3. 3. Prescribir N-acetilcisteína, cada 6 horas asociado a amoxicilina.
4. 4. Prescribir prednisona por vía oral.
Gráfico de respuestas
Comentario

Pregunta relativamente sencilla sobre el tratamiento del asma. Recuerda que los broncodilatadores
de acción corta se emplean a demanda.

Por otra parte, no existe indicación de emplear antibióticos, dado que no tiene signos de infección.

El uso de prednisona por vía oral no estaría justificado sin intentarlo antes por vía inhalatoria, y
menos en este caso, que hasta ahora no ha sido tratada con esteroides.

En este caso clínico, que encaja en el asma moderado persistente, el tratamiento incluye un
betaagonista de acción corta a demanda, uno de acción larga tipo formoterol, y corticoides
inhalados.(R1)

351. El síndrome de Hiper IgE (síndrome de Job) es una inmunodeficiencia de tipo:

1. 1. Defecto de quimiotaxis.
2. 2. Déficit de complemento.
3. 3. Déficit selectivo de IgG con hiperproducción de IgE.
4. 4. Hipergammaglobulinemia.
Gráfico de respuestas

 
 
 
 
Comentario
Pregunta compleja pero que si dominas conceptos básicos sobre los diferentes tipos de
inmunodeficiencias no tendrás ningún problema en resolver. Recuerda que las inmunodeficiencias
se clasifican en varios tipos: celulares, humorales, combinadas, alteración del complemento o de la
función fagocítica. El síndrome de Job o síndrome hiper IgE pertenece a los defectos de la
quimiotaxis. Suelen ser casos esporádicos, aunque existe una forma autosómica dominante con
penetrancia incompleta. Se caracteriza por dermatitis crónica pruriginosa e infecciones bacterianas
sinopulmonares y cutáneas acompañadas de cifras elevadas de IgE y eosinofilia. La manifestación
clínica más típica, como en los déficits de la función fagocítica, es la aparición de abscesos
cutáneos recidivantes por Staphyloccocus aureus.(R1)

352. Mujer de 34 años que, desde los 28, presenta una larga historia de múltiples síntomas
físicos gastrointestinales, dolorosos, cardiopulmonares, pseudoneurológicos y
sexuales. No presenta crisis de angustia y su cuadro no puede explicarse por ninguna
patología orgánica. Estos síntomas le han obligado a acudir con frecuencia al médico y
a tomar fármacos. El diagnóstico es:

1. 1. Trastorno de conversión.
2. 2. Trastorno por somatización (síndrome de Briquet).
3. 3. Síndrome de Ganser.
4. 4. Nosofobia.
Gráfico de respuestas
Comentario

Nos describen un trastorno de somatización, también llamado síndrome de Briquet. Apenas ha sido
preguntado en el MIR, pero es relativamente frecuente en la práctica diaria. Además, te dan
prácticamente la definición, porque nos hablan de síntomas físicos, habiendose descartado
patología orgánica. En caso de habernos descrito síntomas únicamente neurológicos, se trataría
de un trastorno conversivo. El trastorno por somatización es una de las formas de la histeria,
preferentemente en mujeres menores de 30 años, que realizan una búsqueda incesante de
atención médica. Su diagnóstico exige un deterioro significativo a varios niveles en la vida del
paciente.(R2)

353. En ecocardiografía se usa la ley de Bernoulli, ¿cuál es su utilidad?:

1. 1. El estudio de la función diastólica del corazón.


2. 2. La ley de Bernoulli no se usa en ecocardiografía.
3. 3. El estudio de las regurgitaciones valvulares.
4. 4. Establecer el gradiente de presión de una estenosis.
Gráfico de respuestas
Comentario
Pregunta muy difícil que requiere conocimientos de especialista de un tema que no es muy
preguntado en el MIR como son los métodos diagnósticos. La ecocardiografía puede valorar
diámetros de las cavidades, la fracción de eyección y la movilidad de las válvulas, los flujos y áreas
valvulares. La ley de Bernoulli es la que se utiliza para ver gradiente de presiones en las
valvulopatías, por lo que la opción correcta es la 5.(R4)

 
 
 
 
354. De los siguientes enunciados, señale el INCORRECTO.

1. 1. Mayor riesgo de hipermagnesia en diabetes gestacional.


El embarazo gemelar se caracteriza por una mayor incidencia de malformaciones
2. 2.
congénitas.
3. 3. Preeclampsia: duplica la morbimortalidad perinatal.
4. 4. Mayor riesgo de HTA en preeclampsia.
Gráfico de respuestas
Comentario

La respuesta correcta es la número 1, ya que la diabetes gestacional se asocia a hipocalcemia e


hipomagenesemia, el resto de enunciados son correctos.(R1)

355. Un paciente diagnosticado de apendicitis aguda, desarrolla en pocas horas un


estado tóxico con deshidratación, postración, distensión abdominal, así como aumento
del dolor en cuadrante inferior derecho, en intensidad y extensión. Presenta además
fiebre de 38ºC y leucocitosis: 30,000 cel/mm3. Indica la conducta más CORRECTA a
seguir:

1. 1. Apendicectomía inmediata.
Realización de estudios radiográficos para la valoración de la extensión del absceso
2. 2.
apendicular y posponer la cirugía una semana, mientras se le trata con eritromicina.
Aspiración nasogástrica, reposición hidroelectrolítica, antibióticos frente a bacteroides y
3. 3.
cirugía en pocas horas.
4. 4. Descartar megacolon tóxico con Rx abdominal y posterior colectomía si procede.
Gráfico de respuestas
Comentario

Nunca debe olvidar que el tratamiento de una apendicitis aguda ya instaurada es quirúrgico y que
retrasar la intervención puede complicarse con una perforación apendicular. En el contexto de una
apendicitis aguda no tratada, la intensificación del dolor y el aumento de la temperatura por encima

 
 
 
 
de 38ºC con mayor afectación del estado general apuntan hacia una posible perforación. Ante este
cuadro es importante proceder al tratamiento empírico con antibióticos que cubran gram negativos
y anaerobios para evitar en lo posible la evolución a peritonitis aguda y a la rehidratación del
paciente para someterlo a intervención quirúrgica de urgencia.(R3)

356. El herpes circinado múltiple localizado en antebrazos se debe habitualmente a


ciertos factores etiológicos. ¿Puede precisarlos?:

1. 1. Espasmo vascular periférico.


2. 2. Exceso de vello.
3. 3. Dermatofitos.
4. 4. Virus neurotropos.
Gráfico de respuestas
Comentario
Pregunta muy fácil, ya que tan solo te piden que sepas que el herpes circinado o tambien
denominado tiña del cuerpo es producida por algún tipo de dermatofito. Con el nombre de herpes
circinado, se incluyen todas las infecciones por dermatofitos que se localizan el la piel lampiña, con
excepción de las infecciones localizadas en sitios específicos, palmas, plantas y zonas inguinales.
Todas las especies de dermatofitos pertenecientes a los géneros Tricophiton, Microsporum, o
Epidermophiton son capaces de provocarla. Bibliografía: Manual de dermatología Iglesias. 1ª Ed.
Pag. 265(R3)

357. El tratamiento de elección de un carcinoma de células escamosas del canal anal de


5 mm, sin adenopatías locales, ni metástasis en el estudio de extensión, consiste en:

1. 1. Radioterapia.
2. 2. Amputación abdominoperineal y linfadenectomía inginal.
3. 3. Escisión local y linfadenectomía inguinal.
4. 4. Vaporización con láser de CO2.
Gráfico de respuestas
Comentario

El carcinoma de células escamosas o epidermoide anal, es una entidad diferente al cáncer de


recto, que suele manifestarse como una ulceración en dicha región.

El tratamiento si no existe enfermedad metastásica, a cadenas linfáticas o a distancia, de lesiones


superficiales pequeñas (< 3 cm) de la piel perianal consiste en la radioterapia o escisión local
quirúrgica, con semejantes resultados. No se asocia linfadectomía inguinal de rutina (salvo que se
demuestre la afectación ganglionar).

El cáncer escamoso del canal anal y los tumores perianales grandes que invaden el esfínter y el
recto se tratan con una modalidad terapéutica combinada que incluye radiación externa con
quimioterapia simultánea (fluorouracilo y ya sea mitomicina o cisplatino). Se logra control local en
80% de los pacientes.

En la actualidad la cirugía radical (resección abdominoperineal) se reserva para pacientes en los


cuales fracasan la quimioterapia y la radioterapia. La tasa de supervivencia a cinco años es de 60
a 70% para los tumores localizados y superior a 25% para la enfermedad metastásica (etapa IV).

 
 
 
 
El láser y la criocirugía se emplean en entidades que no rompen la membrana basal, como las
neoplasias intraepiteliales, que pueden ser precursoras de esta entidad.(R1)

358. Niña de 13 años que acude a su centro de salud por aparición brusca de fiebre alta
y odinofagia intensa. En la exploración presenta aceptable estado general, la
auscultación cardiopulmonar es normal, el abdomen es blando y depresible, presenta
algunas adenopatías laterocervicales bilaterales y en el examen de la orofaringe se
aprecia una faringe eritematosa, con amígdalas hipertróficas con placas y exudados
bilaterales y lengua saburral. En la piel se observa tenue exantema con micropápulas
muy levemente eritematosas que blanquean a la presión. Teniendo en cuenta su
sospecha diagnóstica, señale cuál de las siguientes afirmaciones es INCORRECTA:

1. 1. La infección está producida por el Streptococo pneumoniae.


Es útil la recogida de exudado faríngeo para cultivo y detección rápida de antígenos
2. 2.
estreptocócicos.
3. 3. El exantema suele resolverse en 3-4 días con descamación foliácea.
4. 4. Si el exantema afecta a la cara, suele respetar el triángulo nasolabial.
Gráfico de respuestas
Comentario

Estamos ante un caso de escarlatina: faringoamigdalitis con exantema micropapular que se palpa
mejor que se ve y desaparece por descamación en pocos días.

La lengua se recubre los primeros días de una capa blanquecina (lengua en fresa blanca o
saburral) y posteriormente se descama y aparece hiperémica (lengua en fresa roja).

Si se afecta la cara, típicamente respeta el triángulo nasolabial (facies de Filatov). Se trata de una
infección debida a Streptococcus pyogenes.

Para confirmar el diagnóstico es útil la detección rápida de antígenos o cultivo, y el tratamiento de


elección es penicilina oral durante 10 días.(R1)

359. What is the disease where the Auer


rods?

1. 1. Chronic lymphatic leukemia.


2. 2. Erythroblastopenia secondary to parvovirus B 19.

 
 
 
 
3. 3. Acute myeloid leukemia.
4. 4. Alterations in erythrocyte deformability.
Gráfico de respuestas
Comentario

Los bastones de Auer ya se ahn preguntado en elaguna ocasión, son inclusiones que se tiñen en
el citoplasma de precursores de granulocitos. Consisten en material granulado, compuesto de
lisosomas fusionados que contienen peroxidasa, enzimas lisosomales y cristales, y que se tiñen
con preferencia al lapislázuli. Por lo general, aparecen caracteristicamente en mieloblastos de la
leucemia mieloide aguda, en especial las de tipo M1, M2, M3 y M4.(R3)

360. Respecto a H. ducreyi, señala cuál de las siguientes afirmaciones es FALSA:

1. 1. Es responsable del chancro blando o chancroide.


2. 2. Provoca una enfermedad de transmisión sexual.
3. 3. Es responsable de conjuntivitis contagiosa.
Una pauta terapéutica adecuada para el tratamiento de esta infección es eritromicina 500
4. 4.
mg cada 6 horas durante 7-10 días.
Gráfico de respuestas
Comentario

Con esta pregunta, podemos repasar los conceptos más preguntados sobre esta infección. El
chancro blando o chancroide es una enfermedad de transmisión sexual causada por
el Haemophilus ducreyi. Tiene un período de incubación de 1- 3 días, tras el que aparece un
chancro blando y doloroso, múltiple, de fondo sucio, acompañado de adenopatías dolorosas
unilaterales, con tendencia a ulcerarse. El tratamiento consiste en 250 mg im de ceftriaxona en
monodosis, o eritromicina durante 7 días. La opción 3 es falsa, pues este agente no es
responsable de conjuntivitis contagiosa.(R3)

361. El Streptococcus viridans es el agente productor más frecuente de:

1. 1. Reumatismo poliarticular agudo.


2. 2. Glomerulonefritis difusa.
3. 3. Cuadros erisipelosos.
4. 4. Endocarditis bacteriana subaguda.
Gráfico de respuestas
Comentario

El agente productor más frecuente de las endocarditis subagudas -opción 4- es el estreptococo


(que ocasiona el 55% de todos los casos en válvulas naturales), y de ellas, el estreptococo viridans
produce el 75% - opción 4- . Debe recordar que la erisipela - opción 3- está producida por el
estreptococo pyogenes, que también puede causar, de forma reactiva, cuadros de glomerulonefritis
posestreptocócica - opción 2- . Un estreptococo causante frecuente de meningitis bacteriana es el
Strep. Pneumoniae (causa más frecuente de meningitis en adultos, de meningitis con fístula de
LCR y de meningitis recurrente). Aprovecho para comentarte otro matiz importante de la respuesta
4: se refiere a cuadro subagudo; si le dijera cuadro fulminante o hiperagudo sería falsa porque la
causa más frecuente sería S. aureus.(R4)

362. Ante una mujer menopáusica desde hace 6 años con metrorragias. ¿Qué es lo
primero que hay que descartar?:

 
 
 
 
1. 1. Miomas en crecimiento.
2. 2. Endometritis por Chlamydia.
3. 3. Adenocarcinoma de endometrio.
4. 4. Cuerpo extraño.
Gráfico de respuestas
Comentario

Es importante que conozca unas nociones básicas sobre las metrorragias postmenopaúsicas.
Debemos estar alerta ante toda hemorragia genital, ya que es el principal síntoma y habitualmente
el más precoz de la mayoría de los tumores genitales. Actualmente la visualización de la cavidad
endometrial por histeroscopía permite obtener muestras de biopsias dirigidas que aumenta la
sensibilidad del legrado-biopsia fraccionado, con lo cual el diagnóstico de elección ha pasado a ser
la biopsia dirigida por histeroscopía. La histerectomía está indicada en las perimenopáusicas en las
que el examen anatomopatológico demuestre una hiperplasia endometrial atípica.(R3)

363. Paciente de 54 años de edad, con antecedentes de HTA en tratamiento con


diuréticos, diabetes mellitus tipo 2 controlada con dieta y ulcus gastroduodenal operado
hace 15 años. Acude a urgencias por cuadro iniciado hace 15 horas y que consiste en
dolor abdominal difuso de tipo cólico, más intenso en zonas inferiores del abdomen; el
dolor no se irradia y no varía con la postura. Hace 7 horas aparecen vómitos abundantes,
al inicio biliosos, siendo ahora de características fecaloideas. Refiere cese de la emisión
de gases y heces desde horas antes de aparecer el cuadro. La exploración física nos
muestra un abdomen distendido difusamente, doloroso a la palpación, sin signos de
irritación peritoneal, con ruidos aumentados de intensidad y sin hernias. La tensión
arterial es de 120/80, el pulso 90 lpm y la temperatura de 37ºC. No hay heces en ampolla
(tacto rectal). Analítica: Hb 15 g, Hcto 42%, leucocitos 8.950 (fórmula 75% segmentados,
15% linfocitos, 1 eosinófilo), glucosa 100 mg/100 ml, urea 80 mg/100 ml. ¿Cuál es el
diagnóstico más probable?:

1. 1. Ileo paralítico.
2. 2. Colecistitis aguda.
3. 3. Obstrucción intestinal mecánica.
4. 4. Obstrucción intestinal estrangulada.
Gráfico de respuestas
Comentario
Pregunta práctica en relación con el tema de la obstrucción intestinal, que es un tema que de cara
al MIR no te debe plantear muchos problemas y del que debes saber su etiología más frecuente y
su cuadro clínico. Aprovecha esta pregunta para quedarte con la forma de presentación más
característica y sus signos y síntomas típicos. Recuerda que la etiología más frecuente depende de
la localización de la obstrucción; en el intestino grueso la causa más frecuente es el cáncer
colorrectal y en el intestino delgado las adherencias y las hernias. Para confirmar el diagnóstico la
prueba de elección es la radiografía simple de abdomen tanto en bipedestación como en decúbito
que pone de manifiesto la distensión de las asas intestinales y los niveles hidroaéreos.(R3)

364. Entre las acciones de la progesterona se encuentran todas las siguientes, EXCEPTO
una. Señálela:

1. 1. Induce la transformación secretora del endometrio.


2. 2. Aumento de la cantidad de moco cervical.
3. 3. Elevación de la temperatura basal.
4. 4. Relajación de la musculatura ureteral.

 
 
 
 
Gráfico de respuestas
Comentario

Es importante que recuerde las acciones de las hormonas en el ciclo genital femenino, algunas
veces han sido preguntadas. La progesterona se fabrica en el cuerpo lúteo durante la fase
secretora, su misión es la maduración del endometrio y provocar las manifestaciones que
beneficien una gestación. Así, preparan las mamas para la lactancia, deprimen la excitabilidad de
las fibras miometriales, puesto que las contracciones uterinas impedirían la gestación (también
relajan el músculo liso digestivo y ureteral), elevan el metabolismo y la temperatura corporal y
disminuyen la cantidad de moco cervical y su contenido en ácido siálico, aumentando su
viscosidad para que no puedan atravesarlo nuevos espermatozoides.(R2)

365. Sobre la frecuencia cardiaca fetal (FCF) en el RCTG, indique la opción INCORRECTA:

1. 1. Su valor normal oscila entre 120 y 160 lpm.


2. 2. A medida que avanza la edad de gestación, la línea de base se aproxima a 160 lpm.
3. 3. La administración de propranolol disminuye la frecuencia cardíaca fetal.
Para considerar que existe un cambio en la línea de base, la variabilidad de la frecuencia
4. 4.
cardíaca fetal se debe mantener más de 15 minutos.
Gráfico de respuestas
Comentario

Conceptos básicos. La frecuencia cardíaca normal en el recién nacido oscila entre 120 y 160 lpm.
Según pasa el tiempo, va reduciéndose conforme el niño crece, hasta alcanzar la del adulto (60-
100). Por tanto, la respuesta incorrecta es la 2, puesto que la frecuencia cardíaca va descendiendo
paulatinamente a medida que avanza la edad gestacional.(R2)

366. Acude a su centro de salud una madre con su hijo recién nacido de 7 días de vida.
Está muy preocupada porque el hijo de una de sus hermanas falleció ayer a los 2 meses
de edad por una muerte súbita y le solicita información acerca del cuadro y cómo puede
ella prevenir que le ocurra a su hijo. Usted afirmaría todo lo siguiente, EXCEPTO:

1. 1. Debe evitar la exposición del niño al humo de tabaco.


Actualmente se considera que el uso de chupón podría ser un factor protector de muerte
2. 2.
súbita.
3. 3. Debe evitar el sobrecalentamiento del entorno y del lactante.
Es aconsejable que el lactante duerma en la misma habitación en la que lo hacen los padres
4. 4.
y, de ser posible, en la misma cama.
Gráfico de respuestas
Comentario

En esta pregunta repasamos la prevención del síndrome de muerte súbita del lactante.

Todas las medidas que se exponen en las respuestas se consideran factores protectores o formas
de prevención del síndrome, excepto la 4, ya que aunque es aconsejable que el niño duerma en la
misma habitación de los padres, se considera peligroso que el niño comparta cama con otros
individuos, tanto por el sobrecalentamiento que puede sufrir el lactante como por el riesgo de
comprimirlo durante el sueño.(R4)

367. A 30-year-old woman who presents to her physician with a 5-month history of
recurrent episodes of bilateral pallor, cyanosis and redness of her fingers after cold

 
 
 
 
exposure along with numbness and local pain. She does not have any other symptoms.
Which of the following is the most likely diagnosis?

1. 1. Chilblains.
2. 2. Acrocyanosis.
3. 3. Raynaud's disease.
4. 4. Livedo reticularis.
Gráfico de respuestas
Comentario
Raynaud's disease. Raynaud's phenomenon is described as the sequence of "PCR" when the
patient is exposed to cold: pallor-cyanosis-rubor, especially in acral parts such as fingers and toes.
Remember that the classic sequence may not be seen in every patient.(R3)

368. La necrosis de los maxilares está relacionada con la administración de:

1. 1. Bifosfonatos.
2. 2. Terramicina.
3. 3. Calcio.
4. 4. Corticoides.
Gráfico de respuestas
Comentario

Por tercer año consecutivo preguntan por este efecto secundario, la necrosis maxilar, que puede
aparecer con la utilización de bifosfonatos. Es un efecto secundario poco habitual, que aparece,
sobre todo, cuando se utilizan a dosis altas, principalmente para el tratamiento de la hipercalcemia
tumoral.(R1)

369. A young male is found unconscious on the floor at a rock music festival. Emergency
services are on their way but you are requested to provide him with health assistance
while they are on their way. The patient is unresponsive, struggling to breathe and
cyanotic. His carotid pulse is present. Which of the following is the BEST first step in
management?

1. 1. Start mouth-to-mouth resuscitation.


Make sure there are no foreign bodies within the patient's mouth. Lift his mandible and
2. 2.
gently tilt his head backwards.
Place the patient on his back on a hard surface and begin chest compressions (at a rate of
3. 3.
80-100 compressions/minute).
4. 4. Electrical defibrillation (200 J).
Gráfico de respuestas
Comentario

Una pregunta que podría resolverse fácilmente, por simple lógica. Ante un caso así, habría que
realizar una rápida valoración cardiorrespiratoria, que además nos ofrecen como dato en el
enunciado. Desde el punto de vista circulatorio, el pulso carotídeo está presente… Pero observe lo
que ocurre con la respiración: existe esfuerzo respiratorio, pero con movimientos de lucha. Ante
este hallazgo, antes de proceder a la respiración boca a boca, es necesario valorar la presencia de
un eventual cuerpo extraño que pudiera estar interrumpiendo el flujo aéreo (respuesta 2
correcta).(R2)

 
 
 
 
370. Señale la pareja INCORRECTA tratamiento-efecto secundario en la hiperplasia
benigna prostática:

1. 1. Finasteride - alopecia.
2. 2. Alfa bloqueantes - hipotensión ortostática.
3. 3. Alfa bloqueantes - eyaculación retrógrada.
4. 4. Finasteride - pérdida de la libido.
Gráfico de respuestas
Comentario

Pregunta que no ts puede permitir fallar el día del ENARM. DebeE recordar que los inhibidores de
la 5- α- reductasa, como la finasterida, no producen alopecia, sino todo lo contrario. De hecho, la
finasteriada se usa en una dosis 5 veces menor que en HBP como tratamiento de la alopecia
androgénica. Como antiandrógeno, es cierto que puede producir disminución de la libido en hasta
un 2% de los casos.(R1)

371. Mujer de 45 años que acude por estreñimiento que al inicio alternaba con periodos
de diarrea, pero que en los últimos meses se ha hecho constante. Como único
antecedente de interés refiere la presencia de hemorroides. El estreñimiento se
acompaña de dolor abdominal y de sensación de flatulencia. No refiere pérdida de peso.
En el estudio de heces aparece sangre y abundante cantidad de moco, pero no existe
pus. Respecto a la enfermedad que sospecha, es cierto:

1. 1. Generalmente la diarrea persiste por la noche y con el ayuno.


2. 2. La exploración física de estos pacientes suele ser normal.
3. 3. El dolor abdominal que calma con la defecación es un dato patognomónico.
Al ser una enfermedad benigna, no está indicado realizar ninguna exploración
4. 4.
complementaria, ya que sería aumentar el gasto sanitario injustificadamente.
Gráfico de respuestas
Comentario

Pregunta básica sobre el colon irritable. Esta pregunta se pregunta en el MIR con cierta frecuencia,
pero en clínica es todavía más común. De hecho, es la enfermedad que más se diagnostica en una
consulta de Gastroenterología.

La característica esencial del síndrome de intestino irritable es el dolor que alivia al defecar, con
alteración en la morfología de las heces y estreñimiento. En ocasiones, puede haber diarrea sin
aumento del volumen líquido como consecuencia del aumento de la frecuencia defecatoria. No
obstante, en general coexiste con estreñimiento. La abundancia de moco en heces también es
típica. Sin embargo, la presencia de sangre en heces, pérdida de peso, anemia o diarrea nocturna
son signos que sugieren organicidad y excluyen este diagnóstico. Lógicamente, en este caso no
podemos valorar la sangre en heces, ya que seguramente es consecuencia de las hemorroides
que nos han mencionado en la pregunta.(R2)

 
 
 
 

372. A patient has undergone a


surgical procedure in order to improve his complete sensorineural deafness. The
physician obtains an X-ray film, as shown in the image. Which of the following options is
the surgical intervention?

1. 1. Insertion of bilateral transtympanic drainage.


2. 2. Stapedectomy with placement of piston prosthesis.
3. 3. Bilateral cochlear implants.
4. 4. Cholesteatoma removal.
Gráfico de respuestas
Comentario

Una pregunta extraordinariamente difícil. Por supuesto, no debe preocuparse si la ha fallado. En


ella, depende completamente de su habilidad para reconocer, en esta radiografía, una estructura
que posiblemente nunca ha visto antes. Si se fija bien, en la región de las órbitas se aprecia, por
transparencia, un macizo óseo en el fondo de las mismas. Corresponde al peñasco del temporal.
Si mira con atención, verá una estructura en forma de espiral, de aspecto arrosariado, que
corresponde a los electrodos del implante. No obstante, como puede imaginar, existía un
procedimiento ganador mucho más fácil. Si la pregunta le dice que hemos hecho “algo” para
remediar una hipoacusia NEUROSENSORIAL, tendrá que elegir una respuesta acorde a esto. Los
drenajes trastimpánicos podrían servir en ciertos tipos de otitis media, cuya hipoacusia es de
transmisión. Lo mismo sucede con la respuesta 2, ya que la estapedectomía es el tratamiento de la
otoesclerosis, también de transmisión. La hipoacusia de los colesteatomas también lo es. Nos
queda, por lo tanto, el implante COCLEAR, que obviamente servirá para solucionar problemas de
CÓCLEA, es decir, oído interno, y por lo tanto una hipoacusia neurosensorial. Un ejemplo más de
cómo lo difícil se convierte en fácil, si no perdemos la perspectiva adecuada.(R3)

373. Una chica de 19 años ha sido ingresada porque, a consecuencia de un aborto


incontrolado, presenta fiebre de 40ºC sin foco aparente. Impresiona de gravedad.
Súbitamente empieza con disnea y taquipnea; en la placa de tórax existen múltiples zonas
de radiotransparencia anormal y la venografía demuestra tromboflebitis pélvica, que
probablemente sea de tipo séptico. ¿Qué tratamiento elegiría usted para esta paciente?:

1. 1. Filtro de Greenfield.
2. 2. Ligadura de la cava.
3. 3. Trombolíticos.

 
 
 
 
4. 4. Embolectomía pulmonar.
Gráfico de respuestas
Comentario

En el examen MIR, lo más preguntado sobre el TEP es la actitud diagnóstico- terapéutica. No


obstante, comprendemos que esta pregunta es más difícil que las habituales, porque nos plantea
una situación excepcional. Aunque el tratamiento estándar del TEP es la anticoagulación con
heparina, en este caso nos presentan una clara indicación de tratamiento invasivo. La ligadura de
la cava inferior se reserva para casos como éste, donde existe una tromboflebitis séptica de origen
pélvico (respuesta 2 correcta). El filtro de Greenfield no es otra cosa que el filtro de cava inferior,
que se emplea para casos en los que la anticoagulación se contraindica, para casos de TEP
recurrente a pesar de anticoagulación adecuada o para grandes trombos flotantes en cava inferior.
Los trombolíticos suelen emplearse en casos de TEP masivo.(R2)

374. Todas las siguientes afirmaciones sobre la nefropatía por analgésicos son ciertas,
excepto una:

1. 1. Se precisa consumir analgésicos en cantidades importantes a lo largo de muchos años.


2. 2. La insuficiencia renal es lentamente progresiva.
3. 3. El daño histológico más importante radica en los glomérulos.
4. 4. La necrosis papilar es una de sus complicaciones.
Gráfico de respuestas
Comentario

La nefropatía por analgésicos ha entrado en alguna ocasión, así que debes conocer lo básico. La
neuropatía por analgésicos produce un daño renal que es, en general, dosis dependiente.
Clínicamente la función renal disminuye gradualmente junto con necrosis crónica de las papilas y
lesión difusa túbulo- intersticial de la corteza renal (el daño NO predomina en los glomérulos).
Recuerda que los efectos de los AINEs sobre el aparato urinario incluyen cinco síndromes: 1)
necrosis tubular aguda tóxica; 2) nefritis intersticial inmunoalérgica aguda; 3) nefritis intersticial
crónica con necrosis papilar; 4) síndrome nefrótico por cambios mínimos; y 5) uroteliomas.(R3)

¿Por qué mecanismo aparece la “hiperinsuflación dinámica” durante el ejercicio en los


enfermos con EPOC?

1. 1. Por alteraciones en la relación “ventilación/ perfusión”.


Por vaciado incompleto de unidades alveolares a causa del aumento de la frecuencia
2. 2.
respiratoria.
Por vaciado incompleto de unidades alveolares a causa de una disminución en la “orden
3. 3.
respiratoria”.
4. 4. Por los problemas mecánicos asociados a una alcalosis respiratoria.
Gráfico de respuestas
Comentario

La hiperinsuflación pulmonar detectada en condiciones pasivas se magnifica cuando aumentan las


demandas ventilatorias de los pacientes, como sucede con la práctica de ejercicio físico. En dichas
situaciones, la espiración deja de ser un proceso pasivo, precisando la contracción de los músculos
espiratorios. Esto, en sujetos sanos produciría una disminución del volumen pulmonar
teleespiratorio, ya que no habría problema para la expulsión del aire. Sin embargo, en un paciente
con EPOC, existe una limitación al flujo espiratorio (recuerda que se trata de una enfermedad
obstructiva…). Por esto, desarrollan un acusado incremento del volumen pulmonar teleespiratorio,

 
 
 
 
alcanzando un nuevo punto de equilibrio a un nivel superior a la capacidad residual funcional. A
este fenómeno, se le denomina hiperinsuflación dinámica. Como la respiración tiene lugar a
mayores volúmenes pulmonares, se incrementa el trabajo respiratorio, porque los músculos
respiratorios tienen que hacer un mayor esfuerzo para llenar los pulmones, puesto que ya estarían
muy llenos. Por ello, la hiperinsuflación dinámica contribuye significativamente a la sensación de
disnea de los pacientes con EPOC.(R2)

375. Las alteraciones de la variabilidad del registro cardiotocográfico se caracterizan por


todo lo siguiente, EXCEPTO:

1. 1. El aumento de variabilidad constituye el signo más precoz de hipoxia fetal leve.


La hipoxia leve produce aumento de la variabilidad que a corto plazo se debe a un
2. 2.
aumento del tono simpático fetal.
3. 3. La variabilidad baja se debe frecuentemente a sueño fetal.
4. 4. El ritmo sinusoidal suele deberse a isoinmunización Rh grave.
Gráfico de respuestas
Comentario

Una pregunta bastante difícil sobre la interpretación del registro cardiotocográfico. No se preocupe
si la ha fallado.

La hipoxia leve produce un aumento de la variabilidad, pero no es debida (a corto plazo) a un


aumento del tono simpático, sino del parasimpático, lo que permite economizar glucógeno a nivel
miocárdico. Si la hipoxia persiste, entonces aparecería el aumento del tono simpático, pero siempre
en segundo lugar, contribuyendo a redistribuir el flujo sanguíneo hacia cerebro, corazón y
suprarrenales.(R2)

376. En las siguientes afirmaciones en relación a la enfermedad inflamatoria pélvica


crónica. Marque lo INCORRECTO.

1. 1. La enfermedad pélvica crónica ocurre en el puerperio tardío.


2. 2. El dolor abdominal y dismenorrea son síntomas de la EIP.
3. 3. La adherencia de los pliegues de las trompas puede deberse a una salpingitis.
4. 4. Ninguna de las anteriores.
Gráfico de respuestas
Comentario

La EIP es una infección de útero, tormpas y ovarios debido a una infección bacteriana ascendente
desde el tracto genital inferior. La causa más frecuente es C. trachomatis y N. gonorrohoeae. El
síntoma más frecuentes es el dolor abdominal bajo, asociado a leucorrea. Entre las secuelas se
encuentra: dolor pélvico crónico, que es el más frecuente, y esterilidad (25%). La opción que no
encaja es la 1, ya que este tipo de patología puede presentarse en cualquier momento, conociendo
los factores de riesgo como: ETS (más importante), uso de DIU y la edad joven.(R1)

377. ¿Que significado tiene hallar en una gammagrafía de perfusión miocárdica un área
hipocaptante (zona "fría")?:

1. 1. Se trata de un error técnico probablemente.


2. 2. No implica nada pues es una prueba muy poco sensible y específica.
Indica la existencia de una zona con intensa angiogénesis (por ejemplo, un
3. 3.
rabdomiosarcoma cardíaco).

 
 
 
 
Tiene el mismo significado que encontrar un área de contractilidad alterada en la
4. 4.
ecocardiografía de stress.
Gráfico de respuestas
Comentario
Un área hipocaptante en una gammagrafía de perfusión miocárdica indica una zona isquémica.
También se denominan zonas frías y se producen porque el flujo sanguíneo coronario a ese nivel
es inadecuado. Para demostrar áreas isquémicas en un eco de estrés, debemos objetivar una
zona que se contrae mal con el estrés (contrayéndose bien basalmente). Por tanto, estas dos
situaciones, como indica la respuesta cinco, es la correcta.(R4)

378. Un paciente de 35 años acude a la consulta por presentar un bulto en el cuello que
presenta desde hace aproximadamente 6 meses, sin que durante este tiempo haya
notado crecimiento del mismo. A la exploración presenta una masa tiroidea dura y pétrea
que no moviliza con la deglución y está adherida a planos profundos. ¿Cuál de las
siguientes afirmaciones NO es correcta?:

1. 1. Tratamiento quirúrgico inicial con biopsia intraoperatoria.


2. 2. La PAAF es obligada.
3. 3. La función tiroidea habitualmente será normal.
4. 4. La gammagrafía tiroidea no aporta datos al diagnóstico.
Gráfico de respuestas
Comentario
Pregunta importante que hace referencia al manejo del nódulo tiroideo. Nos presentan a un varón
joven con un nódulo tiroideo solitario de rápida evolución. Como datos importantes destacan que la
masa es pétrea, adherida a planos profundos y que no se desplaza con la deglución, orientando
hacia un proceso maligno. Recordad que ante un nódulo solitario la primera prueba a solicitar
siempre es la PAAF. La ecografía tiroidea no es siempre obligatoria, dado que si no existen dudas
en la exploración física del tiroides no es necesaria realizarla. La gammagrafía no se solicita nunca,
ya que ante un nódulo hipocaptante, la probabilidad de que se trate de un adenoma folicular es del
80 %, y además los carcinomas foliculares son también hipocaptantes, luego no sirve para el
diagnóstico diferencial. La función tiroidea en los carcinomas foliculares es normal, y sólo se
solicita determinación hormonal en caso de que el paciente presente síntomas de disfunción
tiroidea. Inicialmente la cirugía no se encuentra indicada, y el tipo de la misma dependerá del
resultado de la PAAF.(R1)

379. Mujer con embarazo de 38 semanas de gestación, que ingresa con trabajo de parto.
Durante el periodo de dilatación presenta dolor brusco. A la exploración usted detecta
metrorragia escasa y aumento del tono uterino a la palpación abdominal que resulta muy
doloroso. ¿Cuál sería su diagnóstico?

1. 1. Rotura uterina.
2. 2. Desprendimiento de placenta.
3. 3. Crioamnionitis hemorrágica.
4. 4. Rotura de vasos previos.
Gráfico de respuestas
Comentario

Pregunta muy fácil de un tema que se estudia ampliamente en clase.

 
 
 
 
Dentro del diagnóstico diferencial de las hemorragias del 3er trimestre distinguimos dos grupos: en
las que lo que predomina es el sangrado con un buen estado materno (placenta previa y rotura de
vasos previos) y aquellas que se acompañan de dolor uterino (abruptio y rotura uterina).

En este caso hay dolor predominante sobre la metrorragia, pero nos dicen que hay un aumento del
tono uterino, lo que nos hace orientar nuestro diagnóstico hacia el abruptio y no hacia la rotura
uterina (repuesta 2 correcta).

El tratamiento del abruptio se basa en la terminación de la gestación lo antes posible, utilizando,


como regla general, la cesárea urgente, aunque prefiere utilizarse la vía vaginal si el feto ya está
muerto y las condiciones maternas lo permiten.

En cuanto a las posibles complicaciones del abruptio, es esencial reseñar que es la causa más
común de CID en el embarazo, y que ésta puede causar la muerte materna. Otras complicaciones
son la insuficiencia renal aguda, el útero de Couvelaire y la embolia de líquido amniótico, que es
rara pero grave.

El signo guía de la infección en la embarazada es la fiebre. En este caso no se nos dice que esté
presente y, por el contrario, nos cuentan un cuadro típico del abruptio (respuesta 3 incorrecta).(R2)

380. Tratamiento especifico para trichomoniasis vaginal es:

1. 1. Tetraciclina.
2. 2. Metronidazol.
3. 3. Ketoconazol.
4. 4. Miconazol.
Gráfico de respuestas
Comentario

El tratamiento de elección es el metronidazol. Pueden utilizarse pautas cortas por su fácil


cumplimiento aunque también es posible pautas de siete días, si bien no suele aplicarse como
tratamiento inicial y se reserva para aquellos casos en que el tratamiento en dosis única no ha
resultado eficaz. Al tratarse de una infección de transmisión sexual, es importante el tratamiento de
la pareja sistemáticamente para evitar recidivas. Debe evitarse el consumo de alcohol por su efecto
antabus.(R2)

381. La enfermedad renal poliquística del adulto es un trastorno responsable de


aproximadamente un 10% de los casos de insuficiencia renal en fase terminal. Señale lo
que le parece ERRÓNEO en cuanto a esta enfermedad:

Se hereda con carácter autosómico dominante, estando su gen en el brazo corto del
1. 1.
cromosoma 16.
El parénquima renal intermedio puede ser normal o mostrar nefroesclerosis o nefritis
2. 2.
intersticial.
3. 3. Ocasionalmente se producen episodios de cólico renal por coágulos de sangre.
La hipertensión y la progresión a la insuficiencia renal complican raramente el curso de
4. 4.
esta patología.
Gráfico de respuestas
Comentario

 
 
 
 
La enfermedad poliquística del adulto se caracteriza por la presencia de quistes tubulares renales,
que van creciendo progresivamente, hasta comprimir el resto de estructuras del órgano. A largo
plazo, se produce HTA en un 75% de los pacientes adultos. Esta HTA es secundaria a la isquemia
intrarrenal, por distorsión de la arquitectura y la consecuente activación del sistema renina-
angiotensina (respuesta 5 falsa)

Por otra parte, esta enfermedad asocia aneurismas en el polígono de Willis con cierta frecuencia,
que pueden romperse y producir una hemorragia subaracnoidea. Otra importante asociación son
los quistes hepáticos, que pueden hallarse mediante ecografía en la mayor parte de estos
pacientes.

Por último, es muy característica la ausencia de anemia, e incluso la policitemia, en este tipo de
enfermos. Como sabes, cuando se produce insuficiencia renal crónica por otra causa, es muy
frecuente la asociación con anemia. Esta circunstancia puede no producirse en esta enfermedad,
ya que los quistes son productores de eritropoyetina. Existen, por tanto, dos importantes
distinciones en relación con la insuficiencia renal de otra etiología: menos anemia y más
hipertensión.(R4)

382. El síndrome de Zieve consiste en:

1. 1. Esteatosis hepática alcohólica, anemia hemolítica e hipertrigliceridemia.


2. 2. Hiperesplenismo y esteatosis hepática.
3. 3. Cirrosis biliar primaria, hipergammaglobulinemia y plaquetopenia.
4. 4. Anemia microangiopática, esplenomegalia y hepatitis crónica.
Gráfico de respuestas
Comentario

El síndrome de Zieve es una complicación extremadamente infrecuente de la esteatosis hepática


secundaria al alcohol (hígado graso alcohólico). Se define por la asociación de hipertrigliceridemia,
anemia hemolítica e ictericia en un paciente con esteatosis hepática alcohólica. Los pacientes
suelen tener además dolor abdominal. La trombocitopenia, cirrosis, hiperesplenismo e
hipergammaglobulinemia también pueden manifestarse como parte de este síndrome, aunque no
forman parte de la tríada que lo define.(R1)

383. De los siguientes antiarrítmicos, uno NO aumenta el período refractario del nodo
auriculoventricular:

1. 1. Atenolol.
2. 2. Quinidina.
3. 3. Digoxina.
4. 4. Amiodarona.
Gráfico de respuestas
Comentario
Estamos ante una pregunta muy difícil y poco frecuente en el MIR. Todos los fármacos de esta
pregunta aumentan el tono vagal cardiaco lo que provoca una disminución de la frecuencia
cardiaca y prolongación del periodo refractarioA- V, salvo la quinidina. Esta tiene un mecanismo de
acción muy complejo : bloquea diversas corrientes de salida de k+, lo que explica que prolongue la
duración del potencial de acción y disminuya la contractilidad del corazón, sin embargo a dosis
terapeuticas estas acciones cardiodepresoras son contrarrestadas por sus acciones
antimuscarínicas y por el aumento del tono simpático que su acción vasodilatadora produce. Por
ello, en enfermos con FA puede llegar a acelerar la conducción a través del nodo AV , por lo que
se recomienda digitalizar al paciente.(R2)

 
 
 
 

384. Femenino de 32 años asintomática consulta para una revisión en salud ginecológica
porque desea quedar embarazada. En dicha revisión se le detecta un mioma uterino de 4
cm en parte intramural y en parte subseroso situado en la cara anterior del útero y que
no deforma la cavidad endometrial. ¿Qué actitud le recomendaría?

1. 1. Miomectomía por vía laparotómica.


2. 2. Embolización del mioma por cateterismo arterial.
3. 3. Tratamiento con análogos de Gn-RH durante tres meses antes de intentar el embarazo.
4. 4. Intentar el embarazo sin ningún tratamiento previo.
Gráfico de respuestas
Comentario

En el tratamiento de los miomas el aspecto más importante a tener en cuenta es la presencia o no


de síntomas. En mujeres asintomáticas, como es el caso de la pregunta, no sería necesario el
tratamiento y realizaremos revisiones periódicas para evaluar el crecimiento del mismo.

El hecho de que la paciente esté buscando una gestación, ¿modifica este planteamiento
terapéutico? El papel que juegan los miomas en la fertilidad está poco claro. Por ello, a la hora de
tomar decisiones es importante valorar la localización del mioma. En los miomas submucosos,
dado que se acompañan de una disminución significativa de la tasa de embarazo y de un
incremento del número de abortos, se recomienda tratamiento quirúrgico mediante histeroscopia.

En los miomas intramurales hay mayor controversia pero parece existir consenso en intervenir los
miomas de más de 5 cm y, sobre todo, los que distorsionan la cavidad endometrial. Los miomas
subserosos, ni por su tamaño ni por su localización deben ser extirpados ya que no tienen ninguna
relación con la fertilidad.(R4)

385. La administración o la producción endógena de grandes cantidades de andrógenos


en un niño antes de la pubertad produciría todos los siguientes efectos MENOS:

1. 1. Depósito de sales de calcio en el hueso.


2. 2. Aumento del ritmo de crecimiento.
3. 3. Aumento de la resistencia ósea.
4. 4. Aumento de la talla final del individuo.
Gráfico de respuestas
Comentario
Esta pregunta se puede responder aplicando unos conocimientos básicos sobre la acción
androgénica a nivel óseo. Durante el estirón puberal, en la adolescencia, hay una intensa actividad
del cartílago de crecimiento, marcado por la acción de las hormonas sexuales y el incremento de la
GH. La testosterona estimula, inicialmente, la división celular en la fisis. Los andrógenos tienen un
efecto anabolizante sobre el hueso, a través del estímulo de los receptores de los osteoblastos, por
eso actúan aumentando la matriz ósea, el depósito de sales de calcio y la resistencia ósea. Sin
embargo, las hormonas sexuales aceleran también el cierre epifisario, por lo que, a pesar de
acelerar el ritmo de crecimiento, logran una talla final más baja, pues acortan el periodo de
alargamiento óseo longitudinal. Este hecho determina que se instaure tto en la pubertad precoz
idiopática con análogos de GnRH, así se logra retardar el cierre epifisíario con el fin de mejorar el
pronóstico de talla.(R4)

 
 
 
 
386. ¿Cuál de los siguientes tratamientos es el de elección en las extrasístoles
ventriculares en la fase aguda del infarto de miocardio?:

1. 1. Lidocaína.
2. 2. Potasio intravenoso.
3. 3. Marcapasos temporal.
4. 4. Ansiolíticos.
Gráfico de respuestas
Comentario
Las extrasístoles frecuentes prácticamente son la norma en el infarto agudo de miocardio y no
suelen requerir tratamiento antiarrítmico específico como la lidocaína. El tratamiento es la
monitorización y el tratamiento con betabloqueantes que el paciente ya debe recibir por presentar
un IAM. En caso de otras arritmias que pueden ocurrir en la fase aguda del infarto, la de mayor
repercusión es la fibrilación ventricular primaria que es la causa más frecuente de mortalidad antes
de llegar al hospital. La fibrilación ventricular secundarias (que es la que aparece pasadas las
primeras 48- 72 horas) es muy grave y a diferencia de la FV primaria afecta gravemente al
pronóstico a largo plazo del paciente. Las taquicardias ventriculares monomórficas sostenidas no
suelen ser muy frecuentes en la fase aguda del IAM. El tratamiento es similar al de las TVMS fuera
de la fase aguda del IAM. Si el paciente está hemodinámicamente estable se puede hacer un
intento de cardioversión farmacológica y en ese caso el antiarrítmico de elección es la lidocaína o
la amiodarona iv. Si no cede se realizaría cardioversión eléctrica.(R1)

387. Un paciente intervenido por ulcus duodenal con una antrectomía y


gastroyeyunostomía tipo Billroth II presenta una recaída ulcerosa. La gastrina sérica está
elevada basalmente y aumenta de forma importante tras inyección de secretina. ¿Qué
sospecharía?

1. 1. Adenocarcinoma gástrico.
2. 2. Infección por H. pylori.
3. 3. Antro retenido.
4. 4. Gastrinoma.
Gráfico de respuestas
Comentario

En un paciente sano, la respuesta normal consiste en que la gastrina se inhibe cuando


administramos secretina. Sin embargo, en los que padecen un gastrinoma, se produce una
respuesta paradójica que es precisamente lo contrario: se ELEVA la gastrina cuando
administramos la secretina, que normalmente tendría que inhibirla. Cuando veas este tipo de
respuesta, no lo dudes, es prácticamente seguro que estás ante un gastrinoma (respuesta 4
correcta). En este caso clínico, ya te puedes imaginar el motivo de su recidiva ulcerosa…(R4)

388. Una paciente de 27 años es intervenida de urgencia por un cuadro de dolor


abdominal en fosa ilíaca derecha. En la cirugía se encuentra una tumoración en ciego y
se practica una hemicolectomía derecha. El estudio histopatológico revela la existencia
de un adenocarcinoma estadio B de Dukes. Su padre está diagnosticado de un síndrome
de Muir Torre. La paciente tiene un hijo masculino de 2 años de edad. La conducta
diagnóstico-terapéutica con el hijo debe ser:

1. 1. Colonoscopía a los 23 años de edad y cada 2 - 3 años.


2. 2. Ofrecer la colectomía abdominal total a partir de la mayoría de edad.

 
 
 
 
Screening genético, analizando los genes hMSH2, hMLH1, hPMS1 y hPMH2 en sangre
3. 3.
periférica.
4. 4. Estudio anual de hemorragias ocultas en heces.
Gráfico de respuestas
Comentario

El diagnóstico clínico de esta enfermedad se basa en los criterios de Amsterdam: 3 ó más


familiares afectos de cáncer ce colon (al menos uno que sea familiar de primer grado de los otros
2), uno o más casos en la familia de cáncer colorrectal antes de los 50 años y afectar al menos a
dos generaciones consecutivas.

Como sabe, esta enfermedad se hereda de forma autosómica dominante, con alta penetrancia. El
gen mutante aparece con más frecuencia en los cromosomas 2 (gen hMSH2) y 3 (hMLH1),
afectando a la reparación del ADN. Sin embargo, se han descrito otras mutaciones, por lo que un
test genético negativo no descartaría la enfermedad. Por ello, se recomiendan colonoscopias
periódicas a los familiares en primer grado de estos pacientes. Comenzarán a realizarse a partir
de los 25 años, o 5 años antes que la del familiar más joven afecto de cáncer de colon. A los
familiares de los pacientes con síndrome de Lynch tipo II, también deben realizárseles
mamografías y exploraciones genitales periódicas, dada su asociación al cáncer de mama y
endometrio, entre otros tumores.(R1)

389. Señale la medida terapéutica que NO se utiliza en la hipertensión pulmonar


persistente del recién nacido:

1. 1. Oxido nítrico (NO).


2. 2. Tolazolina.
3. 3. Oxigenación mediante membrana extracorpórea (ECMO).
4. 4. Hipoventilación controlada.
Gráfico de respuestas
Comentario

En el tratamiento de la hipertensión pulmonar del neonato se están ensayando nuevas


modalidades terapéuticas, como el célebre sildenafilo. Pero también se pueden utilizar:
vasodilatadores pulmonares (óxido nítrico, tolazolina, prostaciclina), reposo cardiopulmonar (a
través de la ECMO) y medidas inespecíficas (administración de bicarbonato, hiperventilación
controlada). La hipoventilación provoca el efecto contrario: el exceso de CO2 genera
vasoconstricción pulmonar.(R4)

390. ¿Cuál es la cardiopatía congénita más frecuente en la infancia?

1. 1. Tetralogía de Fallot.
2. 2. Persistencia del conducto arterioso.
3. 3. Coartación de aorta.
4. 4. Comunicación interventricular.
Gráfico de respuestas
Comentario

Pregunta muy importante en el ENARM. La cardiopatía congenita más frecuente es la CIV siempre
que se excluya la válvula bicúspide.

 
 
 
 
La más frecuente de las cianóticas es la D-transposición de los grandes vasos, pero a partir del
año de vida, la cianótica más comun es la tetralogia de Fallot.(R4)

391. Una de las siguientes alteraciones NO es característica de la talasemia grave beta:

1. 1. Aumento de VCM.
2. 2. Aumento de Hb A2.
3. 3. Aumento de Hb F.
4. 4. Malformaciones óseas.
Gráfico de respuestas
Comentario
Se trata de una pregunta bastante sencilla si recordamos que las talasemias cursan típicamente
con disminución del VCM (opción 1 verdadera). El resto de opciones muestran características
típicas de la anemia de Cooley o talasemia major como son el aumento de HbA2 y HbF que tratan
de compensar el déficit de HbA1 al faltar la cadena b. Son típicas las malfomaciones óseas como
consecuencia de la hiperplasia medular y organomegalias (hepatoesplenomegalia) por la
hematopoyesis extramedular y por la hemosiderosis secundaria como ocurre en el caso de la
cardiomegalia, la cual modifica el pronóstico de la enfermedad. Página 11; Manual CTO, 4ª
Edición.(R1)

392. El triple test realizado a las embarazadas mide la alfafetoproteína, el estriol no


conjugado y la HCG. Los resultados que nos hacen sospechar la presencia de un feto
afecto del síndrome de Down son:

1. 1. Alfafetoproteína baja, estriol alto, HCG alta.


2. 2. Alfafetoproteína alta, estriol bajo, HCG alta.
3. 3. Alfafetoproteína baja, estriol bajo, HCG alta.
4. 4. Alfafetoproteína alto, estriol bajo, HCG baja.
Gráfico de respuestas
Comentario

Pregunta fácil que con la siguiente información podrás razonar. Sobre el screening bioquímico
prenatal. Esta pregunta la puede sacar con los conocimientos de clase de primera vuelta.

En el síndrome de Down tenemos una HCG muy elevada, con lo que descartamos la 4.

También sabemos que la alfafetoproteína (a- FP) está descendida; así solo nos quedará elegir
entre la 1 y la 3.

Ahora nos queda el estriol. El estriol es un estrógeno de origen fetal que nos demuestra el
bienestar fetal, ya que para su síntesis intervienen el hígado y las glándulas suprarrenales de dicho
feto.

Así, un estriol bajo indica alteración fetal; la opción que debemos elegir es la 3, que reúne la HCG
alta y la a- FP y estriol bajos.(R3)

393. Acude una mujer de 28 años a su consulta para recoger el resultado de una citología,
que resulta tener células compatibles con un SIL de bajo grado. Se realiza una
colposcopia, que demuestra un área que no capta yodo en la tinción de Schiller. Tras

 
 
 
 
realizar una biopsia de dicha área, se confirma el diagnóstico. ¿Cuál sería la actitud
terapéutica más adecuada?

1. 1. Destrucción amplia de dicho área por crioterapia o termocoagulación.


2. 2. Histerectomía simple.
3. 3. Vigilancia periódica.
4. 4. Histerectomía radical.
Gráfico de respuestas
Comentario

El cáncer de cérvix es un tema muy preguntado, puesto que los métodos de screening poblacional
han demostrado disminuir la mortalidad por esta neoplasia. Los hallazgos colposcópicos que debes
considerar anormales son:

Epitelio yodonegativo o acetoblanco.

Leucoplasia debida a necrosis y/o queratinización.

Neoformación vascular.

Ulceración epitelial.

En el caso que nos presentan, está claro que se trata de una colposcopia anormal, de hecho nos
confirman con biopsia que es un SIL de bajo grado. Ante esto, la actitud debe consistir en la
destrucción de esa zona anómala, sea con crioterapia, láser, termocoagulación… No te confundas
con el SIL de alto grado, en cuyo caso la respuesta correcta hubiera sido la conización.(R1)

394. Todas las afirmaciones sobre la enterocolitis necrosante (ECN) son ciertas,
EXCEPTO:

Su incidencia es del 1 al 5 por 100 de los ingresos de recién nacidos en unidades de


1. 1.
cuidados intensivos.
2. 2. La situación de hipoxia no favorece este proceso.
3. 3. El estrés perinatal es un factor predisponente.
4. 4. El diagnóstico depende de mantener una sospecha constante en dicha patología.
Gráfico de respuestas
Comentario

La enterocolitis necrotizante afecta fundamentalmente a íleon distal y colon proximal.

Suele relacionarse con situaciones de hipoxia y bajo gasto (respuesta 2 correcta).

Clínicamente, se sospecha ante la presencia de distensión abdominal y deposiciones


sanguinolentas.

Existen varios signos radiológicos, pero el más sugestivo es la neumatosis intestinal, que es
diagnóstico de esta enfermedad.

 
 
 
 
El tratamiento consiste en dieta absoluta, sonda nasogástrica y sueroterapia. Debe administrase
además antibioticoterapia, cubriendo gérmenes anaerobios y gramnegativos. En caso de
perforación intestinal o refractariedad a antibióticos, se plantearía tratamiento quirúrgico.(R2)

395. ¿Cuál de las siguientes afirmaciones respecto a la valoración del dolor abdominal
agudo en el anciano es cierta?

Comparado con los jóvenes, en los pacientes ancianos son menos frecuentes los problemas
1. 1.
como la apendicitis o la colecistitis.
Para una misma causa de abdomen agudo, los ancianos tienen el mismo pronóstico que los
2. 2.
jóvenes.
En los ancianos, la forma atípica de presentación de la enfermedad y el retraso en el
3. 3.
diagnóstico conducen a una mayor mortalidad que en los jóvenes.
Ante un dolor abdominal agudo, generalmente los ancianos piden asistencia médica antes
4. 4.
que los jóvenes.
Gráfico de respuestas
Comentario

Pregunta de dificultad media sobre el abdomen agudo, en la que es necesario razonar las
opciones.

La primera opción es falsa porque estos cuadros son más frecuentes en grupos de edad extrema.

Para la misma causa, está claro que los ancianos tienen peor pronóstico que los jóvenes (opción 2
falsa).

Esta presentación más anodina y con menor dolor hace que se retrase la asistencia médica
(opción 4 falsa).

La respuesta correcta es la 3, puesto que todas las características anteriores hacen que la
mortalidad sea mayor en estos pacientes.(R3)

396. Hombre de 55 años remitido por el cardiólogo para valorar si el dolor retroesternal
que padece desde hace un año es de origen esofágico. Ha sufrido diez episodios
nocturnos de dolor , sin asociación con la ingesta, pirosis, ni cortejo vegetativo
acompañante. En una endoscopia se obseva una mucosa normal. Una pHmetría
ambulatoria de 24 horas fue informada como normal. ¿Cual de entre las siguientes
medidas diagnósticas recomendaría a continuación?:

1. 1. Test de Bernstein.
2. 2. Estudio baritado de esófago-estómago.
3. 3. Endoscopia a los tres meses.
4. 4. Manometría esofágica.
Gráfico de respuestas
Comentario
La endoscopia es de utilidad en la enfermedad por reflujo gastroesofágico para determinar si existe
esofagitis y su severidad. No obstante, no está indicada si la clínica que presenta es
exclusivamente pirosis y/o regurgitación ácida, en cuyo caso no se precisan pruebas diagnósticas.
La pHmetría, siendo la prueba de elección, se reserva para casos refractarios al tratamiento,
quirúrgicos o con presentación predominantemente extraesofágica. El test de Bernstein es una
prueba de provocación con ácido muy diluido, que a estos pacientes les provoca dolor.(R1)

 
 
 
 

397. ¿Cuál de las siguientes es FALSA sobre la atención al paciente politraumatizado?:

El acrónimo ABCDE (Airway, Breathing, Circulation, visceral Damage y Extremities) es


1. 1.
la base para priorizar sus lesiones.
La comprobación de una ventilación adecuada es la primera medida a tomar en estos
2. 2.
pacientes.
La Rx AP de pelvis y tórax y lateral de columna cervical son las primeras pruebas
3. 3.
exploratorias complementarias que utilizamos.
Es recomendable tomar 2 vías venosas periféricas cortas de grueso calibre para la
4. 4.
perfusión de líquidos.
Gráfico de respuestas
Comentario

El tema del manejo del politraumatizado es un tema muy útil en el ENARM, y además esta
pregunta nos resume los conceptos más importantes sobre este tema. La repuesta 1, es uno de los
conceptos más importantes, y nos da la respuesta falsa, ya que el primer paso a realizar ante un
accidentado es mantener la vía aérea permeable (airway) con la extracción de cualquier cuerpo
extraño que pueda haber, por lo que la respuesta 2 es la falsa, ya que la comprobación de la
ventilación sería el segundo paso (Breathing). El tercer paso, sería controlar la circulación, por lo
que ante una hipotensión se colocarán 2 vías venosas, para pasar suero o hemoderivados.(R2)

398. En el curso de un parto de nalgas, estando éstas encajadas, la expulsión de meconio


se traduce en:

1. 1. Sufrimiento fetal.
2. 2. Anoxia fetal.
3. 3. Compresión abdominal.
4. 4. Compresión y/o prolapso de cordón.
Gráfico de respuestas
Comentario

Pregunta difícil sobre el parto de nalgas. El meconio puede hacernos pensar que existe sufrimiento
fetal, pero su presencia no siempre tiene este significado. Por ejemplo, en las presentaciones
podálicas (como en este caso) es mucho más frecuente, sobre todo cuando las nalgas están
encajadas, ya que las piernas del feto comprimen el abdomen fetal en cada contracción, lo que
justifica este hallazgo.(R3)

399. Ante un paciente de 37 años con antecedentes de psoriasis en la infancia y que,


desde hace 7 días, presenta un eritema descamativo que afecta a la practica totalidad del
tegumento, el tratamiento más adecuado será:

1. 1. Etretinato.
2. 2. Isotretinoina.
3. 3. Talidomida.
4. 4. Fluticasona tópica.
Gráfico de respuestas
Comentario
Se trata de una pregunta un tanto compleja que requiere un amplio conocimiento del psoriasis. En
principio no te debe preocupar fallarla. El caso clínico describe una eritrodermia (eritema y

 
 
 
 
descamación que afecta a más del 90% de la superficie cutánea) en este caso de etiología
psoriásica. En el tratamiento de la eritrodermia psoriásica no están indicados en principio los
corticoides orales. Suele preferirse metotrexate, acitretino o ciclosporina. Por lo tanto, la opción
correcta es la 1) Pág 173 Dermatology. Bolognia(R1)

400. Una paciente de 45 años consulta por la aparición de contracciones involuntarias en


la muñeca izquierda, que se flexiona contra su voluntad quedando en una contracción
mantenida y dolorosa. El fenómeno desaparece con el sueño. El trastorno la incapacita
considerablemente. El resto de la exploración es normal. En el manejo de la paciente la
opción terapéutica más adecuada es:

1. 1. Benzodiazepinas a dosis de relajante muscular.


2. 2. Anticomiciales del perfil de la carbamazepina.
3. 3. Tizanidina.
4. 4. Infiltración de toxina botulínica.
Gráfico de respuestas
Comentario
Se describe una distonía focal en flexión de la mano. La opción terapéutica más adecuada en una
distonía focal moderada- severa es la infiltración de toxina botulínica. Las benzodiacepinas, el
baclofeno y la tizanidina se emplean en distonías leves. Para distonías moderadas o graves no
focales se puede recurrir a anticolinérgicos(R4)

401. En relación con los hipoglicemiantes orales es cierto que:

1. 1. La acarbosa aumenta la secreción de insulina.


2. 2. La metformina reduce la producción hepática de glucosa.
3. 3. No pueden asociarse a la administración de insulina.
4. 4. Con glibenclamida el riesgo de hipoglicemias es mínimo.
Gráfico de respuestas
Comentario

Los antidiabéticos orales son un grupo de fármacos difíciles de estudiar. No obstante, su


conocimiento es obligado para el ENARM, por lo extendido de su uso y porque se preguntan con
mucha frecuencia. Sobre esta pregunta, deberías saber:

•   R1: la acarbosa no incrementa la secreción de insulina. Sí que son secretagogos las


sulfonilureas y las meglitinidas (repaglinida, nateglinida).
•   R2: la metformina (como otras biguanidas) es normoglucemiante (no hipoglucemiante,
como sí lo serían las sulfonilureas). Disminuyen la resistencia a la insulina en el ámbito
hepático, por lo que se reduce la gluconeogénesis hepática. Dado que tienen cierto efecto
anorexígeno, están especialmente indicadas en pacientes obesos.
•   R3: los hipoglucemiantes orales pueden combinarse con insulina.
•   R4: la glibenclamida pertenece al grupo de las sulfonilureas, de modo que sí es capaz de
producir hipoglucemias. Ha de recordar que las hipoglucemias por sulfonilureas son menos
frecuentes que por insulina, pero más graves y prolongadas (obligan a mantener al
paciente bajo observación al menos 24 horas).

(R2)

 
 
 
 

Características de los antidiabéticos orales

402. Paciente de 49 años con mioma uterino de tamaño equivalente a gestación de 12


semanas, que tiene hipermenorrea y Hb: 9. No existe patología asociada. La paciente está
en lista de espera para histerectomía programada para dentro de 3 – 4 meses. En esta
paciente podría estar indicado el tratamiento preoperatorio con:

1. 1. Ergóticos dopaminérgicos.
2. 2. Inhibidores de fibrinólisis.
3. 3. Análogos de GnRH.
4. 4. Ninguna de las anteriores.
Gráfico de respuestas
Comentario

La repuesta correcta es el uso de análogos de GnRH, ya que disminuyen el volumen y la


vascularización de los miomas. No producen muerte celular, por ello al finalizar el tratamiento,
vuelven a crecer. Las ventajas de su uso previo a la cirugía son las siguientes: disminuyen la
hemorragia intraoperatoria, facilitan la resección laparoscópica en miomas subserosos e inducen
atrofia endometrial.(R3)

 
 
 
 

403. Neonato de 32 semanas de edad


gestacional, ingresado en la UCIN. Presenta a los 16 días de vida, distensión abdominal,
deposiciones hemorrágicas e inestabilidad hemodinámica. En sus antecedentes
personales destaca distrés respiratorio inmediato, que precisó intubación endotraqueal.
Inicia alimentación enteral con fórmula adaptada a los 3 días de vida. ¿Cuál sería su
actitud inicial?

1. 1. Colocar una sonda rectal por posible cólico del lactante.


2. 2. Dieta absoluta, reposición hidroelectrolítica y antibioticoterapia.
3. 3. Observación y repetir la exploración en unas horas.
4. 4. Mantendría la alimentación oral y solicitaría coprocultivo.
Gráfico de respuestas
Comentario

La combinación de deposiciones sanguinolentas y distensión abdominal en un lactante es


prácticamente definitoria de una enterocolitis necrotizante. El resto de los datos son compatibles y
los antecedentes apoyan este diagnóstico (enfermedad de membrana hialina). El tratamiento de
esta enfermedad consiste en dieta absoluta y antibioterapia intravenosa. Respuesta 2 correcta.(R2)

404. Se realiza radiografía de abdomen y se observa la siguiente imagen, junto con la


siguiente biometría hemática: leucocitos 5,000 (neutrófilos 40%, linfocitos 40%, bandas
15%, basófilos 5%), Hb 10 g/dL, plaquetas 150,000/mm3. ¿Qué patología le sugiere?

1. 1. Alergia a las proteínas de la leche de vaca.


2. 2. Megacolón agangliónico.
3. 3. Gastroenteritis.
4. 4. Enterocolitis necrotizante.
Gráfico de respuestas

 
 
 
 
Comentario

Esta pregunta, en realidad, es un regalo. En la pregunta se aportan más datos, pero no son
necesarios. La imagen que puedes ver en la radiografía muestra distensión de las asas
intestinales, muy similar a la que puedes encontrar en el capítulo correspondiente del Manual CTO,
pero insistimos en que no era necesaria. Respuesta 4 correcta.(R4)

405. Señale la afirmación FALSA respecto al manejo de los RNPT:

La administración prenatal de corticoides a la madre tiene más beneficios en los embarazos


1. 1.
de > 34 semanas.
El beneficio óptimo de la administración prenatal de corticoides se observa a las 24-48
2. 2.
horas después de iniciar el tratamiento.
La administración de surfactante exógeno mejora la supervivencia de la EMH, pero no
3. 3.
disminuye la incidencia de displasia broncopulmonar.
La canalización de la vena umbilical puede tener como complicación una trombosis de la
4. 4.
vena porta.
Gráfico de respuestas
Comentario

La respuesta que debe seleccionar es la 1 ya que la administración de surfactante es para los


embarazos < a 34 SDG. El resto de opciónes son verdades, por lo que no debió elegirlas.
Recuerde que el uso de oxígeno suplementario es factor de riesgo para el desarrollo de DBP.(R1)

406. Todas las afirmaciones siguientes sobre el kwashiorkor son ciertas, EXCEPTO:

1. 1. Es una enfermedad de la segunda infancia.


2. 2. Es principalmente una malnutrición calórica.
3. 3. Hay aumento del hígado con infiltración grasa.
4. 4. Los signos clínicos son: detención del crecimiento, pérdida de masa muscular y edema.
Gráfico de respuestas
Comentario

Esta pregunta de malnutrición es importante a la hora de estudiar el kwashiokor pues resume las
principales características de este cuadro. El kwashiokor es un síndrome clínico secundario a un
déficit grave de proteínas. Es la forma más grave y prevalente de malnutrición en el mundo.
Kwashiokor significa "niño depuesto", es decir, niño que deja de mamar, se suele presentar
después del destete. Los datos clínicos mas relevante son crecimiento inadecuado, falta de
energía, perdida de tejido muscular, edemas y aumento de la sensibilidad a las infecciones. La
dermatitis es habitual. El hígado puede aumentar de tamaño y la infiltración grasa es habitual. Se
reducen las proteínas de síntesis hepática. Recuerde que en el marasmo o desnutrición calórica,
predomina la pérdida de peso y tejido adiposo.(R2)

407. Paciente de 32 años que acude a urgencias por fiebre puerperal a los tres días del
parto, loquios malolientes, dolor abdominal y en USG se observa útero vacío. Se le
ingresa con diagnóstico de endometritis puerperal. Se instaura tratamiento con
ampicilina, gentamicina y clindamicina iv. y oxitocina. En la evolución no se observa
mejoría con el tratamiento instaurado, presentando dolor continuo abdominal, fiebre en
agujas sobretodo por las tardes y escalofríos. ¿Cuál sería su sospecha diagnóstica?:

1. 1. Hematoma en la zona de la episiotomía.

 
 
 
 
2. 2. Endmetritis resistente al tratamiento antibiótico.
3. 3. Tromboflebitis pélvica séptica.
4. 4. Fiebre de origen medicamentoso.
Gráfico de respuestas
Comentario

La tromboflebitis pélvica séptica es una complicación puerperal rara, que puede aparecer como
complicación de un parto vaginal o más frecuentemente después de una endometritis postcesárea
(1-2%). El embarazo predispone a la formación de trombos y también el antecedente de una
endometritis. La tromboflebitis pélvica séptica puede presentarse como trombosis aguda de la vena
ovárica (fiebre en agujas, escalofríos y dolor abdominal constante, severo y localizado) o como
fiebre de origen desconocido. El diagnóstico debe considerarse también en aquellas pacientes con
endometritis en las que un tratamiento antibiótico correcto no hace descender la fiebre. El
diagnóstico se realiza a través de la historia clínica y anamnesis detalladas y excluir otras causas
de fiebre puerperal, BH, TAC (con contraste se objetiva la zona trombosada), RMN. El tratamiento
se realiza con heparina y antibióticos de amplio espectro (los mismos empleados para el manejo de
una endometritis) o ligadura de las venas infectadas en los casos de fracaso del tratamiento
médico o cuando aparece un tromboembolismo pulmonar a pesar de la anticoagulación.(R3)

408. Señale el enunciado FALSO en relación a la termorregulación:

1. 1. La temperatura neutral es mayor a menor edad gestacional.


2. 2. La termogénesis química se realiza en el tejido graso pardo.
3. 3. El calor se puede perder por convección, conducción y radiación únicamente.
4. 4. En ambiente térmico neutral el consumo de oxigeno es mínimo.
Gráfico de respuestas
Comentario

El calor se puede perder por convección, conducción, radiación y evaporación, por lo que la
respuesta correcta es la 3.(R3)

409. ¿Cuál es la actuación quirúrgica a seguir en un paciente laparotomizado por


traumatismo abdominal en el que se descubre un hematoma retrohepático de pequeño
tamaño?

1. 1. Apertura del retoperitoneo y drenaje.


2. 2. Apertura del retoperitoneo y taponamiento urgente.
Evitar la apertura del retroperitoneo, excepto si el hematoma aumenta progresivamente de
3. 3.
tamaño.
4. 4. Apertura del retroperitoneo y sutura de las lesiones vasculares presentes.
Gráfico de respuestas
Comentario

Dado que se trata de un hematoma de PEQUEÑO tamaño, en principio no habría necesidad de


drenarlo, ya que difícilmente producirá síntomas por compresión de alguna víscera u ocupación de
espacio. Por otra parte, este tipo de hematomas pueden reabsorberse de forma espontánea, que
es lo que suele ocurrir, por lo que no precisan ningún gesto quirúrgico añadido salvo
complicación.(R3)

410. En relación a la artrosis y sus manifestaciones clínicas, señale la afirmación que NO


es correcta:

 
 
 
 
1. 1. Los nódulos de Heberden aparecen por artrosis de las interfalángicas distales
2. 2. La rizartrosis tiene predominio femenino y suele ser pauci-asintomática.
3. 3. La coxartrosis se manifiesta inicialmente con dolor y limitación de la rotación interna
4. 4. La artrosis de rodilla tiene predominio masculino, siendo en la mayoría de casos bilateral.
Gráfico de respuestas
Comentario
A pesar de ser un tema poco preguntado, en los últimos años las preguntas han ido en aumento.
Debes recordar en la mano, la artrosis trapeciometacarpiana o rizartrosis que afecta al pulgar; los
nódulos de Bouchard de las interfalangicas proximales (regla: la B va antes que la H) y los de
Heberden, a nivel interfalángico distal. Todas ellas muestra un predominio femenino. Recuerda
también la artrosis de rodilla, que al igual que las anteriores tiene predominio femenino y, como
parece lógico suele presentarse de forma unilateral para, con la evolución, hacerse bilateral
(opción 5 falsa). Por último, es importante saber que en la exploración de la artrosis de cadera, el
hallazgo más precoz es la limitación de la rotación interna.(R4)

411. Respecto a la menopausia, la afirmación CORRECTA es:

1. 1. Lo más eficaz es el estradiol oral.


2. 2. El tratamiento debe incluir siempre estrógenos y progestágenos.
3. 3. Si hay contraindicación para los estrógenos, debemos al menos administrar progestágenos.
4. 4. Si aparece tensión mamaria, debemos bajar la dosis de estrógenos.
Gráfico de respuestas
Comentario

La tensión y el dolor mamario son síntomas derivados de un hiperestrogenismo, como también


puede ocurrir en la mastopatía fibroquística. Por tanto, en las pacientes con terapia hormonal
sustitutiva (THS), habría que disminuir la dosis de estas hormonas (respuesta 4 correcta).

Respecto a la opción 1, conviene decir que el estradiol es igualmente eficaz independientemente


de la vía. Lo que puede variar en función de ésta son los efectos secundarios.

En pacientes histerectomizadas, no es necesario añadir progestágenos (respuesta 2 falsa), ya que


no existiría riesgo de cáncer de endometrio, como sucede en otros casos. Por otra parte, carece de
sentido administrar progestágenos solos (opción 3 falsa). Los síntomas climatéricos dependen del
déficit estrogénico. La razón para administrar progestágenos es para evitar el aumento de riesgo
de cáncer de endometrio.(R4)

412. Mujer de 32 años con un nódulo de 2 cm en la mama derecha sospechoso de


malignidad en el ultrasonido mamario. Se realiza tumorectomía y biopsia selectiva del
ganglio centinela y el resultado anatomopatológico informa de un carcinoma ductal
infiltrante, mal diferenciado, con receptores hormonales positivos, Her-2-neu negativo y
ganglio centinela negativo. ¿Qué tratamiento adyuvante añadiría a continuación?:

1. 1. Radioterapia.
2. 2. Radioterapia más quimioterapia.
3. 3. Radioterapia más hormonoterapia.
4. 4. Radioterapia más quimioterapia más hormonoterapia.
Gráfico de respuestas
Comentario

 
 
 
 
La radioterapia del lecho mamario está indicada por haber practicado cirugía conservadora
(tumorectomía). La quimioterapia está indicada porque, si bien la axila es negativa, la paciente
tiene otros factores de mal pronóstico (edad <35 años y tumor indiferenciado). La hormonoterapia
está indicada por tener receptores hormonales positivos y en este caso, al ser premenopáusica, el
fármaco de elección sería tamoxifeno.(R4)

413. Paciente afectado de enfermedad de Addison en tratamiento sustitutivo. Ante la


aparición de una gastroenteritis con intolerancia digestiva y fiebra alta, ¿Cuál sería la
actitud a tomar?:

1. 1. Aumentar la dosis y administrar los gluconcorticoides vía rectal.


2. 2. Seguir con la dosis habitual por vía intravenosa y rehidratación.
3. 3. Reducir la dosis hasta la mejoría y administrar los glucocorticoides vía oral.
4. 4. Aumentar la dosis y administrar los glucocorticoides vía parenteral.
Gráfico de respuestas
Comentario
En esta pregunta se repasa la actitud ante un paciente con insuficiencia suprarrenal primaria en
situación de estrés agudo. Si un paciente con esta patología presenta fiebre o cualquier otra
situación de estrés, se debe incrementar la dosis de glucocorticoide. En situaciones de estrés leve,
se aumentará 3 veces la dosis, y en los casos graves, se multiplicará por 10 la dosis habitual.
Como el paciente presenta una intolerancia digestiva, la vía oral de administración no es la
adecuada, y se deberán aportar los glucocorticoides por vía intravenosa.(R4)

414. En la rosácea, NO encontrará una de las siguientes lesiones:

1. 1. Telangiectasias.
2. 2. Pápulas eritematosas.
3. 3. Pústulas.
4. 4. Comedones.
Gráfico de respuestas
Comentario
La rosácea es una enfermedad crónica de patogenia desconocida que afecta más frecuentemente
a mujeres de edad media. La clínica comienza con episodios de flushing facial en respuesta a
ciertos estímulos (comidas picantes o calientes, alcohol, aumento de temperatura etc.).
Progresivamente aparece eritema persistente así como telangiectasias y papulopústulas, sin
embargo en ningún momento de la evolución de la enfermedad se forman comedones. Otras
complicaciones que pueden observarse son las lesiones oftálmicas (conjuntivitis, blefaritis, iritis) e
hiperplasia de partes blandas, más frecuente en varones, como son el rinofima, otofima y
blefarofima.(R4)

415. En la edad pediátrica, la infección oportunista más frecuente en el SIDA es:

1. 1. Esofagitis por Candida.


2. 2. Infección diseminada por Mycobacterium avium intracellulare.
3. 3. Neumonía por Pneumocystis jirovecii.
4. 4. Infección crónica por virus del Herpes simplex.
Gráfico de respuestas
Comentario

 
 
 
 
Pregunta de dificultad media- alta. La respuesta correcta es la 3. Debemos de saber, en
comparación con el SIDA del adulto, que las infecciones oportunistas infantiles son menos
frecuentes pero de mayor gravedad. Entre ellas, la más frecuente es la neumonía por P. jirovecii,
que además es la causa más frecuente de muerte en estos niños. El tratamiento de elección de
esta patología, como en el adulto, es el trimetoprim/sulfametoxazol.(R3)

416. Respecto a la técnica quirúrgica indicada en un tumor de recto situado a 6 cm del


margen anal y que es susceptible de resección radical, indique la respuesta
INCORRECTA:

1. 1. El mesorrecto debe seccionarse 5 cm debajo del tumor.


2. 2. Debe realizarse una escisión mesorrectal total.
3. 3. No está indicada la resección anterior baja si el paciente asocia incontinencia fecal previa.
4. 4. A veces un reservorio de colon o íleon permite mejorar la continencia.
Gráfico de respuestas
Comentario

En los tumores de recto medio y bajo (hasta aproximadamente 12 cm del margen anal) se debe
realizar como técnica en cirugía radical una resección anterior baja (RAB) que incluye la extirpación
del mesorrecto en su totalidad (EMRT).

Si el tumor se sitúa en el recto alto, por encima de la reflexión peritoneal, es correcto seccionar el
mesorrecto 5 cm por debajo del mismo.

Clásicamente la EMRT asociaba una ileostomía de protección temporal, aunque ya existen centros
donde no lo hacen de rutina y sólo la realizan cuando el riesgo dehiscencia es alto (desnutrición,
ultrabajas…).

No está indicada la RAB si el paciente presenta incontinencia fecal previa al tumor, ya que es una
técnica compleja y con importante riesgo de complicaciones, que no va a mejorar su calidad de
vida, en tales casos se suele optar por una amputación abdominoperineal.

Por último, los reservorios de colon o de ileon (en caso de colectomías totales) mejoran la calidad
de vida de los pacientes sometidos a proctectomía, al mejorar la continencia y el ritmo
defecatorio.(R1)

417. Sería menos probable que la hemoptisis estuviera presentes en:

1. 1. TB pulmonar.
2. 2. Neumonía viral.
3. 3. Adenoma bronquial.
4. 4. Estenosis mitral.
Gráfico de respuestas
Comentario

Se trata de una pregunta fácil sobre hemoptisis. Todas la enfermedades pulmonares pueden
producir hemoptisis y mas en un examen como el ENARM (el puede siempre es nuestro amigo),
sin embargo nos preguntan cual es menos probable y la respuesta correcta es la 2 neumonía viral,
se trata de un proceso inflamatorio, no invasivo, que rara vez producirá hemoptisis.

 
 
 
 
El resto de opciones es probable que la produzcan o incluso que sea el síntoma de debut de la
enfermedad. la TB por invasión y formación de cavernas, el adenoma puede sangrar por el propio
tumor y la estenosis mitral por congestión.(R2)

418. Señale la afirmación CORRECTA respecto al test de Apgar:

1. 1. Se debe practicar únicamente a los RN patológicos o supuestamente patológicos.


2. 2. Entre los parámetros que se valoran se encuentra la frecuencia respiratoria.
3. 3. Un test de Apgar de 4 al minuto de vida implica un alto riesgo de parálisis cerebral.
4. 4. La palidez cutánea generalizada supone una puntuación de 1 en el test de Apgar.
Gráfico de respuestas
Comentario

Los parámetros que se valoran el test de APGAR incluyen el Aspecto, Pulso, Gesto, Actitud y
Respiración.(R2)

419. Niño de 18 meses que presenta diarrea con abundantes evacuaciones, distensión
abdominal y borborigmos. En el examen físico destacan excoriaciones en el área del
pañal. El examen de las heces demuestra un pH bajo; el test de sustancias reductoras es
positivo. El diagnóstico más probable será:

1. 1. Déficit aislado de isomaltasa.


2. 2. Gastroenteritis aguda infecciosa.
3. 3. Déficit de lactasa.
4. 4. Enfermedad celiaca.
Gráfico de respuestas
Comentario

En la enfermedad celiaca los niños presentan diarrea con heces voluminosas y pastosas, puede
acompañarse de estancamiento de la curva pondera y en ocasiones de la talla, con disminución de
la masa muscular, sobre todo en zonas proximales, vómitos, anorexia, palidez, irritabilidad y
distensión abdominal. Otros síntomas: prolapso rectal, hipoplasia del esmalte, úlceras orales y
acropaquias.

La intolerancia a la lactosa cursa con diarrea acuosa, distensión abdominal, meteorismo y eritema
perianal. El diagnóstico se puede hacer por la clínica, si bien se puede recurrir a pruebas
complementarias como la bioquímica fecal o el teste de H2 espirado. En esta patología las heces
son ácidas y la detección de cuerpos reductores es positiva. El tratamiento consiste en la retirada
de la lactosa de la dieta durante un tiempo que varía entre 4 y 6 semanas hasta la remisión del
cuadro. Se realizará la reintroducción paulatina de la misma.(R3)

420. ¿Cuál de los siguientes es el tipo más frecuente de hernia de la pared abdominal?:

1. 1. Hernia umbilical.
2. 2. Hernia crural.
3. 3. Hernia inguinal indirecta.
4. 4. Hernia inguinal directa.
Gráfico de respuestas
Comentario

 
 
 
 
Es habitual encontrar preguntas sobre hernias en el ENARM. Es importante que conozca todos los
tipos posibles y además añadas algunos datos epidemiológicos que seguro serán de mucha
utilidad. La hernia más frecuente es la hernia inguinal indirecta. Recuerde que ésta protruye desde
el orificio profundo del conducto inguinal; es decir los vasos epigástrico quedan de forma medial a
esta hernia. Se dirige hacia el escroto. Es la más frecuente tanto en mujeres como en
hombres.(R3)

421. Paciente femenino de 55 años, con menopausia a los 53 años (diagnóstico clínico y
hormonal) y antecedentes de histerectomía total simple por gran mioma uterino hace 8
años, que acude a consulta por intensa clínica vasomotora, trastornos ocasionales del
sueño, con dificultad para conciliar el sueño y leve atrofia genital. Solicita tratamiento
para mejorar su calidad de vida, por lo que estaría indicado:

1. 1. Estrógenos.
2. 2. Aporte de soja para mejorar la clínica climatérica y calcio oral exclusivamente.
3. 3. THS con estrógenos y gestágenos ya que la combinación evita la patología endometrial
4. 4. Calcio oral y raloxifeno.
Gráfico de respuestas
Comentario

La respuesta correcta es la 1, puesto que los estrógenos son la primera opción terapéutica para
tratar la clínica vasomotora de la menopausia, siempre y cuando no exista contraindicación. En
este caso, no es necesaria la combinación con progetágenos ya que la paciente no tiene útero (no
hay riesgo de cáncer de endometrio). El calcio y los moduladores selectivos de los receptores de
estrógenos se utilizan para la prevención de la osteoporosis.(R1)

422. Ante un varón de 30 años de edad con clínica de hematuria y hemoptisis con
infiltrados pulmonares y patrón p-ANCA +, ¿qué pediría en primer lugar?:

1. 1. c-ANCA.
2. 2. Anticuerpo antimembrana basal glomerular.
3. 3. Biopsia renal.
4. 4. HLA-DRW3.
Gráfico de respuestas
Comentario
Esta pregunta es fácil aunque si no te andas con ojo puedes confundirte. La asociación de
hemoptisis y hematuria nos debe hace pensar en un síndrome de Goodpasture cuya causa más
frecuente es una Enfermedad de Goodpasture, forma debida a la existencia de anticuerpos
circulantes antimembrana basal glomerular. Las opciones 2 y 5 no tienen ninguna relación por lo
que de entrada podemos descartarlas. Las opciones 1, 3, y 4 pueden hacerte dudar. Recuerda que
el 85% de los pacientes presentan HLA DR2 pero esto no significa que todo paciente HLA DR2
tendrá la enfermedad por lo que no nos sirve. La biopsia renal sí que da la confirmación
diagnóstica mediante inmunofluorescencia (vemos un depósito lineal) pero no la podemos pedir de
entrada. Finalmente la petición inicial del título de anticuerpos antimembrana basal glomerular es
una prueba accesible, poco invasiva y que nos puede orientar el diagnóstico por lo que es la
primera prueba a pedir para este paciente.(R2)

423. Con respecto a la llamada "cronología de Murphy" relativa a la sintomatología de la


apendicitis aguda, señale cuál es el orden correcto:

 
 
 
 
1. 1. Epigastralgia, dolor en fosa ilíaca derecha, náuseas y/o vómitos y febrícula.
2. 2. Epigastralgia, febrícula, dolor en fosa ilíaca derecha y náuseas y/o vómitos.
3. 3. Epigastralgia, febrícula, náuseas y/o vómitos y dolor en fosa ilíaca derecha.
4. 4. Epigastralgia, náuseas y/o vómitos, febrícula y dolor en fosa ilíaca derecha.
Gráfico de respuestas
Comentario

La inflamación del apéndice es la patología quirúrgica aguda más frecuente del abdomen y no falta
su presencia en el ENARM. Su historia natural comienza con la obstrucción de la luz, seguido de
distensión, compromiso del drenaje venoso y sobrecrecimiento bacteriano. Esta etapa de
apendicitis focal es percibida por el paciente como dolor visceral mal localizado principalmente
alrededor de la zona periumbilical. La evolución de las manifestaciones pasa por los vómitos y la
afectación sistémica. Cuando la invasión bacteriana se extiende a la pared del apéndice, fase
supurativa, se inflama el peritoneo parietal y entonces el dolor sordo pasa a ser somático y más
localizado, es en este momento cuando el paciente lo percibe como desplazamiento clásico del
dolor a la fosa ilíaca derecha.(R4)

424. Un anciano con TBC e ICC que recibe tratamiento para ambas patologías, desarrolla
un cuadro de artritis aguda en el tobillo derecho que cede a las pocas horas de la infusión
i.v. de colchicina. ¿Cuál de los siguientes fármacos NO suele producir esta
complicación?

1. 1. Isoniacida.
2. 2. Piracinamida.
3. 3. Etambutol.
4. 4. AAS a bajas dosis.
Gráfico de respuestas
Comentario

La isoniacida no es un fármaco que produzca mayor incidencia de hiperuricemia. Se asocia


típicamente a la presencia de hepatitis tóxica, dato fundamental para el examen MIR. Otros efectos
adversos incluyen dolor epigástrico, xerostomía, pelagra, hiperglucemia, acidosis metabólica,
retención urinaria y ginecomastia en los varones. También se han descrito síntomas parecidos a
los del lupus sistémico y artralgias.
También frecuentes suelen serlas neuropatías periféricas caracterizadas por parestesias en las
manos y los pies.(R1)

425. La enterocolitis necrotizante:

1. 1. Se produce por la sobreinfección con Streptococcus pneumoniae.


En estudios experimentales, puede ser prevenida por la alimentación neonatal con leche
2. 2.
materna.
3. 3. Aumenta el riesgo con uso de corticoides.
4. 4. Se acompaña de ascitis, derrame pleural y derrame pericárdico.
Gráfico de respuestas
Comentario

La respuesta correcta es la 2, ya que se puede prevenir con alimentación con leche materna. El
resto de respuestas son incorrectas. Recuerde que el uso de corticoides disminuy el riesgo de
enterocolitis nerotizante.(R2)

 
 
 
 
426. Niño de 2 meses que acude a Urgencias por tos intensa. Dos semanas antes había
comenzado con congestión respiratoria superior y tos leve, etiquetándose el cuadro por
un pediatra como un cuadro catarral. La tos empeoró hasta causar vómitos y episodios
de apnea con cianosis. Su madre también ha presentado tos durante varias semanas.
Durante el examen físico el niño comenzó a toser, persistiendo ésta durante más de 30
segundos, con estridor inspiratorio, y cierto grado de cianosis. Después de dejar de
toser, el niño estaba agotado, con llanto débil. En los exámenes de laboratorio se observó
una leucocitosis de 34,000 con un 85% de linfocitos. ¿Cuál es el agente etiológico más
probable de este cuadro?

1. 1. Adenovirus.
2. 2. Bordetella pertussis.
3. 3. Bordetella parapertussis.
4. 4. Rinovirus.
Gráfico de respuestas
Comentario

El cuadro que nos describen corresponde a una tos ferina. Esta enfermedad es una infección
producida principalmente por Bordetella pertussis.

Afecta preferentemente a menores de un año. Clínicamente produce un pródromo inespecífico, de


tipo catarral, y luego aparece la fase que caracteriza a la enfermedad, que es la de tos paroxística.
Origina accesos repentinos de tos, con series repetitivas de múltiples toses enérgicas,
acompañándose después de un ruido inspiratorio que recibe el nombre de “gallo”, al pasar aire a
través de una glotis cerrada. La tos, en ocasiones, se asocia con vómitos (tos emetizante).

Las complicaciones de la tos ferina son:

•   Neumonía: la más frecuente. Suele ser por sobreinfección bacteriana secundaria.


•   Convulsiones, cuya causa no está bien establecida.
•   Otras: prolapso rectal, hemorragia subconjuntival, hernia umbilical, epistaxis. Estas
complicaciones se deben a la intensa presión positiva intratorácica que se produce en
relación con la tos.

A pesar de tratarse de una infección bacteriana, es característica la presencia de linfocitosis en la


BH, como podemos ver en este caso. El tratamiento antibiótico de la tos ferina es la eritromicina
durante dos semanas.(R2)

427. ¿Cuál de las siguientes afirmaciones NO es cierta, en cuanto a las consecuencias


del sufrimiento fetal en el neonato?:

Durante las horas posteriores el neonato puede aparecer desde hipotónico hasta tener
1. 1.
hipertonía extrema o un tono aparentemente normal.
2. 2. En ningún caso aparecerá edema cerebral difuso secundariamente.
La bradicardia, la apnea y la falta de respuesta a estímulos, son indicadores de
3. 3.
encefalopatía hipóxico-isquémica fácilmente detectables.
4. 4. Puede aparecer actividad convulsiva.
Gráfico de respuestas
Comentario
La encefalopatía hipóxico- isquémica es el resultado de la afectación cerebral difusa que provoca
la hipoxia en los recién nacidos a término. Su clínica puede oscilar desde la hiperexcitabilidad

 
 
 
 
(casos leves) hasta la depresión del SNC (en los casos más graves). Con frecuencia estos niños
nacen deprimidos, y precisa medidas de reanimación profunda. Sus secuelas más importantes son
dos: parálisis cerebral y crisis comiciales.(R2)

428. Señale, entre las siguientes opciones, cuál sería la definición correcta de glucosa
alterada en ayunas:

1. 1. Glucemia tras sobrecarga oral de glucosa menor de 140 mg/dL.


2. 2. Glucemia tras sobrecarga oral de glucosa mayor de 200 mg/dL.
3. 3. Glucemia en ayunas comprendida entre 100 y 125 mg/dL.
4. 4. Glucemia en ayunas mayor de 125 mg/dL.
Gráfico de respuestas
Comentario

Se trata de una pregunta memorística sobre un punto bastante importante como son los criterios
de corte del diagnóstico de la Diabtes Mellitus. Se considera glucemia alterada en ayunas a los
valores comprendidos entre 100 y 125 mg/dl.(R3)

429. Paciente femenino de 65 años, con antecedentes de cáncer de mama, después de


varios años en tratamiento con tamoxifeno, presenta metrorragias. Es estudiada y se
diagnostica un adenocarcinoma de endometrio. La pieza quirúrgica de histerectomía
muestra, tras la estadificación completa y como única focalidad neoplásica, un
adenocarcinoma endometrioide, limitado al endometrio, con aproximadamente un 70%
de patrón de crecimiento sólido. Indíquese el estadio y grado:

1. 1. Ia G2.
2. 2. Ia G3.
3. 3. Ib G3.
4. 4. Ia G1.
Gráfico de respuestas
Comentario

Pregunta específica sobre estadiaje de cáncer de endometrio, no se preocupe si no lo sabe, no


suelen preguntarlo en el ENARM. Una masa tumoral única que no afecta al miometrio sólo puede
ser estadio Ia. El hecho de que tenga un 70% de patrón de crecimiento sólido nos dice que el
tumor está en su mayoría indiferenciado, por lo que corresponde clasificarlo como G3 (respuesta 2
correcta).(R2)

430. Varón de 34 años que acude a urgencias por fiebre de 40 ºC y dolor torácico que
aumenta con la tos y al hacer una inspiración profunda. En la placa de tórax presenta una
condensación homogénea en lóbulo superior derecho que no rebasa la cisura y también
un borramiento del seno costofrénico derecho. Se decide hacerle una prueba diagnóstica
para demostrar con más claridad la presencia de líquido pleural libre; ¿de qué prueba se
trata?

1. 1. Radiografía lateral en decúbito.


2. 2. Radiografía en espiración forzada.
3. 3. Toracocentesis diagnóstica.
4. 4. TAC.
Gráfico de respuestas

 
 
 
 
Comentario

El derrame pleural es un tema clave dentro de las enfermedades de la pleura, del que suelen
preguntar su etiología y tratamiento.

En este caso, se trata de una pregunta general sobre el diagnóstico de derrame, sin entrar en su
diagnóstico etiológico, que se orienta a través de las características del líquido. Ante la sospecha
clínica de derrame pleural, por la presencia de dolor pleurítico (aumenta con la inspiración, los
movimientos...) que es la manifestación más frecuente y que puede acompañarse de disnea y/o
fiebre, así como por encontrar un hemitórax dilatado con disminución de su movilidad, disminución
de la palpación de las vibraciones vocales, percusión mate y silencio auscultatorio, se debe realizar
una Rx de tórax donde podemos encontrar borramiento del seno costofrénico (primero del posterior
y después del lateral), el menisco de Damoisseau. Cuando se sospecha, se confirma si es o no
líquido libre, con una proyección en decúbito lateral sobre el lado afecto. Si el líquido se deposita
en la zona más declive, entonces decimos que es libre. Si no se moviliza, se habla de derrame
loculado, o encapsulado, y la mejor forma de localizarlo es con ecografía torácica.(R1)

431. Mujer de 34 años, nuligesta, diagnosticada de cáncer de ovario izquierdo, bien


diferenciado, con cápsula ovárica íntegra, sin tumor en la superficie y sin ascitis. Señale
cuál de las siguientes respuestas NO es correcta:

La cirugía en el cáncer de ovario tiene como objetivo la estadificación (diagnóstico) y la


1. 1.
extirpación de la mayor cantidad de tejido tumoral posible (tratamiento).
2. 2. En este caso, uno de los tratamientos de elección es la quimioterapia.
Dentro de las posibles vías de diseminación del cáncer de ovario, la más frecuente es la
3. 3.
siembra peritoneal o extensión peritoneal difusa.
Los marcadores tumorales ayudan a la orientación terapéutica, al seguimiento de la
4. 4.
enfermedad y a la valoración de la respuesta al tratamiento.
Gráfico de respuestas
Comentario

La respuesta correcta es la número 2, ya que en estadios IA y IB, G1 y G2, de cáncer de ovario no


es preciso el uso de terapia adyuvante. La respuesta 1 es correcta, puesto que la estadificación del
cáncer de ovario es postquirúrgica y el objetivo de la cirugía es eliminar la mayor parte de tejido
tumoral posible. La respuesta 3 sirve como repaso de las vías de diseminación del cáncer de
ovario, siendo la principal la extensión peritoneal. Es frecuente la diseminación linfática a ganglios
paraaórticos, siendo más infrecuente la diseminación hematógena, que si se da, suele ir precedida
de diseminación peritoneal y linfática. En cuanto a la opción 4: los marcadores tumorales tienen
valor principalmente para el seguimiento de la enfermedad y ver la respuesta al tratamiento de la
misma. Aunque son importantes en la orientación diagnóstica, no son exclusivos de una patología
concreta y pueden verse alterados en otras circunstancias.(R2)

 
 
 
 

432. Primigesta de 40+5 sem


que ingresa por trabajo de parto y 3 cm de dilatación. Se realiza monitorización interna
con amniorrexis artificial previa obteniéndose líquido amniótico claro. Ante la aparición
de la gráfica que muestra en la imagen, decide explorar a la paciente encontrando que la
paciente ha comenzado a sangrar de forma parecida a una menstruación y presenta dolor
a la palpación uterina e hipertonía uterina. Valoración obstétrica: longitudinal, cefálica, 6
cm dilatación y líquido amniótico meconial (+++). ¿Cuál sería la actitud CORRECTA?:

Suspender la infusión de oxitocina iv, colocar a la paciente en decúbito lateral izquierdo y


1. 1.
administración de oxigeno. Vigilancia de gráfica fetal y del estado materno.
Laparotomía urgente por sospecha de rotura uterina para extracción fetal inmediata y
2. 2.
sutura del desgarro uterino.
Realizaría una microtoma de sangre fetal y una comprobación de la coagulación materna
3. 3.
antes de decidir realizar una cesárea ante la sospecha de DPPNI.
4. 4. Extracción fetal inmediata mediante cesárea por la aparición de DPPNI.
Gráfico de respuestas
Comentario

Caso clínico de metrorragia del tercer trimestre de aparición intraparto y que se corresponde con
un Abruptio Placentae.

Los datos fundamentales para el diagnóstico son la aparición de dolor a la palpación uterina,
hipertonía y las alteraciones en la gráfica fetal. Si observa la gráfica verá que aparecen DIPS II
seguidos de una bradicardia fetal con recuperación parcial. Ante la sospecha de DPPNI estará
indicada la realización una cesárea urgente.(R4)

433. ¿Cuál de estas tiene depósitos subepiteliales?:

1. 1. GN membranoproliferativa tipo I.
2. 2. GN mesangial IgA.
3. 3. GN membranosa.
4. 4. Hialinosis focal y segmentaria.
Gráfico de respuestas
Comentario
No se debe fallar ninguna pregunta sobre los aspectos histológicos de las GN (microscopía óptica
y electrónica e inmunofluorescencia), ya que son preguntados frecuentemente en el MIR, por lo
que debes tener una buena tabla para estudiarlos. La microscopía electrónica es la técnica que
nos indica la localización de los depósitos, y en este caso nos dicen que la situación es
subepitelial, por lo que repasando mentalmente tu tabla, debes llegar a la conclusión de que las
GN que tienen depósitos subepiteliales son: GNRP tipos I y II (ya que la GNRP tipo III - opción 5-

 
 
 
 
NO tiene depósitos inmunes, es "pauci- inmune"), GNA post- estreptocócica y GN membranosa,
por lo que LA RESPUESTA CORRECTA ES LA 3.(R3)

434. Mujer de 25 años que consulta por hirsutismo, sin evidencia de claros signos de
virilización, y oligomenorrea. TA: 120/70 mmHg. Se detectan niveles altos de 17-
hidroxiprogesterona en plasma, que aumentan aún más tras el estímulo con ACTH. La
DHEA-s está elevada (al doble de lo normal). Testosterona ligeramente aumentada. LH y
FSH normales. Los 17-cetosteroides en orina están aumentados. ¿Cuál es el diagnóstico
de la paciente?:

1. 1. Déficit parcial de 11-hidroxilasa.


2. 2. Hiperplasia suprarrenal congénita, de inicio en el adulto por déficit de 21-hidroxilasa.
3. 3. Tumor ovárico virilizante.
4. 4. Hirsutismo idiopático.
Gráfico de respuestas
Comentario

Nos presentan una paciente joven con un cuadro de hirsutismo y oligomenorrea secundaria. Como
datos analíticos de interés, destacan niveles elevados de 17- hidroxiprogesterona (que aumentan
tras estímulo con ACTH), elevación de DHEA- s y elevación de 17- cetoesteroides en orina,
compatible con hiperplasia suprarrenal congénita.

Entre las cinco opciones existen dos formas de HSC: respuesta 2 (déficit de 21- hidroxilasa) y
respuesta 1 (déficit de 11- hidroxilasa). El déficit de 11- hidroxilasa se caracteriza por hipertensión
arterial y gran aumento de testosterona en sangre (no es el caso de la paciente que nos
presentan). Solo nos queda entonces la opción 2; recordar que las formas tardías la clínica es muy
similar al que aparece en el síndrome del ovario poliquístico (pero la paciente en esta patología no
presenta niveles elevados de 17- hidroxiprogesterona).(R2)

435. Indique la causa más frecuente de isquemia cerebral:

1. 1. Embolia de origen cardíaco.


2. 2. Arteriosclerosis de los vasos extracraneales e intracraneales.
3. 3. Alteraciones hematológicas.
4. 4. Arteritis infecciosas.
Gráfico de respuestas
Comentario
Pregunta muy importante, directa y sencilla que no se puede fallar. La causa mas frecuente de
isquemia cerebral es la presencia de arteriosclerosis tanto de vasos intracraneales y
extracraneales. Puede producir patología por embolización arterioarterial, o por estenosis u
oclusión in situ. La localización preferente de las placas de ateroma es la bifurcación carotídea y
origen de la arteria carótida interna. Las embolias de origen cardiaco son la causa mas común de
embolia cerebral. Y en segundo lugar, dentro de las causas embolicas se encuentran los émbolos
arterioarteriales. Las demás opciones son causa poco frecuente de ictus isquémicos.(R2)

436. Respecto a los anticonceptivos orales, señale cuál de las siguientes afirmaciones es
FALSA:

1. 1. Los estrógenos disminuyen las LDL y aumentan las HDL.

 
 
 
 
2. 2. Los progestágenos tienen un efecto aterogénico.
3. 3. Los estados de hipercoagulabilidad están relacionados con la dosis de progestágeno.
4. 4. Está aumentada la actividad fibrinolítica.
Gráfico de respuestas
Comentario

El control de la fertilidad es un tema muy preguntado. Lo más importante son los anticonceptivos
hormonales (AH) y el DIU.

Los estrógenos favorecen la lipólisis periférica, disminuyen la actividad de la lipoprotein lipasa,


aumentan las VLDL, HDL y APO-A1 y disminuyen las LDL. Aunque estimula la síntesis hepática de
TG, globalmente la influencia sobre las lipoproteínas se considera favorable y se acepta que son
antiaterogénicos y cardioprotectores. Sin embargo, producen un estado de hipercoagulabilidad
(respuesta falsa- 3).

Los progestágenos aumentan el depósito graso periférico, estimulan la lipoprotein lipasa e inhiben
la lipólisis. Inhiben los efectos estrogénicos, por lo que se consideran aterogénicos.

Los AH inducen una elevación de los factores de coagulación I, II, VII, IX, X y plasminógeno;
aunque también aumentan la actividad fibrinolítica.(R3)

437. Señale cuál de los siguientes fármacos aplicados en oncohematología tiene


propiedades inmunomoduladoras y antiangiogénicas:

1. 1. Imatinib.
2. 2. Melfalán.
3. 3. Lenalidomida.
4. 4. Azacitidina.
Gráfico de respuestas
Comentario
Imatinib es inhibidor de la proteína tirosina –cinasa de la LMC. Melfalán es un alquilante. Rituximab
es AcMo anti- CD20 para neoplasias linfoides B. Azacitidina es agente desmetilante de ADN para
SMD y leucemias agudas. Lenalidomida se emplea en mieloma múltiple fundamentalmente.(R3)

438. Un recién nacido a término, de adecuado peso, es atendido en la sala de partos. Nace
impregnado de líquido amniótico espeso, presenta hipotonía, bradicardia severa y apnea.
Señale la medida inicial de elección en su reanimación:

1. 1. Intubación y aspiración de la vía respiratoria.


2. 2. Aspiración orofaríngea y ventilación con bolsa.
3. 3. Aspiración orofaríngea y estimulación manual de la respiración.
4. 4. Aspiración orofaríngea y masaje cardíaco.
Gráfico de respuestas
Comentario

Pregunta muy importante. Hay una indicación muy concreta para prevenir la aparición del síndrome
de aspiración meconial: presencia de líquido meconial espeso. Los ginecólogos a veces lo
describen de manera extremadamente gráfica: “meconio en puré de chícharos”). Ante esto la
profilaxis se hará a través de aspiración de las secreciones respiratorias bajo visión directa de la
tráquea, con el niño previamente intubado para asegurar la vía aérea (medida más urgente).(R1)

 
 
 
 

 
 
 
 
439. Varón de 45 años, con antecedente de hiperuricemia, ingresado en el hospital hace
12 horas por hemorragia digestiva alta. Presenta de forma brusca inflamación de la rodilla
derecha con febrícula, siendo evidente en la exploración la presencia de derrame
articular. ¿Qué actitud le parece la más correcta?

Realizar artrocentesis para confirmar el diagnóstico e iniciar tratamiento con AINE por vía
1. 1.
oral.
2. 2. Administrar colchicina por vía oral.
3. 3. Combinación de AINES intramusculares y colchicina vía oral.
4. 4. Confirmación de gota por artrocentesis y administración de esteroides intrarticulares.
Gráfico de respuestas
Comentario

Ten mucho cuidado con esta pregunta, hay que leer hasta el último detalle, que para algo nos lo
habrán dado…

El caso clínico que nos presentan parece una artritis microcristalina. Dada la hiperuricemia previa y
la hospitalización, lo más probable es que se trate de una gota. En cualquier caso, habría que
hacer una artrocentesis, para descartar una artritis séptica, que precisaría un tratamiento muy
distinto (antibióticos). Las únicas opciones que realizan artrocentesis son la 1 y la 5.

La razón para elegir la respuesta 5 es el antecedente de HEMORRAGIA DIGESTIVA, que es su


motivo de ingreso. Por ello, estaría contraindicado utilizar AINEs en este momento, precisando
tratamiento local (esteroides intraarticulares), para minimizar el riesgo de resangrado.(R4)

 
 
 
 

440. Niño de 3 años que acude a su consulta


porque los padres refieren que, desde hace meses, presenta diarrea con heces muy

 
 
 
 
voluminosas y pastosas, así como dolor abdominal esporádico. Además, les llama
mucho la atención que el niño tenga episodios de irritabilidad frecuentes. Usted es su
pediatra y, repasando la historia clínica del niño, se encuentra con que ha consultado
varias veces por aftas bucales. Además, repasando sus curvas de peso y talla, observa
un claro estancamiento pondero-estatural desde los 15 meses de vida. En la exploración,
el niño presenta un aspecto fenotípico característico de la enfermedad que usted
sospecha y que se muestra en la imagen. Respecto a esta enfermedad, señale la opción
CORRECTA:

Es más habitual en el sexo masculino (3/1),como casi toda la patología digestiva en edad
1. 1.
pediátrica.
2. 2. Nunca se asocia con déficit de Ig A.
3. 3. Puede asociarse con enfermedades como la artritis idiopática juvenil.
4. 4. Nunca se presenta en adolescentes o adultos.
Gráfico de respuestas
Comentario

El cuadro clínico descrito en la pregunta es muy característico de enfermedad celíaca. Aunque la


presentación de la enfermedad es muy variable, la mayoría de los niños presentan diarrea con
heces grisáceas, abundantes y malolientes como las que los padres describen. También es típico
de la enfermedad, como se describe en el caso clínico, que los niños presenten mal carácter,
irritabilidad, secundarios a la malnutrición. Otros síntomas son anorexia, dolor abdominal y
presencia de aftas orales (descritos en la pregunta).

Estos niños suelen presentar un fenotipo característico como el que muestra la imagen con gran
distensión abdominal y atrofia muscular. Es más frecuente en el sexo femenino (respuesta 1:
falsa). Se puede asociar a otras enfermedades autoinmunes como déficit de IgA (respuesta 2
falsa), la Artritis Idiopática Juvenil (respuesta 3: CORRECTA), la tiroiditis autoinmune, la hepatitis
autoinmune, el LES etc.

Aunque el diagnóstico es más frecuente en la infancia, generalmente entre los 6 meses y los 2
años después de iniciar el gluten en la dieta, existen también casos subclínicos que se
diagnostican a edades más tardías e incluso en la edad adulta (respuesta 4: falsa).(R3)

441. Dada la sospecha diagnóstica que usted tiene con respecto al niño presentado en la
imagen anterior, ¿cuál cree que sería la actitud más CORRECTA?:

Tranquilizar a los padres, dado que se trata de una patología benigna y autolimitada que no
1. 1.
precisa de más estudios.
Realizar una biopsia intestinal como primera medida, dada la importancia del diagnóstico
2. 2.
anatomopatológico en estos casos, y sin necesidad de realizar estudios serológicos.
Solicitaría una determinación de anticuerpos IgA-antitransglutaminasa y, si estos son
3. 3. negativos, quedaría descartada la enfermedad celíaca sin hacer ninguna prueba
complementaria más.
Se debe realizar un estudio serológico incluyendo anticuerpos antitransglutaminasa y
recuento de Ig A total para apoyar el diagnóstico, pero independientemente del resultado
4. 4.
solicitaría una biopsia intestinal que permitiese confirmar el diagnóstico e instaurar el
tratamiento adecuado si se confirma el diagnóstico.
Gráfico de respuestas
Comentario

 
 
 
 
Se trata de una pregunta difícil sobre el diagnóstico y manejo inicial de un niño con sospecha de
enfermedad celíaca.

El caso clínico anterior nos ofrece suficientes datos clínicos como para que el pediatra tenga una
alta sospecha clínica de enfermedad celíaca. Por tanto no se puede no actuar ya que a largo plazo
los pacientes celíacos no tratados (retirada del gluten estricta y de por vida de la dieta) pueden
presentar complicaciones importantes como el linfoma intestinal (respuesta 1: falsa).

En el diagnóstico son muy útiles los anticuerpos antitransglutaminasa (sensibilidad cercana al


100%) por lo que ante la sospecha de la enfermedad deben solicitarse. Si estos son positivos,
apoya fuertemente el diagnóstico aunque para poder establecer el diagnóstico de certeza se
necesita de al menos una biopsia intestinal que demuestre atrofia vellositaria.

Si los anticuerpos son negativos, la clínica es tan sugerente que sigue siendo obligada la
realización de biopsia para descartar la enfermedad celíaca. En otras palabras: si la clínica es
sugerente, o no, pero los anticuerpos antitransglutaminasa son positivos, ó en los casos en que el
niño presente ambas cosas, se debe indicar biopsia (respuesta 4: correcta).

En este caso la clínica es suficientemente sugerente para realizar la biopsia (respuesta 3: falsa). La
biopsia debe realizarse sin haber retirado el gluten de la dieta (si no podría tratarse de un falso
negativo). En situaciones absolutamente excepcionales e infrecuentes (por ejemplo fallo hepático
debido a enfermedad celíaca no tratada) deberá retirarse el gluten de la dieta, antes de realizar la
biopsia intestinal. En nuestro caso no existe urgencia vital para retirar el gluten, por lo que primero
se realizará la biopsia intestinal.(R4)

442. Which of the following is FALSE regarding psoriasis?

In guttate psoriasis, small, red and scaly teardrop-shaped spots appear on the trunk, arms
1. 1.
and legs.
2. 2. Auspitz's sign is of no use in inverse psoriasis.
Discontinuation of systemic corticosteroid therapy can give rise to generalized pustulous
3. 3.
psoriasis.
The pathognomonic nail lesions of psoriasis, such as "oil drop" distal leukonyquia are not
4. 4.
the most frequent.
Gráfico de respuestas
Comentario

Una pregunta de dificultad media sobre la psoriasis. Analicemos todas las opciones:

1.- La psoriasis en gotas aparece en tronco, normalmente a edades muy jóvenes y en relación con
infecciones faríngeas por estreptococo, teniendo mejor pronóstico.

2.- El signo de Auspitz recibe el nombre de signo del rocío hemorrágico y se pone de manifiesto
mediante el raspado metódico de Brocq. Dada la ausencia de descamación de la psriasis de
pliegues, no puede practicarse el raspado y por tanto el signo no es útil.

3.- El tratamiento esteroideo sistémico no debe realizarse en la psoriasis, puesto que su retirada
puede desencadenar formas graves de la enfermedad, como la psoriasis pustulosa.

4.- La respuesta incorrecta es ésta, ya que en la psoriasis no existen lesiones ungueales que
puedan considerarse patognomónicas.(R4)

 
 
 
 
443. Una de las siguientes afirmaciones sobre la patogenia de la preeclampsia es FALSA;
¿cuál?

1. 1. El péptido natriurético auricular aumenta significativamente en la preeclampsia.


2. 2. Las alteraciones hepáticas son frecuentes en la preeclampsia.
3. 3. Existe una disminución de la síntesis de tromboxano A2.
4. 4. La perfusión renal y la filtración glomerular están disminuidas.
Gráfico de respuestas
Comentario

Pregunta más o menos complicada, dado que nos piden recordar la fisiopatología de la
preeclampsia. La secuencia general de la enfermedad está recogida en una tabla resumen dentro
del manual.

La etiología de la preeclampsia es desconocida. En las mujeres con factores predisponentes se


produce una alteración en la inmunotolerancia que produce isquemia y liberación de
vasoconstrictores (como el tromboxano A2), provocando disfunción endotelial sistémica que
conlleva hipoperfusión generalizada (secundario a vasoespasmo e hipercoagulabilidad) a nivel de
distintos órganos. Simplemente con recordar la secuencia básica nos damos cuenta que la
respuesta incorrecta es la opción 3 (el tromboxano aumenta, no disminuye).

Además, se produce hemoconcentración, retención de sodio y edema extracelular secundario a


hipoproteínemia (en un intento de compensar la retención de sodio se produce aumento en la
producción de PAN; respuesta 1 correcta) y afectación hepática secundario a necrosis
hepatocelular (respuesta 2 correcta).(R3)

444. En la cardiopatía isquémica es esencial la utilización de fármcaos antiplaquetarios.


NO es cierto en relación a ellos:

El ácido acetilsalicílico bloquea irreversiblemente la ciclooxigenasa 1, y por lo tanto la


1. 1.
producción de tromboxano A2.
2. 2. El triflusal inhibe irreversiblemente la ciclooxigenasa plaquetaria.
El dipiridamol es un inhibidor de la fosfodiesterasa, aumentando la concentración
3. 3.
intraplaquetaria de AMPc.
4. 4. Abciximab es un inhibidor de la fosodiesterasa.
Gráfico de respuestas
Comentario
Tanto el AAS como el triflusal son inhibidores de la poducción de tromboxano A2 a través de la
inhibición de la ciclooxigenasa. El dipiridamol es un inhibidor de la fosodiesterasa, pero no es un
antiplaquetario de primera linea y sus indicaciones se limitan a la posible utilización en
combinación con AAS. El clopidogrel pertenece al grupo de las tienopiridinas, produciéndose su
efecto a través del antagonismo sobre el receptor plaquetaio difosfato de adenosina. Eptifibatide,
tirofibán y abciximab son antiagregantes por el mecanismo de inhibición de la glicoproteïna
IIb/IIIa.(R4)

445. Masculino de 64 años que ingresa en la unidad coronaria por dolor torácico
prolongado de 2 horas de duración, con elevación de 2 mm del ST de V2 a V4. Se decide
realizar fibrinólisis con estreptocinasa. A los 15 minutos cede el dolor del enfermo y en
el monitor se observa una taquicardia regular de QRS ancho a 150 lpm sin ondas P

 
 
 
 
(previamente el enfermo tenía un QRS estrecho). El enfermo solo nota palpitaciones, con
una TA de 120/70 mmHg. ¿Cuál sería su actitud?

1. 1. Perfusión de lidocaína.
2. 2. Perfusión de procainamida.
3. 3. Observación.
4. 4. Coronariografía urgente.
Gráfico de respuestas
Comentario

Las arritmias que pueden ocurrir en la fase aguda del infarto son variadas. La más importante por
su repercusión es la fibrilación ventricular primaria, que es la causa más frecuente de mortalidad
antes de llegar al hospital. La fibrilación ventricular secundaria (que es la que aparece pasadas las
primeras 48-72 horas) es muy grave y a diferencia de la FV primaria afecta gravemente al
pronóstico a largo plazo del paciente. Las taquicardias ventriculares monomórficas sostenidas no
suelen ser muy frecuentes en la fase aguda del IAM. El tratamiento es similar al de las TVMS fuera
de la fase aguda del IAM. Si el paciente está hemodinámicamente estable se puede hacer un
intento de cardioversión farmacológica y en ese caso el antiarrítmico de elección es la lidocaína,
procainamida o la amiodarona iv. Si no cede, o si fuese hemodinámicamente inestable, se
realizaría cardioversión eléctrica.

Tenga cuidado con esta pregunta y no confundas el caso clínico con un ritmo idioventricular
acelerado (RIVA). Recuerda que el RIVA se produce en relación con la reperfusión de las arterias
coronarias… Pero no alcanza una frecuencia cardíaca por encima de 100, como en este caso. El
RIVA oscila entre 60- 80 lpm. Por encima de 100, se trata de una taquicardia ventricular.(R1)

446. ¿Cuál de las siguientes NO le parece causa de ictericia con dilatación de la vía biliar
(obstructiva)?

1. 1. Ampuloma.
2. 2. Tumor de Klatskin.
3. 3. Pancreatitis crónica.
4. 4. Síndrome de Rotor.
Gráfico de respuestas
Comentario

De entre las que se citan, el síndrome de Rotor no produce ictericia obstructiva. Es un trastorno
caracterizado por un aumento de la bilirrubina a expensas de su fracción directa, debido a un
trastorno en el almacenamiento hepático que altera el drenaje de la bilirrubina directa. El tumor de
Klatskin (tumor a nivel de la bifurcación del conducto biliar común) y el ampular (tumor a nivel
papilar) son tumores de la vía biliar que también producen obstrucción. La pancreatitis crónica, que
a veces cursa con fibrosis a nivel de cabeza, puede comprometer el drenaje biliar.(R4)

447. ¿Qué vacuna se debe administrar a los 18 meses de edad?

1. 1. DPT y polio oral.


2. 2. Pentavalente acelular.
3. 3. Solo polio oral.
4. 4. Solo DT.
Gráfico de respuestas
Comentario

 
 
 
 
Según el esquema nacional de vacunación, a los 18 meses se debe administrar la 4 dosis de
vacuna pentavalente (difteria, tosferina, tétanos, polio e infecciones por H. influenzae tipo B).(R2)

448. ¿Cuál de las siguientes afirmaciones sobre la rubéola es cierta?:

1. 1. Su período de incubación es más breve que el del sarampión.


2. 2. Se acompaña de trombopenia y artritis.
3. 3. La erupción brota tras desaparecer la fiebre.
4. 4. Aparece en forma epidémica, cada 2 a 4 años.
Gráfico de respuestas
Comentario

Pregunta de dificultad media- alta sobre la rubéola. Debemos saber que el período de incubación
de la rubéola es algo mayor que el del sarampión (2- 3 semanas frente 1- 2 semanas), por lo q la
opción 1 es incorrecta. La erupción que brota tras desaparecer la fiebre es característica de el
exantema súbito (opción 3 incorrecta) y la rubéola aparece anualmente, más frecuentemente en
primavera (opción 4 incorrecta). Así llegamos a opción correcta que es la 2 y que debemos
recordar: la rubéola se puede acompañar de artritis (típicamente en mujeres o niños postpúberes,
afectando a las articulaciones de pequeño tamaño, sobre todo metacarpofalángicas) y de
trombopenia, que aparece en 1 de cada 3000 pacientes.(R2)

449. Paciente de 78 años,


diabético insulinodependiente y bronquítico crónico, que hace 7 días empezó con un
cuadro de tos con expectoración mucopurulenta, fiebre y dolor pleurítico izquierdo. Se
automedicó con paracetamol y actualmente acude a Urgencias por agravamiento de su
estado general, persistencia de la fiebre y disnea progresiva hasta hacerse de reposo. En
la exploración física se evidencia matidez a la percusión y silencio en la auscultación en
hemitórax izquierdo. En la imagen nº 9 se muestra la radiografía de tórax.La saturación
es del 88%. De las siguientes afirmaciones sobre diagnóstico y tratamiento de este
enfermo marque la afirmación que considere FALSA:

 
 
 
 
Probablemente el paciente esté afectado de tuberculosis pulmonar que se ha complicado
1. 1.
con una pleuritis tuberculosa, por lo que debe recibir tratamiento antituberculoso.
Probablemente el paciente presente una neumonía bacteriana complicada con un derrame
2. 2. pleural, por lo que debe ser ingresado en el hospital y recibir tratamiento antibiótico
intravenoso.
Dentro de las etiologías que se deben considerar hay que tener en cuenta neumococo y
3. 3.
gramnegativos debido a la comorbilidad asociada del paciente.
Un tratamiento empírico hospitalario que podría iniciarse sería una cefalosporina de 3ª
4. 4. generación (o amoxicilina-clavulánico a dosis elevadas) con un macrólido. Como
alternativa podría utilizarse levofloxacino.
Gráfico de respuestas
Comentario

La radiografía nos muestra una imagen de derrame pleural izquierdo, dada la opacidad en dicho
pulmón en forma de menisco. Ante todo derrame pleural se debe intentar hacer una toracocentesis
para estudiar bioquímicamente y microbiológicamente el líquido. Las principales etiologías y el
tratamiento son los que aparecen en las respuestas 3 y 4 que son correctas.

Si te fijas bien, verás cómo las respuestas 1 y 2 no pueden ser verdaderas al mismo tiempo, por lo
que para resolver la pregunta solo tienes que decidir cuál de las dos es falsa.

Teniendo en cuenta la patología de base de este paciente (DM y broncópata crónico,


probablemente tratado con corticoides), y la evolución del cuadro, es más probable que se trate de
una neumonía bacteriana, que de una tuberculosis (respuesta: 1 falsa). Esta última presentaría
además un cuadro más subagudo e insidioso, con febrícula, pérdida de peso, sudoración nocturna,
etc. Recuerda que en los ancianos con patología de base, el tratamiento se debe realizar de forma
hospitalaria mediante cefalosporinas de 3ª generación y macrólidos o levofloxacino (respuesta: 5
correcta).(R1)

 
 
 
 

 
 
 
 

 
 
 
 

 
 
 
 

 
 
 
 

 
 
 
 

 
 
 
 

 
 
 
 

 
 
 
 

 
 
 
 

 
 
 
 

 
 
 
 

 
 
 
 

 
 
 
 

 
 

También podría gustarte